You are on page 1of 498

02_Pre Calculus_TP.

pdf 1 14/7/2017 4:01:13 PM

Episteme Foundation Textbooks

PRE-CALCULUS &
CALCULUS FOR
PRE-UNIVERSITY AND
UNIVERSITY
STUDENTS
THE SINGAPORE APPROACH
Assoc. Prof. Dr. Asma Ahmad Shariff
Prof. Dr. Ibrahim Mohamed
Dr. Fadzilah Abd. Manaf
Yang C2
C. Sivakumaran
Ooi Liu Qi
Pre-Calculus & Calculus for Pre-University and University Students. The Singapore Approach.
Fundación Episteme
Singapore Asia Publishers Pte Ltd

ISBN: 978-958-56313-1-1

© Singapore Asia Publishers Pte Ltd


© Assoc. Prof. Dr. Asma Ahmad Shariff
© Prof. Dr. Ibrahim Mohamed
© Dr. Fadzilah Abd. Manaf

Edición: julio de 2017

Todos los derechos reservados. Prohibida la reproducción o cita impresa o electrónica total o parcial de esta obra, sin autori-
zación expresa y por escrito de la Fundación Episteme.
All rights reserved. Reproduction or printed or electronic or full or partial citation of this work is forbidden without the express
written permission of the Episteme Foundation.

Carrera 13 # 75 – 75 Segundo piso


www.fundepisteme.com
gerente@fundepisteme.com
Bogotá D.C.

Diseño y diagramación: Singapore Asia Publishers Pte. Ltd.


Impresión: Grafi-impacto

00 Copyright.indd 1 18/7/2017 9:37:27 AM


PRESENTACIÓN

La Fundación Episteme, es una organización privada de la sociedad civil, sin ánimo de lucro, que tiene
como objeto el estímulo, apoyo, mejoramiento y promoción de la educación, tanto a nivel básico (grados
0 a 11) como superior (técnica, tecnológica y universitaria). A partir de los ejes de acceso universal a
la educación, busca mejorar la calidad, el fortalecimiento de las políticas educativas y el apoyo a la
implementación de actividades científicas y tecnológicas, como principio fundamental para el logro de
mayores índices de desarrollo económico y bienestar social.

En un esfuerzo por mejorar la calidad de la educación, la Fundación Episteme viene implementando


desde inicios del año 2017 una alianza estratégica con EDUCARE, Cooperativa de Maestros Públicos
de Singapur, entidad creada con el apoyo del gobierno de ese país que reúne a los mejores profesores
y expertos en pedagogía en distintas áreas del conocimiento incluyendo las matemáticas, quienes han
logrado ubicar a su país en el primer lugar del ranking mundial de las pruebas PISA - 2015, en las cuales
Singapur sigue estando por encima de China (Hong Kong), Canadá, Finlandia, Irlanda, Estonia, Korea,
Japón y Noruega.

Como desarrollo de esta alianza, la semana comprendida entre el 19 y 24 de Junio de 2017, se realizó
en Bogotá una capacitación avanzada en metodologías pedagógicas de las matemáticas por parte del
Profesor Toh Tim Lam (Ph.D), director del área de matemáticas del Instituto Nacional de Educación de
Singapur, entidad encargada de la formación de los maestros de ese país, dirigida al equipo de profesores
de la Universidad Sergio Arboleda.

La Fundación continúa implementando una alianza estratégica con la empresa Singapore Asia Publishers
-SAP-, una de las más antiguas editoriales de Singapur, cuyos expertos diseñaron el presente texto con
la retroalimentación de la decanatura de matemáticas de La Universidad Sergio Arboleda, siguiendo la
filosofía pedagógica del sistema educativo de Singapur.

El libro que hoy presentamos está dirigido a los estudiantes de la Universidad Sergio Arboleda, cada uno
de los capítulos ha sido diseñado para fortalecer el pensamiento y la lógica matemática, a partir de una
aproximación pedagógica basada en una visión práctica que privilegia la aplicación de los conocimientos
matemáticos a la vida diaria.

Miguel Ceballos Arévalo


Fundación Episteme

01 Contents.indd 1 18/7/2017 9:54:10 AM


CONTENTS

Chapter 1 Numerical Systems 1


1.1 Real Numbers 2
1.2 Exponential Expressions 30
1.3 Logarithms 37
1.4 Radicals 41

Chapter 2 Algebraic Expressions 46


2.1 Operations on Algebraic Expressions 47
2.2 Factorisation of Algebraic Expressions 69

Chapter 3 Algebraic Fractions 85


3.1 Simplification of Algebraic Fractions 86
3.2 Operations of Algebraic Fractions 89
3.3 Rationalization of Algebraic Fractions 99

Chapter 4 Equations in One Variable 100


4.1 Linear Equations 101
4.2 Quadratic Equations 108
4.3 Equations with Degree  3 117
4.4 Radical Equations 123
4.5 Exponential Equations 126
4.6 Logarithmic Equations 130
4.7 Applications of Equations 133

Chapter 5 Two-variable Linear Equations 144


5.1 Graphs of Linear Equations 145
5.2 Linear Systems with Two Variables 154
5.3 Graphical Method 155
5.4 Substitution Method 158
5.5 Elimination Method 161
5.6 Equalization Method 164
5.7 Applications of Systems of Equations 166

Chapter 6 Three Linear Equations with Three Variables 175


6.1 Substitution Method 177
6.2 Elimination Method 179
6.3 Inverse Matrix Method 181
6.4 Gauss-Jordan Elimination Method 183
6.5 Applications 185

Chapter 7 Inequalities 188


7.1 Intervals on the Real Number Line 189
7.2 Properties of Inequalities 195
7.3 Linear Inequalities 196
7.4 Quadratic Inequalities 199
7.5 Applications of Inequalities 205

Chapter 8 Graphs of Quadratic Equations 207


8.1 Parabolas 208
8.2 Circles 218
8.3 Ellipses 235
8.4 Hyperbolas 245

Pre-Calculus and Calculus.indb 2 13/7/2017 5:56:50 PM


Chapter 9 Trigonometry 257
9.1 Angles 258
9.2 Pythagorean Theorem 263
9.3 Trigonometric Ratios 267
9.4 Fundamental Trigonometric Identities 274
9.5 Applications of Trigonometry 277

Chapter 10 Functions 281


10.1 Definition of a Relation 282
10.2 Definition of a Function 287
10.3 Domain and Range of a Function 292
10.4 Composite Functions 298
10.5 Inverse Functions 303

Chapter 11 Graphs 309


11.1 Transformation of Graphs 310
11.2 Linear Functions 312
11.3 Quadratic Functions 315
11.4 Absolute Functions 318
11.5 Rational Functions 320
11.6 Radical Functions 324
11.7 Exponential and Logarithmic Functions 327
11.8 Trigonometric Functions 337
11.9 Inverse Trigonometric Functions 345
11.10 Hyperbolic Functions 348

Chapter 12 Limits and Continuity 351


12.1 Introduction to Limits 352
12.2 Properties of Limits 356
12.3 Limits at Infinity 361
12.4 Horizontal and Vertical Asymptotes 368
12.5 Continuity 375

Chapter 13 Differentiation 381


13.1 Geometric Interpretation of Derivatives 382
13.2 Definition of Derivative Functions 383
13.3 The Derivative as a Function 388
13.4 Differentiation Rules 389
13.5 Second and Higher-order Derivatives 398
13.6 Implicit Differentiation 399
13.7 Derivatives of Exponential and Logarithmic Functions 403
13.8 Derivatives of Trigonometric Functions 410
13.9 Derivatives of Inverse Trigonometric Functions 418
13.10 Linearization and Differentials 419
13.11 L’Hôpital’s Rule 424

Chapter 14 Applications of Differentiation 428


14.1 Equations of Tangent and Normal 429
14.2 Related Rates 433
14.3 Maximum and Minimum Values 439
14.4 Graphing Using First and Second Derivatives 453
14.5 Optimization Problems 461
14.6 Applications in Economics and Business 464

Pre-Calculus and Calculus.indb 3 13/7/2017 5:56:50 PM


The Real Number System
Chapter 1:

Numerical Systems

Global Warming: Is it real?

Global warming is the increase in the average temperature of the near-surface air and
oceans of the Earth happening since the mid-twentieth century. During the twentieth
century, global surface temperature increased by 0.74 ± 0.18 ºC or 1.33 ± 0.32 ºF. Signed
numbers like 70.18 and ±0.32 indicate increase or decrease of quantities. These signed
numbers, called integers along with fractions and irrational numbers, are real numbers.
Real numbers are used to measure all quantities in this world like global warming.
Any number that you use in real life is a real number. Any number that is not ‘real’ is
‘imaginary’. Imaginary numbers are those numbers whose squares are negative or less
than zero. An imaginary number b can be expressed as a real number multiplied by the
imaginary unit bi where i2 = –1, such as bi.

Pre-Calculus and Calculus.indb 1 13/7/2017 5:56:50 PM


Chapter 1 • Numerical Systems

1 Numerical Systems
1.1 Real Numbers
1.2 Exponential Expressions
1.3 Logarithms
1.4 Radicals

1.1 Real Numbers


1.1.1 Definition of Real Numbers
1.1.2 Properties of Real Numbers
1.1.3 Operations of Integers
1.1.4 Operations of Fractions
1.1.5 Operations of Decimals
1.1.6 Applications of Real Numbers

1.1.1 Definition of Real Numbers


Real Numbers
The real number system consists of two large sets of numbers – rational
numbers and the irrational numbers.
Rational Irrational Let us look at sets of numbers which are subsets of rational numbers. The
Numbers Numbers first is what we called natural numbers or counting numbers. The symbol
usually used for these numbers is .
 = {1, 2, 3, 4, 5, ...}
The set of natural numbers combined with the number 0 gives us another
set called the whole numbers, W.
W = {0, 1, 2, 3, 4, 5, ...}
Integers consist of the set of natural numbers, the number 0 and the
negative of the natural numbers. The symbol we use is .
 = {..., –5, –4, –3, –2, –1, 0, 1, 2, 3, 4, 5, ...}
 is also known as positive integers, with symbol +. The set of negative
integers is represented by symbol –. Positive integers 2, 4, 6, 8, … are
called even numbers, and positive integers 1, 3, 5, 7, … are called odd
numbers.
Generally, the set of even numbers can be represented as {2n, ∈ }.
E.g. 2(2) = 4
2(50) = 100
4 and 100 are even numbers.
On the other hand, the set of odd numbers can be represented generally as
{2n – 1, ∈ }.
E.g. 2(4) – 1 = 7
2(17) – 1 = 33
7 and 33 are odd numbers.
2

Pre-Calculus and Calculus.indb 2 13/7/2017 5:56:50 PM


Chapter 1 • Numerical Systems

A prime number is a natural number greater than 1 which has 1 and itself
as factors. The first few prime numbers are 2, 3, 5, 7, 11, 13, ...

Rational numbers are formally defined as follows:

Definition 1.1
p
__
Rational numbers are numbers which can be written in the form ​​  q ​​,
where p and q are integers and q ≠ 0. The symbol  is used for rational
numbers.
p
 = ​{x: x = ​  q ​, p, q  , q ≠ 0}​​
__

7
3 ___
__
From the definition above, all numbers in fractional form such as ​​  5 ​​, –​​  10  ​​, 
​ ​17
___
4  ​​ are rational numbers.
__ 7 __ 2 __ 0
Every integer is also a fraction, for example, –7 = –​​  1 ​​ , 2 = ​​  1 ​​ and 0 = ​ ​q ​​,
(q ≠ 0), thus  is a subset of .
Hence, we have  ⊂ W ⊂  ⊂ .
Note
Besides the integers, numbers in decimal form as described below are also
The symbol ∈ means ‘an
rational numbers:
element of’.
(i) Numbers in decimal which terminates, such as 1.325, –10.47, 0.333. The symbol ⊂ means
‘subset of’.
(ii) Numbers where the decimal representations are repeating and
nonterminating, such as –3.134134134..., 0.236572365723657...,
1.33333.... (the dots ‘…’ mean ‘and so on’). These numbers are
sometimes written as –3.134, 0.23657, 1.3 where the bar indicates
the repeating numbers.
These numbers can be written as fractions.

Example 1
Write the following numbers as fractions.
(a) 1.325 (b) 1.3 (c) 0.45

SOLUTION
325 __________
1000 + 325 _____1325
(a) 1.325 = 1 + 0.325 = 1 + ​​ _____
1000  ​​ = ​​  1000 ​​  = ​​  1000 ​​ 

(b) Let x = 1.3 = 1.3333... ——— (1)


Multiply both sides by 10:  10x = 13.333...  ——— (2)
Subtract (1) from (2): 9x = 12
12 ​​ = __
x = ​​ ___ 4
9 ​​  3 ​​ 
(c) Let x = 0.45 = 0.454545... ——— (1)
Multiply both sides by 100: 100x = 45.454545...  ——— (2)
Subtract (1) from (2): 99x = 45
45 ___ 5
x = ​​ ___
99 ​​ = ​​  11  ​​ 
3

Pre-Calculus and Calculus.indb 3 13/7/2017 5:56:50 PM


Chapter 1 • Numerical Systems

Irrational numbers are those numbers which cannot be written as


fractions. We use symbol for irrational numbers.

Examples of irrational numbers are


Note
Decimal representations for irrational numbers are non-terminating and
π non-repeating. Take note that is an irrational number. In calculations,
​​  22
___
7 ​​  
the value of is sometimes written in approximations such as π ≈ 3.142
​​ 22
or π ≈ ___ 7 ​​ .
The sets , W, ,  and are subsets of a bigger set called the real
number set . Figure 1a shows the relationship of all the sets in the real
number system.

Real Numbers

Irrational Rational
Numbers Numbers 

Integers 
(–  0  +)

Whole
Numbers W

Natural
Numbers 
Figure 1a The real number system

Pre-Calculus and Calculus.indb 4 13/7/2017 5:56:50 PM


Chapter 1 • Numerical Systems

Example 2
State whether the following statements are true (T) or false (F).
(a) All real numbers are rational.
(b) All rational numbers are integers.
(c) All natural numbers are rational.
(d) All irrational numbers are real.
π
(e) ​​ __
2 ​​  is a rational number.
(f) 1.212121... is irrational.
(g) {x: x  , x  3} = {0, 1, 2, 3}
(h) {x: x  } = {..., –4, –3, –2, –1, 0, 1, 2, 3, 4, ...}

SOLUTION
(a) F (b) F (c) T (d) T
(e) F (f) F (g) F (h) F

1.1.2 Properties of Real Numbers


Real numbers have properties involving fundamental operations. These
properties make calculations systematic.

The set of real numbers is closed under some operations. This means that
when the operation is performed on real numbers, the result is always a
real number.

Definition 1.2

The closure property states that, for any real number a and b,
a + b is a real number,
a – b is a real number, and
ab is a real number.

In other words, the closure property states that Note


• the sum of two or more real numbers is always a real number, The set of real numbers is
• the difference between two real numbers is always a real number, not closed on division.
• the product of two or more real numbers is always a real number. For instance, 5 ÷ 0 is
undefined, and it is not a real
Definition 1.3 number.

The commutative property states that, for any real number a and b,
a + b = b + a (commutative property of addition)
ab = ba (commutative property of multiplication)

In other words, the commutative property states that


• changing the order of the addends will result in the same sum,
• changing the order of the factors will result in the same product.

Pre-Calculus and Calculus.indb 5 13/7/2017 5:56:50 PM


Chapter 1 • Numerical Systems

Example 3
Complete each statement using the commutative property.
(a) 7 + 3 (b) (7)(3)

SOLUTION
(a) 7 + 3 = 3 + 7 = 10
(b) (7)(3) = (3)(7) = 21

The commutative property does not apply to subtraction and division.


E.g.  5 – 1 = 1 – 5       10 ÷ 2 = 2 ÷ 10
4 ≠ –4 ​​ 15 ​​ 
5 ≠ __
Note
This shows that changing the order of the minuend and the subtrahend
These different results
contradict the rule of gives a different result. Also, changing the order of the dividend and the
commutativity. divisor gives a different quotient.

Definition 1.4

The associative property states that, for any real number a, b and c,
(a + b) + c = a + (b + c) (associative property of addition)
(ab)c = a(bc) (associative property of multiplication)

In other words, the associative property states that


• changing the grouping of the addends will result in the same sum,
• changing the grouping of the factors will result in the same product.

Example 4
Change the grouping of the numbers and evaluate.
(a) (3 + 5) + 4 (b) (5 × 6) × 7

SOLUTION
(a) (3 + 5) + 4 = 3 + (5 + 4)

8+4=3+9
= 12
(b) (5 × 6) × 7 = 5 × (6 × 7)

30(7) = 5(42)
= 210

Pre-Calculus and Calculus.indb 6 13/7/2017 5:56:50 PM


Chapter 1 • Numerical Systems

The associative property does not apply to subtraction and division.


Note
E.g. (10 – 5) – 3 = 10 – (5 – 3)
This shows that changing
the grouping of the numbers
5 – 3 = 10 – 2 involved does not guarantee
2≠8 the same result.
(12 ÷ 6) ÷ 2 = 12 ÷ (6 ÷ 2)

2 ÷ 2 = 12 ÷ 3
1≠4

Definition 1.5

The distributive property states that, for any real number a, b and c,
a(b + c) = ab + ac
(b + c)a = ba + ca

This property states that multiplication can be distributed over addition of


real numbers.
Example 5
Apply the distributive property and evaluate.
(a) 2(2 + 3) (b) (2 + 3)10

SOLUTION
(a) Applying the distributive property,
2(2 + 3) = 2(2) + 2(3)
2(5) = 4 + 6
= 10
(b) Applying the distributive property,
(2 + 3)10 = 2(10) + 3(10)
5(10) = 20 + 30
= 50

Definition 1.6

The identity property states that:


(i) For any real number a, there exists a real number 0, such that
a+0=0+a=a
0 is called the additive identity or identity element for addition.
(ii) For any real number a, there exists a real number 1, such that
a×1=1×a=a
The real number 1 is called the multiplicative identity or identity
element for multiplication.

In other words, the identity property states that


• the sum of 0 and any real number is equal to that real number,
• the product of 1 and any real number is equal to that real number. 7

Pre-Calculus and Calculus.indb 7 13/7/2017 5:56:50 PM


Chapter 1 • Numerical Systems

Example 6
Perform the indicated operations.
(a) 0 + 9 (b) 7 × 1

SOLUTION
(a) 0 + 9 = 9 (b) 7 × 1 = 7

Definition 1.7
Note The inverse property states that:
The reciprocal of zero (0) (i) For any real number a, there exists a real number –a, such that
does not exist. a + (–a) = (–a) + a = 0
The real number –a is called the additive inverse or negative of a.
1
__
(ii) For any non-zero real number a, there exists a real number ​​ a ​​ such
that
__ 1 __ 1
a × ​​  a ​​ = ​​  a ​​ × a = 1
The real number __ ​​  1a ​​is called the multiplicative inverse or reciprocal
of a.

In other words, the inverse property states that


• the sum of a real number and its additive inverse (negative) is 0,
• the product of a real number and its multiplicative inverse (reciprocal)
is 1.

Example 7
Fill in the blanks with the correct number to complete each equation.
(a) 8 + = 0 (b) 20 × =1

SOLUTION 1
___
(a) 8 + –8 = 0 (b) 20 × ​ 20   ​ = 1

1.1.3 Operations of Integers


The set of integers, also known as signed numbers, is composed of whole
numbers together with the negative of natural numbers.
Real numbers can be arranged along a line containing an infinite number of
points called a number line. Every point on the number line corresponds
to a unique real number. The points corresponding to real numbers are
Note ordered and evenly spaced so that points to the right are greater than points
The arrowheads indicate to the left.
that the number line and 3 units 3 units
the numbers continue
indefinitely. –5 –4 –3 –2 –1 0 1 2 3 4 5

negative zero positive


8

Pre-Calculus and Calculus.indb 8 13/7/2017 5:56:51 PM


Chapter 1 • Numerical Systems

Number lines are helpful in understanding and visualizing the fundamental


operations involving integers.

Addition of Integers
Consider the following problem.
A man climbed 5 kilometers up a hill and planted trees for reforestation.
Then, he walked down the hill for 3 kilometers and rest. At what height is
he from his starting point now?
We can solve the problem using a number line.
+5
–3

–5 –4 –3 –2 –1 0 1 2 3 4 5

From 0, move the arrow to 5 and then add –3 by moving 3 units to the left.
The process represents the equation 5 + (–3) = 2. Hence, the mountain
climber is 2 kilometers away from his starting point.
For positive numbers +a : move a units to the right.
For negative numbers –a: move a units to the left.
Example 8
Add the following using the number line.
(a) –3 + 2 (b) –1 + (–4)

SOLUTION
(a) –3 + 2
From 0, move 3 units to the left, then 2 units to the right, ending
with –1.
–3
+2

–5 –4 –3 –2 –1 0 1 2 3 4 5
Hence, –3 + 2 = –1
(b) –1 + (–4)
From 0, move 1 unit to the left, then 4 units to the left, ending with
–5.
–4
–1

–5 –4 –3 –2 –1 0 1 2 3 4 5
Hence, –1 + (–4) = –5

Summary

• To add integers with the same signs, add their absolute values, then
use their common sign.
• To add integers with different signs, find the difference between
their absolute values, then use the sign of the integer with greater
absolute value. 9

Pre-Calculus and Calculus.indb 9 13/7/2017 5:56:51 PM


Chapter 1 • Numerical Systems

Example 9
Find the sum.
(a) 7 + 8
(b) –4 + (–2)
(c) –7 + 2 + (–3)

SOLUTION
Note
(a) 7 + 8
The sum of two or more Find the absolute values of the addends: |7| = 7 and |8| = 8
positive numbers is always Find the sum of the absolute values: 7 + 8 = 15
positive.
Use the common sign to the sum: 7 + 8 = 15
(b) –4 + (–2)
Note Find the absolute values of the addends: |–4| = 4 and |–2| = 2
The sum of two or more Find the sum of the absolute values: 4 + 2 = 6
negative numbers is always Use the common sign to the sum: –4 + (–2) = –6
negative.
(c) –7 + 2 + (–3)
Add the first two addends: Subtract their absolute values, then affix
Note the sign of the number with the greater absolute value.
The sum of a positive –7 + 2 = –5
number and a negative Add the third addend to the sum (–5): Add their absolute values,
number is either positive
then affix the common sign.
or negative, depending on
which number has the greater –5 + (–3) = –8
value. Hence, –7 + 2 + (–3) = –8

Subtraction of Integers
Is subtraction related to addition? Study the following table.
Subtraction Addition
4 – (2) = 2 4 + (–2) = 2
5 – (–1) = 6 5 + (+1) = 6
4 – (–2) = 6 4 + (+2) = 6
–3 – (1) = –4 –3 + (–1) = –4

Observe that subtracting an integer from another integer can be expressed


as adding the opposite of that integer to the other.
Note
When the symbol ‘–’ Summary
is written between two To subtract an integer, add its opposite.
numbers, it is considered as For any numbers a and b, a – b = a + (– b).
subtraction. However, if it is
written in front of a number,
it is considered as a negative
sign.

10

Pre-Calculus and Calculus.indb 10 13/7/2017 5:56:51 PM


Chapter 1 • Numerical Systems

Example 10
Subtract, then illustrate the subtraction process using a number line.
(a) 3 – (–2) (b) –1 – 3

SOLUTION
(a) Find the opposite of –2, then proceed to addition. The opposite of
–2 is 2, and then add it to 3.
Recall that on a number line, adding a positive number n is moving
n units to the right, and adding a negative number n is moving n
units to the left.
Subtraction is just the opposite. To subtract a positive number n,
move n units to the left, and to subtract a negative number n, move
n units to the right.
3
2

–5 –4 –3 –2 –1 0 1 2 3 4 5
Hence, 3 – (–2) = 3 + 2 = 5
(b) –1 – 3 = –1 + (–3) = –4 Note
opposite of 3 Subtracting a positive
proceed to addition number is the same as adding
–3 a negative number.
–1 Subtracting a negative
number is the same as adding
–5 –4 –3 –2 –1 0 1 2 3 4 5 a positive number.

Multiplication of Integers
Consider the following problem.
Suppose that scientists observed that the temperature in a very cold place
steadily decreases by 3 ºC each day during the month of December. What
is the temperature after 5 days if during the first day, the temperature was
–15 ºC?
The process can be written as a repeated addition.
(–3) + (–3) + (–3) + (–3) + (–3)
This addition expression can be simplified as a multiplication expression
5(–3).
5(–3) = (–3) + (–3) + (–3) + (–3) + (–3)
= –15

11

Pre-Calculus and Calculus.indb 11 13/7/2017 5:56:51 PM


Chapter 1 • Numerical Systems

Using a number line, multiplication can be visualized as a series of steps


moving to the right when the number is positive and moving to the left
when the number is negative.
The multiplication process 5(–3) = –15 is illustrated on the number line
below.

–16 –15 –14 –13 –12 –11 –10 –9 –8 –7 –6 –5 –4 –3 –2 –1 0 1

If the temperature on the first day was –15 ºC and decreased by 15 ºC after
5 days, then the temperature on the fifth day is –30 ºC.

Summary
• To multiply integers, multiply their absolute values.
• The product of two numbers with the same sign is positive.
• The product of two numbers with different signs is negative.

Example 11
Multiply using repeated addition, then illustrate the process using a number line.
(a) 4(2) (b) 3(–4) (c) (–3)(–2)

SOLUTION
(a) 4(2) = 2 + 2 + 2 + 2 = 8

0 1 2 3 4 5 6 7 8 9 10
(b) 3(–4) = (–4) + (–4) + (–4) = –12

–12 –11 –10 –9 –8 –7 –6 –5 –4 –3 –2 –1 0 1 2

(c) The multiplication expression 3(–2) means three times of 2 units to the left.
Then, (–3)(–2) means three times of 2 units in the opposite direction.
Hence, (–3)(–2) = 6

–2 –1 0 1 2 3 4 5 6

12

Pre-Calculus and Calculus.indb 12 13/7/2017 5:56:51 PM


Chapter 1 • Numerical Systems

Example 12
Note
Multiply the following.
Multiplication Sign Rules
(a) (15)(–3) (d) (–4)(–15)(–12)
(+)(+) = (+)
(b) (–12)(10) (e) (0)(–23)
(–)(–) = (+)
(c) (–24)(–3) (–)(+) = (–)
(+)(–) = (–)
SOLUTION
(a) (15)(–3)
Find the absolute values: |15| = 15 and |–3| = 3
Multiply the absolute values: (15)(3) = 45
Since the integers have different signs, the product is negative.
Hence, (15)(–3) = –45
(b) (–12)(10)
Find the absolute values: |–12| = 12 and |10| = 10
Multiply the absolute values: (12)(10) = 120
Since the integers have different signs, the product is negative.
Hence, (–12)(10) = –120
(c) (–24)(–3)
Find the absolute values: |–24| = 24 and |–3| = 3
Multiply the absolute values: (24)(3) = 72
Since the integers have the same sign, the product is positive.
Hence, (–24)(–3) = 72
(d) (–4)(–15)(–12)

60
–720 Note
Hence, (–4)(–15)(–12) = –720 Any number multiplied by
zero is equal to zero.
(e) (0)(–23) = 0

Division of Integers
Dividing integers is the inverse operation of multiplying integers.
Note
Division Multiplication
Division Sign Rules
10
___ (+) (–)
10 ÷ 2 = ​​  2 ​​ = 5 5(2) = 10 ___
​​   ​​ = (–)  ​​ ___ ​​ = (+)
(–) (–)
(–10) (–) (+)
(–10) ÷ (–2) = _____
​​   ​​ = 5 5(–2) = –10 ​​ ___  ​​ = (–)  ​​ ___  ​​ = (+)
(–2) (+) (+)

Observe that the quotient of two integers with the same sign is positive.
The quotient of two positive integers is positive. The quotient of two
negative integers is positive.

13

Pre-Calculus and Calculus.indb 13 13/7/2017 5:56:51 PM


Chapter 1 • Numerical Systems

Division Multiplication
–10
(–10) ÷ 2 = ___
​​  2 ​​  = –5 (–5)(2) = –10

10
10 ÷ (–2) = _____
​​ –2 ​​ = –5 (–5)(–2) = 10

Observe that the quotient of two integers with different signs is negative.
a
In general, if a is an integer and b is a non-zero integer, then ​​ __ ​​ = c, which
b
means bc = a.
a is called the dividend, b the divisor and c the quotient.

Summary
• To divide integers, divide their absolute values.
• The quotient of two numbers with the same sign is positive.
• The quotient of two numbers with different signs is negative.

Example 13
Find the quotient of each expression.
–36 144 ​​   –192
(a) ​​ ____
12 ​​   (b) ​​ ____
–24 (c) ​​ _____
–6 ​​ 

SOLUTION
–36
(a) ​​ ____
12 ​​ 
Find the absolute values: |–36| = 36 and |12| = 12
36
Find the quotient of the absolute values: ___
​​ 12 ​​ = 3
Since the integers have different signs, the quotient is negative.
–36
Hence, ____
​​  12 ​​ = –3
144 ​​ 
(b) ​​ ____
–24
Find the absolute values: |144| = 144 and |–24| = 24
​​ 144
Find the quotient of the absolute values: ____
24 ​​ = 6
Since the integers have different signs, the quotient is negative.
Hence, ____ 144 ​​ = –6
​​  –24
–192
(c) ​​ _____
–6 ​​  

Find the absolute values: |–192| = 192 and |–6| = 6


192
Find the quotient of the absolute values: ____
​​  6 ​​  = 32
Since the integers have the same sign, the quotient is positive.
–192
Hence, _____
​​  –6 ​​   = 32

14

Pre-Calculus and Calculus.indb 14 13/7/2017 5:56:51 PM


Chapter 1 • Numerical Systems

Example 14
Find the quotient of each expression.
0 0 25
(a) ​​ ___
10  ​​   (b) ​​ ___
–8  ​​   (c) ​​ ___
0 ​​ 

SOLUTION Note
0 Zero divided by any
(a) ​​ ___
10  ​​ = 0 since 0(10) = 0. non-zero number equals zero.
0
(b) ​​ ___
–8  ​​ = 0 since 0(–8) = 0. Note
25 25
(c) ​​ ___ ___
0 ​​ is undefined. ​​  0 ​​ ≠ 0 since 0(0) ≠ 25. Division by zero is not
possible.

1.1.4 Operations of Fractions


Fractions are rational numbers. Fractions whose numerator is 1 are called
unit fractions.

A fraction whose numerator is less than its denominator is called a


proper fraction. A fraction whose numerator is greater than or equal to
its denominator is called an improper fraction. A number mixed with a
fraction is called a mixed number.
Fractions with the same denominator are called similar fractions.
Fractions with different denominators are called dissimilar fractions.

Types Examples Note


The denominator of a
Unit Fraction ​​  1 ​​ , __
__
2 3 4 5 6 7​​  1 ​​ , __
​​  1 ​​ , __
​​  1 ​​ , __
​​  1 ​​ , __
​​  1 ​​  fraction can never be zero.

• All unit fractions


Proper Fraction 3 __
2 ​​ , __ 5 __ 7 ___ 11
• ​​ __
3 ​​  4 ​​ , ​​  7 ​​ , ​​  9 ​​ , ​​  13  ​​
__ 3 5 __ 6 7 ___ 10 16
Improper Fraction ​​   ​​ , __
​​   ​​ , ​​   ​​ , __
3 5 5 2 3 13 ​​   ​​ , ​​   ​​ , ___
​​   ​​ 

Mixed Number 1 ​​ , 3​​ __


2​​ __ 1 __
1 __ 1
2 4 ​​ , 5​​  6 ​​ , 6​​  4 ​​ 
3 4 __ 5
Similar Fractions ​​  1 ​​ , __
__
6 6 6 6 6 ​​  2 ​​ , __
​​   ​​ , __
​​   ​​ , ​​   ​​ 

3 7
Dissimilar Fractions ​​  1 ​​ , __
__
2 3 4 5 7 9​​  1 ​​ , __
​​  1 ​​ , __
​​  4 ​​ , __
​​   ​​ , __
​​   ​​ 

15

Pre-Calculus and Calculus.indb 15 13/7/2017 5:56:51 PM


Chapter 1 • Numerical Systems

Addition of Fractions
Summary
• To add similar fractions, add their numerators and copy the common
denominator.
• To add dissimilar fractions,
Step 1: Find the lowest common multiple (LCM) of the
denominators. The LCM of the denominators is called the
lowest common denominator (LCD) of the fractions.
Step 2: Change the addend fractions to equivalent fractions with
the LCD as the denominator.
Step 3: Add the similar fractions.

Example 15
Add the following.
–5 __ 7 5 __ 2
(a) ​​ ___
9 ​​ + ​​  9 ​​   (b) ​​ ___
12  ​​ + ​​  3 ​​ 

SOLUTION
–5 __ 7 ______ –5 + 7
(a) ​​ ___
9 ​​ + ​​  9 ​​  = ​​  9 ​​   

2
= ​​ __
9 ​​ 
5 2  ​​ is 12.
(b) The LCD of ___
​​ 12  ​​ and ​​ __
3

12  ​​ + ​​  3 ​​  = ​​  12  ​​ + ​​  3( ​  44 ​ )​​  


5 __ 2 ___ 5 __ 2 ​​​​   __
​​ ___
5 ___ 8
= ​​ ___
12  ​​ + ​​  12  ​​ 
5+8
= ​​ _____
12 ​​ 

13 1
= ​​ ___ ___
12 ​​ or 1​​  12  ​​ 

Example 16
–15 23
Find ____
​​  21 ​​ + ____
​​  –35  ​​ 
.

SOLUTION
–15 ____ 23 ____ –15 ____ –23
​​ ____
21 ​​ + ​​  –35  ​​ = ​​  21 ​​ + ​​  35 ​​ 
–15 –23
The LCD of ____
​​  21 ​​ and ​​ ____
35 ​​ is 105.
 ​   ​ )​​  + ____
​​   ​​ = ​​   ​​​​ (  ​   ​ )​​  
​​   ​​​​ (
–15 –23 ____
____ –15 __ 5 –23 __ 3
​​   ​​ + ____
21 35 21 5 35 3
–75 ____ –69
= ​​ ____
105 ​​ + ​​  105 ​​ 
–75 + (–69)
= ​​ __________
105 ​​   

–144 ​​ 
= ​​ _____
105
48 13
16 = –​​ ___ ___
35 ​​ or –1​​  35 ​​ 

Pre-Calculus and Calculus.indb 16 13/7/2017 5:56:51 PM


Chapter 1 • Numerical Systems

Subtraction of Fractions
Summary
• To subtract similar fractions, find the difference of their numerators
and copy the common denominator.
• To subtract dissimilar fractions,
Step 1: Find the LCD of the fractions.
Step 2: Change the fractions to equivalent fractions with the LCD
as the denominator.
Step 3: Subtract the similar fractions.

Recall that subtraction is the opposite process of addition. Thus,


c a
subtracting ​​ __ ​​ from __
​​   ​​is the same as adding the opposite or negative of
d b
__ c a
​​   ​​ to __
​​   ​​.
d b

Example 17
Subtract the following.
8
2  ​​ – __ 4 __ 5
(a) ​​ __
9 ​​  9 ​​   (b) –​​ ___
28  ​​ – ​​  7 ​​ 

SOLUTION
8 _____
2  ​​ – __ 2–8
(a) ​​ __
9 ​​  9 ​​  = ​​  9 ​​   

–6
= ​​ ___
9 ​​  Note
2(3)
= –​​ ____ ​​  In subtracting fractions, write
3(3)
the subtraction statement as
2  ​​
= –​​ __ an addition statement, then
3
proceed to addition.
4 __ 5
(b) Write the subtraction expression –​​ ___ 28  ​​ – ​​  7 ​​  as an addition statement,
28  ​​ + (​​  –​  7 ​ )​​. 
4 5
–​​ ___ __

4 5
The LCD of –​​ ___ __
28  ​​ and –​​  7  ​​ is 28.
Performing addition,

28  ​​ + (​​  –​  7 ​ )​​  = –​​  28  ​​ + (​​  –​  7 ​ )(​​​​    ​  4 ​ )​​  
4 5 4 5 __ 4
–​​ ___ __ ___ __

4 ____ –20
= –​​ ___
28  ​​ + ​​  28 ​​ 
–4 + (–20)
= ​​ _________
28 ​​   

–24 ​​ 
= ​​ ____
28
6(4)
= –​​ ____ ​​ 
7(4)
6
__
= –​​  7  ​​

17

Pre-Calculus and Calculus.indb 17 13/7/2017 5:56:51 PM


Chapter 1 • Numerical Systems

Example 18
Note
(​  16  ​)​​. 
–5
1  ​​ – ​​  ___
In adding or subtracting
Solve ___
​​  56
fractions, the denominators
must be the same. SOLUTION

56  ​​ – (​​  ​  16 ​ )​​  = ​​  56  ​​ + ​​  16  ​​ 


1 –5 1 ___ 5
​​ ___ ___ ___

56 ( ​  22 ​ )​​  + ___  ​​​​   ​  7 ​ )​​  


​​  16 (
1  ​​​​   __ 5 __ 7
= ​​ ___
2   ​​ + ____ 35
= ____
​​  112 ​​  112  ​​ 
2 + 35
= ​​ ______ 112 ​​   

37
= ____ ​​  112  ​​ 

Multiplication of Fractions
Summary

To multiply fractions, multiply the numerators, then multiply the


denominators, that is,
__ a c _____ (a)(c) ___ ac
​​   ​​ × __
​​   ​​ = ​​   ​​ = ​​   ​​  , where b ≠ 0 and d ≠ 0.

b d (b)(d) bd

Example 19

​  7 ​ )(​​​​    ___
​  6 ​ )(​​​​    –​ ___   ​​ and express the answer in the lowest terms.
Multiply (​​  __ 10  ​)
3 –2 9
Note
The easier way to multiply SOLUTION
​  7 ​ )(​​​​    ___ 10  ​ )​​  = ​​  (7)(6)(10) ​​ 
​  6 ​ )(​​​​    –​ ___
(3)(–2)(–9)
(
fractions is to try to cancel 3 –2 9
common factors from the ​​  __ __________

numerator and denominator. 9


= ​​ ___
70  ​​ 

Division of Fractions
Summary

To divide fractions, multiply the dividend by the reciprocal of the


divisor, that is,
__ a c __ a d ___ ad
​​   ​​ ÷ __
​​   ​​ = ​​   ​​ × __
​​   ​​ = ​​    ​​  , where b ≠ 0 and d ≠ 0.
b d b c bc

Dividing a number by a non-zero number is the same as multiplying it by


the reciprocal of the non-zero number.
The reciprocal of a fraction is called the multiplicative inverse of the
fraction.
​​  1a ​​.
The reciprocal of a is denoted by a–1 and a–1 = __

18

Pre-Calculus and Calculus.indb 18 13/7/2017 5:56:52 PM


Chapter 1 • Numerical Systems

Example 20
Divide the following and express the answers in the lowest terms.

8 ( 16  ​ )​​  
3 3
11 ​​ ÷ ​​  –3​ ___
(a) –​​ __
7 ​​  ÷ (–39) (c) ​​ ___
4 ___
(b) –​​ ___ 1
15  ​​ ÷ ​​  30  ​​ 

SOLUTION
3 3 ___ –1
(a) –​​ __ __
7 ​​  ÷ (–39) = –​​  7  ​​ × ​​  39 ​​ 
1
= ​​ ___
91  ​​ 
4 ___ 1 4 ___ 30
(b) –​​ ___ ___
15  ​​ ÷ ​​  30  ​​ = –​​  15  ​​ × ​​  1 ​​ 
= –8

8 ( 16  ​ )​​  = ​​  8 ​​ ÷ (​​  –​  16 ​ )​​  


3
11 ​​ ÷ ​​  –3​ ___ 11 51
(c) ​​ ___ __ ___

8 ​​ × (​​  –​  51 ​ )​​  


16
​​  11
= ___ ___

22
= –​​ ___
51  ​​

1.1.5 Operations of Decimals


Terminating decimals and repeating decimals are also rational numbers.
The fundamental operations can be performed on decimals.
Addition and Subtraction of Decimals

Summary

To add or subtract decimals:


Step 1: Write the decimals with the decimal points aligned in one
column.
Step 2: Add or subtract as in whole numbers.
Step 3: Affix the decimal point of the sum in the same column as the
addends.

Example 21
Add the following.
(a) 2.3 + 4.65 + 0.875 (b) 3.45 + 1.965 – 2.03

SOLUTION
(a)
2.300 (b) 3.450
4.650 + 1.965
+ 0.875 5.415

7.825 – 2.030
3.385

19

Pre-Calculus and Calculus.indb 19 13/7/2017 5:56:52 PM


Chapter 1 • Numerical Systems

Multiplication of Decimals

Summary

To multiply decimals:
Step 1: M ultiply the decimals as in whole numbers.
Step 2: C ount the total number of decimal places in the factors.
Step 3: From the rightmost digit of the product, count an equal number
of decimal places as those in the two factors.

Example 22
Multiply the following.
(a) 3.72 × 2.3
(b) –8.256 × 0.34
(c) 3.45 × 103

SOLUTION
(a) 1
2
3.72
×2.3
1 116
+7 44

8.556
3.72 × 2.3 = 8.556
(b) 11
1 22
–8.256
×0.34
1 1
33024
Note +2 4768

To find the product of a –2.80704
decimal and any power of –8.256 × 0.34 = –2.807 04
10, move the decimal point
(c) 3.45 × 103 = 3.450 = 3450
to an equal number of places
as the exponent of 10 to the
right if it is positive, and to Division of Decimals
the left if it is negative.
Summary

To divide decimals:
Step 1: Make the divisor a whole number by multiplying both the
divisor and the dividend by the same multiple of 10.
Step 2: D ivide as in whole numbers.
Step 3: Place the decimal point in the quotient in the same position as
the dividend.

20

Pre-Calculus and Calculus.indb 20 13/7/2017 5:56:52 PM


Chapter 1 • Numerical Systems

Example 23
Divide the following. Note
(a) 15.4 ÷ 3.2 (b) 18.975 ÷ 8.25 When dividing a number by
a decimal, always change
SOLUTION the decimal divisor to a
15.4 154
(a) The fraction ____ , equivalent to ​​ ____
​​  3.2 ​​  32 ​​ is obtained by multiplying the
whole number by moving
its decimal point to the end
numerator and the denominator by 10. This makes the divisor a of the divisor. Move the
whole number. decimal point of the dividend
Using long division, the same number of places.
4.8125
3.2 15.4 0000
Note
12 8
Add zeros so that the
2 60
division process is complete.
2 56
40
32 Note
80 Check:
64 4.8125 × 3.2 = 15.4

160 dividend
160 divisor
quotient
0
15.4 ÷ 3.2 = 4.8125
(b) 2.3
Note
8.25 18.97 5 Check:
16 50 2.3 × 8.25 = 18.975
2 47 5 dividend
2 47 5 divisor
quotient
0
18.975 ÷ 8.25 = 2.3

1.1.6 Applications of Real Numbers


While most applications and concepts of mathematics are often associated
with scientific work in specialized fields of study, mathematics could also
be applied in daily life, such as in doing financial transactions at retail
outlets and managing personal finances. Other examples are commission,
hire purchase, interest, money exchange, profit and loss, taxation and
utilities. As you do calculations, concepts of ratio, percentage, proportion
and rate are involved.

21

Pre-Calculus and Calculus.indb 21 13/7/2017 5:56:52 PM


Chapter 1 • Numerical Systems

Ratio and Percentage


Note
Ratio and percentage are basic methods of comparing one quantity with
Ratio and percentage are
another quantity. Ratio is a comparison between two quantities. It is usually
convertible to each other
and are essentially the same
written as a quotient of two numbers. Percentage is a ratio as a fraction of
in arithmetic problems. A 100. It uses the symbol for percent which is %. For example, the ratio of
ratio could also be stated as a adults to children at a party is 4 : 5, which implies that the number of adults
fraction, for example, 4 : 5 is is a multiple of 4 and the number of children is an equivalent multiple of
​​  45 ​​ .
stated as __ 5, such as 16 adults and 20 children.
Here is another example: the percentage of the students in a school who
wear spectacles is 55%. This implies that if the total number of students in
a school is 100, then 55 of those students wear spectacles.

Example 24
Write the following phrases in ratio form.
(a) The ratio of three oranges to seven apples in a basket
(b) The ratio of five cats to 15 mice in a park
(c) The ratio of 50 police officers to 100 citizens in a city
(d) The ratio of 1 centimeter to 1 meter
(e) The ratio of 24 hours to 1 day

SOLUTION
Remember that a ratio can also be expressed as a fraction and a
percentage.
3
(a) The ratio of three oranges to seven apples is 3 : 7 or __
​​ 7  ​​.
5
Note (b) The ratio of five cats to 15 mice in a park is 5 : 15 or ___ ​​ 15  ​​ . We can
Remember that a ratio is a simplify it to the lowest terms by dividing the whole ratio or the
relationship between two
numbers of the same kind or numerator and the denominator by 3. Hence, the ratio is 1 : 3 or __ ​​ 13  ​​.
unit of measure. 50
(c) The ratio of 50 police officers to 100 citizens is 50 : 100 or ____
​​ 100  ​​  .
1
__
Simplifying the ratio, we get 1: 2 or ​​ 2 ​​ .
We can also express the ratio as a percentage. Since it is a fraction
of 100, the ratio is equivalent to 50%.
(d) There are 100 centimeters in a meter. Hence, the ratio of one
centimeter to one meter is 1 : 100 or ____ 1   ​​ 
​​ 100 .
(e) There are 24 hours in a day. Hence, the ratio of 24 hours to 1 day is
​​ 24
24 : 24 or ___
24 ​​ . In lowest terms, the ratio is 1:1 or 1.

22

Pre-Calculus and Calculus.indb 22 13/7/2017 5:56:52 PM


Chapter 1 • Numerical Systems

Example 25
A school has 500 students. If the ratio of girls to boys is 2 : 3, how many
girls are there?

SOLUTION
Total number of units → 2 units + 3 units
= 5 units
5 units → 500
1 unit → 500 ÷ 5
= 100
2 units → 100 × 2
= 200
Hence, there are 200 girls.

Most ratio problems involve a comparison between two quantities. We call


those ratios as two-term ratios. However, there are also ratios that involve
three or more quantities.

Example 26
Marc, Chris and Joseph shared $18 000 in a ratio 2 : 3 : 4. How much
did Joseph receive?

SOLUTION
Total number of units → 2 units + 3 units + 4 units
= 9 units
9 units → $18 000
1 unit → $18 000 ÷ 9
= $2000
4 units → $2000 × 4
= $8000
Hence, Joseph received $8000.
This could be done simply by taking ​​ 49  ​​of $18 000.
__

Proportion
Evaluating ratios may also involve proportion, a mathematical statement
that indicates equality between two ratios. Proportions are important
especially in measuring very small and very large quantities.
a x
Consider this proportion: ​​ __ ​​ = __
​​   ​​  , where b ≠ 0 and y ≠ 0. The quantities
b y
a and y are called the extremes, and the quantities b and x are called the
means. We can easily identify the parts when the ratios are written in the
following form.
means

a:b=x:y

23
extremes

Pre-Calculus and Calculus.indb 23 13/7/2017 5:56:52 PM


Chapter 1 • Numerical Systems

50
​​ 12 ​​  = ​​ ____
In the proportion, __ 100  ​​ 
, the numbers 1 and 100 are the extremes, and
the numbers 2 and 50 are the means. To verify whether the two given ratios
are in proportion, we can apply the fundamental property of proportion.

Definition 1.8

Two ratios are in proportion if and only if the product of the extremes
is equal to the product of the means. In symbols, given a proportion
__ a __x
​​  b ​​ = ​​  y  ​​ , where b ≠ 0 and y ≠ 0,
ay = bx.

Example 27
Determine whether each pair of ratios is in proportion.
3 12 1  ​​ and ​​ __
1  ​​
(a) ​​ __ ___
5  ​​ and ​​  20 ​​   (c) ​​ __
4 2
7 1 50 40
(b) ​​ ___ __
21  ​​ and ​​  3  ​​ (d) ​​ ____ ____
100  ​​ and ​​  100  ​​ 

SOLUTION
3 ___ 12
(a) ​​ __
5  ​​ = ​​  20 ​​ 
3(20) = 5(12)
= 60
Since the product of the extremes is equal to the product of the
means, the two ratios are in proportion.
7 __ 1
(b) ​​ ___
21  ​​ = ​​  3 ​​ 
7(3) = 1(21)
= 21
The two ratios are in proportion.
1  ​​ = __
(c) ​​ __ 1
4 ​​  2 ​​ 
1(2) ≠ 1(4)
Since the product of the extremes is not equal to the product of the
means, the two ratios are not in proportion.
50 ____ 40
(d) ​​ ____
100  ​​ = ​​  100  ​​ 
50(100) ≠ 40(100)
The two ratios are not in proportion.

24

Pre-Calculus and Calculus.indb 24 13/7/2017 5:56:52 PM


Chapter 1 • Numerical Systems

Example 28
Fill in the missing number in the box.

1   ​​ = ___ 2  ​​   6


(a) ​​ ___ ​​  24 (b) ​​ ___ ___
6 ​​ = ​​  36  ​​ 

SOLUTION
×2

1
(a) ​​     ​​ = ___
___ 2  ​​ 
​​  24

×2

× 2 = 24
= 12

×6

6
(b) ​​ ___ ___
6 ​​ = ​​  36  ​​ 
×6
×6=6
=1

Example 29
The ratio of the heights of building A and building B is 7 : 12. If
building A is 20 m, what is the height of building B?

SOLUTION
7 units → 20 m
1 unit → 20 m ÷ 7
20
= ​​ ___
7 ​​ m
20
12 units → ​​ ___
7 ​​ m × 12
240
= ____
​​  7 ​​  m

2
= ​34​ __
7 ​​  m
2  ​​ m.
Hence, the height of building B is 34​​ __
7

Rate
Rate is a ratio between two different quantities. Speed and currency
exchange rates are examples of rate.
Speed is the rate at which distance is covered in a unit time. For example,
the speed of a car traveling along a road is 60 km/h means if the car were
to travel at that exact speed for an hour, it would have covered a distance
of 60 km. With constant speed, the time and the distance traveled can be
calculated when one of the quantities is given.
25

Pre-Calculus and Calculus.indb 25 13/7/2017 5:56:52 PM


Chapter 1 • Numerical Systems

Currency exchange rates state the value of one currency against a unit of
another currency. For example, the exchange rate of the US dollar to the
Singapore dollar is S$1.41 for every US$1.00. This exchange rate may be
inverted to know how much US dollar is required to obtain S$1.00.

Example 30
Darren’s electricity usage last month was 310 kWh at his home.
Electricity is charged at $0.20 per kWh by the service provider. This
month, the rate was increased by 0.75% and Darren reduced his
electricity charges by 5%. How much is his electricity charges this
month inclusive of a 7% goods and services tax?

SOLUTION
Darren’s electricity usage this month after reduction in usage
= 310 – (5% of 310)
​  100   ​ × 310)​​  
= 310 – (​​  ____
5

= 310 – 15.5
= 294.5 kWh
Rate of payment for usage after increase = $0.20 + (0.75% of $0.20)
​  100 ​ × $0.20)​​  
= $0.20 + (​​  ____
0.75

= $0.20 + $0.0015
= $0.2015
Electricity charges to be paid =
 294.5 kWh × $0.2015
= $59.34175
Electricity charges inclusive of GST =
 $59.34175 + (7% of $59.34175)
= $59.34175 + $4.15392
= $63.50 (nearest cent)

Example 31
A water tank can hold 1000 l. It can be filled by a pipe at a rate of
2 l/min. How long does it take to fill a tank?

SOLUTION
Total capacity
_______________________
Time taken (min) =    ​​     ​​
Rate of water to fill the tank
1000 l
= ​​ ______   ​​ 
2 l/min
= 1000 l ​(​  _____ ​ ​​  
2l )
​  1 min
    

= 500 min
Hence, it takes 500 minutes or 8​​ __ 1  ​​hours to fill the tank.
3

26

Pre-Calculus and Calculus.indb 26 13/7/2017 5:56:52 PM


Chapter 1 • Numerical Systems

Exercise 1.1
1. Express each of these numbers as a fraction.
(a) 1.8
(b) 0.21
(c) 1.345
(d) 0.555
(e) 1.651

2. Classify these numbers into rational numbers and irrational numbers:
2 ​​ , 5π, 5.144, e2, ___
–​​ __ ​​  22 ​​ , ​​  __
5 7 √ 7 ​​,  3.142, 0, 5.3215467..., 0.010101...

3. State whether the following statements are true or false.


e
(a) ​​ __
2  ​​is a rational number.
___
(b) ​​√ –4 ​​ is a real number.
__
(c) 2​​√ 2 ​​ is an irrational number.
(d) 1.12345 is an irrational number.
(e) Every irrational number is a real number.
(f) Every rational number is an integer.
(g) Every prime number is a natural number.
__
(h) (​​√ 2 ​​ – 1) is a rational number.

4. Set A is given as

A ={ 6 ​ , 2π, ​ 3 ​,  –5​  3 ​ , –2, 100, 5.123, 4.919191..., 0.12345...}​​. 
2e
​  11
​​  0.11, ___ ___ __ 1

List the elements of A which belong to the following real number sets.
(a) 
(b) 
(c) 
(d)
(e) 

5. Perform the indicated operations. Show the process on a number line.


(a) 7 + 8 (f) –10 – 7
(b) 9 + (–14) (g) –23 – (–14)
(c) –6 + 12 (h) 15 – (–6)
(d) –13 + (–5) (i) –12 + (–7) – 9
(e) 8 – (–9) (j) –5 – 13 + 24 27

Pre-Calculus and Calculus.indb 27 13/7/2017 5:56:52 PM


Chapter 1 • Numerical Systems

6. A
 n airplane flies at an altitude of 460 m. Directly below it, a submarine travels at 235 m below sea
level. How far vertically is the airplane from the submarine?

7. A
 boat traveled 50 km east, then 75 km west, then 20 km east. How far is the boat from its starting
point?

8. Perform the indicated operations.


1 ____ 1 3 4
(a) ​​ ___
10  ​​ + ​​  100   ​​   (d) 9​​ __ __
4 ​​  – 4​​  5 ​​ 
7 __ 5 __ 3 27
1 ​​  × ___
(b) ​​ ___
18  ​​ + ​​  9 ​​  – ​​  2 ​​   (e) 7​​ __
3 ​​  12 ​​ 
3
1 ​​  ÷ 3​​ ___
(c) ​​ __
8 16  ​​ 

5 ​​  3 ​​  ÷ ​​  3( ​  7 ​ )​​  + __


8 __
–2 ​​ × __ 10
4 ​​​​   2 – ___ –3
9. What is the simplified form of  ​​ ___ ​​  12 ​​  ÷ ​​ ___
5 ​​ ?

3 ​​  5 ​​  ÷ (​​  1 – ​ 5 ​ )​​  + ​​  3  ​​ ÷ ​​  15 ​​ ?


3
1  ​​ × __ 4 2 ___ –8
10. What is the simplified form of  –​​ __ __ __

1 ​​  m2. If one side is 6​​ __


11. A rectangular grass lawn has an area of 25​​ __ 2 ​​  m long, how long is the other side?
6 3

12. Perform the indicated operations.


(a) 5.35 + 0.072 + 410.4 (d) (3.452)(–0.203)
(b) 18.974 – 76.451 (e) –7.525 ÷ 0.25
(c) (–8.4 + 0.97) – 6.0472

13. A newspaper boy earned $32.56 and $23.65 for two days respectively. He bought a birthday gift for
his mother which cost $19.80 and a cake which cost $17.60. How much money was he left?

14. Write each mathematical quantity as a ratio in its simplest form.


(a) 300 kilometers covered in 6 hours
(b) 960 kilometers covered in 15 days
(c) 296 meters covered in 4 minutes
(d) 150 meters covered in 30 seconds
(e) $10 paid for 20 apples
(f) $35 paid for 35 strawberries
(g) $15 000 paid for 60 square meters of land
(h) $500 paid for 25 meters of carpet
(i) A container with 140 liters of water to be filled within 28 minutes
(j) A graduated cylinder with 490 milliliters of liquid to be filled within 7 minutes

28

Pre-Calculus and Calculus.indb 28 13/7/2017 5:56:52 PM


Chapter 1 • Numerical Systems

15. State whether each pair of ratios forms a proportion.


6
2 ​​  and ​​ ___
(a) ​​ __
4 20  ​​ 
6 3
(b) ​​ __ __
8 ​​  and ​​  4 ​​ 
4 ​​  and ​​ ___
(c) ​​ __ 24 ​​ 
3 12
5 8
(d) ​​ ___ ___
25  ​​ and ​​  40  ​​ 
1 ​​  and ​​ __
(e) ​​ __ 2 ​​ 
8 4

16. Fill in the missing number in each box.


2   ​​ = ___
(a) ​​ __ ​​  12
6 ​​ 

10
(b) ​​ __ ___
3 ​​ = ​​  6 ​​ 
9 __
(c) ​​ __
4 ​​  = ​​  12 ​​ 
3
(d) ​​ ___ __
8 ​​ = ​​  6 ​​ 
105 7
(e) ​​ __ ​​ = ___
​​  15  ​​ 

17. There are 375 students in a class. There are 15 more boys than girls. If 5 more boys join the class,
what will the ratio of girls to boys be?

18. A rope that is 2.5 meters long is cut into 3 pieces in a ratio of 2 : 3 : 5. What is the difference in length
between the longest piece and the shortest piece?

1 ​​  hours. A truck covered 272 kilometers in 4 hours. Did the two
19. A car covered 578 kilometers in 8​​ __
2
vehicles travel at the same speed?

29

Pre-Calculus and Calculus.indb 29 13/7/2017 5:56:52 PM


Chapter 1 • Numerical Systems

1.2 Exponential Expressions


1.2.1 Integer Index
1.2.2 Rational Index

1.2.1 Integer Index


In this section, we will discuss a real number in index form , where a is
a real number and n is an integer. can be read as ‘nth power of a’, or as
‘a to the nth’.

Definition 1.9

If a is an arbitrary real number and n is a positive integer, then the


number an represents the product of the number a with itself n times,
that is
an = a × a × a × ... × a
a multiplied n times

a is called the base and n is called the index (also known as exponent or
power).
E.g. a1 = a
b3 = b  b  b
5c4 = 5  c  c  c  c
Note (–3)5 = (–3)  (–3)  (–3)  (–3)  (–3) = –243
​  23 ​ )​​​   ​​ = __
​​​ (__
3
8
(–a)4 ≠ –a4 ​​  23 ​​   __
​​  23 ​​   __
​​  23  ​​ = ___
​​  27  ​​ 
(–3)2 ≠ –32
If we multiply a3 and a2, where a is any number and a ≠ 0, we have
a3  a2 = (a  a  a)  (a  a)
=aaaaa
= a5 (by the above definition)
=a 3+2

The illustration shows one of the properties of indices, that is, if a is any
number and p, q are positive integers, then
a p  a q = a p + q
——— (1)
If we divide a by a , 3 2

a3 a  a  a
​​ __2 ​​ = _______
​​  a  a   
​​, a ≠ 0

a
=a
= a3 – 2
This gives us another property of indices, that is
a p
​​ __
a q ​​ = a
 p – q
——— (2)

30

Pre-Calculus and Calculus.indb 30 13/7/2017 5:56:52 PM


Chapter 1 • Numerical Systems

Using equations (1) and (2), we can extend the definition of an to


non-positive indices:
a p
(i) a0 = a p – p = __
a​​   p ​​ = 1, that is, any number raised to the power zero has
the value 1.
(ii) As a–p  a p = a p – p = a0 = 1, we have the definition of a–p, that is
​​  a1 p  ​​. 
a–p = __

Some examples of non-positive indices:


3–1 = __ ​​  11  ​​ = __
​​  1 ​​ 
3 3

​  14 ​ )​​​    ​​ = _____
​​​ (__
–2
​​  1 2 ​​ = _________ ​​  1   ​​  = ___ 1  ​​ = 16
​​  ___
​​  ​  4 ​ )​​​   ​ (​  ​  4 ​ )​   (​  ​  4 ​ )​  ​  16  ​ 
1 1 1
(
__ 1 __ __

​​  1 4 ​​ = ___________________


(–2)–4 = _____ ​​     1  ​​  1  ​​ 
 ​​ = ___ Note
(–2) (–2)  (–2)  (–2)  (–2) 16
70 = 1 0 is not defined.
0

Properties of indices are summarized in the following theorem.

Theorem 1.1

Let a and b be any real numbers and m, n, p and q be any integers.


1. am  am = am + n
2. (am)n = amn = (an)m
3. (ab)m = ambm
​   ​)​​​    ​​ = ___
a m am
4. ​​​ (__ ​​  m  ​​  , where b ≠ 0
b b
a
​​  an1– m  
m
5. ​​ ___
an­ ​​  = a  a = a
m –n­ m–n
= ____ ​​ , where a ≠ 0

6. (a pb q)m = a pmbqm

​  q ​ )​​​    ​​ = ___
m
7. ​​​ (__
​a​​  p​ a pm
​​   qm ​​  , where b ≠ 0
​b​​  ​ b
a –p
b  q
8. ​​ ___   ​​ = __
​​   ​​  , where a ≠ 0, b ≠ 0
b–q a p
​   ​)​​​    ​​ = (​​​  __
​  a ​)​​​    ​​, where a ≠ 0, b ≠ 0
m
a –m b
9. ​​​ (__
b

31

Pre-Calculus and Calculus.indb 31 13/7/2017 5:56:52 PM


Chapter 1 • Numerical Systems

Properties 6 to 9 are obtained from properties 1 to 5 and from the definition


of negative exponents. In the theorem, we stated that m, n, p and q are
integers, but the properties are also true when the indices are rational.

The following are illustrations of the properties of indices:


Note
1. 22  23 = 22 + 3 = 25 = 32
Property 3 does not apply to
π–4  π6 = π–4 + 6 = π2
addition or subtraction:
(a ± b)m ≠ am ± bm 2. (43)2 = 4 3  2 = 46 = 4096
For example,
(x + y)2 = (x + y)(x + y) ​​  16  ​​ = ______
[(5)–3]2 = 5(–3)(2) = 5–6 = __ ​​  1   ​​ 
5 15 625
= x2 + 2xy + y2
≠ x2 + y2 3. (8)3 = (2  4)3 = 2343 = 8  64 = 512
​​  1 
(–2a)–3 = (–2)–3a–3 = ______ 1   ​​ 
= –​​ ___
 ​​ 
(–2)3a3 8a3

​  4 ​ )​​​   ​​ = __
4. ​​​ (__
3
3 33 27
​​  3 ​​ = ___
​​   ​​ 
4 64
2 ​  ​​​   ​​ = _____
​​​ (–​ __
(–2)
3 ) ​​  35 ​​ 
5 5
____32
 = –​​ 
243  ​​ 
76
5. ​​ __4 ​​ = 76  7–4 = 76 – 4 = 72 = 49
7
10–4
​​  1 5 ​​ 
​​  10 ​​ = 10–4  10–1 = 10–4 – 1 = 10–5 = ___
____
10
6. (23  52)2 = 23  2  52  2 = 26  54 = 64  625 = 40 000
(x4y)3 = x4  3y1  3 = x12y3

​  5 ​ )​​​   ​​ = ____


7. ​​​ (___
8
​m​​  4​ m4  8 m32
​​  5  8 ​​ = ___
​​  40 ​​ 
​n​​  ​ n n
​  ​2​​  4 ​​ )​​​   ​​ = ____
​​​ (__
2
​​  24  2 ​​ = __
​​  28 ​​ 
3 32 6

​x​​  ​ x x
4–2  ​​  (–5)3 125
8. ​​ _____ –3 = ​​ 
_____
 ​​  ____
 = –​​ 
16 ​​ 
(–5) 42
x–5 __ y7
​​ ___  
 ​​ = ​​   ​​ 
y–7 x5

9. ​​​ (–​ __ )   (  )​​​   ​​ = ​​ 


–3 3
3 ___ 7 _____ 73 343
7  
 ​ ​​​  ​​ = ​​​  ​  –3   ​    ​​ = –​​ ____
27 ​​ 
(–3)3
​  3  ​)​​​    ​​ = (​​​  __
​  y ​)​​​   ​​ = ____
y –6
​​​ (__
6
3 729
​​  6 ​​ 
y

The properties are used to simplify expressions with indices. In simplifying


an expression involving variables, we change the expression such that
each variable appears only once and express the answers using positive
indices. Let us look at some examples.

32

Pre-Calculus and Calculus.indb 32 13/7/2017 5:56:53 PM


Chapter 1 • Numerical Systems

Example 30
Simplify the following expressions.
​  r –8 ​ 
(d) ​​​ (____ ​​​    ​​
3​s​​  ​)
–4
(a) (3y2)(10y5)

(e) ​​​ [_______ ​​​   ​​


​x​​  ​+ ​y​​  2​]
4
x(y – 1)
(b) (p4q–3)3(2p–2q2)–3 ​  2  ​ 

(f) ​​​ [(​​  _____  ​​​​​    ​ ​​​   ​​


​3b​​  7​ ) ]
–4 2
1   ​​   6​a​​  6​​b​​  9​
(c) ​​ ________ ​   ​  
(m – n)–4
SOLUTION
(a) (3y2)(10y5) = 3(10)y2 + 5 = 30y7
(b) There are two variables p and q.
(p4q–3)3(2p–2q2)–3 = (p4)3(q–3)3(2)–3(p–2)–3(q2)–3
= p12q–92–3p6q–6
= 2–3p12 + 6q–9 – 6 Combine similar variables

= 2–3p18q–15
p18
= ____
​​  15  ​​  Express as positive indices
8q

​​  1   ​​ 
(c) ________ = (m – n)4 Remember, the answer is not m4 – n4
(m – n​)​​  –4​

​  r –8 ​   
(d) (​​​  ____ r–4   ​​  34 81
3​s​​  ​)
–4
_______ ____
​​​  ​​ = ​​  = ​​    ​​ = ____
​​    ​​ 
3 (s )
–4 –8 –4
r s32 r4s32
4

(e) [​​​  _______ ​​​ ] ​​ = ________


4
x(y – 1) x4(y – 1)4
​  2 2 ​  ​​  2  ​​ 
  The answer cannot be simplified further
​x​​  ​+ ​y​​  ​ (​x​​  ​+ ​y​​  2​​)​​  4​

(f) [​​​  (​​  _____  ​​​​​    ​]​​​   ​​ = ( )​​​    ​​


–4 2

)
–8
6​a​​  6​​b​​  9​ 2​a​​  6​​b​​  9​
​  7 ​   ​​​  ​ _____ 7 ​  

​3b​​  ​ ​b​​  ​

= (​​​  _____ ​​​   ​​


2​a​​  ​​b​​  ​)
8
b​ ​​  7​
​  6  9 ​ 

b7(8)
= ________
​​  8 6(8)    ​​ 
2 a b9(8)
= ​​ _________ 1   ​​ 
28a48b72 – 56
= ________
​​  1   ​​ 
256a48b16

33

Pre-Calculus and Calculus.indb 33 13/7/2017 5:56:53 PM


Chapter 1 • Numerical Systems

1.2.2 Rational Index


A number r is a square root of b if r2 = b. A number s is a cube root of b
if s3 = b.
For example,  3 is a square root of 9, since 32 = 9
–3 is also a square root of 9, since (–3)2 = 9
–4 is a cube root of –64 since (–4)3 = –64
Generally, t is an nth root of b if t n = b, where n is a positive integer. In
advanced algebra, it can be proved that equation t n = b has exactly n roots
which may not be all real. Out of these n roots, one of them is called the
principal nth root.
If b is a positive number, then there is only one positive real number t such
that t n = b. This positive number is the principal nth root. For example, 3
is the principal square root of 9. Even though –3 is also a square root of 9,
it is not the principal square root.
If b is a negative number and n is odd, then there is only one negative real
number t such that t n = b. This negative number is the principal nth root.
For example, –4 is the principal cube root of –64.
Note
If b is negative and n is even, then there is no principal nth root in terms
1
__
​​b​​  ​​indicates the principal nth
​  n ​ of real number.
root.
From t × t × t × ... × t = b, we can show that if the laws of indices are also
1
__ 1
__
Thus ​​9​​  ​ 2 ​ ​​ = 3 and –​​9​​  ​ 2 ​ ​​ = –3. n times
1
__
true for rational indices, then the value of t is actually b​​ ​​ ​ n ​​​, since
1
__ 1
__ 1
__ 1
__ n
__
b​​ ​​  ​ n ​​​  ​​b​​  ​ n ​​​  ​​b​​  ​ n ​​​  ... ​​b​​  ​ n ​​​ = ​​b​​  ​ n ​​​
n times
1
__ 1
__
Therefore, ​​b​​  ​ n ​​​ is the nth root of b, and b​​ ​​ ​ n ​​​is the principal nth root.
m
__
Note that ​​b​​  ​  n ​ ​​can be interpreted as ‘the nth root of b to the power m’ or ‘the
m
nth root of bm, that is, b​​ ​​ ​  n ​ ​​ = (​​​  ​b​​  ​ n )​​​​​    ​​  or ​​b​​  ​  n ​ ​​ = (bm​​)​​  ​ n ​​​.
m
__ 1
__ m
__ 1
__

For example, ​8​1​​  2​​ = (​​​  ​81​​  2​)​​​   ​​ = (813​​)​​  2​​ = 729
__3 1
__
3 1
__
​   ​  ​   ​  ​   ​ 

(–343​​)​​  ​​ = [​​​  (–343)​​  3] ​​​​​​​  ​​ = [(–343)2​​]​​  3​​ = 49


2 2
__
​  3 ​  ​  1 ​ 
__
​  1 ​ 
__

34

Pre-Calculus and Calculus.indb 34 13/7/2017 5:56:53 PM


Chapter 1 • Numerical Systems

Example 31
Find the integer representation of each of the following, if they exist.
__3 1
__
​   ​  ​   ​ 
(a) ​​(25)​​  2​​ (d) ​​(–16)​​  4​​
3
​  1 ​ 
__ __
​   ​ 
(b) –​​27​​  3​​ (e) ​​(81)​​  4​​
(c) ​​​ (–​  8 ​ )​​​    ​​
1
–​ __
__ 1 3 ​ 

SOLUTION
(a) ​(25​)​​  ​​ = (​​​  ​25​​  2)​ ​​​   ​​ = 53 = 125
3
__ 1
__
3
​  2 ​ 
​   ​ 

1
__
​   ​ 
(b) –​​27​​  3​​ = –3
1 ​  ​​​   3​​ = ​​(–8)​​  __​ 31 ​ ​​ = –2
1 ​ 
–​ __
(c) ​​​ (–​ __
8)
1
__
​   ​ 
(d) ​​(–16)​​  4​​is not a real number.

(e) ​​(81)​​  4​​ = (​​​  ​81​​  4)​ ​​​   ​​ = 33 = 27


3
__ 1
__
3
​   ​  ​   ​ 

Example 32
Simplify the following.
1
–​ __

(a) ​​​ {[​​  ​(t – 1)​​  3​]​​​    ​}​​​​    ​​


1 ​  –6 4 ​ 
​ __

3
__ 1
–​ __
​  2 ​  2 ​ 
9​ ​​  ​x ​y​​  ​z
4 2
(b) ​​ ________ 1
1 __
__  
 ​​
​   ​  ​   ​ 
​8​​  3​​x​​  2​y2z–1
1 ​ 
​ __
(3x – 2​)​​  ​ 3
(c) ​​ __________8   ​​ 
–​   ​ 
(3x – 2​)​​  3​

SOLUTION
1
–​ __

(a) ​​​ {[​​  ​(t – 1)​​  ​]​​​    ​}​​​​    ​​ = [​​​  (t – ​1)​​  ​]​​​    ​​ = ​​(t – 1)​​  12​​ = ​​(t – 1)​​  2​​
1
4 ​ 
__
1 –6 6 –​  4 ​  6
​  1 ​ 
__
​  3 ​  –​ __
3 ​ 
___
​    ​ 
__

(​​  ​9​​  ​)​​​   ​​x​​  2​ ​y​​  2 ​z


3 1 3 7 5
–​ __ ​  1 ​ 
__
__
​  2 ​  2 ​ 
__
​   ​  –​ __
2 ​ 
7
__
​ 1 ​  –​ __
4 – __ 1 ​  – 2 2 + 1 ​   ​ 
9​ ​​  ​x ​y​​  ​z
________
4 2 2
3 ​x​​  ​ ​y​​  ​z ______
3 2
27​x​​  2​z3
3
(b) ​​  1
__ 1
__  ​​ = ________________
    
​​  1  ​​ 
__ = ________
​​   ​​  = ​​ 
  5  ​​ 
__
2  
​   ​ 
​   ​  ​   ​  ​   ​ 
​8​​  3​​x​​  2​y2z–1 ​8​​  3​ 2​y​​  2​

​  1 ​ 
__
(3x – 2​)​​  ​ 3 1
__ 8
__ 9
__
(c) ​​ __________8 
​   ​  ​   ​  ​   ​ 
 ​​ 
= ​(3x – 2​)​​  3​​​(3x – 2​)​​  3​​ = ​(3x – 2​)​​  3​​ = (3x – 2)3
–​   ​ 
(3x – 2​)​​  3​

35

Pre-Calculus and Calculus.indb 35 13/7/2017 5:56:53 PM


Chapter 1 • Numerical Systems

Exercise 1.2

1. Without using calculator, evaluate the following.


(a)
(b)

(c)

(d)

(e)

(f)

2. Simplify the following.

(a)

(b)

(c)

(d)

(e)

(f)

36

Pre-Calculus and Calculus.indb 36 13/7/2017 5:56:53 PM


Chapter 1 • Numerical Systems

1.3 Logarithms
The number 100 can be expressed as 102, that is 100 = 102. An equivalent
form of 100 = 102 is .
100 = 102 is said to be in index form and is the equivalent
logarithmic form.

Definition 1.10

Let a > 0, a ≠ 1 and x a positive real number such that . Then


y = loga x, that is ‘y is the logarithm of x to base a’. Note
Note that logarithms of negative numbers are not defined. ay = x  y = loga x

Example 33
Write an equivalent logarithmic form of the following.
(a)
(b)
(c)

SOLUTION
(a)
(b)
(c)

Properties of logarithms are summarized in the following theorem. The


properties are used to simplify expressions involving logarithms.

Theorem 1.2

Let a, x and y be any positive real numbers and a ≠ 1.


1. loga xy = loga x + loga y
x
2. loga __
​​  y ​​ = loga x – loga y

3. loga 1 = 0
4. loga a = 1
5. loga xc = c loga x, where c  
6. loga ac = c, where c  
7. ​​a​​  log x​​ = x
a

37

Pre-Calculus and Calculus.indb 37 13/7/2017 5:56:53 PM


Chapter 1 • Numerical Systems

Example 34
Use the properties of logarithms to write the following expressions in
a single logarithm.
(a)

(b)

(c)

(d)

SOLUTION

(a)

(b)

(c)

(d)

38

Pre-Calculus and Calculus.indb 38 13/7/2017 5:56:53 PM


Chapter 1 • Numerical Systems

Example 35
Use properties of logarithms to find the value of the following expressions.
(a)
(b)

(c)

(d)

SOLUTION
(a)

(b)

(c) Note
loga (x ± y) ≠ loga x ± loga y
loga x
_____
​​    
​​≠ loga x – loga y
loga y
(d)

The system of logarithm with base 10 is called the common logarithm


system. Usually the base 10 is omitted, that is, log10 is written as lg. The
system of logarithm to base e is called the natural logarithm or Naperian
logarithm system. We usually write ln to indicate loge. The number e is an
irrational number, e = 2.718281828450...
Scientific calculators can be used to find the logarithm to base 10 and to
base e. To evaluate logarithm whose base is neither 10 or e, we use the
change of base formula.

Theorem 1.3

Let a, b and N be positive real numbers, where a ≠ 1 and b ≠ 1. The


formula to change a logarithm of base a to logarithm of base b is
logb N
loga N = ______
​​   ​​ 
logb a

We usually change the base to 10 or e so that we can use the calculator.

39

Pre-Calculus and Calculus.indb 39 13/7/2017 5:56:53 PM


Chapter 1 • Numerical Systems

Example 36
Find the value of the following logarithms and correct to four decimal
places.
(a)
(b)
(c)

SOLUTION
Change the logarithm to base 10 or e using the change of base formula.
lg 25 ________ 1.397940
(a) log3 25 = ______
​​   ​​ = ​​   ​​ 
= 2.9299
lg 3 0.477121
ln 10 ________2.302585
(b) log2 10 = _____
​​   ​​ ≈ ​​   ​​ 
= 33.219
ln 2 0.693147
lg 70 ________ 1.845098
(c) log5 70 = _____
​​   ​​ = ​​   ​​ 
= 2.6397
lg 5 0.698970

Example 37

Prove the change of base formula: .


SOLUTION
Let . The equivalent index form is .
Taking logarithm to base b on both sides,

Note
From the change of base
formula, it is easy to show
that

Therefore,

Exercise 1.3

1. Without using the calculator, evaluate the following.


(a) log2 32 (d) log4 0.25
__
(e) log3 (​​  ___ 1  ​  ​​  
(b) log3 27​​√ 3 ​​   ​  81 )

(f) log10 (​​  _____ 1   ​ ​​  


(c) log0.5 0.0625 ​  1000 )

2. By changing the base to 10, evaluate the following logarithms. Correct the answers to three decimal
places.
(a) log5 15 (c) log2 75
(b) lo​​g​  __​ 1 ​ ​​​ 7 (d) log8 1.25
2

40

Pre-Calculus and Calculus.indb 40 13/7/2017 5:56:53 PM


Chapter 1 • Numerical Systems

1.4 Radicals
1
__ 1
__ 1
__ 1
__ n
__ 1
__
We have seen that b​​ ​​ ​ n ​​​ × ​​b​​  ​ n ​​​ ×​​b​​  ​ n ​​​× ... × ​​b​​  ​ n ​​​ = ​​b​​  ​ n ​​​ = b, and ​​b​​  ​ n ​​​is the principal
nth root of b. The principal nth root of b can be written in another form as
n
__ 1
__ n
__
  b ​​  is in radical form. The
​​√ b ​​.  ​​b​​  ​ n ​​​ is in index or exponential form and ​​√ 

symbol is called a radical, b is the radicand and n is the index.


Students should be able to convert a number in fractional index form to
radical form and vice versa.
m n
___
__
​​b​​  ​  n ​ ​​ = ​​(bm)​​  ​ n ​​​ = ​​√ 
  b  ​​ 
m 1
__
Note
m ___ __ 1
​​b​​  ​  n ​ ​​ =​​ (​  ​b​​  ​ n ​​) ​​​  ​​ = (​​​  ​√   b ​) ​​​    ​​
m __
m
__ 1
__ n
The number √  ​​ a ​​ or ​​a​​  ​ 2 ​ ​​, where
a > 0, represents only one
___
1
__
​​(bn)​​  ​ n ​​​ = ​​√   bn ​​ = b
n number. __ __1
___
E.g. √ ​​  9 ​​__ = ​​9​​  ​ 2 ​ ​​= 3 and
n
​​ (​  ​b​​  ​ n )​​ ​​​  ​​ = (​​​  ​√   b )​  ​​​  ​​ = b
1
__ n n 1
__
–​​√ 9 ​​ = –​​9​​  ​ 2 ​ ​​ = –3. __
1
__
__ It is incorrect to write ​​√ 9 ​​ = ±3.
When n = 2, we write b​​ ​​ ​ 2 ​ ​​ as ​​√ b ​​.  The index 2 is not written.

Example 38
Write the following in radical form.

​   ​ )​​​    ​​
​  2 ​ 
__

(a) ​​​ (__
y​ ​​  ​ 2 5

8
​  1 ​ 
__
(b) [(x + 2)3​​]​​  4​​
1
__

(c) ​​​ {[​​  ​(3x)​​  2​]​​​    }


​ ​​​    ​​
​  4 ​ 
​  1 ​ 
__
​  1 ​  3
__

SOLUTION _____

√ 
(a) ​​​ (​  8 ​ )​​​    ​​ = ​​  (​​  ​  8 ​ )​​​   ​ ​​ 
​  2 ​ 
__
2
__y​ ​​  2​ 5 5 __ y​ ​​  2​

​  1 ​ 
__ _______
4
(b) [(x + 2)3​​]​​  4​​ = ​​√   (x + 2)3 ​​ 
1
__

(c) ​​​ {[​​  ​(3x)​​  ​]​​​    ​}​​​    ​​ = ​​√ ​√ ​√ 3x ​ ​ ​​


​  4 ​  _____
​  1 ​ 
__
4 3 ____
___
​  1 ​  3
__
2
      

The following are properties of radicals. The properties of radicals follow


directly from properties of indices.

Theorem 1.4

Let m, n and p be__positive integers and assume the roots exist.


___
n n __ n
1. ​​√   ab ​​ = ​​ √ 
  a  ​​  ​​ √ 
  b ​​  Note
n m __
___ ___
mn __ m n __ _____ __
2. ​​√ ​ a ​ ​​ = ​​ a ​​ = ​​ √ ​ a ​ ​​
    
  √ 
__ n __
   
√      
  √ 
n
  a + b  
​​√ 
n __ n
  a ​​ + ​​ √ 
​​≠ ​​√    b ​​ 

√ 
a ​ a ​
n __ ___
√ 
   
__  
_____ __ __
3. ​​ ​   ​ ​​ = ​​   ​​ 
    ​​√ a + b   ​​ a ​​ + √
​​≠ √  ​​  b ​​ 
b ​√ b ​ n
   
pn ___ n ___
4. ​​ √   a pm ​​  = ​​√   am ​​ 
41

Pre-Calculus and Calculus.indb 41 13/7/2017 5:56:54 PM


Chapter 1 • Numerical Systems

In Section 1.1, we learned that irrational numbers are real numbers that
a
cannot be written in the form of  __
​​   ​​  , where b ≠ 0. Some of these irrational
b __
numbers are numbers that contain the radical sign, that is in the form √  ​​ a  ​​,
__ __ __ __
where a > 0, such as √
​​  5 ​​,  2 – √
​​  3 ​​  and √
​​  5 ​​  + √
​​  2 ​​.  These numbers are called
surds.

Rationalization of Surds
When a quotient involves a surd in the denominator, we usually rationalize
the quotient, which is to get rid of the radical sign in the denominator.
Usually this can be done by multiplying the denominator (and numerator)
by the conjugate of the denominator. When we multiply a surd by its
conjugate, we will eliminate the surd.
__ __ __ __ __ __
For example, (​​√ 5 ​​ – ​​√ 2 ​​ )(​​√ 5 ​​ + √
​​  2 ​​)  = (​​√ 5 ​​)  2 – (​​√ 2 ​​ )2 = 5 – 2 = 3
__ __ __ __ __ __
​​√ 5 ​​  – √
​​  2 ​​  is the conjugate of √ 
​​ 5 ​​  + √
​​  2 ​​  and √
​​  5 ​​  + √
​​  2 ​​  is the conjugate of
__ __
​​√ 5 ​​ – ​​√ 2 ​​. 

We rationalize the denominator using the following methods:

For expression of the form


1   ​​, where n > m, we multiply both denominator and numerator by​​
(i) ​​ ____
n ___
  a   
​√  ​
m
n ____
√ 
  a n–m
  
​​.

(ii) ​​ _______1 , we multiply both denominator and numerator by the


__   __ ​​ 
​  a  ​+ √
√ ​  b  ​ __ __ __ __
conjugate of √  ​​ a  ​​ + √
​​  b  ​​, that is √
​​  a  ​​ – ​​√ b  ​​.

__1   ​​, 
(iii) ​​ ______ we multiply both denominator and numerator by the conjugate
​  a  ​ –_____
√ b _____
of ​​√ a – b   ​​  a + b  
​​, that is √ ​​.

Example 39
Rationalize the following expressions.
9__ 1__
(a) ​​ ___   ​​   (b) ​​ ___
5    ​​ 
​  3 ​ 
√ ​√   2 
 ​
SOLUTION __ __
__
9__ ___
___ 9__ ___ √  3 ​  ____
​ __ 9​√ 3 ​ 
(a) ​​     ​​ = ​​     ​​  ​​    ​​ = ​​  3 ​​   = 3​​√ 3 ​​ 
​  3 ​  √
√ ​  3 ​  √ ​  3 ​ 
5
____ 5
__ 5
__ 5
___
1__ ___ 1__ _____   ​2​​ 
​√  5–1
​ ​  ________   ​2​​  ​ ​ 
​√  4
  ​2​​  ​ ​  ____
​√  4
  16 ​ 
​√ 
(b) ​​ ___
5    ​​ = ​​  5    ​​  ​​  5
____   ​​ = ​​  5 __ 5   = ____
__ ​​  ​​  5 __   ​​ = ​​   ​​ 
2  
​√ 
  2 ​  ​√ 
  2 ​  ​√ 
  ​2​​ 
5–1
​ ​  ​√ 
  2 ​  ​√ 
  ​2​​  ​ ​ 
4
​√ 
  ​2​​  ​ ​ 
5

42

Pre-Calculus and Calculus.indb 42 13/7/2017 5:56:54 PM


Chapter 1 • Numerical Systems

Example 40
Rationalize the following expressions. __
__ 1  __ ​​  
(a) ​​ _______ __√ 3 ​   ​​ 
2​
(d) ​​ ______
​  2 ​ + √
√ ​  3 ​  ​  5 ​ – 2
√ Note
__
​  5 
 ​ __ (i)__
__ 1  __ ​​ 
__ __
__√
(b) ​​ _______    ​​   (e) ​​ _______ __
(​​√ a  ​​ __+ √
​​  b ​​)  (​​__
√ a __ ​​ – ​​√ b __​​)  __ __
​  5 ​ – √
√ ​  7 ​  ​  7 ​ – √
√ ​  5 ​ 
__ __ = (​​√ a  ​​)2 + √ ​​  a  ​​​​√ b  ​​ – √
​​  a  ​​​​√ b  ​​ – (​​√ b ​​)  2
​√ 2 ​
__ + 3​√__
 5 ​ 
(c) ​​ ________  ​​  =a–b
​√ 2 ​ – 3​√ 5 ​ 
(ii)__ __
SOLUTION (​​√ x  ​​ +
__
c)(​​√ x ​​ __
– c) __
__ __
= (​​√ x  ​​)2 + c​​√ x ​​ – c​​√ x ​​ – (c)2
_______1 _______ 1 ​
√ 2 ​
   
_______ – ​
√  3 ​ 
(a) ​​  __   __ ​​  = ​​  __   __ ​​   ​​  __ __  ​​  = x – c2
​  2 ​ + √
√ ​  3 ​  √ ​  2 ​ + √ ​  3 ​__  ​√ 2 ​__ – √ ​  3 ​ 
__ ​√__
_________________
= ​​   
  
 2 ​ – √ ​__ 3 ​  __
 ​​
(​  √​  2 ​__ + √ ​  3 ​__) ​  ​  (√
​  2 ​ – √​  3 ​  )​ 
​√ 2 ​ – √ ​  3 ​__ 
= ​​ ___________
__ 2   2 ​​ 
(​​  √ ) ( ​  3 ​ )​​​​   ​
​ __ 2 ​ ​​​​   ​ –__​​  √
​  2 ​ – √
√ ​  3 ​ 
= ​​ _______
2 – 3  ​​   
__ __
​  2 ​ – √
√ ​  3 ​ 
= ​​ _______ –1  ​​   
__ __
= – ​​ (√ ​__ 2 ​ – √ ​__ 3 ​) ​​  
= –​​√ 2 ​​ + √ ​​  3 ​​ 
__ __ __ __
​  5 
__√
_______ ​ __ ​  5 
√  ​ ​  5 ​ + √
√ ​  __
7 ​ 
(b) ​​  = _______
   ​​   _______
​​  __  __ ​​  ​​  __  
 ​​
​√ 5 ​ – √
​  7 ​  ​√ 5 ​ –__√ ​  7 ​ __ ​√ 5 ​ +__ √ ​  7 ​ 
​√ 5 ​ ​   √( ​__ 5 ​ +__√ )
​  7 ​ ​​​   __
= ________________
  
  ​​  __  ​​
(​  √ ​  5 ​
__  – ​
√ __7 ​
  )
  ​​​ (
 ​  ​
√ 5 ​
   +__ √ ​  7 ​__) ​​​  
(​ √
​  5 ​)  (​​ √​  5 ​)  ​+ (​ √ ​  5 ​)  (​​ √​  7 ​)  ​
= _________________
   ​​     __ 2 __ 2 ​​
(​  5 ​) ___
___​​ √ ​​​  ​ – (​​ √ ​  7 ​)  ​​​  ​
√​  25 ​ + √ ​  35 ​ 
= _________
​​  5 – 7 ​​   
___
5+√ ​  35 ​ 
= ​​ _______ –2 ​​___  

–​(5 + √ ​  35 ​) ​
= _________
​​  2 ​​   
___
–5 – √ ​  35 ​ 
= ________
​​  2 ​​   
__ __ __ __ __ __
​√ 2 ​
__ + 3​√__
 5 ​  ________
​  2 ​ + 3​√__
√  5 ​  ________
​  2 ​ + 3​√ __
√ 5 
 ​
(c) ​​ ________ = ​​  __
 ​​   ​​  __
 ​​   
 ​​
​√ 2 ​ – 3​√ 5 ​  √ ​  2 ​ – 3​√ 5 ​  ​√ 2 ​ + 3​√ 5   ​
__ 2 __ __ __ 2
(​​ ​√ 2 
 ​ )​​​  ​+ 2​(3​√ 5 ​ )​​√ 2 ​  + (​​ 3​√ 5 
______________________  ​ )​​​  ​
= ​​        __ 2 __ 2 ​​
(​​ √
​  2 ​ )​​​  ​ – (​​ 3​√ 5 
 ​ )​​​  ​
___
2 + 6​√ 10 ​ + 45
____________
= ​​    2 – ___ 45 ​​ 
47 + 6​√ 10 ​ 
= _________
​​  –43 ​​   
___
–47 – 6​√ 10 ​ 
= __________
​​  43 ​​   

43

Pre-Calculus and Calculus.indb 43 13/7/2017 5:56:54 PM


Chapter 1 • Numerical Systems

__ __ __ ___ __
___ __
__√ 3 ​   ​​ 
2​ 2​√ 3 ​  ______
​  5 ​ + 2 __________
√ 2​√ 15 ​ + 4​√ 3 ​ 
(d) ​​ ______ = ______
​​  __   ​​   ​​  __  
 ​​ = ​​   ​​    = 2​​√ 15 ​​ + 4​​√ 3 ​​ 
5–4
​√ 5 ​ – 2 ​√ 5 ​ – 2 ​√ 5 ​ + 2
__ __ __ __ __ __
_______
__ 1 __ _______
__ 1 __ ​  __

_______7 ​ + ​
  √ __ 5 ​  _______​  7 ​ + √
√ ​  5 ​  _______
​  7 ​ + √
√ ​  5 ​ 
(e) ​​     ​​  = ​​     ​​   ​​   ​​ 
= ​​  7 – 5 ​​   = ​​  2 ​​   
​√ 7 ​ – √
​  5 ​  ​√ 7 ​ – √​  5 ​  ​√ 7 ​ + √ ​  5 
 ​

Using the properties, we can simplify expressions involving surds.


Numbers or expressions involving surds are said to be in simplified form
when
(i) the perfect nth powers are removed __ from ____ the radicand. That is, if we
n ___ n _______ n n a  n ____
  a   
have ​​√  ​​, m > n, then ​​√ 
 a  a      a   
​​ = ​​√   a
​​  ​​√    
​​ = ​​ √  a   
​​
m n m–n n m–n m–n

(ii) the denominator is rationalized.


Example 41
Simplify the following. _____
___ ___ ____ ____
(d) ​​√ ___
3 3
(a) ​​√ 
  27 ​​ + √
​​  32 ​​ – ​​√ 200 ​​   ​√    243 ​ ​​ 

√ 
__ __
a​ ​​  3​
(b) (3​​√ 2 ​​ + 5)(1 – √ 
​​ 2 ​​)  (e) ​​ __
​  5 ​ ​​  
__ b
________
2​√__ 3 ​ – 1
(c) ​​ _______ (f) ​​√   32x7y3z10 ​​ 
5
 ​​  
​  3 ​ + 1

SOLUTION
3
___ ___ ____ 3
__ _____ ______
(a) ​​√ 
  27 ​​ + √
​​  32 ​​ – ​​√ 200 ​​ = ​​√ 
  3  ​​ + √
3
​​ __ 2 × 42 ​​ – ​​√ 2 × 102 ​​ 
__
= 3 + 4​​√ __ 2 ​​ – 10​​√ 2 ​​ 
= 3 – 6​​√ 2 ​​ 
__ __ __ __ __
(b) (3​​√ 2 ​​ + 5)(1 – √ 
​​ 2 ​​ ) = 3​​√ __
2 ​​(  1 – ​​√ 2 ​​)  + 5(1 –__√ 
​​ 2 ​​ )
= 3​​√ 2 ​​
__
 – 3(2) + 5 – 5​​√ 2 ​​ 
__
= (3​​√ 2 ​​ – 5​​__√ 2 ​​ ) + (5 – 6)
= –1 – 2​​√ 2 ​​ 
__ __ __
2​√__ 3 ​ – 1
_______ 2​√ 3 ​ – 1 ______ ​  3 ​ – 1

(c) ​​  = _______
 ​​  ​​  __  ​​  __  ​​ 
 ​​ 
​√ 3 ​ + 1 ​  3 ​
√  + 1 ​  3 ​ – 1
__ __ √ __
2​√ 3 ​
  (  ​√ 3 ​ – 1) – 1(​√ 3 ​ – 1)
____________________
= ​​    – 1 ​​
3  
__ __
2(3) – 2​√ 3 ​ – √
​  3 ​ + 1
= _________________
  
​​  2  ​​ 
__
7 – 3​√ 3 ​ 
_______
= ​​  2 ​​   
_____ ____ _______
______ ____ __ __ 6 __
√  √  √ 
3 3 3
(d) ​​ ​√    243 ​ ​​   = ​​ ​√    27 × 9 ​ ​​    = ​​ 3​√    9 ​ ​​  =√ ​​  3 ​​ ​​ √ 
  9 ​​ 
___ ___ __ ___

√ 
__ __
a​ ​​  ​ _____
__ 3
√ ​a​​  ​a  ​ _____
​  ___ 2
a​√ a  ​
  a​
_____ √ a 
  ​
  ​

___ b 
   
​ a​

_____  ab  ​
(e) ​​ ​  5 ​ ​​  = ​​  4  ​​ = ​​  2 __  ​​ = ​​  2 __  ​​  ​​  __  ​​ = ​​  3 ​​   
b ​√ ​b​​  ​b  ​ b ​√ b  ​ b ​√ b  ​ ​√ b  ​ b
________ 5 __ 5 ____ 5 __ 5 ___
(f) ​​√   32x7y3z10 ​​    x x  ​​ ​​    y  ​​ ​​ √ 
5
= ​​√   2  ​​ ​​ √    z  ​​ 
__ 5 __√
5 5 2 3 10

5
= 2x​​√    x  ​​ ​​ √   y  ​​z
2 3 2

____
= 2xz  ​​√   x y  ​​ 
5
2 2 3

44

Pre-Calculus and Calculus.indb 44 13/7/2017 5:56:54 PM


Chapter 1 • Numerical Systems

Exercise 1.4

1. Without using the calculator, simplify the following.


__ ___
(a) ​​ __ √ 5__ √ 27___
3 75
​   ​ ​​   ​​ ___
​   ​ ​​  
___
(b) 8​​√ 2 ​​ – ​​√ 32 ​​ + 5​​√ 98 ​​ 
___
3​√ ___
12 ​ 
(c) ​​ _____
3   ​​
4​√ 
  54 ​ 
__ __ __
(d) (2​ ​√ 3 ​ – √
​  2 ​​)  (1 – 2​​√ 6 ​​) 
__ __
(e) (√
​​  5 ​​ – ​​√ 2 ​​​​)  ​​  ​​
2

2. Rationalize the following expressions.


2 __ ​​ 
(a) ​​ ______
2+√​  2 
 ​
__ 1  __ ​​ 
(b) ​​ ________
3​√ 3 ​ + √
​  5 ​ 
__4   ​​ 
(c) ​​ ______
​  2 ​ – 3

10 __
(d) ​​ ______
   ​​ 
5–√​  5 ​ 

3. Simplify the following expressions.


____

√ 
__

√ 
x 25y
(a) ​​ ​  y ​ ​​  ​​ ____
__
​  3 ​ ​​  
x
_____
(b) ​​√ x8y2z9 ​​ 
________
______
(c) ​​√ ​√   64x8y27 ​ ​​ 
3

1__
(d) ​​ ___
​  y   ​​​  

2__   ​​ 
(e) ​​ ____
​  3 ​ x

1__ ​​ 
(f) ​​ ____
3  
  x  ​
3​√ 
__ __
2 – ​√ 3 ​
__   2+√
​ __
 3 ​ 
(g) ​​ ______  ​​ – ______
​​   ​​ 
2+√
​  3 ​  2 – ​√ 3 ​ 
__ 1  __ ​​ 
(h) ​​ ________
​  6 ​ – 2​√ 2 ​ 

45

Pre-Calculus and Calculus.indb 45 13/7/2017 5:56:54 PM


Algebra
Chapter 2:

Algebraic Expressions

Historical Note
Diophantus of Alexandria
(≈ AD 201–215 to AD 285–299)

Diophantus, often known as


the father of Algebra, was a
mathematician who lived and
worked in Greece. He was
famous for writing the book
The Arithmetica. This book
was the first known work to
use the structure of modern
algebra, and had an enormous
influence on the development
of the number theory.

Algebra is a generalization of arithmetic concepts and algorithms. It is a useful tool in


modeling real-life situations. An engineer uses algebra to solve physical problems such
as building a bridge or designing a house. Businessmen use algebra to calculate costs,
revenues and profits. Bankers use algebra to calculate interests. Thus, algebra serves as
a foundation of various concepts and applications of mathematics.

Pre-Calculus and Calculus.indb 46 13/7/2017 5:56:55 PM


Chapter 2 • Algebraic Expressions

2 A
 lgebraic Expressions
2.1 Operations on Algebraic Expressions
2.2 Factorisation of Algebraic Expressions

2.1 Operations on Algebraic Expressions


2.1.1 Definition of Algebraic Expressions
2.1.2 Definition of Polynomials
2.1.3 Addition of Polynomials
2.1.4 Subtraction of Polynomials
2.1.5 Multiplication of Polynomials
2.1.6 Special Products
2.1.7 Division of Polynomials

2.1.1 Definition of Algebraic Expressions


The expression 6x is called an algebraic expression.
6x
constant variable

Definition 2.1

An algebraic expression is a combination of one or more variables, or


constants and variables along with at least one mathematical operation.

An algebraic expression is composed of algebraic terms, or simply terms,


which are connected by plus and minus signs.
• There are two terms in the algebraic expression x + 5, and they are x
and 5.
• There is only one term in the algebraic expression 2πr.
Terms involving variables are referred to as variable terms. Terms with
no variables are called constant terms.
Each factor of a term is called a coefficient of the product of all the other
factors in the term. The constant factor of a variable term is called its
numerical coefficient or simply coefficient. The variable factor of a term
is called literal coefficient. For instance, the coefficient of 2xy is 2 and its
literal coefficient is xy.

47

Pre-Calculus and Calculus.indb 47 13/7/2017 5:56:55 PM


Chapter 2 • Algebraic Expressions

Example 1
For each algebraic expression, determine the number of terms, the
numeral coefficient, and the literal coefficient of each term.
(a) 3x2 – 5x + 2 (b) 2x2y – 3xy2

SOLUTION
(a) There are three terms in the expression 3x2 – 5x + 2.
3x2 – 5x + 2
This is a constant term.

This is a variable term. Its numerical


coefficient is –5 while the literal
coefficient is x.

This is a variable term. Its numerical


coefficient is 3 while the literal
coefficient is x2.
(b) There are two terms in the expression 2x2y – 3xy2.
2x2y – 3xy2
The numerical coefficient is –3. The
literal coefficient is xy2.

The numerical coefficient is 2. The


literal coefficient is x2y.

In the algebraic expression 3ab – 7ab, it can be seen that both terms have
the same literal coefficients. Such terms are called similar terms.

Definition 2.2

Similar terms are terms that have the same literal coefficient, that is,
the terms have same variables raised to the same power. Otherwise they
are called dissimilar terms.

48

Pre-Calculus and Calculus.indb 48 13/7/2017 5:56:55 PM


Chapter 2 • Algebraic Expressions

Example 2
Determine if each pair of terms is similar.
(a) 3a and 2b (d) 5 and –8
(b) 6 j and 4j
2
(e) 5x2y and 9xy2
(c) 7xy and –xy

SOLUTION
(a) The literal coefficients of 3a and 2b are different since the variables
a and b are different. Hence, they are dissimilar terms.
(b) The literal coefficients of 6 j2 and 4j are different since j2 and j have
Note
different exponents. Hence, they are dissimilar terms.
–xy = –1xy
(c) The expressions 7xy and –xy are similar terms since their literal
coefficients are alike.
(d) All constants such as 5 and –8 are similar terms.
(e) The expressions 5x2y and 9xy2 are dissimilar terms since their literal
coefficients x2y and xy2 have variables raised to different exponents.

2.1.2 Definition of Polynomials


Consider the algebraic expressions below.
3 3 a–b
2x2 –​​ __
2  ​​ab
_____
​​   ​​ 
2  
__
5x – 3x + 2
2 2
x + √ 
​​ 2 ​​  10

These algebraic expressions are called polynomials.


Observe that:
• all the exponents of the variables are whole numbers,
• there are no negative exponents, hence there are no variables in the
denominator,
• there are no fractional exponents, hence there are no variables inside
radical symbols.

Here are some examples of algebraic expressions that are not polynomials:
2x + 1
______ __ 3
​​  y + 2 ​​  ​​  x ​+ 2​ x0.5 + y
______ __
4x–2 – 5 + 1 ​​√ 3x + 1 ​​  –​​√ x  ​​

Definition 2.3

A polynomial is an algebraic expression where each term is a constant,


a variable, or a product of constants and variables with non-negative
integers as exponents.

49

Pre-Calculus and Calculus.indb 49 13/7/2017 5:56:55 PM


Chapter 2 • Algebraic Expressions

Example 3
Determine whether the given algebraic expression is a polynomial.
Note __
(a) x2 + 3x – 2 (c) 45 (e) 3​​√ x  ​​ – 4
All polynomials are algebraic 5 x
expressions, but not all (b) ​​ __
x ​​ – 4x + 1 (d) ​​ __
3  ​​ – 2x
algebraic expressions are
polynomials. SOLUTION
(a) The algebraic expression x2 + 3x – 2 is a polynomial since all the
exponents of the variables are whole numbers.
5
(b) The algebraic expression __ ​​  x ​​ – 4x +1 is not a polynomial since the
5
variable in the first term __ ​​ x ​​is in the denominator.
(c) The constant 45 is a polynomial.
x
(d) The algebraic expression __ ​​  3  ​​ – 2x is a polynomial since all variable
Note terms have non-negative exponents. __
(e) The algebraic
__
expression 3​​√ x  ​​– 4 is not a polynomial since in the
Any constant is a polynomial.
term 3​​√ x  ,​​ the variable x is inside a radical symbol.

Polynomials have special names depending on the number of terms.

Definition 2.4

A monomial is a polynomial with one term.


A binomial is a polynomial with two terms.
A trinomial is a polynomial with three terms.
A multinomial is a polynomial with four or more terms.
Note
Example 4
A multinomial is more often
called as polynomial. Classify the following polynomials as monomial, binomial or trinomial.
2x + y
(a) x2 – 3 (d) 4x2y3z (g) ​​ ______
3 ​​  
(b) –10 (e) 9y – 24y + 16
2

3xy
(c) 5x4 (f) ​​ ___
4 ​​  
SOLUTION
(a) The polynomial x2 – 3 has two terms. It is a binomial with one
variable.
(b) The polynomial –10 is a constant and has only one term. It is a
monomial.
(c) The polynomial 5x4 has only one term. It is a monomial with one
variable.
(d) The polynomial 4x2y3z has only one term. It is a monomial with
three variables.
(e) The polynomial 9y2 – 24y + 16 has three terms. It is a trinomial.
3xy
(f) The polynomial ___ ​​  4 ​​  has only one term. It is a monomial with two
variables.
2x + y
(g) The polynomial ​​  ______ 3 ​​ 
 is a binomial. It is a simplified form of
2x __
___ y
50 ​​  3 ​​ + ​​  3  ​​which has two terms.

Pre-Calculus and Calculus.indb 50 13/7/2017 5:56:55 PM


Chapter 2 • Algebraic Expressions

Definition 2.5

The degree of a term is the sum of the exponents of every variable in


the term.

For instance, the term 5x4 has a degree of 4.


Consider the term 4x2y3z which has three variables.
What is the degree of 4x2y3z?
The exponent of the variable x is 2.
The exponent of the variable y is 3.
The exponent of the variable z is 1.
The degree of 4x2y3z is determined by adding the exponents of the variables,
that is, 2 + 3 + 1 = 6
Hence, the degree of 4x2y3z is 6.

Example 5
Determine the degree of each monomial.
3
(a) –4x6 (c) ​​ __
4  ​​
(b) 5ab3c5 (d) 0

SOLUTION Note
(a) The degree of –4x6 is 6. The degree of a non-zero
(b) The degree of 5ab3c5 is 1 + 3 + 5 = 9. constant is zero.
3
(c) The monomial __​​  4 ​​  is a non-zero constant. Its degree is 0.
Note
(d) The degree of the monomial 0 is not zero. A zero polynomial has
The constant 0 has no degree.
no degree.

A polynomial is a sum or difference of one or more terms. The degree of


a polynomial is the same as the degree of its term with the highest degree.

Definition 2.6

The degree of a polynomial is the highest degree of any of its


non-zero terms.

A polynomial is written in descending order when its terms are arranged


from the term with the highest degree to the term with the lowest degree.

51

Pre-Calculus and Calculus.indb 51 13/7/2017 5:56:55 PM


Chapter 2 • Algebraic Expressions

Example 6
Determine the degree of each polynomial.
(a) 2x4 – 3x2 + 5 (b) 3x2y + 10xy – 5xy2

Note SOLUTION
The degree of a polynomial
(a) The degree of the term 2x4 is 4.
in one variable is the greatest The degree of the term –3x2 is 2.
exponent in the polynomial. The degree of the constant term 5 is 0.
Since the highest degree among the terms is 4, the degree of
2x4 – 3x2 + 5 is 4.
(b) The degree of 3x2y is 3
The degree of 10xy is 2.
The degree of –5xy2 is 3.
The highest degree among the terms is 3. Hence, the degree of
3x2y + 10xy – 5xy2 is 3.

2.1.3 Addition of Polynomials


Method 1: Add the polynomials vertically.
Align similar terms, add their numerical coefficients and retain the literal
coefficients.
x2 + 2x + 3
+ 2x2 – x – 1
3x2 + x + 2
Method 2: Add the polynomials horizontally.
Group the similar terms, add their numerical coefficients and retain their
literal coefficients.
(x2 + 2x + 3) + (2x2 – x – 1) = (x2 + 2x2) + (2x – x) + (3 – 1)
= 3x2 + x + 2

Example 7
Add 3x3 – 8x2 + 5x – 10 and 2x2 – 7x3 + 10x + 8.

Note SOLUTION
(3x3 – 8x2 + 5x – 10) + (2x2 – 7x3 + 10x + 8)
To add polynomials, combine
= (3x3 – 7x3) + (–8x2 + 2x2) + (5x + 10x) + (–10 + 8)
similar terms.
= –4x3 – 6x2 + 15x – 2

Example 8
Find the sum of 6x5 + 4x2y3 + 8x3y2 – 5x4y + xy4 and
6x2y3 + 10x4y – 7xy4 + 3x3y2 – y5.
Note SOLUTION
Write similar terms in one 6x5 + 8x3y2 + 4x2y3 – 5x4y + xy4
column and add. + 3x3y2 + 6x2y3 + 10x4y – 7xy4 – y5
6x5 + 11x3y2 + 10x2y3 + 5x4y – 6xy4 – y5
52

Pre-Calculus and Calculus.indb 52 13/7/2017 5:56:56 PM


Chapter 2 • Algebraic Expressions

2.1.4 Subtraction of Polynomials


Subtraction is the process of adding the additive inverse, that is,
x – y = x + (–y)
Hence, to subtract a polynomial, write the subtraction statement to its
equivalent addition statement by changing the subtrahend to its additive
inverse. To get the additive inverse, change all the signs of the terms in the
subtrahend. Then, add by combining similar terms.

Example 9
Find the difference between 5x4 – 3x3 + 2x – 4 and 2x2 + 2x3 + x – 5.

SOLUTION
(5x4 – 3x3 + 2x – 4) – (2x2 + 2x3 + x – 5)
Find the additive inverse of the subtrahend by changing the signs of the
terms, and then add the two polynomials.
(5x4 – 3x3 + 2x – 4) + (–2x2 – 2x3 – x + 5)
Add by combining similar terms.
5x4 + (–3x3 – 2x3) + (–2x2) + (2x – x) + (–4 + 5)
= 5x4 – 5x3 – 2x2 + x + 1

Example 10
Subtract 0.3x3 – 2x2y + xy2 – 0.2y3 from 2x3 + 0.5x2y + y3.

SOLUTION
Subtracting vertically, write similar terms in one column,
2x3 + 0.5x2y + y3
– (0.3x – 2x y + xy – 0.2y3)
3 2 2

and add the additive inverse,


2x3 + 0.5x2y + y3
+ –0.3x + 2x y – xy + 0.2y3
3 2 2

1.7x3 + 2.5x2y – xy2 + 1.2y3

Example 11
Subtract x2 – y2 from y2 – x2.

SOLUTION
Subtracting horizontally:
(y2 – x2) – (x2 – y2) = (y2 – x2) + (–x2 + y2)
= y2 + y2 + (–x2) + (–x2)

= 2y2 – 2x2

53

Pre-Calculus and Calculus.indb 53 13/7/2017 5:56:56 PM


Chapter 2 • Algebraic Expressions

2.1.5 Multiplication of Polynomials


Multiplying a Monomial and a Monomial
To multiply a monomial and another monomial, use the product law and
the power law of exponents.
Below is an example of multiplying a monomial by a monomial. We need
to use the properties of real numbers and laws of exponents.
(3x)(2x) = (3)(2)(x)(x) Commutative property
= 6x1 + 1 Product law of exponents
= 6x2

Example 12
Find the following product.
(a) (–3x2y)(2xy2)2
​  21 ​​ x​​  2​)​​( 4y2)(–3x)
(b) ​​ (__

(c) ​​ (__ ​ (​​​​    __


​  2 ​​ x​​  –1​​y​​  –2) ​ ​​  
​  52 ​ xy​ ​​  3)
3

(d) (0.2xyz)(0.25xz ) 2

(e) (–xy2)(–xy–2)(–xy2)

SOLUTION
(a) (–3x2y)(2xy2)2 = (–3x2y)(22x2y2 × 2)
= (–3x2y)(4x2y4)
= –12x4y5
​  21 ​ x2)​​( 4y2)(–3x) = (​​  __
(b) ​​ (__ ​  21 ​ )​​( 4)(–3)(x2 + 1)(y2)
= –6x3y2
​  2 ​ x–1y–2)(​​​​    __
(c) ​​ (__ ​  52 ​ xy3)​​  = (​​  __ ​  2 ​ )(​​​​    __
​  5 ​ )​​x 
3 3 2 –1 + 1 –2 + 3
y
3
= __ ​​  5 ​​ x0y
3
= ​​ __ 5  ​​y
(d) (0.2xyz)(0.25xz2) = (0.2)(0.25)(x1 + 1)(y)(z1 + 2)
= 0.05x2yz3
(e) (–xy2)(–xy–2)(–xy2) = –x1 + 1 + 1y2 – 2 + 2
= –x3y2

54

Pre-Calculus and Calculus.indb 54 13/7/2017 5:56:56 PM


Chapter 2 • Algebraic Expressions

Multiplying a Polynomial and a Monomial


To multiply a monomial and a polynomial, use the distributive property to
multiply the monomial with each term of the polynomial.
Using the distributive property of multiplication over addition,

2x(3x + 1) = (2x)(3x) + (2x)(1)
= 6x2 + 2x
Example 13
Multiply the following.
(a) 3x2(5x2 – x + 9)
(b) –2a2bc2(3a3b2c – 7ab3c2 + 5a2bc3)

SOLUTION
(a) 3x2(5x2 – x + 9) = 3x2(5x2) – 3x2(x) + 3x2(9)
= 15x4 – 3x3 + 27x2
(b) –2a2bc2(3a3b2c – 7ab3c2 + 5a2bc3)
= (–2a2bc2)(3a3b2c) – (–2a2bc2)(7ab3c2) + (–2a2bc2)(5a2bc3)
= –6a5b3c3 + 14a3b4c4 – 10a4b2c5

Product of Two Binomials


The product of two binomials can be obtained using the method below.

(x + a)(x + b) = x(x) + (x)(b) + (a)(x) + (a)(b)


= x2 + bx + ax + ab
= x2 + (a + b)x + ab
A more general form of the method is derived below.

(ax + by)(cx + dy) = (ax)(cx) + (ax)(dy) + (by)(cx) + (by)(dy)


= acx2 + adxy + bcxy + bdy2
= acx2 + (ad + bc)xy + bdy2

Example 14
Multiply the following.
(a) (x + 3)(x + 5) (b) (2x – 3)(3x + 4)

SOLUTION

(a) (x + 3)(x + 5) = x2 + (3 + 5)x + (3)(5)
= x2 + 8x + 15

(b) (2x – 3)(3x + 4) = (2x)(3x) + [(2)(4) + (–3)(3)]x + (–3)(4)
= 6x2 + (8 – 9)x – 12
= 6x2 – x – 12

55

Pre-Calculus and Calculus.indb 55 13/7/2017 5:56:56 PM


Chapter 2 • Algebraic Expressions

2.1.6 Special Products


Square of a Binomial
You can also find a special product for the square of a binomial.
first term
second term

(x + y)2 = (x + y)(x + y)

= (x)(x) + (x)(y) + (y)(x) + (y)(y)


= x2 + xy + xy + y2
= x2 + 2xy + y2
square of the second term
twice the product of the first term and
the second term
square of the first term

Theorem 2.1

The square of a binomial sum (x + y)2 is a trinomial which is the square


of the first term of the binomial plus twice the product of the first term
and the second term plus the square of the second term of the binomial.
(x + y)2 = x2 + 2xy + y2

The square of a binomial difference (x – y)2 is a trinomial which is similar


to the square of a binomial sum (x + y)2 except for the negative sign of the
middle term.
(x – y)2 = (x – y)(x – y) = [x + (–y)][x + (–y)]
= (x)(x) + (x)(–y) + (–y)(x) + (–y)(–y)
= x2 – xy – xy + y2
= x2 – 2xy + y2

Theorem 2.2

The square of a binomial difference (x – y)2 is a trinomial which is the


square of the first term of the binomial minus twice the product of the
first term and the second term plus the square of the second term of the
binomial.
(x – y)2 = x2 – 2xy + y2

56

Pre-Calculus and Calculus.indb 56 13/7/2017 5:56:56 PM


Chapter 2 • Algebraic Expressions

The product of a binomial to itself can be illustrated geometrically by


finding the area of a square whose side has dimension x + y.
x y

x x2 xy

y xy y2

The area of the square is A = (x + y)2.


This is equal to the sum of the four areas x2, xy, xy, and y2.
Hence, A = (x + y)2 = x2 + 2xy + y2.

Example 15
Multiply the following.
​ 4 ​ y)​​​   ​​
(b) ​​​ (7 – __
2
3
(a) (x + 8)2 (c) (–3a2x + 2by2)2

SOLUTION
(a) (x + 8)2 = x2 + 2(x)(8) + 82
= x2 + 16x + 64
​ 4 ​ y)​​​   ​​ = 72 – 2(7)​​ (__
(b) ​​​ (7 – __ ​  4 ​ y)​​  + (​​​  __
​  4 ​ y)​​​​   ​​
2 2
3 3 3

9 2
​​ 21
= 49 – ___ ___
2 ​​ y + ​​  16  ​​ y
(c) (–3a2x + 2by2)2 = (–3a2x)2 + 2(–3a2x)(2by2) + (2by2)2
= 9a4x2 – 12a2bxy2 + 4b2y4

Example 16
​​  21 ​​ .
Find the area of a square whose side is 3x + __

SOLUTION
The area A of the square is given by
​ 12  ​)​​​   ​​
A = (​​​  3x + __
2

= (3x)2 + 2(3x)​​ (__ ​  21 ​ )​​  + (​​​  __


​  21 ​ )​​​   ​​
2

= 9x2 + 3x + __ ​​  41 ​​ 

57

Pre-Calculus and Calculus.indb 57 13/7/2017 5:56:56 PM


Chapter 2 • Algebraic Expressions

Product of the Sum and Difference of Two Terms


You can derive a special product in multiplying the sum and difference of
two terms.

(x + y)(x – y) = (x)(x) + x(–y) + (y)(x) + (y)(–y)
= x2 – xy + xy – y2
Note
= x2 – y2
Inspect the two binomial
factors.
Theorem 2.3
Are the first terms the same?
Are the second terms the The product of the sum and difference of two terms is the difference of
same? the squares of these terms.
Is one of the binomials a sum (x + y)(x – y) = x2 – y2
of two terms and the other a
difference of two terms?
Example 17
Find the following product.
(a) (5x + 3)(5x – 3)

3 ​ s)(​​​​    2r + ​ 3 ​ s)​​  
(b) ​​ (2r – ​ __
1 1
__

(c) (3y2 + 4y)(3y2 – 4y)

SOLUTION
(a) (5x + 3)(5x – 3) =  (5x)2 – (3)2
= 25x2 – 9
(b) ​​ (2r – ​ __ 1 ​ s ​​  = (2r)2 – ​​​  __
1 ​ s ​​​​    2r + ​ __ 1 ​ s ​​​   ​​
3 )( 3 ) (3 )
2
​ 

​​  91 ​​ s2
= 4r2 – ___
(c) (3y2 + 4y)(3y2 – 4y) = (3y2)2 – (4y)2
= 9y4 – 16y2

2.1.7 Division of Polynomials


Recall addition of similar fractions.
Note a __ b _____ a+b
​​ __
c ​​ + ​ ​c ​​ = ​​  c   
​​ 
If a, b, and c are real numbers
and c ≠ 0,
The process can be reversed as shown below.
a + b __
_____ a b
​​  c   ​​ = ​​  c ​​ + __
  ​ ​c ​​ a + b __
_____ a b
​​  c   ​​ = ​​  c ​​ + __
  ​ ​c ​​

58

Pre-Calculus and Calculus.indb 58 13/7/2017 5:56:56 PM


Chapter 2 • Algebraic Expressions

Dividing a Polynomial by a Monomial


To divide a polynomial by a monomial, divide each term in the polynomial
by the monomial.

Example 18
Divide the following.
12x8 – 24x6 + 12x4 – 18x2
_____________________
(a) ​​        ​​
6x2
32x6y3 – 24x4y4 – 8x2y5
__________________
(b) ​​        ​​
–8x2y2

SOLUTION
12x8 – 24x6 + 12x4 – 18x2 ______
_____________________ 12x8 ____ 24x6 12x4 ____ 18x2
(a) ​​      2 ​​ = ​​  – ​​  2 ​​ + ____
2 ​​  ​​  2 ​​ – ​​  2 ​​ 
6x 6x 6x 6x 6x
= 2x8 – 2 + (–4x6 – 2) + 2x4 – 2 + (–3x2 – 2)
= 2x6 – 4x4 + 2x2 – 3
32x6y3 – 24x4y4 – 8x2y5 ______
__________________ 32x6y3 ______ 24x4y4 ______
8x2y5
(b) ​​       2 2 ​​ = ​​  2 2 ​​ 
– ​​  2 2 ​​ 
– ​​  2 2   ​​ 
–8x y –8x y –8x y –8x y
= –4x6 – 2y3 – 2 + 3x4 – 2y4 – 2 + x2 – 2y5 – 2
= –4x4y + 3x2y2 + y3

Division of Two Polynomials


A polynomial p(x) can be divided by a polynomial g(x) if the degree of
p(x) is greater than or equal to the degree of g(x). Otherwise, the division
is not possible. We can divide p(x) by g(x) using the long division
method. This procedure is similar to the procedure in dividing two
integers. Let us look at the process of dividing p(x) = 4x3 + 3x2 + 2x – 4 by Note
g(x) = x2 – x + 1. Polynomial p(x) is called the dividend and polynomial
The dividend and divisor
g(x) is called the divisor. The division is done in the following steps.
must be in descending order.
Step 1: Write the dividend and divisor as below.

)
x2 – x + 1 4x3 + 3x2 + 2x – 4

Step 2: Divide the first term of the dividend by the first term of the
divisor, and write the result over the first term of the dividend.
4x3 ÷ x2 = 4x
4x
x2
– x + 1 4x )
3
+ 3x2
+ 2x –4

Step 3: Multiply the result obtained in Step 2 by the divisor and write the
result below the dividend.
4x(x2 – x + 1) = 4x3 – 4x2 + 4x
4x
)
x2 – x + 1 4x3 + 3x2 + 2x –4
4x3 – 4x2 + 4x

59

Pre-Calculus and Calculus.indb 59 13/7/2017 5:56:56 PM


Chapter 2 • Algebraic Expressions

Step 4: Subtract the result in Step 3 from the dividend, then bring down
the next term in the dividend.
(4x3 + 3x2 + 2x) – (4x3 – 4x2 + 4x) = 7x2 – 2x
4x
2 3
)
x – x + 1 4x + 3x + 2x – 4
2

4x3 – 4x2 + 4x
7x2 – 2x – 4

Step 5: Repeat Steps 2 to 4 using as the dividend.
7x ÷ x = 72 2

7(x2 – x + 1) = 7x2 – 7x + 7
(7x2 – 2x – 4) – (7x2 – 7x + 7) = 5x – 11

4x + 7
2
)
x – x + 1 4x + 3x + 2x – 4
3 2

4x3 – 4x2 + 4x
7x2 – 2x – 4
7x2 – 7x + 7
5x – 11

The process stopped at this point. It is not possible to divide further since
the degree of the dividend, 5x – 11 is now less than the degree of the
divisor, x2 – x + 1. In the division above, 4x + 7 is called the quotient and
5x – 11 is the remainder. Usually symbol q(x) is used for quotient and r(x)
for the remainder. Thus, q(x) = 4x + 7 and r(x) = 5x – 11.

Example 19
Find the quotient and remainder when p(x) = x3 – 2x2 – 6x + 5 is divided
by g(x) = x – 3.

SOLUTION
Using long division,

x2 + x – 3
)
x – 3 x – 2x2 – 6x + 5
3

x3 – 3x2
x2 – 6x
x2 – 3x
– 3x + 5
– 3x + 9
–4

The quotient q(x) = x2 + x – 3 and remainder r(x) = –4.

60

Pre-Calculus and Calculus.indb 60 13/7/2017 5:56:56 PM


Chapter 2 • Algebraic Expressions

Example 20
Use long division to divide p(x) = x4 + 2x2 – 8x + 3 by g(x) = x2 – 3x + 1.
State the quotient and remainder.

SOLUTION
Note that the polynomial p(x) has no x3 term, that is, the coefficient of x3
is zero. In this case, we need to add 0x3 term in p(x) to avoid confusion.
x2 + 3x + 10
2
)
x – 3x + 1 x + 0x + 2x2 – 8x + 3
4 3

x4 – 3x3 + x2
3x3 + x2 – 8x
3x3 – 9x2 + 3x
10x2 – 11x + 3
10x2 – 30x + 10
19x – 7

The quotient is q(x) = x2 + 3x + 10 and the remainder is r(x) = 19x – 7.

We have shown that when p(x) is divided by divisor g(x), we obtain


the quotient q(x) and the remainder r(x). The division can be written in
equation form as
____p(x) r(x)
​​   ​​ = q(x) + ____
​​    ​​ 
g(x) g(x)
Multiplying by throughout, we obtain
p(x) = q(x)g(x) + r(x)
When a polynomial p(x) of degree n is divided by divisor g(x) of degree
m, n  m, then
(a) the quotient will be of degree n – m
(b) the remainder (if it is not zero) will be of degree less than m.


: degree : degree 1

: degree : degree



: degree 2 : degree 0

: degree 3 : degree 1
The remainder is a constant (a polynomial of degree 0) when the divisior
is linear.

61

Pre-Calculus and Calculus.indb 61 13/7/2017 5:56:56 PM


Chapter 2 • Algebraic Expressions

Example 21
Find q(x) and r(x) such that x3 + 4x2 – 3x – 1 = q(x)(2x – 1) + r(x).

SOLUTION
q(x) is the quotient, and r(x) is the remainder when p(x) is divided by
g(x).
Using long division,
9 3
​​  21 ​​ x2 + __
__
​​  4 ​​ x – ​​ __
8  ​​
2x – 1 x3 + 4x2 – 3x – 1
x3 – __ ​​  21 ​​ x2
__ 9
​​  2 ​​ x2 – 3x
9 2 __ 9
​​ __ 2  ​​x – ​​  4 ​​ x
3
– ​​ __
4  ​​x – 1
3 3
– ​​ __ __
4 ​​ x + ​​  8  ​​
11 ​​ 
–​​ ___
8
9 3 11 ​​ .
​​  21 ​​ x2 + __
Therefore, q(x) = __ ​​  4 ​​  x – __
​​  8 ​​  and r(x) = –​​ ___
8

Example 22
Determine q(x) and r(x) such that
6x4 + x3 – 25x2 – 4x + 4 = q(x)(3x – 1) + r(x).

SOLUTION
Using long division,
2x3 + x2 – 8x – 4
3x – 1 6x + x3 – 25x2 – 4x + 4
4

6x4 – 2x3
3x3 – 25x2
3x3 – x2
– 24x2 – 4x
– 24x2 + 8x
– 12x + 4
– 12x + 4
0

Therefore, q(x) = 2x3 + x2 – 8x – 4 and r(x) = 0.

62

Pre-Calculus and Calculus.indb 62 13/7/2017 5:56:56 PM


Chapter 2 • Algebraic Expressions

In Example 22, the remainder is zero. In this case, the polynomial p(x)
is said to be completely divisible by g(x), i.e. g(x) is the exact divisor of
p(x). Thus, q(x) and g(x) become factors of p(x).
p(x) = q(x)g(x) + 0
= q(x)g(x)
Now, we look at a shortened form of the long division, called the synthetic
division. We are going to discuss synthetic division when the divisor is
linear, which is in the form of x + a.
Let us look at the following long division where p(x) = 2x4 + 3x3 – x – 5 is
divided by g(x) = x + 2.
2x3 – 1x2 + 2x – 5
x + 2  2x4 + 3x3 + 0x2 – 1x – 5
2x4 + 4x3
–1x3 + 0x2
–1x3 – 2x2
2x2 – 1x
2x2 + 4x
–5x – 5
–5x – 10
5
The above long division can be shortened to three lines below, where the
numbers in the shortened form are the numbers in bold, in the above long
division.
2 2 3 0 –1 –5
4 –2 4 –10
2 –1 2 –5 5
We can obtain the shortened form with the following steps.
Step 1: Write the value of a, the constant term of the divisor followed by
coefficients of the polynomial p(x) in one line.
2 2 3 0 –1 –5
a

Step 2: Bring down the first coefficient of p(x) to the third line.
2 2 3 0 –1 –5

2
Step 3: Multiply the first coefficient by a and write the product on the
second line as shown below.

2 2 3 0 –1 –5
4
2
63

Pre-Calculus and Calculus.indb 63 13/7/2017 5:56:57 PM


Chapter 2 • Algebraic Expressions

Step 4: Subtract 4 from 3 and write the value on the third line.
2 2 3 0 –1 –5
4 (subtract)

2 –1
Step 5: Repeat Steps 3 and 4:
(a) Multiply –1 by a and write the product on the second line.
(b) Subtract –2 from 0.

2 2 3 0 –1 –5
4 –2
2 –1 2

Step 6: Repeat Steps 3 and 4 until the division is completed, as shown


below.

2 2 3 0 –1 –5
4 –2 4 –10
2 –1 2 –5 5

Alternative Method:
The process can be made a little faster and less prone to sign errors by
rewriting g(x) = x + a as g(x) = x – (–a), and adding instead of subtracting
for Step 4. Using the same example, we now write g(x) = x + 2 as
g(x) = x – (–2).
The shortened division becomes
–2 2 3 0 –1 –5
g(x) = x + 2
–4 2 –4 10 (add)
= x – (–2) 2 –1 2 –5 5

Note The last number on the third line represents the value of the remainder, and
the rest of the numbers are the coefficients of the quotient.
Since g(x) is linear, the
remainder is a constant, and –2 2 3 0 –1 –5 coefficients of dividend
the degree of the quotient is –4 2 –4 10
3 (degree of p(x) – degree of
g(x)). 2 –1 2 –5 5 remainder

coefficients of quotient

Thus, the quotient q(x) is 2x3 – x2 + 2x – 5 and the remainder r(x) is 5. This
is called the synthetic division.

64

Pre-Calculus and Calculus.indb 64 13/7/2017 5:56:57 PM


Chapter 2 • Algebraic Expressions

Example 23
Divide using synthetic division. State the quotient and remainder.
(a) (3x4 – x – 4) ÷ (x + 1)
(b) (2x5 – 17x2 + 3) ÷ (x – 2)
SOLUTION
(a) p(x) = 3x4 – x – 4, g(x) = x + 1 = x – (–1)
–1 3 0 0 –1 –4
–3 3 –3 4
3 –3 3 –4 0
The quotient q(x) = 3x3 – 3x2 + 3x – 4 and the remainder r(x) = 0.
(b) p(x) = 2x5 – 17x2 + 3, g(x) = x – 2
2 2 0 0 –17 0 3
4 8 16 –2 –4
2 4 8 –1 –2 –1
The quotient q(x) = 2x4 + 4x3 + 8x2 – x – 2 and the remainder
r(x) = –1.
The next example shows how we divide p(x) by g(x) when g(x) is in the
form ax + b, where a and b are constants. To use the synthetic division,
we rewrite the divisor as a​(​  x + __ ​ ​​,  and assume we are dividing by x + __
​ a )
b b
​​  a ​​
instead of ax + b. In this case, the quotient obtained must be divided by a.
Let us discuss this in the following example.

Example 24
Divide p(x) by g(x) using synthetic division. State the quotient and
remainder.
(a) p(x) = 5x3 + 2x2 – 3x + 1, g(x) = 3x + 1
(b) p(x) = 8x3 + 4x + 3, g(x) = 2x – 1

SOLUTION
(a) Write the divisor as g(x) = ​3(​  x + __ ​ 13  ​)​​ and assume we are dividing by
​​  31 ​​ .
x + __
Using synthetic division,
1
–​​ __
3 ​​  5 2 –3 1
5 1 28
–​​ __ __ ___
3 ​​  –​​  9 ​​  ​​  27 ​​ 
1  ​​ 28 ___ 55
5 ​​ __
3 –​​ ___
9 ​​  ​​  27 ​​ 

65

Pre-Calculus and Calculus.indb 65 13/7/2017 5:56:57 PM


Chapter 2 • Algebraic Expressions

For the above division we have,


5x3 + 2x2 – 3x + 1 = (​​  5x2 + __ ​  9 ​ )(​​​​    x + __
​ 13  ​)​​  + ___
28 55
​ 31 ​ x – ___ ​​  27 ​​  ——— (1)
which can be rewritten as
5x3 + 2x2 – 3x + 1 = (​​  5x2 + __ ​  9 ​ )(​​​​    __  ​​  + ___
)
28 3x + 1 55
​ 31 ​ x – ___ ​  3 ​   ​​  27 ​​ 

​​  31( ​  9 ​ )​​( 3x + 1) + ___


28 55
= __  ​​​​   5x2 + __ ​ 31 ​ x – ___ ​​ 27 ​​ 

= (​​​​  __ ​ 27 ​ )(​​  3x + 1) + ___


5 28 55
​  3 ​ x2 + __ ​ 91 ​ x – ___ ​​ 27 ​​  ——— (2)
Equation (1) is the equation form when p(x) is divided by x + __ ​​  31 ​​ ,
Note
and equation (2) is the equation form when p(x) is divided by
The quotient 3x + 1. The quotient and remainder when p(x) is divided by 3x + 1,
__ 5 __ 1 ___ 28 5 28 55
q(x) = ​​   ​​ x2 +​​   ​​ x – ​​   ​​ is
3 9 27 is q(x) = __ ​​  91 ​​ x – ​​ ___
​​  3 ​​ x2 + __ ___
27 ​​ and r(x) = ​​  27 ​​ .
obtained by dividing the
coefficients of the quotient (b) p(x) = 8x3 + 4x + 3, g(x) = 2x – 1
by 3. ​ 21 ​ )​​ and assume we are dividing by
Write the divisor as g(x) = 2​​ (x – __
​​  21 ​​ .
x – __
Using synthetic division,
​​  21 ​​ 
__
8 0 4 3
4 2 3
8 4 6 6

8x2 + 4x + 6
The quotient q(x) = ​​  __________
2 ​​  = 4x2 + 2x + 3 and the remainder

r(x) = 6.

66

Pre-Calculus and Calculus.indb 66 13/7/2017 5:56:57 PM


Chapter 2 • Algebraic Expressions

Exercise 2.1
1. Add the following polynomials.
(a) 3x3 – 4x2 + 5x; 4x2 – 7x3 + 10
(b) 3a2b3 – 5a3b2 + 7a4b + 9ab4; 5ab4 – 6a2b3 + 8a3b2 – a4b

2. Subtract the following polynomials.


(a) (8y4 – 7y3 + 6y + 15) – (6y3 – 5y2 – 9y + 3)
(b) (3a – 6b + 18ab) – (4a2 – 11b + 19ab)
(c) (5m3n2 – 10m4n + 9m4n – 6m2n3) – (5m4n – 6mn4 + m2n3)

3. Perform the indicated operations.


(a) (10x4 – 3x2 + 3x) + (2x3 – 11x2 + 5) – (6x4 – 4x3 + 5x2 – x – 8)
(b) (6a2 – 5a3 + 6a5 – 7) – [(3a3 – 2a2 + 7a – 4) + (8a4 – 5a + 3a2)]
(c) (4r2 + 15rs – 10s2) + (5s2 – 8rs + r2) – (16rs + 8r2 – 5s2)
(d) (12m3 – 9n3 + 4mn2 – 7mn2) – [(m2 + 3mn2) – (m3 + 5n3)]
(e) –3p2q(8p2q2 – 6p3q + 5pq3 – 9)
4x2y4 – 12x4y5 + 16x4y2
(f) ​​ ___________________
       ​​
2x2y2
(g) 5x(2x3 – 5x + 6) + 3x(7x + 1)
(h) 6a2(3a – 5) – 3a(2a – 3)
3x2y – 15x2y + 24x2y2 __________________
12x3y2 – 8x2y3 + 6x3y3
(i) ​​ _________________
   3xy  ​​
  – ​​ 
       ​​
2x2y2
(j) (48x4 – 36x3 + 18x2 – 45x + 15) ÷ 3x

4. Find the special product.


(a) (5x2 + 2y2)(5x2 – 2y2)
(b) (6a – 7b)2
(c) (9x2y + xy2)2
(d) (5r – 11s)(2r + 8s)
(e) (4x2 – y2)(5x2 – 9y2)
(f) (3v + 4)(9v2 – 12v + 16)
(g) (7a2 – 3b2)(49a4 + 21a2b2 + 9b4)

5. Multiply the following.


(a) 5x(x2 – 3x +2)
(b) –2a2(3a2 + a – 5)
(c) (y – 10)(y + 10)
(d) (8b + 3c)(8b – 3c)
(e) (y + 7)2
(f) (3r – 5s)2
(g) (x – 2)(x2 + 2x + 4)
(h) (a + 1)(a2 – a + 1)
(i) (p + 5)(p – 6)
(j) (5m – 2n)(3m + 4n)

67

Pre-Calculus and Calculus.indb 67 13/7/2017 5:56:57 PM


Chapter 2 • Algebraic Expressions

6. Find the special product.


(a) ​​ (2x + __ ​ 5 ​ )(​​​​    2x + __ ​ 5 ​ )​​  
3 3

(b) ​​​ (3​m​​  2​ + __ ​ ​​​​​   ​​


​  14 ​​ n​​  3)
2

(c) [(3x – 2y) + 5]2


(d) ​​ (5x + __ ​ 12  ​y)(​​​​    2x – __ ​ 15  ​)​​  
(e) [(5a – b) – 1][(5a – b) + 10]
(f) [a – (b + 2)][a + (b + 2)]

7. Use long division to find the quotient and remainder when polynomial p(x) is divided by
polynomial g(x).
(a) p(x) = x3 – x2 + 2x – 1,
g(x) = x3 + 1
(b) p(x) = 3x4 – 5x2 + x – 6,
​​  12 ​​ 
g(x) = x – __
(c) p(x) = 6x3 + 5x2 – 4x – 1,
g(x) = 2x – 3
(d) p(x) = x4 – x3 + 2x2 – x + 2,
g(x) = x2 – x + 1
(e) p(x) = 3x4 + 2x3 – x2 – x – 6,
g(x) = x2 – 3

8. Determine q(u) and r(u) such that f (u) = q(u)g(u) + r(u).


(a) f (u) = u3 + 2u2 – 5u + 8,
g(u) = u – 1
(b) f (u) = u4 + 3u2 – u,
g(u) = u + 2
(c) f (u) = 2u5 + u2 + 3,
g(u) = u + 1
(d) f (u) = 5u3 + 2u2 – 3u + 1
g(u) = 3u + 1
(e) f (u) = 6u3 + 5u2 – 4u – 1
g(u) = 2u – 3

9. Using synthetic division to find the quotient and remainder when polynomial p(x) is divided by g(x).
(a) p(x) = 2x3 – 3x2 + x – 1; g(x) = x – 2
(b) p(x) = x3 – 2x2 + 3; g(x) = x + 2
(c) p(x) = 3x5 – 2x3 + 3; g(x) = 2x + 1
(d) p(x) = 7x6 + x5 + 12x2 + 5x – 3;
g(x) = x – 1

68

Pre-Calculus and Calculus.indb 68 13/7/2017 5:56:57 PM


Chapter 2 • Algebraic Expressions

2.2 Factorisation of Algebraic Expressions


2.2.1 Common Factors
2.2.2 Factoring by Grouping
2.2.3 Perfect Squares
2.2.4 Difference of Squares
2.2.5 Trinomials
2.2.6 Sum and Difference of Cubes

Factorisation is the process of getting the factors of a given polynomial.


It is the reverse of finding the product of polynomials. Factoring the
product results to the original polynomials that were multiplied together
to get the product.

Definition 2.7

A polynomial is said to be factored completely if it is written as a product


consisting only of prime or irreducible factors.

A polynomial is prime or irreducible if it cannot be expressed as a


product of at least two polynomials. The factors of a prime or irreducible
polynomial are only 1 and itself or –1 and itself.
For example, the binomial x + 2 can only be expressed as a product of the
factors (x + 2) and 1. Hence, the polynomial x + 2 is prime or irreducible
since it cannot be expressed as a product of two polynomials.
On the other hand, the binomial 4x2 + 10x can be expressed as the product of 2x
and 2x + 5. Hence, 4x2 + 10x is factorable.
Examples of prime polynomials are x2 + 2, a2b – c2, x + y + z and x3 + y3.

2.2.1 Common Factors


The first step in factoring a polynomial is to determine if there is a monomial
factor that is common to all terms of the polynomial. It is advisable to look
for the greatest common factor (GCF) of the terms.
The greatest common factor of a polynomial with more than one term is
a monomial with the greatest coefficient and degree that is common to all
the terms.
Let us look at the process to find the greatest common factor of the
polynomial 12x3 + 30x2.
Step 1: Find the GCF of the numerical coefficients.
GCF of 12 and 30 = 6
Step 2: Find the GCF of the literal coefficients.
GCF of x3 and x2 = x2
Step 3: The product of the GCF of the numerical coefficients and the
GCF of the literal coefficients is the greatest common monomial
factor of the polynomial.
GCF of the polynomial 12x3 + 30x2 = 6x2
69

Pre-Calculus and Calculus.indb 69 13/7/2017 5:56:57 PM


Chapter 2 • Algebraic Expressions

Once the greatest common monomial factor of a polynomial has been


determined, the polynomial can then be factored. The GCF of the terms of
12x3 + 30x2 is 6x2. This is the first factor. The second factor is solved by
dividing each term of 12x3 + 30x2 by 6x2.
6x2(2x) + 6x2(5)
12x3 + 30x2 _____________
​​ __________
2 ​​ = ​​   
   ​​ 
6x 6x2
6x2(2x + 5)
= _________
​​   ​​  
6x2
= 2x + 5
Note
This factoring uses the Thus, the factors of 12x3 + 30x2 are 6x2 and 2x + 5.
distributive property: Hence, polynomial 12x3 + 30x2 written in factored form is
ax + ay = a(x + y)
12x3 + 30x2 = 6x2(2x + 5).

Summary

The following steps show the process of factoring by finding the greatest
common monomial factor.
Step 1: Find the GCF of the terms of the polynomial to get the first
factor.
Step 2:  F
 ind the second factor by dividing the given polynomial by
the GCF.
polynomial = (GCF)​​ (__________ )​​  
polynomial
​  GCF ​   

Example 25
Find the greatest common monomial factor of each polynomial and
write it in factored form.
(a) 3x + 18 (c) 2x4 – 10x3 – 6x2
(b) –9r2s + 15rs2 (d) 5t5 + 30t3

SOLUTION
(a) Greatest common monomial factor: 3
The other factor: x + 6
Factored form: 3(x + 6)
(b) Greatest common monomial factor: –3rs
The other factor: 3r – 5s
Factored form: –3rs(3r – 5s)
(c) Greatest common monomial factor: 2x2
The other factor: x2 – 5x – 3
Factored form: 2x2(x2 – 5x – 3)
(d) Greatest common monomial factor: 5t3
The other factor: t2 + 6
Factored form: 5t3(t2 + 6)

70

Pre-Calculus and Calculus.indb 70 13/7/2017 5:56:57 PM


Chapter 2 • Algebraic Expressions

Example 26
Factor the following polynomials completely.
(a) 20x4y5 + 8x3y6
(b) 18a6b2c2 – 6a4b2c
(c) –12r3s3 + 24r3s2 – 60r2s3

SOLUTION
(a) The GCF of the numerical coefficients of 20x4y5 and 8x3y6 is 4,
while the GCF of the literal coefficients is x3y5. Thus, the greatest
common monomial factor is 4x3y5.
The other factor is obtained by dividing 20x4y5 + 8x3y6 by 4x3y5.
20x4y5 + 8x3y6 _________________
____________ 4x3y5(5x) + 4x3y5(2y)
​​     ​​ =
    
​​   ​​ 
4x3y5 4x3y5
4x3y5(5x + 2y)
____________
=    ​​   ​​ 
4x3y5
= 5x + 2y
Hence, 20x4y5 + 8x3y6 = 4x3y5(5x + 2y)
(b) The greatest common monomial factor of the given polynomial is
6a4b2c.
3a2c(6a4b2c) – 6a4b2c
18a6b2c2 – 6a4b2c ___________________
​​ ______________
   4 2   ​​ =   
  ​​       ​​
6a b c 6a4b2c
6a4b2c(3a2c – 1)
______________
=    ​​    ​​ 
6a4b2c
= 3a2c – 1
Hence, 18a6b2c2 – 6a4b2c = 6a4b2c(3a2c – 1)
(c) Since the first term is negative, make the GCF negative. Then, get Note
the other factor by dividing the given polynomial with the GCF.
Factoring out the negative of
–12r2s2(rs – 2r + 5s)
–12r3s3 + 24r3s2 – 60r2s3 _________________
​​ ____________________
      2 2 ​​ =    ​​      ​​ the GCF is usually desirable
–12r s –12r2s2 when the terms in the
= rs – 2r + 5s remaining factor are not in
descending order.
Hence, –12r3s3 + 24r3s2 – 60r2s3 = –12r2s2(rs – 2r + 5s)

Example 27
Identify the greatest common factor of 125x3y2z2 + 75x2y3z2 – 25x2y3z3
and factor the polynomial completely.

SOLUTION
The greatest common factor of 125x3y2z2 + 75x2y3z2 – 25x2y3z3 is 25x2y2z2.
125x3y2z2 + 75x2y3z2 – 25x2y3z3 _________________
25x2y2z2(5x + 3y – yz)
​​ _________________________
       2 2 2 ​​ =    ​​      ​​
25x y z 25x2y2z2
= 5x + 3y – yz
Hence, 125x3y2z2 + 75x2y3z2 – 25x2y3z3 = 25x2y2z2(5x + 3y – yz)

71

Pre-Calculus and Calculus.indb 71 13/7/2017 5:56:57 PM


Chapter 2 • Algebraic Expressions

2.2.2 Factoring by Grouping


There are instances in which there is no common factor among the terms
of a polynomial. Examine the polynomial below.
2x + 2y + xz + yz
The terms of the given polynomial do not have a common factor. However,
notice that the first two terms both have a numerical coefficient of 2 and
the last two terms have a literal coefficient of z. The terms can then be
grouped so that each group has a common factor.
Pair up the two terms with
common factor 2 in one group and
2x + 2y + xz + yz = (2x + 2y) + (xz + yz) the other two terms with common
= 2(x + y) + z(x + y) factor z in another group.
= (x + y)(2 + z)
Factor each group. In this case, the
common factor of the two groups
is (x + y).

Summary

The following steps show the process of factoring by grouping.


Step 1: Find any greatest monomial factor that is common among all
terms of the given polynomial.
Step 2: Group pairs of terms so that the terms in each group have a
common factor.
Step 3: Find the GCF in each group. If there is a common binomial
factor, then factor the polynomial.
Step 4: Write the polynomial as a product of the common factor and
the other remaining factors.

72

Pre-Calculus and Calculus.indb 72 13/7/2017 5:56:57 PM


Chapter 2 • Algebraic Expressions

Example 28
Factor the following polynomials completely.
(a) 2rv + 3sv – 2rw – 3sw
(b) 5xy – 10xz + y – 2z
(c) 2x3 – 10x2 – 4x + 20
(d) 10a3b2 – 10a2b2 + 5a2b – 5ab
(e) 4rs2t + 8rs2 + 12rt2 + 24rt

SOLUTION
(a) The first two terms have common monomial factor v, and the other
two terms have common factor w.
Grouping the first two terms and the other two terms,
2rv + 3sv – 2rw – 3sw = (2rv + 3sv) – (2rw + 3sw)
= v(2r + 3s) – w(2r + 3s)
= (2r + 3s)(v – w)
(b) 5xy – 10xz + y – 2z =
 (5xy – 10xz) + (y – 2z)
= 5x(y – 2z) + (y – 2z)
= (y – 2z)(5x + 1)
(c) 2x3 – 10x2 – 4x + 20 = 2(x3 – 5x2 – 2x + 10)
= 2[(x3 – 5x2) – (2x – 10)]
= 2[x2(x – 5) – 2(x – 5)]
= 2(x – 5)(x2 – 2)
(d) 10a3b2 – 10a2b2 + 5a2b – 5ab = 5ab(2a2b – 2ab + a – 1)
= 5ab[(2a2b – 2ab) + (a – 1)]
= 5ab[2ab(a – 1) + (a – 1)]
= 5ab(a – 1)(2ab + 1)
(e) 4rs2t + 8rs2 + 12rt2 + 24rt = 4r(s2t + 2s2 + 3t2 + 6t)
= 4r[(s2t + 2s2) + (3t2 + 2t)]
= 4r[s2(t + 2) + 3t(t + 2)]
= 4r(t + 2)(s2 + 3t)

73

Pre-Calculus and Calculus.indb 73 13/7/2017 5:56:57 PM


Chapter 2 • Algebraic Expressions

2.2.3 Perfect Squares


In Section 2.1.6, you have learnt to find the special product for the square
of a binomial.
(x + y)2 = x2 + 2xy + y2
(x – y)2 = x2 – 2xy + y2
Reversing the process, you can factor a perfect square trinomial by
expressing it as the square of a binomial.

Theorem 2.4

Factoring a perfect square trinomial will give:


x2 + 2xy + y2 = (x + y)2
x2 – 2xy + y2 = (x – y)2

Example 29
Factor the following polynomials.
(a) x2 + 6x + 9
(b) 9c2 – 12cd + 4d2
(c) 16r4 + 24r2s3 + 9s6

SOLUTION
(a) x2 + 6x + 9 = (x)2 + 2(x)(3) + (3)2
= (x + 3)2
(b) 9c2 – 12cd + 4d2 = (3c)2 – 2(3c)(2d) + (2d)2
= (3c – 2d)2
(c) 16r4 + 24r2s3 + 9s6 = (4r2)2 + 2(4r2)(3s3) + (3s3)2
= (4r2 + 3s3)2

Example 30
A classroom has a square floor with an area of (36x2 + 108x + 81) square
units. What is its length?

SOLUTION
The area of a square is given by A = l 2.
A = l 2
= 36x2 + 108x + 81
= (6x)2 + 2(6x)(9) + (9)2
__
= (6x + 9)2
________
​ ​​  (6x + 9)2 ​​ 
​  l 2  ​​ = √

l = 6x + 9
Hence, the length of the floor is (6x + 9) units.

74

Pre-Calculus and Calculus.indb 74 13/7/2017 5:56:57 PM


Chapter 2 • Algebraic Expressions

2.2.4 Difference of Squares


The special product of the sum and the difference of two terms is the
difference of the squares of the terms.
(x + y)(x – y) = x2 – y2

Reversing the process gives the technique for factoring the difference of
two squares.

Theorem 2.5

Factoring the difference of two squares will give:


x2 – y2 = (x + y)(x – y)

The technique for factoring the difference of two squares can be illustrated
using a square as shown below.
Step 1: Cut out a square and label its sides as x.
x

Step 2: Cut out a small square with sides y from the square with sides x.
x–y y
y

x
x–y

x
The area of the remaining piece is x2 – y2.
Step 3: 
Cut out the remaining piece in half to form two congruent
trapezoids.
x–y
y
y
x
x–y

x
Step 4: Rearrange and fit the two trapezoids to form a rectangle.
x+y

x–y x–y

x+y

75

Pre-Calculus and Calculus.indb 75 13/7/2017 5:56:58 PM


Chapter 2 • Algebraic Expressions

The width of the rectangle is x – y and its length is x + y. Therefore,


the area of the rectangle formed by the two trapezoids is given by
A = lw = (x + y)(x – y).
The area of the rectangle is equal to the area of the remaining
piece.
x2 – y2 = (x + y)(x – y)

area of the area of the


remaining piece rectangle

Example 31
Factor the following expressions completely.
(a) x2 – 100
(b) 25r2 – 36s4
(c) (x + 2y)2 – 9w2
(d) x4 – y8z4

SOLUTION
(a) x2 – 100 = x2 – (10)2
= (x + 10)(x – 10)
(b) 25r – 36s4 = (5r)2 – (6s2)2
2

= (5r + 6s2)(5r – 6s2)


(c) (x + 2y)2 – 9w2 = (x + 2y)2 – (3w)2
= [(x + 2y) + 3w][(x + 2y) – 3w]
Note = (x + 2y + 3w)(x + 2y – 3w)
The sum of squares x2 + y2 (d) x4 – y8z4 = (x2)2 – (y4z2)2
is prime and cannot be = (x2 + y4z2)(x2 – y4z2)
factored. = (x2 + y4z2)​​ [(x)2 – (y2z)2] ​​
= (x2 + y4z2)(x + y2z)(x – y2z)

Example 32

Factor ___ 1  ​​ a8b4 – (c + 2)4 completely.


​​  16

SOLUTION
1 ​​ a​​  4​​b​​  2​ ​​​   ​​ – [(c + 2)2]2
( )
2
1  ​​ a8b4 – (c + 2)4 = ​​​  __
​​ ___ ​ 
16 4
= [​​  ​  4 ​​ a​​  4​​b​​  2​+ (c + 2​)​​  2​][​​​​   __
__ 1 ​  41 ​​ a​​  4​​b​​  2​– (c + 2)2]​​  

​  41 ​​ a​​  4​​b​​  2​+ (c + 2​)​​  2​][​​​​   (​​  __


= [​​  __ ​  21 ​​ a​​  2​b)​​​   ​– (c + 2)2]​​  
2

​  41 ​​ a​​  4​​b​​  2​+ (c + 2)2](​​​​    __


= [​​  __ ​  21 ​​ a​​  2​b + c + 2)[​​​​   __ ​  21 ​​ a​​  2​b – (c + 2)]​​  

= (​​  __ ​  41 ​​ a​​  4​​b​​  2​ + c2 + 4c + 4))(​​​​    __ ​  21 ​​ a​​  2​b + c + 2)(​​​​    __ ​  21 ​​ a​​  2​b – c – 2)​​  

76

Pre-Calculus and Calculus.indb 76 13/7/2017 5:56:58 PM


Chapter 2 • Algebraic Expressions

2.2.5 Trinomials
To factor trinomials of the form ax2 + bx + c, consider the reverse process
of multiplying two binomials.
The product of mx + n and px + q gives mpx2 + (mq + np)x + nq, which is
a trinomial.
Its reverse gives mpx2 + (mq + np)x + nq = (mx + n)(px + q).
The trinomial mpx2 + (mq + np)x + nq could be simplified to:
mpx2 + (mq + np)x + nq Note
If the last term c > 0, then n
ax2 + bx + c and q should have the same
sign.
where mp = a, mq + np = b and nq = c. The trinomial could then be
simplified to ax2 + bx + c.
To factor a trinomial of the form ax2 + bx + c, where a = 1, we will use the
cross method.
Consider the trinomial x2 + 5x + 6.
Step 1: Draw a cross.

Step 2: Place the squared term under the left of the horizontal line, the
middle term under the right, and the constant term between the
squared and the middle term.
squared constant middle
term term term

x2 +6 +5x

Step 3: Think of a pair of factors of the squared and constant terms. Write
the factors above the horizontal line.
x +3

x +2
x 2
+6 +5x

77

Pre-Calculus and Calculus.indb 77 13/7/2017 5:56:58 PM


Chapter 2 • Algebraic Expressions

Step 4: Cross-multiply the factors as shown below and write the products
in the last column.
x +3 +3x

x +2 +2x
x 2
+6 +5x

Step 5: Add the products in the last column. If the sum matches the middle
term (+5x), then the factors of the first and the constant terms are
correct. However, if the sum does not match the middle term,
think of another possible pairs of factors of the squared and
constant terms, and repeat Steps 3 to 5.
Hence, x2 + 5x + 6 = (x + 3)(x + 2)

Example 33
Factor x2 – 8x + 15 completely.

SOLUTION
squared term constant term
middle term

x2 +15 –8x
Think of the factors of x2 and +15. For +15, the possible factors are 1
and 15, –1 and –15, 3 and 5, –3 and –5. The only possible combination
that will give –8x is –3 and –5.

x –5 –5x

x –3 –3x
x2 +15 –8x
Hence, x2 – 8x + 15 = (x – 5)(x – 3).

78

Pre-Calculus and Calculus.indb 78 13/7/2017 5:56:58 PM


Chapter 2 • Algebraic Expressions

Example 34
Factor the following polynomials completely.
(a) x2 + 4x + 3 (d) x2 – 13x + 42
(b) x + 3x – 10
2
(e) x2 + 8x – 48
(c) x – 6x – 7
2

SOLUTION
(a) x2 + 4x + 3
x +3 +3x

x +1 +x
x2 +3 +4x

Hence, x2 + 4x + 3 = (x + 3)(x + 1)
(b) x2 + 3x – 10
x +5 +5x

x –2 –2x
x2 –10 +3x

Hence, x2 + 3x – 10 = (x + 5)(x – 2)
(c) x2 – 6x – 7
x +1 +x

x –7 –7x
x2 –7 –6x

Hence, x2 – 6x – 7 = (x + 1)(x – 7)
(d) x2 – 13x + 42
x –7 –7x

x –6 –6x
x2 +42 –13x

Hence, x – 13x + 42 = (x – 7)(x – 6)
2

(e) x2 + 8x – 48
x +12 +12x

x –4 –4x
x2 –48 +8x

Hence, x2 + 8x – 48 = (x + 12)(x – 4)

79

Pre-Calculus and Calculus.indb 79 13/7/2017 5:56:58 PM


Chapter 2 • Algebraic Expressions

Now, consider a trinomial of the form ax2 + bx + c, where a ≠ 1. When the


coefficient of x2 is not 1, we can still use the cross method as shown in the
following examples. There will be more combinations of factors that we
have to test because the squared term is not just x2.

Example 35
Factor the following polynomials completely.
(a) 2x2 + 7x + 3 (d) 4x2 – 19x – 5
(b) 4x2 – x – 14 (e) 6x2 + 2x – 20
(c) 8x2 – 79x + 63 (f) 42x2 – 40x – 18

SOLUTION
(a) 2x2 + 7x + 3
2x +1 +x

x +3 +6x
2x 2
+3 +7x

Hence, 2x2 + 7x + 3 = (2x + 1)(x + 3)
(b) 4x2 – x – 14
4x +7 +7x

x –2 –8x
4x2 –14 –x

Hence, 4x2 – x – 14 = (4x + 7)(x – 2)
(c) 8x2 – 79x + 63
8x –7 –7x

x –9 –72x
8x 2
+63 –79x

Hence, 8x2 – 79x + 63 = (8x – 7)(x – 9)
(d) 4x2 – 19x – 5
4x +1 +x

x –5 –20x
4x2 –5 –19x

Hence, 4x2 – 19x – 5 = (4x + 1)(x – 5)

80

Pre-Calculus and Calculus.indb 80 13/7/2017 5:56:58 PM


Chapter 2 • Algebraic Expressions

(e) 6x2 + 2x – 20
3x –5 –10x

2x +4 +12x
6x2 –20 +2x

Hence, 6x2 + 2x – 20 = (3x – 5)(2x + 4)
(f) 42x2 – 40x – 18 = 2(21x2 – 20x – 9)
7x –9 –27x

3x +1 +7x
21x2 –9 –20x

Hence, 42x – 40x – 18 = 2(7x – 9)(3x + 1)
2

2.2.6 Sum or Difference of Cubes


The sum of two squares cannot be factored, but the sum or the difference
of two cubes is factorable.

Theorem 2.6
Note
The expressions x2 + xy + y2
Factoring the sum or difference of two cubes will give: and x2 – xy + y2 are not
perfect square trinomials.
x3 + y3 = (x + y)(x2 – xy + y2)
Hence, they cannot be
x3 – y3 = (x – y)(x2 + xy + y2) factored.

Example 36
Factor the following polynomials completely.
(a) r3 + 27 (c) 2m6 – 54n9
(b) 8c + 64d
6 3
(d) 64c9 – 125d 12

SOLUTION
(a) r3 + 27 = r3 + 33
= (r + 3)[(r)2 – (r)(3) + (3)2]
= (r + 3)(r2 – 3r + 9)
(b) 8c6 + 64d 3 = (2c2)3 + (4d)3
= (2c2 + 4d)[(2c2)2 – (2c2)(4d) + (4d)2]
= (2c2 + 4d)(4c4 – 8c2d + 16d 2)
(c) 2m6 – 54n9 = 2(m6 – 27n9)
= 2[(m2)3 – (3n3)3]
= 2{(m2 – 3n3)[(m2)2 + (m2)(3n3) + (3n3)2]}
= 2(m2 – 3n3)(m4 + 3m2n3 + 9n6)
(d) 64c9 – 125d 12 = (4c3)3 – (5d 4)3
= (4c3 – 5d 4)[(4c3)2 + (4c3)(5d 4) + (5d 4)2]
= (4c3 – 5d 4)(16c6 + 20c3d 4 + 25d 8)
81

Pre-Calculus and Calculus.indb 81 13/7/2017 5:56:58 PM


Chapter 2 • Algebraic Expressions

Example 37
Christopher wants to construct a rectangular pool with a base area of
(5x + 10px + 15qx) cm2.
(a) Factor 5x + 10px + 15qx.
(b) The length of the pool area is 5x cm. Find the perimeter of the base
of the pool given that x = 5, p = 8, and q = 4.

SOLUTION
(a) 5x + 10px + 15qx = 5x(1 + 2p + 3q)
(b) Area of the pool =
 5x + 10px + 15qx
= 5x(1 + 2p + 3q)
Length = 5x
Width = 1 + 2p + 3q
The perimeter of a rectangle is given by the formula P = 2l + 2w.
P=  2l + 2w
= 2(5x) + 2(1 + 2p + 3q)
= 10x + (2 + 4p + 6q)
= (10x + 2 + 4p + 6q) cm
Given that x = 5, p = 8 and q = 4,
P=  [10(5) + 2 + 4(8) + 6(4)] cm
= (50 + 2 + 32 + 24) cm
= 108 cm
The perimeter of the base of the pool is 108 cm.

Example 38
Simplify the following polynomials using the appropriate factoring
technique.
(a) (x + y)2 – (x – y)2 (b) (p + 4)3 + (p – 4)3

SOLUTION
(a) Factor (x + y)2 – (x – y)2 using the technique for the difference of
two squares. Let (x + y) be the first term and (x – y) be the second
term.
(x + y)2 – (x – y)2 = [(x + y) + (x – y)][(x + y) – (x – y)]
= (x + y + x – y)(x + y – x + y)
= (2x)(2y)
= 4xy
(b) Factor (p + 4)3 + (p – 4)3 using the technique for the sum of two
cubes. Let (p + 4) be the first term and (p – 4) be the second term.
(p + 4)3 + (p – 4)3
= [(p + 4) + (p – 4)][(p + 4)2 – (p + 4)(p – 4) + (p – 4)2]
= (p + 4 + p – 4)[(p2 + 8p + 16) – (p2 – 16) + (p2 – 8p + 16)]
= (2p)(p2 + 8p + 16 – p2 + 16 + p2 – 8p + 16)
= (2p)(p2 + 48)
= 2p3 + 96p

82

Pre-Calculus and Calculus.indb 82 13/7/2017 5:56:58 PM


Chapter 2 • Algebraic Expressions

Example 39
Factor the following polynomials completely.
(a) 9a2 – b2 + 2bc – 2c2 (b) x2 + 2xy – 3y2 + 3x – 3y
Note
SOLUTION Group the last three terms
(a) 9a2 – b2 + 2bc – c2 = 9a2 – (b2 – 2bc + c2) together since they form a
= 9a2 – (b – c)2 perfect square trinomial.
= (3a)2 – (b – c)2
= [3a + (b – c)][3a – (b – c)] Note
= (3a + b – c)(3a – b + c)
x +3y +3xy
(b) x2 + 2xy – 3y2 + 3x – 3y = (x2 + 2xy – 3y2) + (3x – 3y)
= [(x + 3y)(x – y)] + (3x – 3y) x –y –xy
= (x + 3y)(x – y) + 3(x – y)
= (x – y)(x + 3y + 3) x2 –3y2 +2xy

Exercise 2.2
1. Factor the following polynomials completely.
(a) 4x2y – 8xy3 + 6xy4
(b) 8x2 – 2y2
(c) ab – bc + da – dc
(d) 4a2 – 4ab + b2
(e) 8r3 + 125s3
(f) c2 + 2c – 3
(g) 64m3 – n3

2. Factor the following polynomials completely.


(a) 3x2 – 13x + 12
(b) 16(a + b)2 – 9c2
(c) 18x4 – 32y6
(d) c2x6 – c2y12
(e) 4mp – 6np – 12mq + 18nq
(f) 125p6 – 8q6
(g) 21b2 – 43b – 14

3. Factor the following polynomials completely.


(a) 25r4 + 30r2s2 + 9s4
(b) (x + 2)2 – (y + 1)2
(c) r3 – s3 – r2 + s2
(d) m2 – 2mn + n2 – mp + np
(e) a2 + ab – 2b2 – a3 + b3

83

Pre-Calculus and Calculus.indb 83 13/7/2017 5:56:58 PM


Chapter 2 • Algebraic Expressions

4. Factor the following polynomials completely.


(a) 9v4 + 12v2w2 + 4w4
(b) (r + s)2 – (2 + rs)2
(c) m4 – (3m + 4)2
(d) 21x4 + 4x2(5z – 3y) – 12(5z – 3y)2
(e) (x – 4y)3 + 64
(f) r4s3 – 16s3
(g) 25c8 + 20c4d 4 + 4d 8
(h) 2r2 + 5rs – 3s2 + 8r3 – s3

5. Factor 1 + mn – m – n completely.

6. Factor 5x4 + 5x3 – 100x2 completely.

7. If 6a2b2 is factored out from 24a2b2 – 18a3b2, what is the binomial left?

8. What is the factored form of 6a2 – 16a + 8?

9. The area of a rectangle is (3x2 + 7x – 6) square units and its length is (3x – 2) units. What is its width
in terms of x?

10. The volume of a cuboid is (a3 – 5a2 + 6a) cubic units. If one of the dimensions is a units, what are the
two other dimensions in terms of a?

84

Pre-Calculus and Calculus.indb 84 13/7/2017 5:56:58 PM


Algebra
Chapter 3:

Algebraic Fractions

Climatology

Global temperature has increased tremendously through time. The temperature increase over
the last 50 years is due to the increase in the atmosphere of greenhouse gas concentrations
like water vapor, carbon dioxide, methane, and ozone in the atmosphere. The largest cause of
greenhouse gas concentration is the burning of fossil fuels leading to the emission of carbon
dioxide. The Earth’s temperature is likely to increase by 1.1 ºC to 6.4 °C by the end of the
twenty-first century.
Converting units of temperature from degrees Celsius to degrees Fahrenheit or vice versa
5([ºF] – 32)
makes use of equations involving algebraic fractions. The formula [ºC] = ​​ __________
9 ​​  allows

you to convert temperature readings from degrees Fahrenheit to degrees Celsius. How much
will the Earth’s temperature increase in degrees Fahrenheit, by the end of the twenty-first
century?

Pre-Calculus and Calculus.indb 85 13/7/2017 5:56:58 PM


Chapter 3 • Algebraic Fractions

3 A
 lgebraic Fractions
3.1 Simplification of Algebraic Fractions
3.2 Operations of Algebraic Fractions
3.3 Rationalization of Algebraic Fractions

3.1 Simplification of Algebraic Fractions


Simplifying algebraic fractions is equivalent to reducing fractions to their
lowest terms.
A fraction is in its lowest terms if its numerator and denominator have no
common factors other than 1 or –1.
To reduce a fraction to its lowest terms, factor the numerator and the
denominator and cancel the common factors.
15 (3)(5) 5
​​  24 ​​ = ​​ __________
___
= ​​ __ ​​ 
  ​​ 
(3)(2)(2)(2) 8
15 5
Hence, ___
​​  24 ​​ reduced to its lowest terms is __
​​ 8 ​​ .

Example 1
Reduce the following fractions to their lowest terms.
6 8
(a) ​​ ___
52  ​​   (c) ​​ ___
48  ​​ 
5 72
(b) ​​ ___
70  ​​   (d) ​​ ___
48 ​​ 

SOLUTION
6 _________ (2)(3) 3 3
(a) ​​ ___
52  ​​ = ​​  (2)(2)(13) = ______
  ​​  ​​  = ___
   ​​  ​​    ​​ 
(2)(13) 26
5 ________ 5
(b) ​​ ___
70  ​​ = ​​  (2)(5)(7) ​​  1   ​​ 
= _____
   ​​  ​​  1  ​​ 
= ___
(2)(7) 14
(2)(2)(2)
8 ____________
(c) ​​ ___
  ​​ = ​​    
48 (2)(2)(2)(2)(3) ​​  1   ​​ 
  ​​ = _____ ​​  1 ​​ 
= __
(2)(3) 6
(2)(2)(2)(3)(3) __
72 ____________ 3
(d) ​​ ___
48 ​​ =   
​​   ​​ = ​​   ​​ 
(2)(2)(2)(2)(3) 2

An algebraic fraction, as with a fraction, is in its lowest terms if its


numerator and denominator do not have any common factor other than
1 or –1.
Canceling the common factors of the numerator and the denominator of
an algebraic fraction is equivalent to removing a factor that is equal to 1.
​   ​)(​​​​   __
​   ​ ​​  = ​​   ​​
b c) b
c ac a c __ a
Since __ ​​   ​​ = (​​  __
​​  c ​​= 1 when c ≠ 0, then ___
86 bc

Pre-Calculus and Calculus.indb 86 13/7/2017 5:56:59 PM


Chapter 3 • Algebraic Fractions

Summary

The following steps show the process of simplifying algebraic fractions.


Step 1: Factor both the numerator and denominator completely.
Step 2: 
Cancel the common factors of the numerator and the
denominator.

Example 2
Simplify the following algebraic fractions.
y2 – 1 10x – 20y
(a) ​​ _____
y + 1 ​​   (d) ​​ ________  ​​ 

x2 – 4y2
x3 – x2 –x – 3
(b) ​​ ______
x – 1 ​​   (e) ​​ ______
x + 3 ​​ 
x2 – 4 x3 – 8
(c) ​​ ________   ​​   (f) ​​ __________    ​​ 
x2 – x – 2 x2 + 3x – 10

SOLUTION
y2 – 1 ___________ (y – 1)(y + 1) Note
(a) ​​ _____
y+1  
 ​​ = ​​     y + 1 ​​  The numerator y2 – 1 is a
(y – 1)(y + 1)
___________ difference of two squares.
=    ​​  y + 1 ​​  Hence, y2 – 1 = (y – 1)(y + 1)
=y–1
x – x _______
3 2 x2(x – 1)
(b) ​​ ______
x–1  ​​ = ​​  x – 1 ​​   
x2(x – 1)
= _______ ​​  x – 1 ​​   

= x2
x2 – 4 (x – 2)(x + 2) Factor the difference of two squares.
(c) ​​ ________   ​​  ___________
​​ 
=     ​​
x –x–2
2
(x – 2)(x + 1) Factor a general trinomial where a =1.
(x
___________ – 2)(x + 2)
=    ​​   ​​
(x – 2)(x + 1)
x+2
= _____
​​  x + 1   ​​ The term x cannot be canceled.

10x – 20y _____________ 10(x – 2y)


(d) ​​ ________  ​​  = ​​    
     ​​ Factor the difference of two squares.
x2 – 4y2 (x – 2y)(x + 2y)
10
= ​​ ______ x + 2y      ​​
–x – 3 _______ –(x + 3)
(e) ​​ ______
x + 3 ​​  = ​​  x + 3 ​​   

= –1 Factor the difference of two


x –8
3 (x – 2)(x + 2x + 4)
2 cubes.
(f) ​​ __________    ​​  = ________________
   ​​      ​​
x + 3x – 10
2
(x – 2)(x + 5) Factor a general trinomial
(x – 2)(x2 + 2x + 4)
________________ where a =1.
=   
​​      ​​
(x – 2)(x + 5)
x2 + 2x + 4
= _________
​​  x + 5 ​​   

87

Pre-Calculus and Calculus.indb 87 13/7/2017 5:56:59 PM


Chapter 3 • Algebraic Fractions

Exercise 3.1
1. Simplify the following algebraic fractions.

x2 – 49y2
(a) ​​ ________ ​  2 x – 25  ​​ 
2
__________
x – 7y   ​​
  (f) ​
x + 9x + 20
x2 – 81y2 –x2 – 5x
(b) ​​ ________
x + 9y   ​​   (g) ​​ _______
x + 5 ​​ 

3x3 – 6x2 –h3 – 10h2


(c) ​​ ________
5x – 10 ​​   (h) ​​ _________ ​​ 

2h + 20
4x3 – 12x2
(d) ​​ _________
2x – 6 ​​  
  ​  m – 4n 
________
(i) ​  ​​ 
12n – 3m
r – 1  ​​  
2
(e) ​​ _________ ​  7b – 14a ​​ 
________ 
r2 + 4r – 5 (j) ​
8a – 4b

2. Simplify the following algebraic fractions.

x + xy x2 + x – 2
(a) ​​ ________
  ​​   (f) ​​ ________  ​​ 
ab + aby (x + 2)2
ax + ay a2 + 6a + 9
(b) ​​ _______
ax – az ​​   (g) ​​ ____________
     ​​
4a2 + 14a + 6
x2 – 3x 10(x – y)
(c) ​​ __________  ​​   (h) ​​ ________2  ​​ 
x + 4x – 21
2
30(x – y)
x2 (a + b)3
(d) ​​ ______   ​​   (i) ​​ ____________
     ​​
x – xy
2
(a + b)2(a – b)

9m + 12 x2 – (y + 2)2
(e) ​​ ________
3mn + 4n   
​​ (j) ​​ __________  ​​ 
xy – y2 – 2y

88

Pre-Calculus and Calculus.indb 88 13/7/2017 5:56:59 PM


Chapter 3 • Algebraic Fractions

3.2 Operations of Algebraic Fractions


3.2.1 Multiplication of Algebraic Fractions
3.2.2 Division of Algebraic Fractions
3.2.3 Addition and Subtraction of Algebraic Fractions with the Same Denominator
3.2.4 Addition and Subtraction of Algebraic Fractions with Different Denominators
3.2.5 Combination of Operations of Algebraic Fractions

3.2.1 Multiplication of Algebraic Fractions


Recall how the product of two fractions is derived. When multiplying __ ​​ 25 ​​ 
3
and __
​​  7 ​​ , we multiply the numerators together and then the denominators.

​  25 ​ )(​​​​    __
​  7 ​ )​​  = _____
(2)(3)
(
3
​​  __ ​​   ​​ 
(5)(7)
6
= ___
​​  35  ​​ 

The same method is used when multiplying algebraic fractions.

Definition 3.1

The product of two algebraic fractions is the product of their numerators


over the product of their denominators.
​   ​)(​​​​    __
​   ​ ​​  = ___
a c ac
( b d ) bd
​​  __ ​​   ​​  , where b ≠ 0 and c ≠ 0.

When multiplying algebraic fractions, always cancel the common factors


(if any).

Example 3
Multiply the following algebraic fractions and simplify the product.
​​​​    ​    ​ ​​   )(​​​​    _______  ​​  
33b )( 10a3 ) 10x2 )
(a) ​​ (____ (c) ​​ (_____
5a ____ 11b 5x 3​x​​  2​+ 9x
​   3 ​  ​  x +  3 ​  ​   ​ 

(b) ​​ (_____ ​ ​​​  ​    


)(  2 ​)​​   (d) ​​ (_______ ​​​​    _______ ​​  
x  + x + 1 x  + x – 2 )
)(
x – 5y ___________ ​x​​  2​– ​y​​  2​ ​x​​  3​– 1 7​x​​  3​
​  x – y ​  ​  2   ​  ​  2    ​ 
x – 4xy – 5y
2

SOLUTION
​​​​    ​    ​ ​​  = _______
33b )( 10a3 ) 330a3b3
(a) ​​ (____
5a ____ 11b 55ab
​   3 ​  ​​    ​​ 
55ab
= ​​ ____________
      ​​
(55ab)(6a2b2)
= ​​ _____ 1   ​​ 
6a2b2
Note
(b) ​​ (_____  
)( x2 – 4xy – 5y2 ) (x – y)(x2 – 4xy – 5y2) ​​  
x – 5y ___________ ​x​​  2​– ​y​​  2​ (x – 5y)(x2 – y2)
__________________
​  x – y ​  ​​
​  ​ ​        ​ ​​ =   
​​    
x +y +xy
(x – 5y)(x + y)(x – y)
_________________
= ​​     ​​
(x – y)(x – 5y)(x + y) x –5y –5xy
(x – 5y)(x + y)(x – y)
_________________ x2 –5y2 –4xy
= ​​     ​​
(x – 5y)(x + y)(x – y)
=1
89

Pre-Calculus and Calculus.indb 89 13/7/2017 5:56:59 PM


Chapter 3 • Algebraic Fractions

 (​​​​    _______  
10x ) (x + 3)(10x2)
(5x)(3x2 + 9x)
(c) ​​ (_____
​  x +  3 ​)
5x 3​x​​  2​+ 9x ____________
​  2 ​  ​​ =   
​​     ​​
(5x)(3x)(x + 3)
____________
= ​​        ​​
(10x2)(x + 3)
3(15)(x2)(x + 3)
____________
=    ​​   ​​
Note 2
(10)(x2)(x + 3)
3
= __ ​​  2 ​​ 
x –1 –x


x  + x + 1 )( x  + x – 2 ) [
(d) ​​ (_______   ][ (x + 2)(x – 1) ​]​​  
​x​​  3​– 1 7​x​​  3​
_______ (x – 1)(​x​​  2​ + x + 1) ___________
_______________ 7x​​  3​
x +2 +2x ​  2   ​ 
​​​​  ​     ​
  ​​ = ​​  ​ 
       ​  ​​​​  ​    
2
x +x+1
2

x2 –2 +x 7x3
= _____
​​  x + 2  ​​ 

3.2.2 Division of Algebraic Fractions


Dividing algebraic fractions is the same as dividing fractions. Recall how
the quotient of two fractions is derived.
2
The quotient of  ​​ __ 1
__ 2
__ 1
__
3 ​​  and ​​  5 ​​  is the product of  ​​ 3 ​​  and the reciprocal of  ​​ 5 ​​ .
2 ​​​​    __
​    ​)​​  
(​   ​ )(
5
3 1 ​​  2 ​​  ÷ ​​ __
__
3 5
1 ​​  = ​​  __

(2)(5)
= _____
​​   ​​ 
(3)(1)
10
= ___
​​  3 ​​ 
The same method is used when dividing algebraic fractions.

Definition 3.2

The quotient of two algebraic fractions is obtained by multiplying the


dividend by the reciprocal of the divisor.
​   ​ )(​​​​    __
​   ​  ​​  
b c)
a c a d
​​   ​​  ÷ ​​ __  ​​ = (​​  __
__
b d
(a)(d)
= _____
​​   ​​ 
(b)(c)
ad
= ___
 ​​   ​​  ,
bc
where b ≠ 0 and c ≠ 0.

90

Pre-Calculus and Calculus.indb 90 13/7/2017 5:56:59 PM


Chapter 3 • Algebraic Fractions

Example 4
Divide the following algebraic fractions and simplify the quotients.
4 ____ 16
(a) ​​ ___
6x   ​​ ÷ ​​  12x 3 ​​ 
15 5a
(b) ​​ _____    ​​ ÷ ​​ ____   
​​
10ab3 30b
x + 5 _________ x2 + 6x + 5
(c) ​​ _____
35  ​​   ÷ ​​ 
7x    ​​ 

SOLUTION

)​​  
4   ​ ​​​​    ____
 = ( ) (
4   ​​ ÷ ____16 12​x​​  3​
(a) ​​ ___
6x 12x ​​   3  ​​ ​​  ___
​  6x 16 ​  ​ 
48x3
= ​​ ____ 96x ​​ 
(48x)(x2)
= ​​ ________ ​​ 
(48x)(2)
x2
= __ ​​  2 ​​ 

   ​ ​​​​    ____
​   ​  ​​  
10ab3 30b ( 10ab3 )( 5a )
15 5a 15 30b
(b) ​​ _____ ÷ ​​ ____   
   ​​  ​​ = ​​  _____
​ 
450b
= ​​ ______   ​​ 
50a2b3
(50b)(9)
= ​​ __________   ​​ 
(50b)(a2b2)
9
= ​​ ____    ​​ 
a2b2
Note
​​​​    ​  2
)( ​​  
x + 6x + 5 )
= (​​  _____
x + 5 _________x2 + 6x + 5 x + 5 _________ 7x
(c) ​​ _____
35  ​​  ÷ ​​ 
7x    ​​  ​  35 ​  
     ​ 
x +5 +5x
(7)(x)(x + 5)
________________
=    ​​     ​​
(7)(5)(x + 5)(x + 1) x +1 +x
(x)
= ________
​​     ​​  x2 +5 +6x
(5)(x + 1)
x
= ______ ​​  5x + 5 ​​ 

91

Pre-Calculus and Calculus.indb 91 13/7/2017 5:56:59 PM


Chapter 3 • Algebraic Fractions

Example 5
Divide the following algebraic fractions and simplify the quotients.
x x3 + 8
(a) ​​ _____ _____
x – 2 ​​ ÷ ​​  x2 – 4 ​​ 
x2 + 5xy ____________ x2 – 25y2
(b) ​​ _______ 3 ​​  ÷ ​​       ​​
x + 8y
3
x – 2xy + 4y2
2

SOLUTION
x x3 + 8
(a) ​​ _____

x – 2 x – 4 ​​ 
 ​​ _____
 ÷ ​​  2

 ​ ​​  
Factor the difference of two squares.
)( x + 8)
​​​​    _____
x x​ ​​  2​– 4
= (​​  _____
​  x – 2 ​  ​  3
Factor the sum of two cubes.

)[​​​​        ​ ​​  
(x + 2)(x2 – 2x + 4) ]
x (x + 2)(x – 2)
= (​​  _____
​  x – 2 ​  ________________
​    
x
= _________
​​  2    ​​ 
x – 2x + 4

x2 – 25y2
x2 + 5xy ____________
(b) ​​ _______  
 ​​ ÷ ​​       ​​
x3 + 8y3 x2 – 2xy + 4y2
Note

  ​​  
x3 + 8y3 is a sum of two cubes.
=(
x + 8y3 )( x2 – 25y2 ) Factor the difference of two squares.
x​ ​​  2​+ 5xy ____________ x​ ​​  2​– 2xy + 4​y​​  2​
Hence, ​​  _______
​  3  
 ​ ​​​​    
​   ​  
x3 + y3 = (x + y)(x2 – xy + y2)

= [​​  ___________________ ​​​​    _____________


2  ​][    ​]​​  
x(x + 5y) x​ ​​  2​– 2xy + 4​y​​  2​
​   
   ​   
(x + 2y)(x – 2xy + 4y ) (x – 5y)(x + 5y)
2

x
= ​​ _____________
      ​​
(x + 2y)(x – 5y)
x
= ​​ _________________
      ​​
x2 – 5xy + 2xy – 10y2
x
= ____________
​​        ​​
x2 – 3xy – 10y2

3.2.3 Addition and Subtraction of Algebraic


Fractions with the Same Denominator
Fractions with the same denominators are called similar or like fractions.
To add similar fractions, add the numerators and write the sum over the
common denominator.

​​  71 ​​  and ​​ __


Consider adding __ 2 ​​ . Add the numerators and copy the denominator.
7

​​  1 ​​  + __
__
7 7 ​​  2 ​​  = _____
7​​  1 + ​​
2  

3
= __
​​   ​​ 
7
Algebraic fractions with the same denominators are added the same way.

92

Pre-Calculus and Calculus.indb 92 13/7/2017 5:56:59 PM


Chapter 3 • Algebraic Fractions

Definition 3.3

Algebraic fractions with the same denominators are added (or


subtracted) by adding (or subtracting) the numerators and writing the
sum or the difference over the common denominator.
__ a b _____ a+b a b _____ a–b
​​  c ​​ + __ ​​ and __
​​  c ​​ = ​​  c   
  ​​  c ​​ – __
​​  c ​​ = ​​  c   
 ​​, where c ≠ 0.

Example 6
Add or subtract the following algebraic fractions and simplify the results.
3y + 8 ___ 2y
(a) ​​ ______
7 ​​   + ​​   ​​ 
7
5b 2
4b2 – 7
(b) ​​ _____  ​​ – ______
​​   ​​ 
b–8 b–8
2x2 – 4y2 ____ 2x2
(c) ​​ ________
x – y    ​​ + ​​  x – y   
  ​​
7m2­ + 3 ___________
3m + 7 – 4m2 _______
3m2 – 5
(d) ​​ _______
m+5  ​​ +
  ​​     m+5  ​​ –
  ​​  m + 5 ​​ 
2a2 + 7a _____
3a2 + 5a – 6 _______ a
(e) ​​ __________
a + 2 ​​ – ​​  a + 2 ​​ 
  + ​​  a + 2 ​​ 

SOLUTION
3y + 8 ___ 3y + 8 + 2y
2y __________
(a) ​​ ______
7  ​​  + ​​   ​​ = ​​ 
7 7 ​​   

5y + 8
= ______
​​  7 ​​   

5b2 4b2 – 7 ____________ 5b2 – (4b2 – 7)


(b) ​​ _____  ​​ – ______
​​   ​​ 
= ​​     ​​ 
b–8 b–8 b–8
5b2 – 4b2 + 7
= ___________
​​   ​​   
b–8
b2 + 7
= _____ ​​   ​​ 
b–8
2x2 – 4y2 ____ 2x2 ____________ 2x2 – 4y2 + 2x2
(c) ​​ ________
x–y    
​​
  + ​​  x–y   
​​
  =    ​​  x – y  ​​

4x – 4y
2 2
= ________ ​​  x – y    ​​

4(x – y)(x + y)
____________
=    ​​  x – y  ​​ 
= 4(x + y)
= 4x + 4y

93

Pre-Calculus and Calculus.indb 93 13/7/2017 5:56:59 PM


Chapter 3 • Algebraic Fractions

7m2­ + 3 ___________
3m + 7 – 4m2 _______
3m2 – 5
(d) ​​ _______
m+5  ​​ +
  ​​     m+5  ​​ –
  ​​  m + 5 ​​ 
7m2 + 3 + 3m + 7 – 4m2 – 3m2 + 5
____________________________
=    
​​  m + 5 ​​
  

3m + 15
= _______
​​  m + 5 ​​ 

3(m + 5)
= ​​ ________
m + 5 ​​ 

=3
2a2 + 7a _____
3a2 + 5a – 6 _______ a 3a2
+ 5a – 6 – 2a2 – 7a + a
(e) ​​ __________
a+2  ​​  

  ​​  a+2  ​​ +
  ​​   
a+2  ​​ = ​​  ______________________
   a  
+ 2 ​​
a2 – a – 6
= ________
​​  a + 2 ​​   

(a – 3)(a + 2)
= ___________
  
​​  a + 2 ​​ 
=a–3

3.2.4 Addition and Subtraction of Algebraic


Fractions with Different Denominators
Recall that in order to add fractions with different denominators, we need
Note to write all the given fractions in the lowest common denominator (LCD).
The LCD is the lowest This is done by finding the LCD of the fractions and finding the equivalent
common multiple (LCM) fractions with the LCD as the denominator.
of the denominators.
​​  18 ​​  and __
Consider adding __ ​​  16 ​​ . The LCD of the two fractions is 24. Now, find
the equivalent of each fraction using the LCD as the denominator. The
fractions become
​  18 ​ )( ​​​​  __
​​  18 ​​  = (​​  __ ​  3 ​ ) ​​  and ​​ __
6 (​  6 ​ )( ​​​​  ​  4 ​ ) ​​
__ 3 1 ​​  = ​​  __
1 __ 4

3 4  ​​ 
= ___
​​  24  ​​   = ___
​​  24

Now that the two fractions have the same denominators, add the numerators
and write the sum over the common denominator.
3
​​  18 ​​  + __
__
​​  16 ​​  = ___​​  24  ​​ + ___ 4  ​​ 
​​  24
3+4
= _____
​​  24 ​​   
7
= ___
​​  24  ​​ 

94

Pre-Calculus and Calculus.indb 94 13/7/2017 5:56:59 PM


Chapter 3 • Algebraic Fractions

The same method is used for adding or subtracting algebraic fractions with
different denominators.

Definition 3.4

Algebraic fractions with different denominators can be added or


subtracted using the following properties:
a c ___ ad bc _______ ad + bc
(i) ​​ __ ​​ + __ ​​   ​​ = ​​    ​​ + ___
​​   ​​ = ​​      ​​,

b d bd bd bd
a c ___ ad bc _______ ad – bc
(ii) ​​ __ ​​ – __
​​   ​​ = ​​    ​​ – ___
​​   ​​ = ​​     ​​,

b d bd bd bd
where b ≠ 0 and d ≠ 0.

The expression bd is the lowest common denominator (LCD). It is the


product of the distinct factors of the two denominators. It becomes the
common denominator of the equivalent algebraic fractions. Only then, the
algebraic fractions can be added or subtracted.

Example 7
Find the LCD of the following pairs of algebraic fractions.
2a 3a 2 , _______ 10 3
(a) ​​ ___ ___
21 ​​ and ​​  45 ​​   (c) ​​ _____
x – 5 ​​  ​​  and ​​ __2  ​​ 
   ​​ 
x(x – 5) x
5 2 ​​ 5
(b) ​​ ______ and ​​ ____
   ​​  y – x   (d) ​​ _________    ​​   ​​  2   ​​ and ​​ _____
, _____ 1   ​​ 
y2 – x2 x2 + 2x + 1 x + 1 x2 – 1

SOLUTION
(a) Factors of 21: 3 and 7
Factors of 45: 3, 3 and 5
LCD: (3)(3)(5)(7) = 315
(b) Factors of y2 – x2: (y + x) and (y – x)
Factor of y – x: y – x
LCD: (y + x)(y – x) = y2 – x2
(c) Factor of x – 5: x – 5
Factors of x(x – 5): x and x – 5
Factors of x2: x and x
LCD: (x)(x)(x – 5) = x2(x – 5)
(d) Factors of x2 + 2x + 1: (x + 1) and (x + 1)
Factor of x + 1: x + 1
Factors of x2 – 1: (x + 1) and (x – 1)
LCD: (x + 1)(x + 1)(x – 1) = (x + 1)2(x – 1)

95

Pre-Calculus and Calculus.indb 95 13/7/2017 5:56:59 PM


Chapter 3 • Algebraic Fractions

Example 8
Find the sum or difference and simplify the result.
2a ___ a x + 3 _____ x–3
(a) ​​ ___
21 ​​ – ​​  45  ​​   (c) ​​ _____
x – 4 ​​ + ​​  x + 3   ​​
5 10 3
(b) ​​ ______   ​​  + ____ ​​  2    
​​ 2
(d) ​​ _____ _______
– __
x – 5 ​​ – ​​  x(x –  5) ​  ​  2  ​​ 
y2 – x2 y – x x

SOLUTION

21 ​​ – ​​  45  ​​ = (​​  ​  21 ​ )(​​​​    ​  15 ​ )​​  – (​​  ​  45  ​ )(​​​​    ​  7 ​ )​​  


2a ___ a 2a ___ 15 a __ 7
(a) ​​ ___ ___ ___

30a 7a
= ____
​​  315 ​​ – ____ ​​  315  ​​ 
30a – 7a
= ​​ ________ 315 ​​   

23a
= ​​ ____315 ​​ 
​ ​​​​    _____  ​ ​​  
y2 – x2 y – x y2 – x2 ( y – x )( y + x )
5 5 y+x
(b) ​​ ______   ​​  + ____​​  2     ​​ = ______ ​​     ​​  + ​​  ____​  2     ​   
5 2y + 2x
= ​​ ______ 2   2 ​​  + ___________
​​    ​​ 
y – x (y – x)(y + x)
5 + 2y + 2x
= ​​ __________  ​​   
y2 – x2
 ​​ = (​​  _____ )( ​​​​    ​  x + 3   ​)​​  + (​​  _____  ​)(​​​​    ​  x – 4   ​)​​  
x + 3 _____ x–3 x + 3 _____ x+3 x – 3 _____ x–4
(c) ​​ _____
x – 4 ​​ + ​​  x + 3  ​  x – 4 ​  ​  x + 3 
(x + 3)2 (x – 3)(x – 4)
= ___________
​​       ​​ + ___________   
​​   ​​
(x – 4)(x + 3) (x – 4)(x + 3)
(x2 + 6x + 9) + (x2 – 7x + 12)
= ________________________
   ​​      ​​
(x – 4)(x + 3)
2x2 – x + 21
= ​​ ___________
     ​​
(x – 4)(x + 3)

x – 5 )( x2 ​ )​​  – (​​  ​  x(x –  5) ​  )(​​​​   ​  x )​ ​​  – (​​  ​  x2  ​ )(​​​​    ​  x – 5 


2   ​​ – _______
(d) ​​ _____ ​​ 
10
   ​  – __
​ 
3
 ​​
   = ( ​​  _____
​  2   ​ ​​​​    __ ​ 
x​ ​​  2​ _______ 10 __x __3 _____ x–5
 ​)​​  
x – 5 x(x – 5) x 2

​​  – ​​  __________ ​​  – ​​  _______ ​​  


x (x – 5) ] [ (x)(x)(x – 5) ] [ x2(x – 5) ]
= [​​  _______
2​x​​  2​ 10x 3(x – 5)
​  2    ​  ​     ​  ​   ​ 

= [​​  _______ ​​  – ​​  _______ ​​  – ​​  _______ ​​  


x (x – 5) ] [ x2(x – 5) ] [ x2(x – 5) ]
2​x​​  2​ 10x 3x – 15
​  2    ​  ​     ​  ​    ​ 

Note 2x2 – 10x – 3x + 15


________________
=    ​​      ​​
x2(x – 5)
2x2 – 13x + 15
x –5 –10x = ​​ ____________
    ​​ 
x2(x – 5)
2x –3 –3x (x – 5)(2x – 3)
____________
= ​​     ​​ 
x2(x – 5)
2x 2
+15 –13x
2x – 3
= ______
​​  2 ​​   
x

96

Pre-Calculus and Calculus.indb 96 13/7/2017 5:57:00 PM


Chapter 3 • Algebraic Fractions

3.2.5 Combination of Operations of Algebraic


Fractions

Example 9
Simplify the following algebraic fractions.
(a) ​​(____ y) (​  ​x​​  2​– ​y​​  2 ​​  ​x​​  ​+ ​y​​  ​) y2
x
​  x 1– y   ​ – _____ 1    
​  x + ​ ​​ ÷ ​​ ______ 1 – ______ ​  2 1  2 ​ ​​ + __ ​​    ​​ 

(b) ​​(__ ​  13  ​ )​​ ÷ (


​x​​  ​ ​x​​  4​) x + 1
x
​  1x ​ + __ ​  12  ​ + __
​​ __ ​  1  ​  ​​ + _____
​​     ​​ 
​x​​  ​

SOLUTION
(a) ​​(____ y) (​  ​x​​  2​– ​y​​  2 ​​  ​x​​  ​+ ​y​​  ​) y2
x
​  x 1– y   ​ – _____ 1    
​  x + ​ ​​ ÷ ​​ ______ 1 – ______ ​  2 1  2 ​ ​​ + __ ​​    ​​ 
x + y – (x – y) ______________ x2 + y2 – (x2 – y2) __ x
= ​​ ____________
      ​​ ÷ ​​       ​​ + ​​    ​​ 
(x – y)(x + y) (x2 – y2)(x2 + y2) y2
2y 2y2 x
= ______
​​  2  2 ​​  ÷ ​​ _____________
      ​​ + __ ​​    ​​ 
x – y (x2 – y2)(x2 + y2) y2
2y (x2 – y2)(x2 + y2) __ x
= ______ ​​  2  2 ​​  × _____________
  
​​   ​​  + ​​  2  ​​ 
x –y 2y2 y
x2 + y2 __ x
= ______ ​​  y    ​​ + ​​  2  ​​ 

y
x2y + y3 + x
= ​​ __________  ​​   
y2
(b) ​​(__ ​  13  ​ )​​ ÷ (
​x​​  ​ ​x​​  4​) x + 1
x
​  1x ​ + __ ​  12  ​ + __
​​ __ ​  1  ​  ​​ + _____
​​     ​​ 
​x​​  ​
x2 + 1 _____ x2 + 1 _____ x
= _____ ​​  3 ​​   ÷ ​​  4 ​​   + ​​     ​​ 
x+1
x x
x2 + 1 _____ x4 x
= _____ ​​  3 ​​   × ​​  2    ​​  + _____
​​     ​​ 
x x +1 x+1
x
= x + _____ ​​  x + 1 ​​ 
x2 + x + x
= ________ ​​  x + 1 ​​   

x2 + 2x
= ______​​  x + 1 ​​ 
x(x + 2)
= _______ ​​  x + 1 ​​   

97

Pre-Calculus and Calculus.indb 97 13/7/2017 5:57:00 PM


Chapter 3 • Algebraic Fractions

Exercise 3.2

1. Perform the indicated operations and simplify the result.


​  5x ​ )(​​​​    ____  )​​   (d) ​​ (_____  ​ ​​​​    ​  ​​  
b + 2 )( 3b – 6 )
(a) ​​ (___
3​x​​  3​ 20​x​​  5​ ​b​​  2​– 4 __________ ​b​​  2​+ 3b + 2
​  6 ​
  ​   ​  

​  5 ​ )(​​​​    _____
(b) ​​ (___ ​​   (e) ​​ (______ ​​​​    ​  2 ​​  
35xy ) m –1 m –m+1 )
)(
7xy 9 ​m​​  3​+ 1 __________ ​m​​  2​– 2m + 1
​    2 ​  ​  2  ​   ​ 
3y

(c) ​​ (_____  ​ ​​​​    ​ 


x – 9 )( x + 1 )
 ​​   (f) ​​ (______  
)[ x2 + 2x + 4  ]​​  
x + 1 _______ 4x + 12 ​x​​  3​– 8 _________ 8(x – 2​)​​  3​
​  2    ​
  ​  2x – 4   ​ ​​​​  ​   ​ 

2. Perform the indicated operations and simplify the result.


3x – 15y x2 – 2xy – 3y2 x3 – 27y3 ____________
x2 + 3xy + 9y2
(a) ​​ ____________
      ​​ ÷ ​​  ___________
    ​​   (c) ​​ ________
 ​​ ÷ ​​        ​​
x2 – 2xy – 15y2 x2 – 9y2 x2 – 9y2 x2 + 3xy
y2 – y + 2 _______ 6y + 6 x2 – 2x x2 – 2x – 8
(b) ​​ ________  ​​ 
÷ ​​    ​​   (d) ​​ _________ ÷ ​​ _________
  ​​   ​​ 

y2 + y – 2 6y + 12 x2 – 2x + 4 x3 + 8

3. Perform the indicated operations and simplify the result.


x + 5 ______ 3x – 8 6 x–4
(a) ​​ _____
7 ​​ 
 + ​​ 
7 ​​  
  (f) ​​ _____ _____
x – 2 ​​ + ​​  x   
​​ 

4y – 3 ______
2y – 6 5x – 2 _____ x–1
______
​ 
(b) ​  ​​ 
  ​   ​​   (g) ​​ ______  
 ​​ 
+ ​​    ​​
3 –​ 3

x2 – 16 x + 4
5x 2x + 6
(c) ​​ _____ ______
x –  2 ​​ – ​​  x – 2 ​​   (h) ​ ​  b2 + 9 
_____ ​ ​ 3   ​​ 
– _____
 ​​ 
b –9 b+3
m + 11 ______ 3m + 9
(d) ​​ ______
m + 5 ​​ 
+ ​​  m + 5 ​​   (i) ​ ​  2 5  2 ​​ 
__________ ​ ​   3 
+ _____________ ​​
x – xy – 6y x2 + 7xy + 10y2
3 5y
​  x –x 1 
_____
(e) ​ ​ ​x +2 ​​
​​ + _____ 3   (j) ​​ ______
5y + 2 ​​  – _______
​​  2    ​​ 
x 25y – 4

4. Perform the indicated operations and simplify the result.


1
(a) ​​ _____ _____ 2 _____ 1
x + 1 ​​ – ​​  x – 3 ​​ + ​​  x – 2 ​​ 
4x
(b) ​​ _____ ​​  2   ​​ + _____
– _____
   ​​  ​​  2   ​​ 
x2 – 1 x + 1 x – 1

(c) ​​ [_____________ ​​​​    ​  ​​  ÷ ______


3x2 – 27 ]( x2 + 2x + 4 ) x3 – 8
12(​x​​  2​+ 5x + 6) _________ 5x – 15 4x – 8
  
​   ​     ​  ​​   ​​ 

(d) ​​ (    
)( x + 2xy – 3y )  
​x​​  2​– 6xy + 9​y​​  2​ ____________
___________ ​x​​  3​+ 27​y​​  3​ x2 – 3xy + 9y2
____________
​  2 ​  ​​​​  ​        ​ ​​ ÷   
​​      ​​
x – 9y
2 2 2
x2 + 3xy

)(​​​​    __________ ​​  
2x + 5x + 2 )
(e) ​​ (_____
5 3 ​x​​  2​ + x – 2
​  x – 2 ​ – _____
​  x – 1 ​  ​  2   ​ 

)​​  ÷ ( ​​  
x + 6x + 5 )
(f) ​​ (_____
3 ​x​​  2​– 25
​  x –2 1 ​ 
​  x + 1 ​ – _____ ​​  _________
​  2   ​ 

(g) ​​(_____ )​​ ÷ ​​  x2 – 4 ​​ 


3
4   ​ – _____ _____ 5 x+4
​  x + 2 ​  x – 2 ​  + _____
​​  2 ​​ 

x

98

Pre-Calculus and Calculus.indb 98 13/7/2017 5:57:00 PM


Chapter 3 • Algebraic Fractions

3.3 Rationalization of Algebraic Fractions


In Chapter 1, Section 1.4, we learnt about rationalization of surds. When
a quotient involves a surd in the denominator, we usually rationalize
the quotient to get rid of the radical sign. The same concept is used in
rationalization of algebraic fractions.
Let us look at some examples.

Example 10
Rationalize the following expressions.
__2
(a) ​​ ______ __ 1 __ ​​ 
(b) ​​ ________
​  x  ​+  3 ​​  
√ 2​√ x  ​+ √
​  y  ​
SOLUTION
__
__2   ​​  __2   ​​   x  ​– 3
​ __
(a) ​​ ______ ______ ______

=
​  x  ​+ 3 √​​  
​  x  ​+ 3 √ ​​  ​  x  ​– 3  ​​

__
2​√ x  ​– 6
= _______
​​  x – 9 ​​   
__ __
2​√ x  ​– √
​  y  ​
__ 1  __ ​​ 
(b) ​​ ________ ​​  __ 1  __ ​​ 
= ________  ________
​​  __ __   ​​
2​√ x  ​+ √
​  y  ​ 2​√ __ x  ​+ √​ __ y  ​ 2​√ x  ​– √
​  y  ​
2​
√ x 
  ​
  – ​
√ y 
  ​

= ________
​​  4x – y    ​​

Exercise 3.3
1. Rationalize the following expressions.
x
(a) ​​ _________
_____   ​​  
​  x + 1 ​ – x

x +_____
4
(b) ​​ _________
   ​​ 
5–√​  4 + x ​ 
__x
2
(c) ​​ ______
​  x ​ + 1 ​​ 

​  2x
__________
______
   ​​ 
(d) ​
3+√
​  2x + 1 ​ 

  _____ 3x 
(e) ​​ _____________ _____ ​​
​  x + 2 ​ + √
√ ​  x + 1 ​ 

99

Pre-Calculus and Calculus.indb 99 13/7/2017 5:57:00 PM


Equations and Inequalities
Chapter 4:

Equations in One Variable

Natural Habitat
Loss of habitat is one of the greatest threats to wildlife. Human activities such as dumping
toxic substances, polluting lakes, rivers, and seas, affected forests and reefs.
Global imaging of Earth’s satellites show that about 6 million hectares of tropical forests are
lost each year and about 3 million hectares of forests are visibly degraded. These tropical
forests contain over 50% of our plant and animal species.
Thus, the loss of tropical forest habitat has a proportionally big effect on the loss of plants
and animals. People must realize that these trees, plants, and animals play roles in Earth’s
ecosystems, and hence, must be protected.
Relationships between habitat and species can be modeled using equations. In this chapter,
you will learn more about equations which are important tools of mathematics.

Pre-Calculus and Calculus.indb 100 13/7/2017 5:57:00 PM


Chapter 4 • Equations in One Variable

4 E
 quations in One Variable
4.1 Linear Equations
4.2 Quadratic Equations
4.3 Equations with Degree  3
4.4 Radical Equations
4.5 Exponential Equations
4.6 Logarithmic Equations
4.7 Applications of Equations

4.1 Linear Equations


A linear equation in one variable has exactly one solution.
A linear equation is also called a first degree equation since its degree is
1, that is, the highest power of its variable is 1.

Definition 4.1

A linear equation in one variable is an equation that can be written


in the form
ax + b = 0
where a and b are real numbers and a ≠ 0.

To solve an equation means to find its solution. A better method to solve an


equation is to change the given equation to a simpler equivalent equation
using properties of equality. Equations having the same solutions are
called equivalent equations.
The following equations are equivalent since they have the same solution.
7x + 3 = –18        14x + 16 = – 26
7x = –18 – 3 14x = –26 – 16
7x = –21 14x = –42
x = –3 x = –3
You can use the following properties of equality to solve equations.

Theorem 4.1

The addition property of equality (APE) states that adding the same
number to both sides of an equation results in an equivalent equation.
For any real numbers a, b and c,
if a = b, then a + c = b + c

101

Pre-Calculus and Calculus.indb 101 13/7/2017 5:57:00 PM


Chapter 4 • Equations in One Variable

Example 1
Use the addition property of equality to solve the following equations.
(a) x – 5 = 7
(b) 5x = 4x + 10

SOLUTION
(a) x–5=7
x–5+5=7+5 Add 5 to both sides (APE)

Note x = 12
(b) 5x = 4x + 10
Adding –4x to both sides is
the same as subtracting 4x
5x + (–4x) = 4x + (–4x) + 10 Add (–4x) to both sides (APE)
from both sides (APE). x = 10

Note Theorem 4.2


An equation is solved The multiplication property of equality (MPE) states that multiplying
when you get an equivalent both sides of an equation by the same non-zero number results in an
equation of the form x = a.
equivalent equation.
For any numbers a, b and c, where c ≠ 0,
if a = b, then ac = bc

Example 2
Use the multiplication property of equality to solve the following
equations.
(a) 2x = 10
1  ​​x = –6
(b) ​​ __
3
SOLUTION
(a) 2x = 10
​  21 ​ )​​( 2x) = (​​  __
​​ (__ ​  21 ​ )​​( 10) ​​ 21 ​​  (MPE)
Multiply both sides by __

x=5
1  ​​x = –6
(b) ​​ __
3
​  31 ​ x)​​  = (3)(–6)
(3)​​ (__ Multiply both sides by 3 (MPE)

x = –18

102

Pre-Calculus and Calculus.indb 102 13/7/2017 5:57:00 PM


Chapter 4 • Equations in One Variable

Below are other properties of equality which are useful in solving


equations.

For any real numbers a, b and c, the following properties hold.


(i) Reflexive Property
a=a
(ii) Symmetric Property
If a = b, then b = a.
(iii) Transitive Property
If a = b and b = c, then a = c.
(iv) Substitution Property
If a = b, a may be replaced by b, or b may be replaced by a, in any
equation or expression.

Example 3
Solve 5(x – 3) = 2x – 9.

SOLUTION
5(x – 3) = 2x – 9
5x – 15 = 2x – 9 Distributive property
5x – 15 – 2x = 2x – 9 – 2x APE
3x – 15 = –9
3x – 15 + 15 = –9 + 15 APE
3x = 6
​  31 ​ )​​( 3x) = (​​  __
​​ (__ ​  31 ​ )​​( 6) MPE

x=2
The equation 5(x – 3) = 2x – 9 is called a conditional equation since it
has only one solution.

Example 4
Solve –5(x – 1) = 3 – 5x.

SOLUTION
–5(x – 1) = 3 – 5x
–5x + 5 = 3 – 5x Distributive property
–5x + 5x + 5 = 3 – 5x + 5x APE
5=3
Note that –5(x – 1) = 3 – 5x becomes 5 = 3, which is always false for
all values of x.
Hence, the equation –5(x – 1) = 3 – 5x has no solution and is called an
inconsistent equation.

103

Pre-Calculus and Calculus.indb 103 13/7/2017 5:57:00 PM


Chapter 4 • Equations in One Variable

Summary

The following steps show the process of solving linear equations in one
variable.
Step 1: Simplify both sides of the equation as needed.
(i) Use the distributive property to remove any grouping
symbols.
(ii) Clear the equation of fractions by multiplying both sides by
the lowest common denominator (LCD).
(iii) Clear the equation of decimals by multiplying both sides by
a power of 10 determined by the decimal number with the
most places.
(iv) Combine like terms.
Step 2: A
 pply the addition property of equality (APE) so that all
variable terms are on one side and all constants are on the
other side, then simplify.
Step 3: A
 pply the multiplication property of equality (MPE) to make
the coefficient of the remaining variable 1.
Step 4: Check the solution in the original equation.

Example 5
Find the solution of 4 – 3(y – 1) – 4(y + 2) = 5(7 – 2y).

SOLUTION
4 – 3(y – 1) – 4(y + 2) = 5(7 – 2y)
4 – 3y + 3 – 4y – 8 = 35 – 10y Distributive property
–7y – 1 = 35 – 10y
–7y – 1 + 10y = 35 – 10y + 10y APE
3y – 1 = 35
3y – 1 + 1 = 35 + 1 APE
3y = 36
​​ (__ ​  31 ​ )​​( 3y) = (​​  __
​  31 ​ )​​( 36) MPE

y = 12

Observe that adding 10y on both sides of –7y – 1 = 35 – 10y results in


–7y – 1 + 10y = 35 – 10y + 10y
3y – 1 = 35
This is the same as changing the sign of –10y resulting to 10y, then moving
the term 10y to the other side of the equation.
–7y – 1 = 35 – 10y
–7y – 1 + 10y = 35
3y – 1 = 35
Moving the negative or additive inverse of a term to the other side is a
shortened procedure of adding the same term to both sides of an equation.
This shortened process for APE is popularly known as transposition of
104 terms.

Pre-Calculus and Calculus.indb 104 13/7/2017 5:57:01 PM


Chapter 4 • Equations in One Variable

For example, to solve 3y – 1 = 35:


Transpose –1 to the other side of the equation.
Note
3y – 1 = 35 To transpose a term is
to move the negative (or
3y = 35 + 1
additive inverse) of the
3y = 36 term to the other side of the
1 ​​  is the same as dividing
Observe further that multiplying both sides by ​​ __ equation.
3
both sides by 3 (MPE).
3y
___ 36 Note
​​  3 ​​ = ___
​​  3 ​​ 
By MPE, multiplying both
y = 12 ​​ 1a ​​  is
sides of an equation by __
Example 6 the same as dividing both
sides by a, for any non-zero
3 5
​​  32 ​​ (x – 1) = __
Solve __ ​​ 4  ​​x + ​​ __
6  ​​. real number a.

SOLUTION
2  ​​(x – 1) = __ 3 5
​​ __ 3 ​​ 4  ​​x + ​​ __ 6  ​​
​  32 ​ (x – 1)]​​  = 12​​ (__
12​​ [__ ​ 6  ​)​​  
3 5
​  4 ​ x + __ MPE

8(x – 1) = 12​​ (__ ​  4 ​ x)​​  + 12​​ (__ ​  6 ​ )​​  


3 5 Distributive property

8x – 8 = 9x + 10
8x – 8 – 9x = 10 Transpose 9x
–x – 8 = 10
–x = 10 + 8 Transpose 8
– x = 18
(–1)(–x) = (–1)(18) MPE
x = –18

To solve an equation with fractions, clear the equation of fractions by


multiplying both sides by the LCD.

Example 7
15 5
Solve ______ – 2 = ______
​​  3x – 1 ​​  ​​  3x – 1 ​​ 
.

SOLUTION
15 5
​​ ______ – 2 = ______
3x – 1 ​​  ​​  3x – 1 ​​ 

– 2)​​  = (3x – 1)​​ (______ )​​  


Multiply both sides by
(3x – 1)​​ (______
15 5
​  3x – 1 ​  ​  3x – 1 ​  the LCD, 3x – 1

15 – 2(3x – 1) = 5
15 – 6x + 2 = 5 Distributive property
17 – 6x = 5
–6x = 5 – 17 Transpose 17
–6x = –12
–6x ____ –12
​​ ____ –6 ​​ = ​​  –6 ​​ 
MPE

x=2
105

Pre-Calculus and Calculus.indb 105 13/7/2017 5:57:01 PM


Chapter 4 • Equations in One Variable

To solve an equation with decimals, clear the equation of the decimals


by multiplying both sides by a power of 10 equal to the most number of
significant decimal places.

Example 8

Solve 0.3t + 1.16 = 0.2t + 5.

SOLUTION
0.3t + 1.16 = 0.2t + 5
(100)(0.3t + 1.16) = (100)(0.2t + 5) MPE
30t + 116 = 20t + 500 Distributive property
30t + 116 – 20t = 500 Transpose 20t
10t + 116 = 500
10t = 500 – 116 Transpose 116
10t = 384
384
10t ____
___
​​  10 ​​ = ​​  10 ​​  MPE

t = 38.4

Example 9

Solve 0.5x + 0.7 = 0.2(x – 1).

SOLUTION
0.5x + 0.7 = 0.2(x – 1)
10(0.5 + 0.7) = 10[0.2(x – 1)] MPE
5x + 7 = 2(x – 1)
5x + 7 = 2x – 2 Distributive property
5x + 7 – 2x = –2 Transpose 2x
3x + 7 = –2
3x = –2 – 7 Transpose 7
3x = –9
3x ___
___ –9
​​  3 ​​ = ​​  3 ​​  MPE

x = –3

106

Pre-Calculus and Calculus.indb 106 13/7/2017 5:57:01 PM


Chapter 4 • Equations in One Variable

Exercise 4.1
1. Solve the following equations.
(a) 4x = 9x – 25 (f) –5(–4 + 3x) = 2(2x – 9)
(b) –3x + 15 = –6x (g) 4x – 3(x + 5) = 2x + 5(–6 – x)
(c) 7x – 13 = 10x + 14 (h) 7(–4) – 3(2x + 1) = 13
(d) –12x + 8 = 3x – 22 (i) 3(y – 7) + 2(4 – 2y) = 40
(e) 6(x + 2) = 2x – 8 (j) 20 – 2(z – 5) = 5(10 – 3z)

2. Solve the following equations.


x x 5x – 4
(a) ​​ __ __
3  ​​ – 6 = ​​  2  ​​ + 4 (f) ​​ ______
3 ​​  = –2x – 9

7x –5x 2x – 11 _______
–3x + 4
(b) ​​ __ ___
4 ​​ + 2 = ​​  3 ​​  – 4 (g) ​​ ______
5 ​​ 
 = ​​ 
2 ​​  

4x __ 1 __ x __ 3 7x + 2 _______–5x + 3
(c) ​​ ___
3 ​​ + ​​  2 ​​  = ​​  3  ​​ – ​​  4  ​​ (h) ​​ ______
4 ​​  = ​​ 
–6 ​​  

x 4x
(d) –​​ __ ___
5  ​​ + 3 = ​​  15  ​​ + 1 (i) 1.7y = 5.4 + 1.43y
–3x + 2
(e) ​​ _______ 4 ​​   = 2x + 7 (j) 0.25x + 1 = 0.05x – 2.2

3. Find the solution of each equation.


(a) 0.3(a – 15) = 4 – 0.2(5 – 0.7a)
(b) 0.125(8y – 2) + 0.05 = 2 – 0.24(3 – 5y)
4x – 5 x+3
(c) ​​ ______
2 ​​   – 3 = ​​ 
_____
4 ​​  

3x + 7 2x – 9
(d) ​​ ______
–2 ​​   = 5 – ​​ 
______
6 ​​   

3x – 1 x
(e) ​​ ______
4 ​​   + 2x – 1 = ​​    ​​
__
5
2x + 9 3x + 4
(f) ​​ ______ 7 ​​   – x – 2 = ​​ 
______
14 ​​ 

3 ___ 4 ___ 6
(g) ​​ ___
2x   ​​ – ​​  5x   ​​ = ​​  20  ​​ 
–9 2
(h) ​​ ___ ___
4x  ​​ – 5 = ​​  3x  ​​ 
5 1
(i) ​​ _____
x – 3 ​​ + 2 = ​​  x – 3 ​​ 
_____

7
(j) ​​ ______
2x + 1 ​​  – 3 = ______ ​​  2x 2+ 1 ​​ 

107

Pre-Calculus and Calculus.indb 107 13/7/2017 5:57:01 PM


Chapter 4 • Equations in One Variable

4.2 Quadratic Equations


4.2.1 Solving Quadratic Equations by Factoring
4.2.2 Solving Quadratic Equations by the Quadratic Formula
4.2.3 Imaginary Roots
Note
Definition 4.2
The term quadratic is
derived from the Latin word A quadratic equation in one variable is a second-degree equation
quadratus, which means
written in the standard form ax2 + bx + c = 0, where a, b and c are real
‘squared’.
numbers and a ≠ 0.

Examples of quadratic equations in one variable are as follows:

This signifies that the equation is quadratic.

3x2 + 2x – 5 = 0

The quadratic equation has


only one variable, x.

This signifies that the equation is quadratic.

5m2 – 2m + 10 = 0

The quadratic equation has


only one variable, m.

When a quadratic equation in one variable is written in standard form, you


can easily identify the numerical coefficients of each term. For example,
the equation, x2 + 5x – 6 = 0 is already in standard form. The numerical
coefficients of each term of the equation are indicated below.
x2 + 5x – 6 = 0

The constant term is –6.

The numerical coefficient of 5x is 5.

The numerical coefficient of x2 is 1.

108

Pre-Calculus and Calculus.indb 108 13/7/2017 5:57:01 PM


Chapter 4 • Equations in One Variable

4.2.1 Solving Quadratic Equations by Factoring


The following example recaps some factoring techniques that we have
learnt in Chapter 2, Section 2.2. These are helpful in solving quadratic
equations.

Example 10
Factor each polynomial.
(a) 2x2 + 4x (d) 6x2 – 15x – 9
(b) x2 + 6x + 9 (e) 3x2 – 108
(c) x2 – 4x – 21

SOLUTION
(a) Factor the binomial by common monomial factoring.
2x2 + 4x = 2x(x) + 2x(2)
= 2x(x + 2) Note
(b) The quadratic trinomial x + 6x + 9 is a perfect square.
2
Recall that
x2 + 6x + 9 = x2 + 2(x)(3) + 32 (x ± y)2 = x2 ± 2xy + y2.
= (x + 3)2
(c) Factor the polynomial using the cross method.
x –7 –7x

x +3 +3x
x2 –21 –4x
Hence, x2 – 4x – 21 = (x – 7)(x + 3).
(d) Apply common monomial factoring and the cross method.
6x2 – 15x – 9 = 3(2x2 – 5x – 3)
2x +1 +x

x –3 –6x
2x 2
–3 –5x
Hence, 6x2 – 15x – 9 = 3(2x + 1)(x – 3).
(e) 
Apply common monomial factoring and the special product
formula for the difference of two squares.
3x2 – 108 = 3(x2) – 3(36)
= 3(x2 – 36)
= 3(x + 6)(x – 6)

109

Pre-Calculus and Calculus.indb 109 13/7/2017 5:57:01 PM


Chapter 4 • Equations in One Variable

Now, you can apply the factoring techniques and the zero product property
to find the roots of the given quadratic equation.

Theorem 4.3

The zero product property states that, if the product of two algebraic
expressions is zero, then at least one of the factors is equal to zero.
If AB = 0, then A = 0 or B = 0.

Summary

The following steps show the process of solving quadratic equations by


factoring.
Step 1: Write the equation in standard form ax2 + bx + c = 0.
Step 2: Factor the quadratic trinomial of the equation.
Step 3: Apply the zero product property.

Example 11
Determine the roots of the equation x2 – x – 20 = 0.

SOLUTION
Note Step 1: The given equation is already in standard form.
x2 – x – 20 = 0
Remember that the quadratic
trinomial of the equation Step 2: Factor the quadratic trinomial of the equation.
must be in standard form x2 – x – 20 = 0
before the zero product (x – 5)(x + 4) = 0
property is applied.
Step 3: Apply the zero product property.
x – 5 = 0  or  x + 4 = 0
x = 5 x = –4

Example 12
Solve the following quadratic equations.
(a) 3x2 + 2x = 8 (b) 5x2 – 5x = 0

SOLUTION
(a) Rewrite the equation in standard form.
3x2 + 2x = 8
3x2 + 2x – 8 = 0
Factor the quadratic trinomial using the cross method.
x +2 +6x

3x –4 –4x
3x2 –8 +2x
Hence, (x + 2)(3x – 4) = 0.
110

Pre-Calculus and Calculus.indb 110 13/7/2017 5:57:01 PM


Chapter 4 • Equations in One Variable

Find the values of x by applying the zero product property. Note


x + 2 = 0   or   3x – 4 = 0
To check if the solution
x = –2 3x = 4
satisfies the given equation,
​​  34 ​​ 
x = __ substitute the acquired values
in the equation.
4  ​​  .
Hence, x = –2 or ​​ __
3
(b) Extract the common factor 5x.
5x2 – 5x = 0
5x(x – 1) = 0
5x = 0  or  x – 1 = 0
x = 0 x=1
Hence, x = 0 or 1.

4.2.2 Solving Quadratic Equations by the


Quadratic Formula
A formula that can be used to solve any quadratic equation in one variable Note
can be derived from the equation ax2 + bx + c = 0 where a ≠ 0. Here are
the steps. When finding an equivalent
quadratic equation that has
Step 1: Assuming a is positive, multiply both sides of the equation by ​​ __ 1 ​​.
a a leading coefficient of 1,
Then, simplify the equation. you need to multiply both
1 ​​(ax2 + bx + c) = __ sides of the equation by the
​​ __ a ​​  1a ​​(0) reciprocal of the leading
​​ __ 1 ​​(ax2) + __​​  1a ​​(bx) + __ ​​  1a ​​(c) = __ ​​  1a ​​(0) coefficient.
a
b c
x2 + __ ​​  a ​​x + __ ​​  a ​​ = 0
c
Step 2: Move the constant term, __ ​​ a ​​to the right-hand side of the equation
c
by adding –​​ __ a ​​to both sides.
b c
x2 + __ ​​  a ​​x = –​​ __ a ​​
Step 3: Add the square of one-half of the coefficient of x to both sides of the
Note
equation.
b b2 c ___ b2 b2
x2 + __
​​  a ​​x + ___​​  2  ​​ = –​​ __ a  ​​ + ​​  2  ​​  The expression ___​​  2  ​​ is the
4a
4a 4a
__ b ___ b 2
b 2
_______ – 4ac square of one-half of the
x2 + ​​  a ​​x + ​​  2  ​​ = ​​   ​​    coefficient of x.
4a 4a2
[​​​  (​  ​  2 ​ )(​​​    ​  a ​)​] ​​​​   ​​ = (​​​  ​  2a  ​)
  ​​​​   ​​
2 2
Step 4: Factor the perfect square trinomial. Then, find the values of x by __ 1 __ b ___ b
finding the square root of both sides.
b2
= ___
​ 2a   ​)​​​   ​​ = _______
​​  2  ​​ 
​​​ (x + ___
2
b b2 – 4ac 4a
​​  2 ​​   
4a _______ b2
The expression ___
√ 
​​  2  ​​ is added
b ​b​​  2​– 4ac 4a
x + ___ ​​  2a   ​​ = ±​​ _______ ​   ​ ​​  

_______4a2 to both sides of an equation
b ​√ ​b​​  2​– 4ac   ​ to produce a perfect square
x + ___ ​​  2a   ​​ = ± ​​ _________ 2a ​​  
  trinomial.
_______
b _________ ​√ ​b​​  2​– 4ac  

x = –​​ ___ 2a _______    ​​ ± ​​ 
2a ​​   

–b ± √ ​  ​b​​  2​– 4ac   ​


x = _____________
  
​​  2a ​​  111

Pre-Calculus and Calculus.indb 111 13/7/2017 5:57:01 PM


Chapter 4 • Equations in One Variable

Theorem 4.4

Given the standard form ax2 + bx + c = 0 where a ≠ 0, the roots are given
by the quadratic formula: _______
​  ​b​​  2​– 4ac  
–b ± √
_____________ ​
x =   
​​  2a ​​ 

Summary

The following steps show the process of solving quadratic equations


using the quadratic formula.
Step 1: Write the quadratic equation in the standard form
ax2 + bx + c = 0.
Step 2: Determine the numerical values for a (numerical coefficient
of x2), b (numerical coefficient of x) and c (constant term).
Step 3: Substitute the values of a, b and c in the quadratic formula and
evaluate the expression.

Example 13
Find the roots of 3x2 – 9x = 0 using the quadratic formula.

SOLUTION
Step 1: Write the given quadratic equation in standard form.
3x2 – 9x – 0 = 0
Step 2: I dentify the values of a, b and c.
a = 3, b = – 9 and c = 0
Step 3: Substitute the values of a, b and c in the quadratic formula and
evaluate the expression. _______
_____________–b ± ​√ ​b​​  2​– 4ac   ​
x = ​​    2a ​​  
____________
–(–9) ± √ ​    
(–9​)​​  2​– 4(3)(0) ​
x = ____________________
   ​​      ​​
___
2(3)
9±√ ​  81 ​ 
x = _______
​​  6 ​​   

9±9
x = _____ ​​  6 ​​   

9+9 9–9
x = _____
​​  6 ​​  or  x  
  = ​​ _____ 6 ​​ 

18 0
x = ​​ ___ 6 ​​   x = ​​ __
6 ​​ 
x = 3 x=0
Hence, the roots are 0 and 3.

112

Pre-Calculus and Calculus.indb 112 13/7/2017 5:57:01 PM


Chapter 4 • Equations in One Variable

The quadratic formula can be used to solve any type of quadratic equation
in one variable, regardless whether the given equation is factorable.
Note
If a quadratic equation cannot
Example 14 be easily factored, we may
Find the roots of x2 + 2x – 4 = 0 using the quadratic formula. use the quadratic formula.

SOLUTION
Step 1: The given equation is in standard form.
x2 + 2x – 4 = 0
Step 2: The values of a, b and c are 1, 2 and –4 respectively.
Step 3: Substitute the values of a, b and c in the quadratic formula.
Then, evaluate the expression. _______
–b ± ​√ ​b​​  2​– 4ac  
_____________ ​
x = ​​    2a  ​​ 
____________
–2 ± √​  (2​
   )​​  2​– 4(1)(–4) ​
x = ____________________
   ​​   ​​ 
___
2(1)
–2 ± √​  4 + 16 ​ 
x = _______
  
​​   ​​ 
2___
–2 ± √​  20 ​ 
x = ________ ​​  2 ​​  

__
–2 ± 2​√ 5  ​
x = ________ ​​  2 ​​__   
2(–1 ± √ ​  5 ​ )
x = _________ ​​  2 ​​   
__
x = –1 ± √  ​​ 5 ​​ 
__ __
​​ 5 ​​ and –1 – √ 
Hence, the roots are 1 + √  ​​ 5 ​​. 

Example 15
Find the roots of 6x2 + 15x = –6x – 9 using the quadratic formula.

SOLUTION
Express the equation in standard form. Move all the terms to the
left-hand side of the equation by applying the addition property of
equality.
6x2 + 15 = –6x – 9
6x2 + 15x + 6x + 9 = –6x – 9 + 6x + 9
6x2 + 15x + 6x + 9 = 0
6x2 + 21x + 9 = 0
Identify the values of a, b and c.
a = 6, b = 21 and c = 9

113

Pre-Calculus and Calculus.indb 113 13/7/2017 5:57:01 PM


Chapter 4 • Equations in One Variable

Substitute the values of a, b and c in the quadratic formula. Then,


evaluate the expression. _______
_____________ –b ± √ ​  ​b​​  2​– 4ac  

x =    ​​   ​​ 
2a____________
–21 ± √ ​    
(21​)​​  2​– 4(6)(9) ​
x = ____________________
   ​​      ​​
___
2(6)
–21 ± √ ​  441 – 216 ​ 
x = _______
   ​​  12 ​​ 
__
–21 ± √ ​  225 ​ 
x = ________ ​​  12 ​​   
–21 ± 15
x = ________ ​​  12 ​​   
–21 ± 15 –21 – 15
x = ________
​​  12 ​​  or  x  
  = ________
​​  12 ​​   

–6 –36
x = ___ ​​  12 ​​   x = ​​ ____
12 ​​ 
x = –​​ __ 1  ​​ x = –3
2
Hence, the roots are –3 and –​​ __ 1  ​​.
2

4.2.3 Imaginary Roots


Consider the equation x2 = −4. When the ___ square roots of the equation are
extracted, the resulting equation is x = ±​​√ −4. ​​ 
In this case, the roots of the equation are not real numbers. To resolve this
problem, mathematicians formulate a new kind of number. We call such
numbers as imaginary numbers.

Definition 4.3

An imaginary number is a number that can be expressed as the product


of a real number and an imaginary
___ unit i. The imaginary unit i is defined
by i2 =−1, or simply i = √ 
​​ −1 ​​ .

Examples of imaginary numbers are i, 5i, –2i and 15i.


Note You can simplify –4 as follows:___ ______
Since imaginary numbers are ​​  −4 ​​ = √
√ ​​  (4)(–1) ​​
__

___
not real numbers, they cannot
= (​​ √​  4 ​)  (​​​​ √
​  −1 ​) ​​
be plotted on a real number
line. = (2)(i)
= 2i
Hence, the square root of –4i is 2i. The roots of the equation x2 = −4 are
–2i and 2i.

114

Pre-Calculus and Calculus.indb 114 13/7/2017 5:57:01 PM


Chapter 4 • Equations in One Variable

Example 16
Find the roots of 5x2 + 80 = 0.

SOLUTION
Translate 5x2 + 80 = 0 into the form X 2 = d.
5x2 + 80 = 0
5x2 = –80
5x2 80
​​ ___ ___
5 ​​ = –​​  5 ​​ 
x2 = –16____
x = ±​​√ _______
–16 ​​ 
= ±​​√ (16)(–1) ​​ 
= ±4i
Hence, the roots of 5x2 + 80 = 0 are –4i and 4i. The equation has two
imaginary roots.

Exercise 4.2
1. Solve the following quadratic equations by factoring.
(a) 2x(5x – 3) = 0 (e) x2 – 5x + 6 = 0

(b) (7x – 5)(7x + 5) = 0 (f) x2 + 4x – 21 = 0

​  23 ​ x – 6)​​  = 0
(c) (4x – 3)​​ (__ (g) x2 + 10x + 16 = 0

(d) x2 – x = 0 (h) 2x2 + 3x – 2 = 0

2. Solve the following quadratic equation by factoring.


(a) 3x2 – 5x – 2 = 0 (e) (x + 5)(x – 5) = 39
9x2
(b) 15x2 + 4x – 3 = 0 (f) ​​ ___
16 ​​ – 1 = 0
(c) 6x2 + 23x + 7 = 0 (g) (3x – 4)(x – 7) = – 20

(d) 5x2 – 26x + 5 = 0

3. It is given that one of the solutions of the equation x2 + kx + 42 = 0 is 6.


(a) What is the value of k?
(b) What is the other solution of the equation?

4. Given that x = 8 is a solution of the equation x2 + mx = 104, find


(a) the value of m,
(b) the other solution of the equation.

115

Pre-Calculus and Calculus.indb 115 13/7/2017 5:57:01 PM


Chapter 4 • Equations in One Variable

5. Find the roots of the following quadratic equations using the quadratic formula.
(a) x2 + 25x = 0 (f) x2 = 4x + 45
(b) x2 – 36x = 0 (g) 2x2 – 10x + 35 = x2 + 10
(c) x2 + 14x + 48 = 0 (h) x(x – 7) = 60
(d) x2 – 19x + 90 = 0 (i) x2 + 7x = 8
(e) x2 – x – 30 = 0 (j) 2x2 – 5x + 1 = 0

6. Solve the equation –(3x – 4)2 = 8x – 7(x2 + 2x – 3) + 63.

7. Find the roots of the following quadratic equations.


(a) (x + 1)2 = 1
36
(b) (x – 2)2 – ___
​​  25 ​​ = 0

(c) (2x + 5)2 + 1 = 0

(d) 3x2 + 6x + 8 = 0

116

Pre-Calculus and Calculus.indb 116 13/7/2017 5:57:01 PM


Chapter 4 • Equations in One Variable

4.3 Equations with Degree  3


Solving linear equations is a one step process, but solving quadratic
equations involves factorization.
Similarly, equations with degree  3 could be found by factorization. To
carry out the factorization, factor theorem has to be used to determine the
linear factors of the equation. Once the equation is factored as completely
as possible into linear or quadratic factors, the usual methods could be
used to find the roots.
Theorem 4.5

The factor theorem states that, when the polynomial P(x) is divided by
a linear function ax – b, if the division results in a remainder P​(​  __ ​ ​​  = 0,
​  a )
b

the function ax – b is a factor of the polynomial P(x).


Example 17
Consider polynomial x3 – 3x2 – x + 3. Show that x – 3 is one of the
factors of the polynomial. Then, find the other two factors.
SOLUTION
 f (3) =
 33 – 3(32) – 3 + 3
=0
Hence, x – 3 is one of the factors of the polynomial.
Since x – 3 is a factor, then x3 – 3x2 – x + 3 = (x – 3)q(x), where q(x) is
quadratic. We can use long division to find q(x).
x2 – 1
x – 3 x – 3x2 – x + 3
3

x3 – 3x2
–x + 3
–x + 3
0
From the long division, we have the quotient q(x) = x2 – 1 = (x + 1)(x – 1).
Hence, the other two factors are x + 1 and x – 1.

Example 18
7
x = –​​ __
4  ​​is a root of the polynomial equation 4x – 33x + bx + 175 = 0,
3 2

where b is a real number.


Find (a) the value of b,
(b) all the roots of the equation.
SOLUTION
(a) Let p(x) = 4x3 – 33x2 + bx + 175

4  ​​is a root, p​(​  –​  4 ​ )​​  = 0


7 7
Since x = –​​ __ __

  ( 4 ) ​​ + b​(
4​(​​  –​ __
4) 4 ​ )​​ + 175 = 0
3 2
7 __7 7

 ​ ​​​  ​​ – 33​ ​​  –​   
 ​ ​​​  ​  –​ __
343 _____ 1617 __ 7
–​​ ____
16 ​​ – ​​  16 ​​   – ​​   ​​ b + 175 = 0
4
7 840
​​ __ ____
4 ​​ b = ​​  16 ​​ 
b = 30 117

Pre-Calculus and Calculus.indb 117 13/7/2017 5:57:01 PM


Chapter 4 • Equations in One Variable

(b) Use long division to divide p(x) = 4x3 – 33x2 + 30x + 175 by
7
g(x) = x + __​​  4 ​​  .
4x2 – 40x + 100
7
x + __
​​  4 ​​  4x3 – 33x2 + 30x + 175
4x3 + 7x2
–40x2 + 30x
–40x2 – 70x
100x + 175
100x + 175
0
The other two roots are obtained by solving q(x) = 0, that is
4x2 – 40x + 100 = 0
x2 – 10x + 25 = 0
(x – 5)(x – 5) = 0
The roots of the equation are x = –​​ __ 4  ​​ and x = 5.
7

In Example 18, there are two similar roots. x = 5 is called a double root,
or a root with multiplicity 2. If a particular root occur k times, then it is a
root of multiplicity k.

Example 19
x = 1 is a root of multiplicity 2 of the polynomial equation p(x) = 0,
where p(x) = x4 – 2x3 – 2x2 + 6x – 3. Factor p(x) completely.

SOLUTION
x = 1 is a root of multiplicity 2, hence we use (x – 1)(x – 1) = x2 – 2x + 1
as the divisor.
x2 –3
x – 2x + 1 x – 2x – 2x + 6x – 3
2 4 3 2

x4 – 2x3 + x2
–3x2 + 6x – 3
–3x2 + 6x – 3
0
From the division, we have the quotient q(x) = x – 3, which can be 2

factored further as __ __
q(x) = x2 – 3 = (x – √ 
​​ 3 ​​)  (x + √ 
​​ 3 ​​) 
Hence, p(x) can be factored completely as __ __
p(x) = (x – 1)(x – 1)(x – √  ​​ 3 ​​)  (x + √ 
​​ 3 ​​) 

118

Pre-Calculus and Calculus.indb 118 13/7/2017 5:57:02 PM


Chapter 4 • Equations in One Variable

In the examples that we have discussed, the roots are all real numbers.
This is not necessarily true. Before we look at examples where not all
roots are real, we look at another theorem.

Theorem 4.6

If an imaginary number is a root of polynomial p(x) with real


coefficients, then the conjugate of z, that is, z is also a root of p(x).

Note that when an imaginary number is multiplied by its conjugate, the


result is always a real number.
(a + bi)(a – bi) = a2 + b2

Example 20
3
x = __
​​  2 ​​  is a root of the polynomial p(x) = 2x3 – 7x2 + 10x – 6. Find the
other two roots.

SOLUTION
3
Using long division, divide p(x) by x – ​​ __
2 ​​  .
2x2 – 4x + 4
3
x – __
​​  2 ​​  2x3 – 7x2 + 10x – 6
2x3 – 3x2
–4x2 + 10x
–4x2 + 6x
4x – 6
4x – 6
0
The other two roots of p(x) are the solutions of 2x2 – 4x + 4 = 0.
Using the quadratic formula,
_______
​  b2 – 4ac  
–b ± √ ​
x = _____________
  
​​    ​​ 
2a _____
–(–4) ± √
____________ ​  16 – 32 ​ 
=    ​​  4  ​​ 
___
4±√​  –16 ​ 
= ​​ _______ 4 ​​  

___
4​√ –1 ​ 
= 1 ± ​​ _____ 4 ​​  

=1±i
The other two roots are x = 1 + i and x = 1 – i, which are imaginary
roots.

119

Pre-Calculus and Calculus.indb 119 13/7/2017 5:57:02 PM


Chapter 4 • Equations in One Variable

Example 21
Solve 2x4 + 4x3 – x2 – x + 2 = 0.
SOLUTION
Let the polynomial P(x) = 2x4 + 4x3 – x2 – x + 2. This is a degree 4
polynomial.
The trial and error method is used to find linear factors of the polynomial.
To test whether (x – 1) is a factor.
Substituting x = 1, P(1) =  2+4–1–1+2
=6
 (x – 1) is not a factor.
To test whether (x + 1) is a factor.
Substituting x = –1, P(–1) =  2–4–1+1+2
=0
 (x + 1) is a factor.
To test whether (x – 2) is a factor.
Substituting x = 2, P(2) =  2(16) + 4(8) – 1(4) – 1(2) + 2
= 60
 (x – 2) is not a factor.
To test whether (x + 2) is a factor.
Substituting x = –2, P(–2) =  2(16) + 4(–8) – 1(4) – 1(–2) + 2
=0
 (x + 2) is a factor.
Hence, P(x) = (x + 1)(x + 2)q(x), where q(x) is the quotient.
As P(x) is degree 4 and (x + 1)(x + 2) is degree 2, Q(x) must be degree 2.
Let q(x) = ax2 + bx + c, where a ≠ 0.
P(x) ≡ (x + 1)(x + 2)(ax2 + bx + c)
2x4 + 4x3 – x2 – x + 2 ≡ (x2 + 3x + 2)(ax2 + bx + c)
Comparing coefficients of x4 : a = 2
Comparing constants : 2 = 2c
c=1
Let x = 1,
2 + 4 – 1 – 1 + 2 = (1 + 3 + 2)(2 + b + 1)
6 = (6)(3 + b)
3+b=1
b = –2
Hence, P(x) = (x + 1)(x + 2)(2x2 – 2x + 1).
Solving (x + 1)(x + 2)(2x2 – 2x + 1) = 0,
(x + 1) = 0, (x + 2) = 0 or (2x2 – 2x + 1) = 0.
For x + 1 = 0, x = –1
For x + 2 = 0, x = –2
For 2x2 –_________
2x + 1 = 0,
___ ___
2±√​  4 – 4(2)(1) ​ _______
2±√ ​  –4 ​  ________
2 ± 2​√ –1 ​  __
______________
x = ​​    4 ​​  = ​​  4 ​​   = ​​ 
4 ​​   = ​​  21 ​​  ± ​​ __
1  ​​i
2
1 1
The solutions to the equation are x = –1, –2, ​​ 2  ​​ – ​​  2 ​​ i and ​​ __
__ __ 1 __ 1
2 ​​  + ​​  2 ​​ i.
120

Pre-Calculus and Calculus.indb 120 13/7/2017 5:57:02 PM


Chapter 4 • Equations in One Variable

Sometimes, we can reduce a equation of degree  3 to a quadratic equation.


Let us look at the following example.

Example 22
Determine the roots of the equation x4 – 5x2 + 4 = 0.
SOLUTION
Let t = x2.
x4 – 5x2 + 4 = 0
(x2)2 – 5(x2) + 4 = 0
t2 – 5t + 4 = 0
The obtained equation is a quadratic equation in terms of t. Find the
roots of the obtained quadratic equation by factoring.
t2 – 5t + 4 = 0
(t – 4)(t – 1) = 0
t – 4 = 0   or   t – 1 = 0
t = 4 t=1
Substituting t = x2,
x2 = 4  
__ or x2 = 1 __
x = ±​​√ 4 ​​   x = ±​​√ 1 ​​ 
x = ±2 x = ±1
Hence, the roots of the equation are –2, –1, 1 and 2.

Exercise 4.3
1. For P(x) = 3x4 + 8x3 + 14x2 – 19x – 28, it is given that (3x – 4) and (x + 1) are factors.
(a) Factor P(x) completely.
(b) Solve the equation P(x) = 0 to find its roots.

2. The polynomial P(x) = 6x3 + px2 – 29x + 10 has a factor of 3x + 5.


(a) Find p.
(b) Solve the equation P(x) = 0 to find its roots.

3. Solve the equation 6x3 + 7x2 – 34x – 35 = 0.

4. The polynomial P(x) = 3x3 + ax2 + bx + 22 is exactly divisible


__ by __x2 – 2.
(a) Find the values of a and b by writing x2 – 2 as (x – ​√2 
 ​)(x + √
​ 2 
 ​).
(b) Solve the equation P(x) + (x – 2)(2x + 3) = 0.
2

121

Pre-Calculus and Calculus.indb 121 13/7/2017 5:57:02 PM


Chapter 4 • Equations in One Variable

___
72 ___
18
5. Solve the equation 17 – ​  2 ​ = x2 + 2x – ​  x ​ which has four distinct integer roots.
x

6. Solve the equation 2x3 – 7x2 – 42x + 72.

7. The polynomial P(x) is 4x4 – 20x3 – 3x2 + 70x + 49. A, B and C are positive integers such that
P(x) = [Ax2 – Bx – C]2.
(a) Find the values of A, B and C.
(b) What are the roots of the equation P(x) = Ax2 – Bx – C?
(c) Find the roots of the equation 4x8 – 20x6 – 3x4 + 70x2 + 49 = 0. Correct your answers to 2 decimal
places.

8. Solve the equation 9x4 + 241x2 + 525 = 125 by reducing it to a quadratic equation.

122

Pre-Calculus and Calculus.indb 122 13/7/2017 5:57:02 PM


Chapter 4 • Equations in One Variable

4.4 Radical Equations


A radical equation_____ is an equation which _____ involves
_____unknown under a
__ __
​​  x  ​​= 3 + ​​√ x – 1 ​​
radical. √  and ​​√ x ​​  – √
​​  x + 1 ​​
 = ​​√ x + 2 ​​
 are examples of
equations involving radicals. To solve a radical equation, leave one of the
terms with radical on one side and all other terms on the other. Square both
sides of the equation and simplify. If there are more than one term with
radical, we need to repeat the process until all the radicals are removed.
Solutions obtained need to be checked whether they are the true solutions
of the original equation. Values which do not satisfy the given equation are
rejected. They are called extraneous solutions.

Example 23
Solve______
the following radical equations. _____
__
(a) ​​√ 3x – 2 ​​ = 4 _____
(c) ​​√______
​​  y + 2 ​​______
 y  ​​ – √  =1 _____
__
(b) ​​√ u  ​​+ 1 = √ 
​​ u + 7 ​​   (d) ​​√ 3 – 3x   ​​ – √
​​  2x + 5 ​​  ​​ 2 – x  
= √  ​​

SOLUTION
______
(a) ​​√ 3x – 2 ​​ 
=4
Squaring both sides,
______ 2
​​​ (​√ 3x – 2 ​  
)​​​   ​​ = 42
3x – 2 = 16
3x = 18
x=6
_______ ___
​​  3(6) – 2 ​​ 
Check: √ = √ 
​​ 16 ​​ = 4
Hence, x = 6.
__ _____
(b) ​​√ u  ​​+ 1 = √  u + 7 ​​ 
​​ _____
__
​​√ u  ​​ = √ ​​  u + 7 ​​ – 1
Squaring both_____ sides,
__
(​​√ u  )​​ = (​​√ u + 7 ​​ – 1)_____
2 2

u = (u + 7) – 2​​_____
√ u + 7 ​​ + 1
_____
u = u + 8 – 2​​√ u – 7 ​​

u + 7 ​​ = 8
2​​√ _____
​​√ u + 7 ​​ = 4
Squaring both sides again,
_____ 2
​​​ (​√ u + 7 ​) ​​​   ​​ = 16
u + 7 = 16
u=9
__ _____
​​ 9 ​​ + 1 = 4 = √ 
Check: √  ​​ 9 + 7 ​​ 
Hence, u = 9.

123

Pre-Calculus and Calculus.indb 123 13/7/2017 5:57:02 PM


Chapter 4 • Equations in One Variable

__ _____
​​  y + 2 ​​ = 1
(c) ​​√ y  ​​ – √ _____
__
​​√ y  ​​= 1 + √ ​​  y + 2 ​​ 
Squaring both sides, _____
__
(​​√ y  )​​ 2 = (​​​  1 + √ + 2 ​) ​​​   ​​
2
​  y_____
y = 1 + 2​​√ y + _____ 2 ​​ + (y + 2)
_____
y = y + 3 + 2​​√ y + 2 ​​ 
–2​​√ y + 2 ​​ = 3
Squaring both sides again,
_____
(​​​  –2​√ y + 2 ​) ​​​   ​​ = 32
2

4(y + 2) = 9
4y = 1
y = ​​ __ 1 ​​ 
__ 4
_____
1
√  √ 
Check: ​​ ​  4 ​ ​​  – ​​ __
__

__ _____
​​  21 ​​  – __
​  41 ​  + 2 ​​ = __
3
​​  2 ​​  ≠ 1
Hence, √ ​​  y  ​​ – √ ​​  y + 2 ​​ = 1 has no solution.
______ ______ _____
(d) ​​√ 3 – 3x  
​​ – √
​​  2x + 5 ​​  ​​ 2 – x  
= √  ​​
Squaring both sides,______ ______ 2 _____ 2
(​​​  √
​  3 – 3x  ​– ​
√  2x + 5 ​) 
​​​   ​​ = (​​​  ​√ 2 – x ) 
​​​​   ​​
______ ______
(3 – 3x) – 2​​√ 2x + 5 ​​​​√   3 – 3x   ​​ + (2x + 5) = 2 – x
______ ______
8 – x – 2​​√ ______2x + 5 ​​​​   3 – 3x  
√______ ​​= 2 – x
 2x + 5 ​​​​√
–2​​√______   3 – 3x  
______
​​ = –6
​​√ 2x + 5 ​​​​√   3 – 3x   ​​ = 3
Squaring both sides again,
______ ______ 2
​​​ (​√ 2x + 5 ​​√   3 – 3x   )​​​​   ​​ = 32
(2x + 5)(3 – 3x) = 9
6x – 6x2 + 15 – 15x = 9
6x2 + 9x – 6 = 0
2x2 + 3x – 2 = 0
(2x – 1)(x + 2) = 0
x = __ ​​  21 ​​   or  –2
Check:
________ ________ __ __ _____

√  √ 
 – ​​ 2​ (​   ​ )​​  + 5 ​​  √  √   √ 
__
​​ 3 – 3​ (​  2 ​ )​ ​​ 
1 1 3 3
__ __
2
__
​​  6 ​​ = –​​ ​  2 ​ ​​  ≠ ​​ 2 – __
= ​​ ​  2 ​ ​​  – √ __
​ 21 ​ ​​  
________ ________ _______
​​√ 3 – 3(–2) ​​  – ​​√ 2(–2) + 5 ​​  =3–1=2=√ ​​  2 – (–2) ​​ 
Hence, x = –2 is the only solution.

124

Pre-Calculus and Calculus.indb 124 13/7/2017 5:57:02 PM


Chapter 4 • Equations in One Variable

Example 24
3
______
Solve ​​√ 
  3x – 4 ​​ 
= 2.
SOLUTION
Since the equation involves finding cube root, raise both sides to the
power of three to eliminate the radical.
______
3
  3x – 4 ​​ 
​​√  =2
______
(​​√   3x – 4 ​​) = 23 3 3
Raise both sides to power of 3
3x – 4 = 8
3x = 12 Add 4 to both sides
x=4 Divide both sides by 3

Example 25
______
5
Solve ​​√   x2 + 7 ​​ 
= 2.
SOLUTION
Since the equation involves finding 5th root, raise both sides to the
power of five to eliminate the radical.
______
5
​​√   x  + 7 ​​ 
2
=2
x + 7 = 25 2 Raise both sides to power of 5
x2 + 7 = 32
x2 = 25 Subtract 7 from both sides
x = ±5 Square root both sides
Hence, x = –5 and 5.

Exercise 4.4
1. Solve the following equations.
_____ ____ ____ _____
(a) ​​√ 7 + x ​​ = x – 5 (e) ​​√ t + 7 ​​ = √ 
​​ t + 2 ​​ + √ 
​​ 2t – 3 ​​ 
___________
__ __ _______
(b) ​​√ 17 + 6​√ 2 
 ​x ​​  ​​ 2 ​​x  (f)
= 3 + √  x + √ 
​​ 4 – 15x  
​​ = 4
_____ _______ ____ ___
(c) 2​​√ 1 – x ​​ = √
​​  16 – 3x ​​ 
– 1 (g) ​​√ 27x ​​ + √
​​  3x ​​ = 12
__ _____ _______ __
(d) ​​√ x ​​ + √
​​  3x – 2 ​​ 
= √ 
​​ 6x + 10 ​​   (h) x – ​​√ x ​​ = 6

2. Solve the following equations.


3
_____ 4
_____
  x – 2 ​​ 
(a) 4​​√  + 1 = 9   x + 1 ​​ 
(d) ​​√  +4=8
__ 5
_____
3
  x ​​ + 1 = –2
(b) ​​√    x – 2 ​​ 
(e) ​​√  +3=4
2

3
______ 3
_____
(c) –2​​√ 
  2x + 5 ​​ 
+ 7 = 15   x + 4 ​​ 
(f) ​​√  –2=0
2

125

Pre-Calculus and Calculus.indb 125 13/7/2017 5:57:02 PM


Chapter 4 • Equations in One Variable

4.5 Exponential Equations


Exponential equations are equations in which the index involves
variables. Examples of exponential equations are 2x = 5x, 32x – 3x + 2 = 0 and
5x = 5(53x – 1). These equations are solved by the following methods:
(i) Method of inspection. This method is usually used when each side of
the equation can be written in the form of indices with the same base.
(ii) 
Taking logarithm on both sides of the equations and using the
properties of logarithms.
(iii) Using a suitable substitution of the form to obtain an equation
in y which is then solved.

Example 26
Solve the following exponential equations.
(a) 25x – 2 – 42 – x = 0
(b) ​​5​​  x ​​  53x = ___
2 1  ​​ 
​​  25
(c) (3x)4 = 81

SOLUTION
(a) 25x – 2 – 42 – x = 0
25x – 3 = (22) 2 – x
25x – 3 = 24 – 2x
Equating the index,
5x – 3 = 4 – 2x
7x = 7
x=1
(b) ​​5​​  x ​​  53x = ___
2 1  ​​ 
​​  25
​​5​​  x + 3x​​ = 5–2
2

Equating the index,


x2 + 3x = –2
x + 3x + 2 = 0
2

(x + 2)(x + 1) = 0
x = –2 or –1
(c) (3x)4 = 81
34x = 34
Equating the index,
4x = 4
x=1

126

Pre-Calculus and Calculus.indb 126 13/7/2017 5:57:02 PM


Chapter 4 • Equations in One Variable

Example 27
Solve the following exponential equations, correcting your answers to
three decimal places.
(a) 3x = 5
(b) 10x – 1 – 0.5 = 0
(c) 25(5x) = 2

SOLUTION
(a) 3x = 5
It is not possible to write both sides of equation in index form with
same base. Therefore, we use the second method, that is, to take
logarithm (base 10 or base e) on both sides of equation.
lg 3x = lg 5
x lg 3 = lg 5
lg 5
x = ____
​​    ​​
lg 3
x ≈ 1.465
(b) 10x – 1 – 0.5 = 0
10x – 1 = 0.5
Taking logarithm to base 10 on both sides,
lg 10x – 1 = lg 0.5
(x – 1) lg 10 = lg 0.5
Note
x – 1 = lg 0.5 lg 10 = log10 10 = 1
x = 1 + lg 0.5
x ≈ 0.699
(c) 25(5x) = 2
5x + 2 = 2
Taking logarithm to base e on both sides,
ln 5x + 2 = ln 2
(x + 2) ln 5 = ln 2
x ln 5 + 2 ln 5 = ln 2
x ln 5 = ln 2 – 2 ln 5
ln 2 – 2 ln 5
x = __________
​​   ​​   
ln 5
x ≈ – 1.569

127

Pre-Calculus and Calculus.indb 127 13/7/2017 5:57:02 PM


Chapter 4 • Equations in One Variable

Example 28
Using a suitable substitution, change the following equations to
quadratic equations and solve them.
(a) 9x – 10(3x) + 9 = 0
(b) 4x – 2x + 2 = 12
9
(c) 2x + 21 – x = __
​​  2 ​​ 

SOLUTION
(a) 9x – 10(3x) + 9 = 0
(32)x – 10(3x) + 9 = 0
(3x)2 – 10(3x) + 9 = 0
Let 3x = y    y2 – 10y + 9 = 0
(y – 9)(y – 1) = 0
y = 9  or  y = 1
 3x = 9 or 3x = 1
3x = 32 3x = 30
x = 2 x=0
Hence, x = 0 or 2.
(b) 4x – 2x + 2 = 12
(22)x – 2x22 – 12 = 0
(2x)2 – 4(2x) – 12 = 0
Let 2x = y    y2 – 4y – 12 = 0
(y – 6)(y + 2) = 0
y = 6  or  y = –2
 2x = 6 or 2x = –2
Taking lg on both sides of 2x = 6,
lg 2x = lg 6
x lg 2 = lg 6
lg 6
x = ____​​   
 ​​
lg 2
x = 2.585  (3 d.p.)
There is no solution for 2x = –2.
Hence, x = 2.585.
9
(c) 2x + 21 – x = __ ​​  2 ​​ 
9
2x + __ ​​  22x  ​​ = __ ​​  2  ​​
2(2x)2 + 4 = 9(2x)
2(2x)2 – 9(2x) + 4 = 0
Let 2x = y   2y2 – 9y + 4 = 0
(2y – 1)(y – 4) = 0
y = __ ​​  21 ​​   or  y = 4

 2x = __ ​​  21 ​​   or 2x = 4
2x = 2–1 2x = 4
x = –1 x=2
Hence, x = –1 or 2.
128

Pre-Calculus and Calculus.indb 128 13/7/2017 5:57:02 PM


Chapter 4 • Equations in One Variable

Exercise 4.5
1. Solve the following equations.
(a) 5x__– 2 = 625 __
2x + 1
(b) ​​​ (√
​  3 ​ )​​​​    ​​ – 9​​√ 3 ​​ = 0
(c) 33y – 19 y – 1 = 812y
(d) 32x + 2 + 3x + 3 – 10 = 0
(e) 22x + 1 + 4 = 9(2x)
(f) 22t – 1 + 2t + 1 = 16

2. Solve for x. Correct your answers to 3 decimal places.


(a) 5x – 1 = 7
9
(b) 21 – x = __
​​  4 ​​ 
(c) 4(2x) = 3x  5x

3. A radioactive substance decays such the mass m of substance after time t is given by the function,
m = Me–0.0578t where M is the initial mass of the substance and t is the time in minutes.
(a) What fraction of the substance of initial mass M grams remains after 24 minutes of decay?
(b) How long does it take for the substance to become ___ 1  ​​ of its initial mass?
​​ 20

129

Pre-Calculus and Calculus.indb 129 13/7/2017 5:57:02 PM


Chapter 4 • Equations in One Variable

4.6 Logarithmic Equations


Equations that contain logarithms of an unknown are called logarithmic
equations. Examples of logarithmic equations are log2 (3x – 1) = 3 and
. The properties of logarithms are used to simplify
and solve logarithmic equations. Each solution obtained must be checked
and extraneous solutions are discarded. Logarithms are not defined for
negative numbers.

Example 29
Solve the following logarithmic equations.
(a) log6 (x + 3) + log6 (x – 2) = 1
(b) log2 (5y – 7) = 3
(c) lg (2t3 + 1) = 1 + 3 lg t
(d) ln (x + 1) + ln x = ln (3x – 1)
SOLUTION
(a) log6 (x + 3) + log6 (x – 2) = 1
log6 (x + 3)(x – 2) = 1
(x + 3)(x – 2) = 61
x2 + x – 12 = 0
(x + 4)(x – 3) = 0
x = –4  or  x = 3
Check:
For x = – 4, log6 (–4 + 3) = log6 (–1)
log6 (–4 – 2) = log6 (–6)
log6 (–1) and log6 (–6) are not defined.
For x = 3, log6 (3 + 3) + log6 (3 – 2) = 1
Hence, x = 3.
(b) log2 (5y – 7) = 3
5y – 7 = 23
5y = 15
y=3
(c) lg (2t3 + 1) = 1 + 3 lg t
lg (2t + 1) – 3 lg t = 1
3

lg (2t3 + 1) – lg t3 = 1
lg (​​  ______  ​​  = 1
t )
2​t​​  3​+ 1
​  3 ​  
2t3 + 1
​​ ______  ​​ 
 = 101
t3

2t3 + 1 = 10t3
8t3 = 1 __
t = ​​  __ √  ​  81 ​ ​​  
3

​​  21 ​​ 
= __

130

Pre-Calculus and Calculus.indb 130 13/7/2017 5:57:02 PM


Chapter 4 • Equations in One Variable

(d) ln (x + 1) + ln x = ln (3x – 1)
ln [x(x + 1)] = ln (3x – 1)
x(x + 1) = 3x – 1
x2 + x = 3x – 1
x – 2x + 1 = 0
2

(x – 1)2 = 0
x=1

Example 30
Solve the following logarithmic equations.
(a) 4 log4 x = logx 4
(b) 3(log3 y + logy 3) = 10
(c) 3 – 2 log25 u = 2 logu 5
SOLUTION
(a) 4 log4 x = logx 4
Using the change of base formula, logx 4 = _____ ​​  1    
​​  .
log4 x
Thus, ​​  1    
4 log4 x = _____ ​​  .
log4 x
Multiply throughout by log4 x,
4 (log4 x)2 = 1
4 (log4 x)2 – 1 = 0
Factor the left side,
(2 log4 x – 1)(2 log4 x + 1) = 0
2 log4 x – 1 = 0  or  2 log4 x + 1 = 0
2 log4 x = 1 2 log4 x = –1
log4 x = ​​  2 ​​   __ 1 log4 x = –​​ __ 1  ​​
2
1
__
​   ​  1 ​ 
–​ __
x = ​​4​​  2​​ x = ​​4​​  2​​
x = 2 1  ​​ 
x = ​​ __
​  1 ​ 
__
​4​​  2​
​​  21 ​​ 
x = __
1  ​​ or 2.
Hence, x = ​​ __
2
(b) 3(log3 y + logy 3) = 10
​​  1    
Using the change of base formula, logy 3 = _____ ​​  .
log3 y
​ ​​  = 10.
( l​og​  3​​ y )
​  1    
Thus, 3​​  log3 y + _____
Multiply throughout by log3 y,
3(log3 y)2 + 3 = 10 log3 y
3(log3 y) – 10 log3 y + 3 = 0
2

131

Pre-Calculus and Calculus.indb 131 13/7/2017 5:57:02 PM


Chapter 4 • Equations in One Variable

Factor the left side,


(3 log3 y – 1)(log3 y – 3) = 0
3 log3 y – 1 = 0   or   log3 y – 3 = 0
log3 y = __ ​​  31 ​​   log3 y = 3
__ 1
​   ​ 
y = ​​3​​  3​​ y = 33
y ≈ 1.442 y = 27
Hence, y = 1.442 or 27.
(c) 3 – 2 log25 u = 2 logu 5
Using the change of base formula,
log5 u _____ log5 u 1 ​​  .
log25 u = ______
​​    = ​​  2 ​​ 
 ​​  _____
 and logu 5 = ​​      
log5 25 log5 u
2 log5 u _____
Thus, 3 – _____
​​  2 ​​   = ​​  2     ​​  .
log5 u
Multiply throughout by log5 u,
3(log5 u) – (log5 u)2 = 2
(log5 u)2 – 3(log5 u) + 2 = 0
Factor the left side,
(log5 u – 1)(log5 u – 2) = 0
log5 u – 1 = 0  or  log5 u – 2 = 0
log5 u = 1 log5 u = 2
u=5 1
u = 52
u = 5 u = 25
Hence, u = 5 or 25.

Exercise 4.6
1. Solve the following logarithmic equations. Correct your answers to 3 decimal places, if applicable.
(a) log5 y = 2 (e) log2 x = 4 – 4 logx 2

(b) ln (x – 5) = 1 (f) (lg x)2 = lg x3

(c) lg u = 2 lg 3 + lg (5u – 2) (g) 5 – 4 log3 x = logx 3


1 ​​  log 4 = log (t – 1)
(d) log3 (3t) – ​​ __ (h) 2x – 2lg x = 0
2 3 3

2. A bank offers to its depositors an annual compound interest rate of 3%. A deposit of $10 000 was
made at the beginning of the year 2010.
(a) Write a formula to express the value of the deposit after n year.
(b) After how many years will the deposit exceed the value of $12 000?
(c) Find the number of years taken for the value of the deposit to double.

132

Pre-Calculus and Calculus.indb 132 13/7/2017 5:57:02 PM


Chapter 4 • Equations in One Variable

4.7 Applications of Equations


4.7.1 Number Problems
4.7.2 Geometry Problems
4.7.3 Investment Problems
4.7.4 Motion Problems
4.7.5 Age Problems
4.7.6 Mixture Problems
4.7.7 Work Problems

4.7.1 Number Problems


Example 31
What three consecutive odd integers give a sum of 87? Note
Represent the integers
SOLUTION in terms of variables and
Step 1: Define variables. algebraic expressions.
Let: x = smallest odd integer
x + 2 = middle odd integer
x + 4 = largest odd integer
Step 2: Translate into an equation.
The sum of three consecutive odd numbers is 87.
x + (x + 2) + (x + 4) = 87
Step 3: Solve the equation.
x + x + 2 + x + 4 = 87
3x + 6 = 87
3x = 87 − 6 Transpose 6
3x = 81
3x ___ 81
​​ ___ 3 ​​ = ​​  3 ​​  MPE

x = 27
Hence,
the smallest odd integer is x = 27,
the middle odd integer is x + 2 = 27 + 2 = 29,
the largest odd integer is x + 4 = 27 + 4 = 31.

Example 32
A positive number is one-fourth of another number. The difference
between the greater number and the smaller number is 21. What are the
two numbers?

SOLUTION
Step 1: Define variables.
Let: x = greater number
​​ __ 1  ​​x = smaller number
4
Step 2: Translate into an equation.
greater number – smaller number = 21
x − __ ​​ 14  ​​x = 21
133

Pre-Calculus and Calculus.indb 133 13/7/2017 5:57:02 PM


Chapter 4 • Equations in One Variable

Step 3: Solve the equation.


x − __ ​​ 41 ​​ x = 21

​ 14  ​x)​​  = 4(21)
(4)​​ (x – __ MPE

4x − x = 84 Distributive property
3x = 84
3x ___ 84
​​ ___ 3 ​​ = ​​  3 ​​ 
MPE

x = 28
Hence,
the greater number is x = 28,
the smaller number is __ ​​ 14  ​​x = ​​ __
1  ​​(28) = 7.
4

4.7.2 Geometry Problems


Example 33
Mr. Lee bought a rectangular lot whose length is 5 m more than its
width. If its perimeter is 70 m, find the dimension of the lot.

SOLUTION
Step 1: Define variables.
x+5


Let: x = width of the lot
x +5 = length of the lot
Step 2: Translate into an equation.
The perimeter is 70 m.
P = 2l + 2w
70 = 2(x + 5) + 2x
Step 3: Solve the equation.
70 = 2(x + 5) + 2x
70 = 2x + 10 + 2x
70 − 10 = 4x
60 = 4x
60 ___ 4x
​​ ___ 4 ​​ = ​​  4 ​​ 
15 = x
Hence,
the width of the lot is x = 15 m,
the length of the lot is x + 5 = 15 + 5 = 20 m.

134

Pre-Calculus and Calculus.indb 134 13/7/2017 5:57:03 PM


Chapter 4 • Equations in One Variable

Example 34
If the height of a triangle is decreased by 2 cm, it will be half the base
length. Given that the area of the triangle is 15 cm2, find the height of
the triangle.

SOLUTION
Step 1: Define variables.
Let x be the height of the triangle.

base length
If the height is decreased by 2 cm, it will be half the base
length.
base length = 2(x + 2)
Step 2: Translate into an equation.
Since the area of the triangle is equal to 15 cm2,
​​ __ 1
2 ​​ bh = A
​​ __ 1  ​​[2(x + 2)](x) = 15
2
x2 + 2x = 15
x2 + 2x – 15 = 0
Step 3: Solve the quadratic equation using the quadratic formula.
Determine the values of a, b and c.
a = 1, b = 2 and c = –15
Substitute them in the _______ quadratic formula.
_____________ –b ± √ ​  ​b​​  ​– 4ac  
2

x = ​​    2a  ​​ 
____________
–2 ± √ ​  (2​
   )​​  2​– 4(1)(–15) ​
x = ____________________
   ​​      ​​
___
2(1)
–2 ± √ ​  4 + 60 ​ 
x = _______
   ​​  2__  ​​ 
–2 ± √ ​  64 ​ 
x = ________ ​​  2 ​​   
–2 ± 8
x = ________ ​​  2 ​​   
–2 + 8 –2 – 8
x = ________
​​  2 ​​  or  x  
  = ​​ ________
2 ​​   
6 –10
x = ​​ ___ 2 ​​   x = ​​ ____
2 ​​  
x = 3 x = –5
Since the height of the triangle must be non-negative, the
height is 3 cm.

135

Pre-Calculus and Calculus.indb 135 13/7/2017 5:57:03 PM


Chapter 4 • Equations in One Variable

4.7.3 Investment Problems


Interest is an amount obtained from investing money for some time.
Simple interest I paid on the principal P at an interest rate r after a time
t is calculated by
I = Prt,  where I = interest
P = principal
r = rate
t = time
If a principal P is invested at a simple interest rate r, the total amount A
after a time t is given by
A = P + I or A = P + Prt.

Example 35
Mrs. Peterson is retiring from her job. She plans to invest a part of her
5 million dollars retirement money at 6% simple interest and the rest
at 10% simple interest for 1 year. How much should she invest at each
rate in order to gain a total of $420 000 annual interest from these two
investments?

SOLUTION
Step 1: Define variables.
Let: x = the amount invested at 6%
5 000 000 − x = the amount invested at 10%
Investment Principal, P Rate, r Time, t Interest, I
Invested at 6% x 0.06 1 0.06x
Invested at 10% 5 000 000 – x 0.10 1 0.10(5 000 000 – x)

Step 2: Translate into an equation.


Interest gained at 6% + Interest gained at 10% = Total interest
0.06x + 0.10(5 000 000 − x) = 420 000
Step 3: Solve the equation.
0.06x + 0.10(5 000 000 − x) = 420 000
100[0.06x + 0.10(5 000 000 − x)] = 100(420 000)
6x + 10(5 000 000 − x) = 42 000 000
6x + 50 000 000 − 10x = 42 000 000
50 000 000 − 42 000 000 = 4x
8 000 000 ___ 4x
​​ _________ 4 ​​  = ​​  4 ​​ 

2 000 000 = x
5 000 000 − x =  5 000 000 − 2 000 000
= 3 000 000
Hence, she should invest 2 million dollars at 6% simple interest and
3 million dollars at 10% simple interest.

136

Pre-Calculus and Calculus.indb 136 13/7/2017 5:57:03 PM


Chapter 4 • Equations in One Variable

Example 36
Suppose you deposit $10 000 in a bank at an annual simple interest rate
of 2%.
(a) How much interest will it gain in 1 year and 6 months?
(b) How much will there be in your savings account after 1 year and
6 months?

SOLUTION
Step 1: Define variables.
P = 10 000
Note
r = 2% = 0.02 ​​ 12 ​​  or 0.5 years
6 months = __
t =  1 year and 6 months
= 1.5 years
Step 2 and 3: Translate into an equation and solve.
I = Prt
I = (10 000)(0.02)(1.5)
I = 300
(a) It will gain an interest of $300 after 1 year and 6
months.
(b) A = P + I
= 10 000 + 300
= 10 300
Your savings account will have $10 300 after 1 year and
6 months.

4.7.4 Motion Problems


The distanced travelled d by a vehicle moving at an average speed s
after time t can be modeled as
d = st,  where d = distance Note
s = speed
d = st
t = time
d
s = __ ​​  t ​​ 
Example 37
d
t = __
​​  s ​​ 
A van leaves City A to travel towards City B at 64 km/h. A car leaves
City B to travel towards City A at 56 km/h. City A and City B are about
480 km apart. How many hours will it take before these two vehicles
meet?

SOLUTION
Step 1: Define variables.
Let t = time in hours it will take for the van and car to meet
Vehicle Speed (km/h) Time (h) Distance (km)
van 64 t 64t
car 56 t 56t
137

Pre-Calculus and Calculus.indb 137 13/7/2017 5:57:03 PM


Chapter 4 • Equations in One Variable

Step 2: Translate into an equation.


distance traveled by van + distance traveled by car = total distance
64t + 56t = 480
Step 3: Solve the equation.
64t +56t = 480
120t = 480
120t ____ 480
​​ ____ 120 ​​ = ​​  120 ​​ 
t=4h
Hence, the van and the car will meet after 4 h.

Example 38
An object is launched from the ground. The height h(t), in meters, of an
object as it travels above the ground is given by h(t) = –16t2 + 64t + 190
where t is the time in seconds. Find the time taken for the object to
strike the ground.

SOLUTION
Step 1: The variable has already been defined.
h(t) = –16t2 + 64t + 190
Step 2: Translate into an equation.
For the object to strike the ground, h(t) = 0.
–16t2 + 64t + 190 = 0
Step 3: Solve the equation.
Use the quadratic formula to solve the quadratic equation.
Simplify the equation first.
Divide both sides of the equation by 2.
–8t2 + 32t + 95 = 0
Based on the quadratic equation in the standard form, a = –8,
b = 32 and c = 95.
Substitute the values in the quadratic formula and evaluate the
expression. _______
​  b2 – 4ac  
–b ± √
_____________ ​
t =    ​​    ​​ 
2a______________
​  (32)
–32 ± √    2
– 4(–8)(95) ​
______________________
t =   
​​      ​​
2(–8)
___________
​  1024 + 3040 ​ 
–32 ± √
t = _________________
  
​​   ​​ 
–16
_____
​  4064 ​ 
–32 ± √
___________
t = ​​  –16 ​​   

≈ 5.98 or –1.98
Since t represents time, we reject –1.98.
Hence, it takes 5.98 seconds for the object to strike the ground.

138

Pre-Calculus and Calculus.indb 138 13/7/2017 5:57:03 PM


Chapter 4 • Equations in One Variable

4.7.5 Age Problems


Example 39
Kate sent a birthday invitation to Lara saying:
“I am inviting you to my birthday party! My age will be 6 years less
than thrice my age 10 years ago. How old will I be on my birthday?”

SOLUTION
Step 1: Define variables.
Birthday age Age 10 years ago
Kate’s age x x – 10

Step 2: Translate into an equation.


Kate’s age will be 6 years less than thrice her age 10 years ago.
x = 3(x − 10) − 6
Step 3: Solve the equation.
x = 3(x − 10) − 6
x = 3x − 30 − 6
x = 3x − 36
x − 3x = −36
−2x ____ −36
​​ ____ 2 ​​  = ​​  2 ​​  
x = 18
Hence, Kate will be celebrating her 18th birthday.

4.7.6 Mixture Problems


Linear equations are helpful in determining the mixing concentrations of
solutions so as to achieve the desired concentration. Concentration refers
to the amount of a solution that is pure.
For example, a 250 ml solution that has 70% isopropyl alcohol has
(70%)(250) = (0.70)(250)
= 175 ml of isopropyl alcohol
and the rest is another chemical, usually water.
Changing concentrations of solutions or mixtures is based on the equation:
Q = cA
where Q
 is the quantity of a substance in a solution or mixture,
c is the percent of concentration,
A is the amount of solution.

139

Pre-Calculus and Calculus.indb 139 13/7/2017 5:57:03 PM


Chapter 4 • Equations in One Variable

Example 40
How many milliliters of 40% alcohol solution must be added to 50 ml
of 70% alcohol solution to make a 50% alcohol solution?

SOLUTION
Step 1: Define variables.
Let x be the number of milliliters of 40% alcohol solution to be added.
% of Amount of Quantity of
concentration, solution, substance,
c A Q
70% alcohol
0.7 50 0.7(50)
solution
40% alcohol
0.4 x 0.4x
solution
50% alcohol
0.5 x + 50 0.5(x + 50)
solution

Step 2: Translate into an equation.


amount of alcohol amount of alcohol amount of alcohol
+ =
in the 70% solution in the 40% solution in the 50% solution
0.7(50) + 0.4x = 0.5(x + 50)
Step 3: Solve the equation.
Note 0.7(50) + 0.4x = 0.5(x + 50)
When combining two 10[0.7(50) + 0.4x] = 10[0.5(x + 50)]
solutions with different 7(50) + 4x = 5x + 250
concentrations, the 350 + 4x = 5x + 250
concentration of the resulting 4x − 5x = 250 − 350
solution will always
−x = −100
be a value between the
concentrations of the two (−1)(−x) = (−1)(100)
mixed solutions. x = 100
Hence, 100 ml of 40% alcohol must be added.

4.7.7 Work Problems


Work problems are usually solved using equations involving fractions.
The whole task is considered as one job and the rate of doing work is
​​  1x ​​, where x is the number of time units one has to finish the
represented by __
job.
In work problems, the amount of work done W is the product of the rate
r and time t, that is,
W = rt,  where W = work
r = rate
t = time

140

Pre-Calculus and Calculus.indb 140 13/7/2017 5:57:03 PM


Chapter 4 • Equations in One Variable

Example 41
Peter can paint the entire school fence in 4 days. His helper can finish
the same task in 6 days. How long will it take for them to finish the job
if they work on it together?

SOLUTION
Step 1: Define variables.
Let x be the number of days Peter and his helper take to finish
the job.
Rate, Time, Work,
r t W
x
Peter ​​  1 ​​ 
__
4 x ​​ __
4  ​​
1 x
His helper ​​ __
6 ​​  x ​​ __
6  ​​

Step 2: Translate into an equation.


work done by Peter + work done by his helper = 1 job
__ x x
​​  4  ​​ + ​​ __
6  ​​
= 1

Step 3: Solve the equation.


x __ x
​​ __ 4  ​​ + ​​  6  ​​ = 1
​  6  ​)​​  = 12(1)
x x
12​​ (__ ​  4  ​ + __

​  4  ​)​​  + 12​​ (__ ​  6  ​)​​  = 12


x x
12​​ (__
3x + 2x = 12
5x = 12
5x ___ 12
​​ ___ 5 ​​ = ​​  5 ​​ 
12 ​​ 
x = ​​ ___
5
2 ​​ 
= 2​​ __
5
2
Hence, Peter and his helper will take 2​​ __
5 ​​  days to finish the job.

141

Pre-Calculus and Calculus.indb 141 13/7/2017 5:57:03 PM


Chapter 4 • Equations in One Variable

Exercise 4.7
1. The sum of three consecutive integers is 105. Find the three integers.
1 ​​  of all the students take Mathematics and __
2. In a certain private school, 20 more than ​​ __ ​​  14 ​​  of those who
2
1 ​​ of all the students take Science, how many students are there
take Mathematics also take Science. If ​​ __
6
in the school?

3. A 40-year-old mother has a 10-year-old daughter. In how many years will the mother be twice as old
as her daughter?

4. A
 student travels from the school to his house at 35 km/h. It takes him 30 minutes longer than his
return trip from his house to the school at 42 km/h. Find the distance between the school and his
house.

5. A
 student can finish a Mathematics assignment in 3 hours without the use of a calculator. However,
his friend can finish the same assignment in 2 hours with the use of a calculator. How many hours
will it take if they work on the same assignment together?

6. How many kilograms of rice worth $30 per kilogram should be mixed with 80 kilograms of rice
worth $40 per kilogram to get a mix to sell at $35 per kilogram?

7. Leon gets test scores of 82, 85, 88 and 90 in four examinations. If the final examination determines ​​ __ 1 ​​ 
3
of the course grade and the other examinations determine the other __ ​​ 23 ​​ , what must he get in the final
examination to get an average score of 90 for the course?

8. T
 he width of a rectangle is 8 m shorter than its length. If the perimeter of the rectangle is 44 m, find
its dimensions.

9. C
 hristine is 9 years older than Angel. Six years from now, Christine’s age will be thrice the age of
Angel 3 years ago. Find their present ages.

10. 7 multiply by a number gives 30 more than twice the number. What is the number?

​​ 13 ​​  of a number, the result is 15. What is the number?


11. If 9 is added to __

12. Two cars, 288 km apart, start at the same time moving towards each other and meet after 12 h. If the
speed of one car is 5 km/h faster than that of the other car, find the speed of each car.

13. Kate is 9 years younger than Angel. Four years ago, Angel’s age was four times that of Kate’s age.
What are their ages?

14. A car traveling at 70 km/h left City A at 8 am. Another car traveling at 80 km/h left 2 h later, and soon
passed the first car. At what time did the second car pass the first?

15. Two students decide to jog towards each other at speeds of 2 km/h and 4 km/h. They are 30 km apart.
In how many hours will they meet?
142

Pre-Calculus and Calculus.indb 142 13/7/2017 5:57:03 PM


Chapter 4 • Equations in One Variable

16. Two college students working together can complete a task in 6 h. How long will it take one student
to do the task alone if the other student can do the job in 10 h?

17. How many liters of 30% alcohol solution must be added to 40 liters of 10% alcohol solution to
produce a mixture of 25% alcohol?

18. Chris is thrice as old as David 8 years from now. Find their present ages if Chris is four times as old
as David.

19. Find four consecutive odd integers whose sum is 216.

20. Ferdie has $70 000, part of which he invests at 9% simple interest and the rest at 14% simple interest.
If he obtains $9500 annually from these two investments, how much does he invest at each rate?

21. An improvised rocket is launched at 14.7 meters per second from the ground. The equation for the
height h of the rocket at time t seconds after launch is h(t) = 4.9t2 + 14.7t, where h is in meters. When
does the rocket strike the ground?

22. A rectangular cardboard has an area of 105 cm2. Its length is 8 cm more than its width. Find the
dimensions of the cardboard.

23. Concentric circles are circles with a common center. The area of a circle is equal to πr2, where r
represents the radius of the circle. The diagram below shows two concentric circles. The inner circle
has a radius of x cm. The shaded region has a width that is 2 cm less than the radius of the inner circle.
The shaded region has an area of 39π cm2.

(a) Form an equation in x and show that it reduces to 3x2 – 8x – 35 = 0.


(b) Find the area of the inner circle in terms of π.

24. While Elizabeth is reading her history book, she notices that the product of the facing pages of her
book is 20 306. Find the page numbers.

25. The width and the length of a rectangle are (3x – 1) cm and (4x + 5) cm respectively. Given that the
area of the rectangle is 65 cm2, form an equation in x and find the perimeter of the rectangle.

26. Kelvin’s mother gives him a certain number of $1 coins every year for his coin box. The number of
coins that she gives is according to Kelvin’s age. At present, he has $465 altogether. How old is he
now if his mother started giving coins for his coin box when he was one year old?

143

Pre-Calculus and Calculus.indb 143 13/7/2017 5:57:03 PM


Equations and Inequalities
Chapter 5:
Two-variable Linear
Equations

Investments

Investors want to invest their money wisely. Some investors are torn between low risk
and high risk investments. Low risk investments offer stability and security but usually
pay the lowest interest. High risk investments offer a higher interest but are very unstable
and unpredictable. Some investors choose both investments. They put part of their money
on a high risk investment and the rest on a low risk investment to make sure that one part
remains safe. Questions of this type, where you will be asked how much should be placed
on each investment, involve systems of linear equations.

Pre-Calculus and Calculus.indb 144 13/7/2017 5:57:03 PM


Chapter 5 • Two-variable Linear Equations

5 T wo-variable Linear Equations


5.1 Graphs of Linear Equations
5.2 Linear Systems with Two Variables
5.3 Graphical Method
5.4 Substitution Method
5.5 Elimination Method
5.6 Equalization Method
5.7 Applications of Systems of Equations

5.1 Graphs of Linear Equations


Consider the linear equation y = 50x. Notice that the variable x can take
any real number as its value. Hence, there are infinitely many ordered
pairs that can satisfy the linear equation.
Since a linear equation is a relation, it can be presented as a graph. A graph Note
offers a convenient way of representing an infinite number of ordered Take note that the graph of
pairs that can satisfy a linear equation. a linear equation is a straight
line.
Summary

The following steps show the process of sketching the graph of a linear
function.
Step 1: Find at least two ordered pairs that can satisfy the linear
equation.
Step 2: Plot the points that correspond to the ordered pairs on the
rectangular coordinate plane.
Step 3: Draw a straight line that passes through these points.

Example 1
Draw the graph of y = 3x + 6.
SOLUTION
First, find at least two ordered pairs that satisfy the linear equation.
When x = –2, y =  3(–2) + 6
= –6 + 6
=0
When x = 0, y =  3(0) + 6
=0+6
=6
When x = 1, y =  3(1) + 6
=3+6
=9

145

Pre-Calculus and Calculus.indb 145 13/7/2017 5:57:03 PM


Chapter 5 • Two-variable Linear Equations

Table of values:
x –2 0 1
y 0 6 9
Next, plot the points (−2, 0), (0, 6), and (1, 9) on a rectangular coordinate
plane. Finally, draw a straight line that passes through these points.
y
12 y = 3x + 6

8 (1, 9)

(0, 6)
4
(–2, 0) x
–6 –4 –2 0 2 4 6
–4

–8

–12

The straight line formed is the graph of the equation y = 3x + 6.

Example 2
Draw the graph of y = –2x + 4.
SOLUTION
Find two ordered pairs that can satisfy the linear equation.
When x = 0, y =
 –2(0) + 4
=0+4
=4
When x = 2, y =
 –2(2) + 4
= –4 + 4
=0
Table of values:
x 0 2
y 4 0

146

Pre-Calculus and Calculus.indb 146 13/7/2017 5:57:03 PM


Chapter 5 • Two-variable Linear Equations

Plot the points on a rectangular coordinate plane, then draw a straight


line that passes through these points.
y
6
(0, 4)
4

2
(2, 0) x
–6 –4 –2 0 2 4 6
–2

–4
y = –2x + 4
–6

x-intercept and y-intercept of a Line


The graph of a linear equation intersects either the x-axis or y-axis or both.
The two points where the graph of a linear equation intersects the x-axis
and the y-axis are the easiest to plot.
The x-intercept is the x-coordinate of a point where the graph intersects
the x-axis.
The y-intercept is the y-coordinate of a point where the graph intersects
the y-axis.
To obtain the x-intercept of the equation, let y = 0 and find the value of
x in the given linear equation. To obtain the y-intercept of the equation, let
x = 0 and find the value of y in the given linear equation.

Example 3
Find the x and y-intercepts of 3x + 4y = 12 and draw its graph.
SOLUTION
To find the x-intercept, substitute y = 0 into the equation and find the
value of x.
3x + 4y = 12
3x + 4(0) = 12
3x + 0 = 12
x = 4
To find the y-intercept, substitute x = 0 into the equation and find the
value of y.
3x + 4y = 12
3(0) + 4y = 12
0 + 4y = 12
y = 3
The x-intercept is 4. Therefore, the graph intersects the x-axis at (4, 0).
The y-intercept is 3. Therefore, the graph intersects the y-axis at (0, 3).
Plot the two points and draw a straight line passing through these points.
147

Pre-Calculus and Calculus.indb 147 13/7/2017 5:57:04 PM


Chapter 5 • Two-variable Linear Equations

y
3x + 4y = 12 6

4
(0, 3)
2
(4, 0) x
–6 –4 –2 0 2 4 6
–2

–4

–6

Example 4
1  ​​x + 3.
Draw the graph of y = –​​ __
2
SOLUTION
Find the x-intercept. Substitute y = 0 and find the value of x in the given
equation.
y = –​​ __ 1  ​​x + 3
2
0 = –​​  21 ​​ x + 3
__

x = 6
Find the y-intercept. Substitute x = 0 and find the value of y in the given
equation.
y = –​​ __ 1
2 ​​ x + 3
1 ​​ (0) + 3
y = –​​ __
2
y = 3
Hence, the x-intercept is 6 and the y-intercept is 3.
Plot the points and draw a straight line passing through these points.
y
6
1  ​​x + 3
y = –​​ __ 4
2 (0, 3)
2
(6, 0)
x
–6 –4 –2 0 2 4 6
–2

–4

–6

148

Pre-Calculus and Calculus.indb 148 13/7/2017 5:57:05 PM


Chapter 5 • Two-variable Linear Equations

Gradient of a Line
The gradient of a line is the steepness of the slope of the line.
The gradient of line is defined as
rise
gradient = ​​ ____
run ​​ 
vertical change
= _______________
  
​​     ​​
horizontal change
change in y
= __________
​​    
​​
change in x
y –y
= ______
​​  x2 – x1  ​​
2 1

run = x2 – x1
(x2, y2)
rise = x2 – x1

(x1, y1) x

Definition 5.1

The gradient m of a line that passes through points P (x1, y1) and
Q (x2, y2) is y2 – y1
______
m = ​ ​x – x    ​​  ,
2 1
where x2 ≠ x1.

The graph of a line can be described given its gradient.


(i) If the gradient of a line is positive, then the graph of the line rises
from left to right.
y
The value of x
increases.

The value
of y increases.

149

Pre-Calculus and Calculus.indb 149 13/7/2017 5:57:05 PM


Chapter 5 • Two-variable Linear Equations

(ii) If the gradient of a line is negative, then the graph of the line falls
from left to right.
y

The value of x
decreases.

The value of y
increases. x

(iii) If the gradient of a line is zero, then the graph of the line is a horizontal
line.
y

The value of y
is constant.

(iv) If the gradient of a line is undefined, then the graph of a line is a


vertical line.
y

The value of x
is constant.

150

Pre-Calculus and Calculus.indb 150 13/7/2017 5:57:05 PM


Chapter 5 • Two-variable Linear Equations

Example 5
Find the gradient of the line passing through the given points. Then,
draw its graph.
(a) (–2, 3) and (3, –1)
(b) (4, –1) and (3, 5)
(c) (–5, –2) and (1, 4)
(d) (0, 1) and (4, 2)
SOLUTION
(a) Let (x1, y1) = (–2, 3) and (x2, y2) = (3, –1).
y – y _______ –1 – 3 __
m = ______
​​  x2 – x1 ​​ 
= ​​    = ​​  –4 ​​ = –​​ __
 ​​  4  ​​
2 1 3 – (–2) 5 5
To draw the graph, plot the two points first, then draw a line passing
through these points.
y

2
x
–4 –2 0 2 4
–2

–4

(b) Let (x1, y1) = (4, –1) and (x2, y2) = (3, 5).
y – y _______ 5 – (–1) __ 6
m = ______
​​  x2 – x1 ​​ 
= ​​  3 – 4 ​​ 

= ​​  –1  ​​ = –6
2 1

2
x
–4 –2 0 2 4
–2

–4

151

Pre-Calculus and Calculus.indb 151 13/7/2017 5:57:05 PM


Chapter 5 • Two-variable Linear Equations

(c) Let (x1, y1) = (–5, –2) and (x2, y2) = (1, 4).
y – y _______ 4 – (–2) __ 6
m = ______
​​  x2 – x1 ​​ 
= ​​   ​​ 
= ​​   ​​  = 1
2 1 1 – (–5) 6
y

2
x
–4 –2 0 2 4
–2

–4

(d) Let (x1, y1) = (0, 1) and (x2, y2) = (4, 2).
y2 – y1 _______
m = ​​ ______ = ​​  42 – 1
x2 – x1 ​​ 
__ 1
– 0 ​​ = ​​  4 ​​ 
y
4

2
x
–4 –2 0 2 4
–2

–4

Exercise 5.1
1. Determine the gradient of the line in each graph.
(a) y (b) y

4 4

2 2
x x
–4 –2 0 2 4 –4 –2 0 2 4
–2 –2

–4 –4

152

Pre-Calculus and Calculus.indb 152 13/7/2017 5:57:06 PM


Chapter 5 • Two-variable Linear Equations

(c) y (e) y

4 4

2 2
x x
–4 –2 0 2 4 –4 –2 0 2 4
–2 –2

–4 –4

(d) y (f) y

4 4

2 2
x x
–4 –2 0 2 4 –4 –2 0 2 4
–2 –2

–4 –4

2. Draw the graphs of the following functions.


(a) x = –7 (d) 3x + 15y = 30
x
(b) y = 8 (e) y = –​​ __
3  ​​ + 15
(c) 2x – y = 10 1 ​​ x + __
(f) ​​ __ ​​  12 ​​ y = 8
4

153

Pre-Calculus and Calculus.indb 153 13/7/2017 5:57:06 PM


Chapter 5 • Two-variable Linear Equations

5.2 Linear Systems with Two Variables


A linear equation in two variables x and y is of the form ax + by = c, where
a, b and c are constants and both a and b are not zero. Thus, a system of
two linear equations with variables x and y is of the form
ax + by = c
dx + ey = f
where a, b, c, d, e and f are constants. A solution of the system is a pair
of values for x and y, usually written (x, y) which satisfies both equations.
This solution is also called a simultaneous solution of the given equations.

y
The graph of each equation in the system is a line. Therefore, we have a
pair of lines which is either (i) intersecting, (ii) parallel or (iii) coincident
(that is, identical).
(i) If the lines intersect, the point of intersection (x, y) is the solution
x
0 of the system. The system is said to be consistent. In this consistent
a b
system, we find that __​​   ​​≠ __
​​   ​​.
(i) I ntersecting lines, system has
only one solution.
d e
y
(ii) If the lines are parallel, the system of equations has no solutions.
a b __c  
The system is said to be inconsistent. In this case, __ ​​    ​​ = __
​​   ​​≠ ​​   ​​ .
d e f
If the lines are coincident, the equations are called dependent
(iii) 
x equations. The two equations are actually the same line and every
0
point on the line represents a solution. Since there are infinite numbers
(ii) P
 arallel lines, system has no of points on the line, the number of solutions is infinite. The system is
solution.
a b __c  
said to be consistent and in this case, __
​​   ​​ = __
​​   ​​ = ​​   ​​ .
y d e f

Example 6
Without solving, determine whether the given system consists of
x
0 intersecting lines, parallel lines or coincident lines.
(iii) C
 oincident lines, system has
(a) 2x + 3y = 12 (c) 2x – y – 3 = 0
infinite number of solutions. x – 3y = –3 6x – 3y – 9 = 0
(b) 4x + 2y = 3
2x + y = 1

SOLUTION
(a) For the given system, a = 2, b = 3, c = 12, d = 1, e = –3 and f = –3.
a 2 __ b
We find that __ ​​   ​​ = __
​​   ​​  ≠ ​​   ​​ , therefore the system contains intersecting
d 1 e
lines.
a 4 __ b c
(b) ​​ __ ​​ = __
​​   ​​  = ​​   ​​≠ __
​​   ​​  , therefore the system contains parallel lines.
d 2 e f
a 2 __ b c
(c) ​​ __ ​​ = __
​​   ​​  = ​​   ​​ = __
​​   ​​  , therefore the system contains coincident lines.
d 6 e f

154

Pre-Calculus and Calculus.indb 154 13/7/2017 5:57:07 PM


Chapter 5 • Two-variable Linear Equations

5.3 Graphical Method


The solution of a system of linear equations can be found by graphing both
the equations in the same coordinate plane. The point common to both
lines is a solution of the system, and the coordinates of this point satisfy
both linear equations.

Example 7
Solve the system of linear equations by graphical method.
x – 2y = –2
3x – y = 4

SOLUTION
Write the given equations in the form y = mx + c, where m is the gradient
and c is the y-intercept. Then, draw the graph of each equation.
__ 1
x – 2y = –2  y = ​ ​2 ​​x + 1
3x – y = 4  y = 3x – 4
The graph of the first equation is a line with y-intercept = 1 and
__ 1
gradient = ​​  2  ​​. The graph of the second equation is a line with
y-intercept = –4 and gradient = 3.
y
6 3x – y = 4

4 x – 2y = –2

2
x
–6 –4 –2 0 2 4
–2

–4

–6

The two lines intersect at exactly one point. The point of intersection
is (2, 2). Hence, the system is consistent and independent, and its
solution is (x, y) = (2, 2).

155

Pre-Calculus and Calculus.indb 155 13/7/2017 5:57:07 PM


Chapter 5 • Two-variable Linear Equations

Example 8
Solve the system of linear equations by graphical method.
5x + 2y = 10
5x + 2y = 6

SOLUTION
Write the given equations in the form y = mx + c and draw the graph of
both equations.
__ 5
5x + 2y = 10  y = –​​  2 ​​x + 5
__ 5
5x + 2y = 6  y = –​​  2 ​​x + 3
The graph of the first equation is a line with y-intercept = 5 and
5
gradient = –​​ __
2 ​​ . The graph of the second equation is a line with
5
y-intercept = 3 and gradient = –​​ __ 2 ​​ .
y
6
5x + 2y = 6
4
5x + 2y = 10

2
x
–6 –4 –2 0 2 4
–2

–4

–6

Note that the lines are parallel since they have the same gradient but
different y-intercepts. Hence, there is no common point and the system
is inconsistent.

156

Pre-Calculus and Calculus.indb 156 13/7/2017 5:57:07 PM


Chapter 5 • Two-variable Linear Equations

Example 9
Solve the system of linear equations by graphical method.
6x + y = 5
12x + 2y = 10

SOLUTION
Write the given equations in the form y = mx + c and draw the graph of
both equations.
6x + y = 5  y = –6x + 5
12x + 2y = 10  y = –6x + 5
Note that each equation can be written as y = –6x + 5. This means that
the two lines represent the same line. Hence, the system is consistent
and dependent.
y
6
6x + y = 5

2 12x + 2y = 10

x
–6 –4 –2 0 2 4
–2

–4

–6

Exercise 5.3
1. Solve the system of linear equations by graphical method.
(a) 4x – y = 5 (f) 2x – 5y = 4
x + 2y = 8 4x – 10y = 0
(b) x – 3y = 0 (g) 3x – y = 5
2x – y = 0 9x – 3y = 15
​​  13 ​​ x – 5
(c) y = __ (h) –x + y = 7
y = –5 x–y=1
​​  12 ​​ x + 2
(d) y = __ (i) 2x + y = 3
y = 4 y = –2x + 7
(e) x + y = 4 (j) 14x – 2y = 2
2x + 2y = 8 y = 7x + 1

157

Pre-Calculus and Calculus.indb 157 13/7/2017 5:57:07 PM


Chapter 5 • Two-variable Linear Equations

5.4 Substitution Method


The substitution method involves translating the system into an equation
that has only one variable.

The following steps show the process of finding the solution of a linear
Note system by the substitution method.
If the given equations involve
Step 1: Select an equation and express one variable in terms of the other.
a variable with coefficient
1 or −1, express that variable Step 2: Substitute the expression derived in Step 1 into the other equation.
in Step 1 to avoid working The result is an equation that involves only one variable.
with fractions. Step 3: Solve the equation obtained in Step 2.
Step 4: Substitute the value obtained in Step 3 into any of the two
equations in the given system and find the value of the other
variable.

Example 10
Find the solution set of the following system of linear equations by the
substitution method.
6x + 2y = 5
3x + y = 2

SOLUTION
Work on the second equation since the coefficient of y is 1.
Express y in terms of x.
3x + y = 2
y = 2 – 3x
Substitute y = 2 – 3x into the first equation.
6x + 2y = 5
6x + 2(2 – 3x) = 5
6x + 4 – 6x = 5
6x + 4 – 6x – 4 = 5 – 4
0=1
The resulting equation, 0 = 1, is a false statement. There are no values
of x and y for which 0 = 1. Thus, the given system of equations has no
solution. The graph of this system is a pair of parallel lines.
The solution set of the given system of equations is an empty set { },
and the system is inconsistent.

158

Pre-Calculus and Calculus.indb 158 13/7/2017 5:57:07 PM


Chapter 5 • Two-variable Linear Equations

Example 11
Solve the following system of linear equations by the substitution
method.
5x – y = 6
10x – 2y = 12
SOLUTION
Work on the first equation since the coefficient of y is –1.
Express y in terms of x.
5x – y = 6
–y = 6 – 5x
y = –6 + 5x
Substitute y = –6 + 5x into the second equation.
10x – 2y = 12
10x – 2(–6 + 5x) = 12
10x + 12 – 10x = 12
12 = 12
The last equation, 12 = 12, is an identity. This means that the given
system of equations has infinitely many solutions. The solution set is
the set of values (x, y) that satisfy the equation y = –6 + 5x.
The solution set is given by {(x, y) | y = –6 + 5x}. Thus, the system is
consistent and dependent.

Example 12
Solve the following system of linear equations by the substitution
method.
x + 2y = 5
2x + 5y = 12

SOLUTION
Note that the first equation has a variable with coefficient 1. Work on
this equation and express x in terms of y.
x + 2y = 5
x = 5 – 2y
Substitute x = 5 – 2y into the second equation.
2x + 5y = 12
2(5 – 2y) + 5y = 12
10 – 4y + 5y = 12
10 + y = 12
y = 2
Find the value of x by substituting y = 2 into the first equation.
x + 2y = 5
x + 2(2) = 5
x+4=5
x=1
Hence, the solution is (x, y) = (1, 2) and the system is consistent and
independent.
159

Pre-Calculus and Calculus.indb 159 13/7/2017 5:57:07 PM


Chapter 5 • Two-variable Linear Equations

Exercise 5.4
1. Solve the following systems of linear equations by the substitution method.
(a) 10x + y = –5
4x + 3y = 11
(b) 2x – 7y = 8
x + 2y = –7
(c) 5x – 2y = 2
10x – 4y = 4
(d) 3x + 2y = 10
–3x – 2y = 2
(e) 9x + 2y = 10
y = 6x – 2
(f) 10x – 3y = 11
y = 5x – 4

2. Solve the following systems of linear equations by the substitution method.


(a) 3x + 2y = 18
3x – 8y = –12
(b) 4x + y – 2 = 0
35
3x – 4y + ___​​  2 ​​ = 0

(c) 4t – s = 9
5t – s = 0
(d) 5t – 3s – 3 = 0
5t + 15s – 21 = 0
(e) 2u + 3v = 12
5u – v = 13

160

Pre-Calculus and Calculus.indb 160 13/7/2017 5:57:07 PM


Chapter 5 • Two-variable Linear Equations

5.5 Elimination Method


This method uses the fact that two systems of equations are equivalent if
they have the same solutions. In this method, one variable is eliminated so
that we can get the value of the other variable. This is done by transforming
the given system into an equivalent system so that one of the variables can
be easily eliminated.
Given a system of equations, an equivalent system of equations results if
(i) two equations are interchanged,
(ii) an equation is multiplied or divided by a non-zero constant,
(iii) a constant multiple of one equation is added to another equation.

The following steps show the process of finding the solution of a linear
system by the elimination method.
Step 1: Arrange the terms of both equations in this format:
ax + by = c
dx + ey = f
where a, b, c, d, e and f are constants.
Step 2: If needed, multiply either one equation or both equations by an
appropriate non-zero number so that the sum of the coefficients
of x or y is 0.
Step 3: Add the equations in Step 2. This will result in an equation in one
variable.
Step 4: Solve the equation obtained in Step 3.
Step 5: Substitute the value found in Step 4 into any of the given
equations and find the value of the other variable.

Example 13
Solve the system of linear equations by the elimination method.
5x + 2y = 12
2x – 3y = 1

SOLUTION
5x + 2y = 12 ——— (1)
2x – 3y = 1 ——— (2)
Choose a variable to eliminate. Suppose we eliminate the variable x. To
eliminate x , multiply equation (1) by 2 and equation (2) by –5.
2(5x + 2y) = 2(12)
10x + 4y = 24 ——— (3)
–5(2x – 3y) = –5(1)
–10x + 15y = –5 ——— (4)
Add equations (3) and (4).
10x + 4y = 24
+ –10x + 15y = –5
0 + 19y = 19
y=1
161

Pre-Calculus and Calculus.indb 161 13/7/2017 5:57:07 PM


Chapter 5 • Two-variable Linear Equations

Substitute y = 1 into equation (1) and find the value of x.


5x + 2y = 12
5x + 2(1) = 12
5x + 2 = 12
5x = 10
x = 2
Hence, the solution is (x, y) = (2, 1), and the system is consistent and
independent.

Example 14
Solve the system of linear equations by the elimination method.
3x + 4y = 2
7x – 6y = –3

SOLUTION
3x + 4y = 2 ——— (1)
7x – 6y = –3 ——— (2)
Suppose we eliminate the variable y. To eliminate y, multiply equation
(1) by 3 and equation (2) by 2.
3(3x + 4y) = 3(2)
9x + 12y = 6 ——— (3)
2(7x – 6y) = 2(–3)
14x – 12y = –6 ——— (4)
Add equations (3) and (4).
9x + 12y = 6
+ 14x – 12y = –6
23x + 0 = 0
x = 0
Substitute x = 0 into equation (1) and find the value of y.
3x + 4y = 2
3(0) + 4y = 2
0 + 4y = 2
4y = 2
y = ​​ __ 2  ​​
4
y = ​​ __ 1 ​​ 
2

2 ​ )​​ and the system is consistent and


Hence, the solution is (x, y) = (​​  0, ​ __
1
independent.

162

Pre-Calculus and Calculus.indb 162 13/7/2017 5:57:07 PM


Chapter 5 • Two-variable Linear Equations

Example 15
Solve the following system of linear equations by the elimination
method.
3x + y = 3
6x + 2y = 6

SOLUTION
3x + y = 3 ——— (1)
6x + 2y = 6 ——— (2)
Suppose we eliminate the variable x. To eliminate x, multiply equation
(1) by –2.
–2(3x + y) = (–2)(3)
–6x – 2y = –6 ——— (3)
Add equations (3) and (2).
–6x – 2y = –6
+ 6x + 2y = 6
0=0
Note that 0 = 0 is an identity equation. This means that the given
system of linear equations has infinitely many solutions. Therefore, the
solution set is given by {(x, y) | 3x + y = 3}, and the system is consistent
and dependent.

Exercise 5.5
1. Solve the following systems of linear equations by the elimination method.
(a) 2x – 3y = –15 (d) 6x + 9y = 7
x + 2y = 10 3x – 3y = 1
(b) 3x – 5y = –19 (e) 10x – 2y = 8
2x + 3y = 0 5x – y = 4
(c) 5x + 4y = –7 (f) 6x – 6y = 5
10x + 3y = 1 3x – 3y = 1

2. Solve the following systems of linear equations by the elimination method.


(a) 2x – 9y = –30 (d) –5s + 3t + 10 = 0
9x + 4y = 132 4s + 2t – __ ​​  23 ​​  = 0

(b) 3x + 2z – 2 = 0 (e) 2y – 3z = 0
x + 3z – 2 = 0 3y – 4z = 1
(c) 2u + v = 3
2u – v = –1

163

Pre-Calculus and Calculus.indb 163 13/7/2017 5:57:07 PM


Chapter 5 • Two-variable Linear Equations

5.6 Equalization Method


The equalization method involves insolating the same unknown in the
system of equations and then equating the expressions.

Example 16
Solve the system of linear equations by equalization method.
x + 2y = 1
3x + 7y = 2
SOLUTION
x + 2y = 1 ——— (1)
3x + 7y = 2 ——— (2)
Express x in terms of y for both equations.
From equation (1),
x = 1 – 2y ——— (3)
Form equation (2),
3x = 2 – 7y
2 – 7y
x = ​​ ______
3 ​​  
  ——— (4)

Equating the right side of equations (3) and (4).


2 – 7y
1 – 2y = ______
​​  3 ​​   

3 – 6y = 2 – 7y
y = –1
Substitute y = –1 into equation (3) to find the value of x.
x = 1 – 2(–1)
=3
Hence, the solution is (x, y) = (3, –1) and the system is consistent and
independent.

164

Pre-Calculus and Calculus.indb 164 13/7/2017 5:57:07 PM


Chapter 5 • Two-variable Linear Equations

Example 17
Solve the system of linear equations by equalization method.
2x + 5y = 29
5x – 2y = 29

SOLUTION
2x + 5y = 29 ——— (1)
5x – 2y = 29 ——— (2)
Express 2x in terms of y for both equations.
From equation (1),
2x = 29 – 5y ——— (3)
From equation (2),
5x = 29 + 2y
2x = ​​ __ 2
5 ​​ (29 + 2y)
58 + 4y
2x = ​​ _______ 5 ​​  
  ——— (4)
Equating the right side of equations (3) and (4).
58 + 4y
29 – 5y = ​​ _______5 ​​   

145 – 25y = 58 + 4y
87 = 29y
y = 3
Substitute y = 3 into equation (3) to find the value of x.
2x = 29 – 5(3)
x=7
Hence, the solution is (x, y) = (7, 3) and the system is consistent and
independent.

Exercise 5.6
1. Solve the following systems of linear equations by the elimination method.
(a) 5x + y = 8 (d) 4m – n – 7 = 0
3x + y = 6 3m + 2n – 8 = 0
(b) 2p – 5q = 9 (e) 7a + 3b = 12
2p + 3q = 7 4a – 2b = 5
(c) 3s – 2t – 5 = 0 (f) 3x – 4y – 1 = 0
2t – 5s – 9 = 0 2x – 5y + 4 = 0

165

Pre-Calculus and Calculus.indb 165 13/7/2017 5:57:07 PM


Chapter 5 • Two-variable Linear Equations

5.7 Applications of Systems of Equations


5.7.1 Number and Geometry Problems
5.7.2 Investment Problems
5.7.3 Mixture Problems
5.7.4 Uniform Motion and Work Problems

5.7.1 Number and Geometry Problems


Example 18
The sum of two numbers is 10. If one number is subtracted from the
other, the difference would be 2. Find the two numbers.

SOLUTION
Step 1: Define variables.
Let x be the first number and y be the second number.
Step 2: Translate into a system of linear equations.
x + y = 10 ——— (1)
x – y = 2 ——— (2)
Step 3: Solve the system of linear equations.
Using the elimination method,
x + y = 10
+ x – y = 2
2x = 12
x=6
Substitute x = 6 into equation (1) to find the value of y.
x + y = 10
6 + y = 10
y=4
Hence, the two numbers are 4 and 6.

Example 19
The ones digit of a two-digit number is 2 more than twice its tens digit.
If the digits are reversed, the new number would be 7 more than twice
the original number. Find the original number.

SOLUTION
Step 1: Define variables.
Let x be the tens digit and y be the ones digit of the original
number. Thus, the original number is 10x + y. The new number
is 10y + x.
Step 2: Translate into a system of linear equations.
y = 2 + 2x ——— (1)
10y + x = 7 + 2(10x + y) ——— (2)
Simplify equation (2) first.
10y + x = 7 + 2(10x + y)
10y + x = 7 + 20x + 2y
10y – 2y + x – 20x = 7
8y – 19x = 7 ——— (3)
166

Pre-Calculus and Calculus.indb 166 13/7/2017 5:57:07 PM


Chapter 5 • Two-variable Linear Equations

Step 3: Solve the system of linear equations.


Substitute equation (1) into equation (3) to find the value of x.
8y – 19x = 7
8(2 + 2x) – 19x = 7
16 + 16x – 19x = 7
16 – 3x = 7
–3x = 7 – 16
–3x = –9
x=3
Substitute x = 3 into equation (1) to find the value of y.
y= 2 + 2x
= 2 + 2(3)
=2+6
=8
Hence, the original number = 10(3) + 8
= 38

Example 20
The length of a rectangular lot is 10 m shorter than thrice its width. Its
perimeter is 60 m. Find the dimensions of the rectangular lot.

SOLUTION
Step 1: Define variables.
Let x be the width of the rectangle and y be its length.
Step 2: Translate into a system of linear equations.
y = 3x – 10 ——— (1)
2x + 2y = 60 ——— (2)
Step 3: Solve the system of linear equations.
Substitute equation (1) into equation (2) to find the value of x.
2x + 2y = 60
2x + 2(3x – 10) = 60
2x + 6x – 20 = 60
2x + 6x = 60 + 20
8x = 80
x = 10
Substitute x = 10 into equation (1) to find the value of y.
y=
 3x – 10
= 3(10) – 10
= 30 – 10
= 20
Hence, the length of the rectangle is 20 m and its width is 10 m.

167

Pre-Calculus and Calculus.indb 167 13/7/2017 5:57:07 PM


Chapter 5 • Two-variable Linear Equations

5.7.2 Investment Problems


Problems involving investments can be solved using systems of linear
equations. Given the principal amount of investment P, the annual rate
r and the time t in years, the simple interest I is given by
I = Prt

Example 21
Mrs. Mary invested $80 000 in two different banks. The first bank
offered 4% annual simple interest, while the other offered 6% annual
simple interest. After one year, the total interest earned from those two
investments amounted to $4400. How much was invested at each rate?

SOLUTION
Step 1: Define variables.
Let x = amount that Mrs. Mary invested at 4% annual simple
interest and
y = a mount that she invested at 6% annual simple interest.
Step 2: Translate into a system of linear equations.
The sum of the two investments is $80 000.
Thus, x + y = 80 000.
The interest earned from the two banks can be solved using the
formula I = Prt.
The interest from the first bank in one year, which is 4% annual
simple interest, is
I = Prt = x(0.04)(1) = 0.04x
The interest from the second bank in one year, which is 6%
annual simple interest, is
I = Prt = y(0.06)(1) = 0.06y
The total interest from the two banks is
0.04x + 0.06y = 4400
Step 3: Solve the system of linear equations.
x + y = 80 000 ——— (1)
0.04x + 0.06y = 4400 ——— (2)
We use the elimination method. To eliminate x, multiply equation
(1) by –0.04.
–0.04(x + y) = (–0.04)(80 000)
–0.04x – 0.04y = –3200 ——— (3)
Add equations (2) and (3).
0.04x + 0.06y = 4400
+ –0.04x – 0.04y = –3200
0.02y = 1200
1200
y = _____
​​  0.02 ​​ 
y = 60 000

168

Pre-Calculus and Calculus.indb 168 13/7/2017 5:57:07 PM


Chapter 5 • Two-variable Linear Equations

Substitute y = 60 000 into equation (1) to find the value of x.


x + y = 80 000
x + 60 000 = 80 000
x = 80 000 – 60 000
x = 20 000
Hence, the amount invested at 4% annual simple interest was
$20 000, while the amount invested at 6% annual simple interest
was $60 000.

5.7.3 Mixture Problems


Example 22
In a chemistry experiment, Ruben must make 10 l of 48% acetic acid
solution. He has 30% acetic acid solution and 60% acetic acid solution.
How much of each solution does he need?

SOLUTION
Step 1: Define variables.
Let x = amount of 30% acetic acid solution (in l) and
y = amount of 60% acetic acid solution (in l).
Organize the given information in a table.
Amount of Percentage of Amount of
Type of Solution
Solution (l) Acetic Acid Acetic Acid (l)
30% solution x 30% = 0.30 0.30x
60% solution y 60% = 0.60 0.60y
Mixture
x + y = 10 48% = 0.48 0.48(10) = 4.8
(48% solution)

Step 2: Translate into a system of linear equations.


The sum of the amounts of solutions x and y is 10.
Thus, x + y = 10.
The sum of the amount of 30% acetic acid solution and the
amount of 60% acetic acid solution is equal to the amount of
the acetic acid solution of the desired mixture.
Thus, 0.03x + 0.06y = 4.8.
Step 3: Solve the system of linear equations.
x + y = 10 ——— (1)
0.03x + 0.60y = 4.8 ——— (2)
 We use the elimination method. To eliminate x, multiply
equation (1) by −0.30.
x + y = 10
–0.30(x + y) = (–0.30)(10)
–0.30x – 0.30y = –3 ——— (3)

169

Pre-Calculus and Calculus.indb 169 13/7/2017 5:57:07 PM


Chapter 5 • Two-variable Linear Equations

Add equations (2) and (3).


0.30x + 0.60y = 4.8
+ –0.30x – 0.30y = –3
0.30y = 1.8
1.8
y = _____
​​  0.30  ​​ 
y=6
Substitute y = 6 into equation (1) to find the value of x.
x + y = 10
x + 6 = 10
x=4
Hence, Ruben needs 4 l of 30% acetic acid solution and 6 l
of 60% acetic acid solution to make 10 l of 48% acetic acid
solution.

5.7.4 Uniform Motion and Work Problems


Example 23
Two buses left the same station and went in opposite directions. Bus
A was traveling 30 km/h faster than bus B. After two hours, the buses
were 340 km apart. Find the speeds of the two buses.

SOLUTION
Step 1: Define variables.
Let x be the speed of bus A and y be the speed of bus B.
Step 2: Translate into a system of linear equations.
Recall that the formula for uniform motion problems is d = st,
where d is the distance, s is the speed and t is the time.
Bus A was traveling 30 km/h faster than bus B.
Thus, x = y + 30.
After 2 hours, the buses were 340 km apart. This means that the
sum of the distances covered by the two buses after two hours
was 340 km.
Distance covered by bus A after 2 hours = 2x
Distance covered by bus B after 2 hours = 2y
Thus, 2x + 2y = 340.
Step 3: Solve the system of linear equations.
x = y + 30 ——— (1)
2x + 2y = 340 ——— (2)
We use the substitution method. Substitute equation (1) into
equation (2) to find the value of y.
2x + 2y = 340
2(y + 30) + 2y = 340
2y + 60 + 2y = 340
4y + 60 = 340
4y = 280
y = 70
170

Pre-Calculus and Calculus.indb 170 13/7/2017 5:57:07 PM


Chapter 5 • Two-variable Linear Equations

Substitute y = 70 into equation (1) to find the value of x.


x = y + 30
= 70 + 30
= 100
Hence, the speed of bus A was 100 km/h, while the speed of
bus B was 70 km/h.

Example 24
Jerald started painting a treehouse. After the fourth day, he fell sick, so
Jane had to continue his work. Jane finished it after 2 days. Suppose
2
Jerald and Jane could finish the work together in 2​​ __
3 ​​  days, how long
would each of them take to finish the work alone?

SOLUTION
Step 1: Define variables.
Let x = n umber of days Jerald can finish painting the treehouse
alone and
y = n umber of days Jane can finish painting the treehouse
alone.
The given information in the problem can be summarized in the table
below.
Number of Days to
Work Done in a Day
Finish the Job

Jerald x 1 ​​
​​  __
x

Jane y 1 ​​
​​  __
y

Jane and 2  ​​ 3 ​​ 


1   ​​ = ​​  __
2​​  __ ​​ ___
Jerald 3 2
__
2​    ​ 8
3
Step 2: Translate into a system of linear equations.
The sum of Jerald’s and Jane’s working rate is equal to the
work done in a day when they work together.
3
​​  1x ​​ + __
Thus, __ ​​  1y ​​ = __
​​  8 ​​   . ——— (1)
Moreover, Jerald worked for 4 days, and Jane continued and
finished the work in 2 days. Jerald then finished __ ​​ 4x ​​of the work,
while Jane finished __ ​​ 2y .​​
Thus, __ ​​  4x ​​ + __
​​  2y ​​= 1. ——— (2)

171

Pre-Calculus and Calculus.indb 171 13/7/2017 5:57:07 PM


Chapter 5 • Two-variable Linear Equations

Avoid working with variables in the denominator. Thus, we


​​  1x ​​ and b = __
let a = __ ​​  1y .​​ Hence, equations (1) and (2) become the
following system of linear equations.
3
a + b = __ ​​  8 ​​
  ——— (3)
4a + 2b = 1 ——— (4)
Step 3: Solve the system of linear equations.
We use the elimination method. Multiply equation (3) by –4.
(–4)(a + b) = (–4)​​ (__ ​  8 ​ )​​  
3

3
–4a – 4b = –​​ __ 2  ​​ ——— (5)
Add equations (4) and (5).
4a + 2b = 1
3
+ –4a – 4b = –​​ __ 2  ​​
–2b = –​​ __ 1 ​​ 
2

b = __ ​​  41 ​​ 

Substitute b = __ ​​  41 ​​  into equation (3) to find the value of a.


3
a + b = __ ​​  8 ​​ 
1 __ 3
a + ​​ __
4 ​​  = ​​  8 ​​ 
3 1
a = __ ​​  8 ​​  – __
​​  4 ​​ 

a = ​​ __ 1  ​​
8
Now, go back to the original variables and find x and y.
a = ​​ __ 1 ​​        b = __ ​​  1y ​​
x
​​ __ 1 __ 1 1 __
__ 1
8 ​​  = ​​  x ​​ ​​  4 ​​  = ​​  y ​​
x = 8 y=4
Hence, Jerald would finish painting the treehouse alone in 8
days, while Jane would finish the same painting alone in 4 days.

172

Pre-Calculus and Calculus.indb 172 13/7/2017 5:57:08 PM


Chapter 5 • Two-variable Linear Equations

Exercise 5.7
1. The sum of two numbers is 23. If one number is subtracted from the other number, their difference
would be 7. Find the numbers.

2. The sum of two numbers is 35. Twice the first number is 5 less than the second number. Find the
numbers.

3. The sum of the digits of a two-digit number is 11. The ones digit is 1 less than twice the tens digit.
Find the number.

4. The sum of the digits of a two-digit number is 13. The ones digit is 3 more than the tens digit. Find
the number.

5. The length of a rectangular picture frame is 5 cm longer than its width. Its perimeter is 50 cm. Find
the dimensions of the frame.

6. The length of a rectangular swimming pool is 4 m shorter than twice its width. Its perimeter is 40 m.
Find the dimensions of the swimming pool.

7. Larry’s age is 5 years less than thrice his daughter’s age. Five years ago, the sum of their ages was
49. Find Larry’s present age.

8. Alex is 5 years younger than his brother, Aaron. The sum of their ages is 27. Find their ages.

9. The sales of the school cafeteria for a particular day showed that twice the number of chicken
sandwiches sold exceeded the number of ham sandwiches sold by 20. The total number of sandwiches
sold for that day is 100. How many sandwiches of each type were sold?

10. A total of 270 students participated in a dance. The number of boys exceeded the number of girls by
30. How many boys and girls participated in the dance?

11. Roland inherited a certain amount of money from his grandfather. He invested part of it in a trust fund
earning 8% annual simple interest. The rest of the amount, which was twice the first investment, was
invested in a trust fund earning 10% annual simple interest. The total annual interest from the two
investments was $8400. How much was invested at each rate?

12. Mrs. Jones invested $50 000 in two different banks. The first bank offered 5% annual simple interest,
while the other bank gave 8% annual simple interest. The total annual interest earned from the two
investments was $3340. How much was invested at each rate?
6
13. Working together, Matthew and Eugene could finish cleaning the entire house in ​​ ___
5 ​​  hours. However,
Matthew worked alone for an hour last weekend. Then, Eugene continued the cleaning alone for
one and a half hours. Based on the given information, how long would each of them take to finish
cleaning the house alone?

173

Pre-Calculus and Calculus.indb 173 13/7/2017 5:57:08 PM


Chapter 5 • Two-variable Linear Equations

14. Working together, Ronie and Romeo could finish their art project in 2 days. However, Ronie did the
project alone on the first day. Then, Romeo continued working on it for the next 4 days. Based on the
information, how long would each of them take to finish the art project alone?

15. Daniel, a chemist, was asked to prepare 10 l of 40% ethyl alcohol solution by mixing a solution of
70% alcohol and a solution of 20% alcohol. How many liters of each solution should be used?

16. Anna was asked to prepare 20 l of 30% muriatic acid solution by mixing the following: 40% muriatic
acid solution and 20% muriatic acid solution. How many liters of each solution should she use?

17. Two buses left the terminal at the same time and traveled in opposite directions. Bus A was traveling
20 km/h faster than bus B. After two hours, both buses covered a total distance of 280 km. Find the
speeds of the two buses.

18. Two trains left the same station and traveled in opposite directions. The speed of train B was 20 km/h
less than twice the speed of train A. After an hour, the trains were 100 km apart. Find the speeds of
the two trains.

19. Tickets for the drama club presentation were sold at $100 for VIP and $80 for non-VIP. 200 tickets
were sold and a total sales of $16 800 were made. How many VIP and non-VIP tickets were sold?

20. John’s team made a total score of 49 in a basketball game. The number of two-point shots exceeded
thrice the number of three-point shots by 2. How many three-point shots and two-point shots were
made?

21. The sum of two numbers is 30. If thrice the smaller number is 10 less than twice the bigger number,
find the two numbers.

22. The sum of the digits of a two-digit number is 10. If the digits of the number are reversed, the new
number formed is 1 less than twice the original number. Find the number.

174

Pre-Calculus and Calculus.indb 174 13/7/2017 5:57:08 PM


Equations and Inequalities
Chapter 6:

Three Linear Equations


with Three Variables

Historical Note

Johann Carl Friedrich Gauss (1777–1855) was a great mathematician,


astronomer and physicist. At a young age, he already made a remarkable
impression when he added all the integers from 1 to 100 using a pattern.
He made significant contributions to the number theory. He was the one
who formulated the prime number theorem. His work on geometry led some
mathematicians after him to the discovery of hyperbolic geometry. Also,
he introduced another way of solving systems of linear equations as part
of his proof in one of his theorems. The method is known as the Gaussian
elimination.

Pre-Calculus and Calculus.indb 175 13/7/2017 5:57:08 PM


Chapter 6 • Three Linear Equations with Three Variables

Three Linear Equations with


6
Three Variables
6.1 Substitution Method
6.2 Elimination Method
6.3 Inverse Matrix Method
6.4 Gauss-Jordan Elimination Method
6.5 Applications

A system of three linear equations in three variables x, y and z is of the


form:
ax + by + cz = d
ex + fy + gz = h
ix + jy + kz = l, where a, b, c, ..., l are constants.
Each equation in the system represents a plane. If the system has two or
more parallel planes, the system is inconsistent and has no solution. If the
three planes coincide (that is, all three equations are identical), or all three
intersect at a common line, then the system is consistent and has infinite
number of solutions. If the three planes intersect at a single point (for
example, the point where a ceiling and two walls meet), then the system
is consistent and the point of intersection is the solution of the system. We
are going to discuss the method of finding the solution for the case where
the planes intersect at a single point.
We can solve the system with three linear equations with the following
methods:
(i) Substitution
(ii) Elimination
(iii) Inverse matrix
(iv) Gauss-Jordan elimination
Let us look at some examples.

176

Pre-Calculus and Calculus.indb 176 13/7/2017 5:57:08 PM


Chapter 6 • Three Linear Equations with Three Variables

6.1 Substitution Method


Example 1
Solve the following system of equations using the substitution method.
x + y – z = –4
2x + 5y – 4z = –16
x – 2y + 2z = 1

SOLUTION
x + y – z = –4 ——— (1)
2x + 5y – 4z = –16 ——— (2)
x – 2y + 2z = 1 ——— (3)
Express either of the given equations for one variable in terms of the
others.
From equation (1),
x + y – z = –4
x = –4 – y + z
Substitute x = –4 – y + z into equations (2) and (3).
From equation (2),
2(–4 – y + z) + 5y – 4z = –16
–8 – 2y + 2z + 5y – 4z = –16
3y – 2z = –8 ——— (4)
From equation (3),
(–4 – y + z) – 2y + 2z = 1
–3y + 3z = 5 ——— (5)
Solve equations (4) and (5) by another substitution.
From equation (4),
3y – 2z = –8
3y = –8 + 2z
–8 + 2z
y = _______
​​  3 ​​     ——— (6)

Substitute equation (6) into equation (5),

–3​​ (_______ )​​  + 3z = 5
–8 + 2z
​  3 ​   

8 – 2z + 3z = 5
z = –3
Substitute z = –3 into equation (6),
–8 + 2(–3)
y = _______
​​  3 ​​ 

14 ​​ 
= –​​ ___
3
14 ​​ and z = –3 into equation (1),
Substitute y = –​​ ___
3
14 ​  ​​ – (–3) = –4
x + (​​  –​ ___
3)
7
x = –​​ __
3 ​​ 
Hence, the point of intersection of the system is (x, y, z) = (​​  –​ __3 ​ , –​  3 ​,  –3).​​  
7 ___ 14
177

Pre-Calculus and Calculus.indb 177 13/7/2017 5:57:08 PM


Chapter 6 • Three Linear Equations with Three Variables

Exercise 6.1
1. Solve the following system of equations using the substitution method.
(a) 3x + 8y – z = –18
2x + y + 5z = 8
2x + 4y + 2z = –4

(b) –3x + 2y – 2z = –10


2x + y + z = 4
x – 2y + 3z = 7

(c) –2x – 4y + 5z = –2
–5x + 2y + 3z = 19
3x – 3y + 2z = –15

(d) x + y – z = –1
4x – 3y + 2z = 16
2x – 2y – 3z = 5

(e) –40x1 + 16x2 + 9x3 = 7


13x1 – 5x2 – 3x3 = –2
5x1 – 2x2 – x3 = –1

(f) x + y + z = 2
3x + 2y – z = 8
2x – 3y – 4z = 0

178

Pre-Calculus and Calculus.indb 178 13/7/2017 5:57:08 PM


Chapter 6 • Three Linear Equations with Three Variables

6.2 Elimination Method


Example 2
Find the point of intersection of the three planes in the following system
using the elimination method.
x+y+z=2
3x + 2y – z = 8
2x – 3y – 4z = 0

SOLUTION
x + y + z = 2 ——— (1)
3x + 2y – z = 8 ——— (2)
2x – 3y – 4z = 0 ——— (3)
Looking at the equations, it will be easier to eliminate variable z (than
the other two variables).
Adding equations (1) and (2), we have
x+y+z=2
+ 3x + 2y – z = 8
4x + 3y = 10
We need to take another pair of equations and eliminate z from these
equations. Let us take equations (1) and (3). (You may also take
equations (2) and (3)).
Multiply equation (1) by 4 and add to equation (3).
4x + 4y + 4z = 8
+ 2x – 3y – 4z = 0
6x + y = 8
Now, solve the system
4x + 3y = 10 ——— (4)
6x + y = 8 ——— (5)
Using elimination method again, we now eliminate y.
Multiply equation (5) by 3.
6x + y = 8
18x + 3y = 24 ——— (6)
Subtract equation (6) from equation (4) to find the value of x.
4x + 3y = 10
– 18x + 3y = 24
–14x = –14
x = 1
Substitute x = 1 into equation (5) to obtain the value of y.
6(1) + y = 8
y = 2
To find the value of variable z, substitute x = 1 and y = 2 into
equation (1).
x + y + z = 2
z = 2 – x – y
=2–1–2
= –1
Hence, the point of intersection of the system is (x, y, z) = (1, 2, –1). 179

Pre-Calculus and Calculus.indb 179 13/7/2017 5:57:08 PM


Chapter 6 • Three Linear Equations with Three Variables

Exercise 6.2
1. Solve the following system of equations using the elimination method.
(a) 2u – 3w = 16
2v + w = 4
u __ v
​​ __
6  ​​ – ​​  6  ​​ = 1
s + 2t + 2u = 2
(b)
s + 3t + u = 4
s – 7t + 4u = –9
(c) 2x – 3y + z = 0
5x + 2y – 4z – 3 = 0
x – 7y – 12z + 18 = 0
(d) –3a + 4b – c + 2 = 0
a – 2b + 3c – 4 = 0
2a + b – 4c – 3 = 0
(e) 2x + 4y – 10z = –2
3x + 9y – 21z = 0
x + 5y – 12z = 1
(f) 5x – 3y + 2z = 13
2x + 4y – 3z = –9
4x – 2y + 5z = 13

180

Pre-Calculus and Calculus.indb 180 13/7/2017 5:57:08 PM


Chapter 6 • Three Linear Equations with Three Variables

6.3 Inverse Matrix Method


A general system of linear equations is a set of finite linear equations with
the same number of unknown variables in the following form.

where are unknown variables and a and b are constants.


We can write the system of linear equations in matrix form:

and further express in compact form by a single matrix equation,


AX = b
If A–1 exist, then
A–1AX = A–1b
IX = A–1b A–1A = I
X=A b –1

That means, we need the inverse matrix of A. Then, we multiply the inverse
matrix with the column vector b to give the required solution.

Example 3
Solve the following system of equations using inverse matrix.
x + y + z = 2
3x + 2y – 2z = 8
2x – 3y – 4z = 0

SOLUTION
In matrix form, AX = b,

​​​​    y​  ​​  ​​  = ​​  8​ 


​ ​​  ​​  
1 1 1 x 2
[2 –3 –4][z] [0]
​​  3​ 
​ ​  2​ ​  ​  –2​
​ 
​ 

To find A–1, we use the elementary row operations.

​ ​  2​ ​  ​  –2​ ​  ​  0​ ​  1​ ​  0​​ ​​  


1 1 1 1 0 0
[2 –3 –4 0 0 1]
​​    
3​ 

Reduce a21 to zero: R2 ← R2 – 3R1


Reduce a31 to zero: R3 ← R3 – 2R1

​  –1​ ​  ​  –5​ ​  ​  –3​ ​  ​  1​ ​  0​​ ​​  


1 1 1 1 0 0
[0 –5 –6 –2 0 1]
​​  0​ 
​  

181

Pre-Calculus and Calculus.indb 181 13/7/2017 5:57:09 PM


Chapter 6 • Three Linear Equations with Three Variables

Reduce a12 to zero: R1 ← R1 + R2


Reduce a32 to zero: R3 ← R3 – 5R2

​ ​  –1​ ​  –5​ ​  –3​ ​  1​ ​  ​  0​​ ​​  


1 0 –4 –2 1 0
[0 0 19 13 –5 1]
​​    0​ 

Reduce a13 to zero: R1 ← R1 + ___ 4  ​​ R


​​  19 3

5
Reduce a23 to zero: R2 ← R2 + ___ ​​  19  ​​ R3

⎢  ⎥ 
⎡ 14 ​  –​ ___1 ___ 4⎤
1 0 0 ___ ​  19 19  ​  ​  19  ​ 
​  ​ 
​​     ​  ​  ​  ​  8 ​  ___ 6​  ​  ___5 ​​ ​​  
0 –1 0 ​ ___ 19  ​  –​  19  ​  ​  19  ​ 
⎣0 0 19 13 –5 1 ⎦
Reduce a22 to one: R2 ← –R2

⎢  ⎥ 
Reduce a33 to one: R3 ← ___ 1  ​​ R
​​  19 3

⎡ 14 1 ___ 4 ⎤
1 0 0 ​ ___ ___
19 ​  –​  19  ​  ​  19  ​ 
​    19 ​   ​ ​  ​  19  ​ ​  ​  –​  19  ​​ ​ ​ ​​  
8 ___ 6 5
​​    ​  1​ ​  0​ ​  –​ ___
0​  ___

⎢  ⎥ 
13
___ 5 ___
___ 1
⎣0 0 1 ​  19 ​  –​  19  ​  ​  19  ​ ⎦
⎡ ___ 14 ​  –​ ___ 1 ___ 4 ⎤
​  19 19  ​  ​  19  ​ 
​     19 ​   ​ ​  ​  19  ​ ​  ​  –​  19  ​​ ​ ​ ​​  
8 ___
___ 6 5
___
Hence, A–1 = ​​    
–​ 
___13 5 ___
___ 1
⎣ ​  19 ​  –​  19  ​  ​  19  ​ ⎦

⎢  ⎥ 
Since AX = b, then X = A–1b.
⎡ ___ 14 ​  –​ ___ 1  ​  ___ 4  ​ ⎤
 –1
​  19 19 19 ​ 

​​  ​  ​​  ​​  = ​​​     ​ ​​  ​​  


x 2
[z] 19 ​   ​ ​  ​  19  ​ ​  ​  –​  19  ​​ ​ ​ ​​​    [
0]
​    
y 8
___ ___ 6 5
___
–​  ​​​​  8​ 

⎢  ⎥ 
___13 5 ___
___ 1
⎣ ​  19 ​  –​  19  ​  ​  19  ​ ⎦
⎡ ___ 20 ⎤
​  19 ​ 

​​  y​  ​​  ​​  = ​​  ​  ___


x
[z] ​  19 ​  ​​ ​​  
32

14
___
⎣–​  19 ​ ⎦
20 32 14
The solution is x = ​​ ___ ___
19 ​​ , y = ​​  19 ​​ , z = –​​  19 ​​ .
___

Exercise 6.3
1. Write the following system of linear equations in the form of AX = b and solve using inverse matrix
method.
(a) x + y + z = 2 (b) x1 – 7x2 + 4x3 = 9
3x + 2y – z = 8 x1 + 2x2 + 2x3 = 2
2x – 3y – 4z = 0 x1 + 3x2 + x3 = 4
182

Pre-Calculus and Calculus.indb 182 13/7/2017 5:57:09 PM


Chapter 6 • Three Linear Equations with Three Variables

6.4 Gauss-Jordan Elimination Method


An alternative method is the Gauss-Jordan elimination method. We put the
system in the following form: [A | b]
Reduce A using the elementary row operations to identity matrix, I. Hence,
the solution can be readily obtained from the final matrix.

Example 4
Solve the following system of equations using the Gauss-Jordan
elimination method.
x1 – 7x2 + 4x3 = 9
x1 + 2x2 + 2x3 = 2
x1 + 3x2 + x3 = 4
SOLUTION
Put the system into the following form.
[A | b]
Our aim is to reduce A into an identity matrix.

​​  
1 –7 4 9
[1 3 1 4]
​ ​  2​ ​  ​  2​ ​  2​​ 
​​  1​ 

Reduce a21 to zero: R2 ← R2 – R1


Reduce a31 to zero: R3 ← R3 – R1

​  9​  ​  –2​ ​  ​  –7​​​ ​​  


1 –7 4 9
[0 10 –3 –5]
​​    
​  
0​ 

7
Reduce a12 to zero: R1 ← R1 + __ ​​  9 ​​ R2
10
Reduce a32 to zero: R3 ← R3 – ___ ​​  9 ​​R  2

⎢  ⎥ 
⎡ 32 ⎤
22 ​  ___
1 0 ​ ___ 9 ​  9 ​ 
​  ​  9​ ​  –2​  ​  –7​​ ​​  
0​ 
​​    
7 ___
__ 25
⎣0 0 –​  9 ​  ​  9 ​ ⎦

Reduce a13 to zero: R1 ← R1 + ___ ​​  22  3


7 ​​R

⎢  ⎥ 
18
Reduce a23 to zero: R2 ← R2 – ___ ​​  7 ​​R  3
⎡ 86 ⎤
1 0 0 ___ ​  7 ​ 

  ​  9​ ​  0​ ​  ​  –​ ___ ​ ​ ​​  


99
​  
​​    
0​  7 ​
7 ___
__ 25
⎣0 0 –​  9 ​  ​  9 ​ ⎦

183

Pre-Calculus and Calculus.indb 183 13/7/2017 5:57:09 PM


Chapter 6 • Three Linear Equations with Three Variables

​​ 91 ​​ R2
Reduce a22 to one: R2 ← __

⎢  ⎥ 
9
Reduce a33 to one: R3 ← –​​ __
7 ​​ R3
⎡ 86 ⎤
___
1 0 0 ​   ​ 
7
​  ​  1​ ​  0​ ​  –​ ___
​​  0​ 
   ​ ​ ​​  
11 ​
7 
25
___
⎣0 0 1 –​  7 ​ ⎦
Hence, we have the solution as follows:
86 25
11 ​​ , x = –​​ ___
x1 = ___
​​  7 ​​ , x2 = –​​ ___
7 3 7 ​​ 

Exercise 6.4
1. Solve the following matrix equations using the Gauss-Jordan elimination method.

​  ​​  ​​  = ​​  –2​ 


​​​​    b​  ​  ​​  ​​  
1 0 2 a
[ 2 1 0][c] [ 3 ]
–1
(a) ​​  –1​ 
​  ​  2​ ​  3​​ 

​​ ​​​​    q​  ​​  ​​  = ​​  –3​


​  ​  ​  ​​  
[2 2 –2][ r ] [14]
4 1 0 p 8
(b) ​​  1​ 
​ ​  0​ ​  2​ 

184

Pre-Calculus and Calculus.indb 184 13/7/2017 5:57:09 PM


Chapter 6 • Three Linear Equations with Three Variables

6.5 Applications
Example 5
A charity foundation distributes aids to three groups of recipients:
students, the poor, the sick. The table below shows the number of
recipients for the year 2002, 2003 and 2004.

Number of recipients
Year Students Poor Sick Total ($)
2002 10 100 50 85 000
2003 15 120 70 119 000
2004 18 105 100 136 250
Assume that the values of each aid are x, y and z (in $) respectively per
year.
(a) Obtain the system of linear equations based on the information given.
(b) Rewrite the linear equations in (a) in matrix form.
(c) How many students, the poor and the sick receive the aids each
year?

SOLUTION
(a) The system of linear equations is
10x + 100y + 50z = 85 000
15x + 120y + 70z = 119 000
18x + 105y + 100z = 136 250
(b) In matrix form,

​  120​ ​  70​​​ ​​​​    y​  ​​  ​​  = ​​  ​ 119 000​​ 


​ ​​  
10 100 50 x 85 000
[18 105 100][z] [136 250]
​​  15​ 
​    

(c) Using Gauss-Jordan elimination method:

​  120​ ​  70​ ​  ​  119 000​​​ ​​  


10 100 50 85 000
[18 105 100 136 250]
​​     15​ 
​  

15
Reduce a21 to zero: R2 ← R2 – ___ ​​  10 ​​ R1
18
Reduce a31 to zero: R3 ← R3 – ___ ​​  10 ​​ R1

​  ​  –30​ ​  –5​ ​  –8500​​​ ​​  


10 100 50 85 000
[ 0 –75 10 –16 750]
0​ 
​​  ​  
  

185

Pre-Calculus and Calculus.indb 185 13/7/2017 5:57:09 PM


Chapter 6 • Three Linear Equations with Three Variables

10
Reduce a12 to zero: R1 ← R1 + ___
​​  3 ​​R
 2
5
Reduce a32 to zero: R3 ← R3 – ___
​​  2 ​​ R2

⎢  ⎥ 
⎡ 170 000 ⎤
100 _______
____
10 0 ​   ​
3    ​   ​
3   
​​   ​  
0​ ​  ​  –30​ 
   ​  ​ ​​  
–5​  ​  ​  –8500​ ​
45
___
⎣0 0 ​  2 ​  4500 ⎦

40
Reduce a13 to zero: R1 ← R1 – ___
​​  27  ​​R3

​​  92 ​​ R3
Reduce a23 to zero: R2 ← R2 + ___
⎡10 0 0 50 000⎤
⎢ 
​​  ​   
0
  ​  ​  –30
45
⎥ 
​  ​  0​  ​  –7500 ​​​ ​​  
⎣0 0 ___ ​  2 ​  4500 ⎦

Reduce a11 to one: R1 ← ___ 1  ​​ R


​​ 10 1

1
Reduce a22 to one: R2 ← –​​ ___
30  ​​ R2
Reduce a33 to one: R3 ← ___ 2  ​​ R
​​ 45 3

​  1​ ​  0​ ​  250​ ​​ ​​  


1 0 0 5000
[0 0 1 200 ]
​​    
​  
0​ 

Therefore, (x, y, z) = (5000, 250, 200).


Hence, 5000 students, 250 of the poor and 200 of the sick receive
the aid each year.

186

Pre-Calculus and Calculus.indb 186 13/7/2017 5:57:09 PM


Chapter 6 • Three Linear Equations with Three Variables

Exercise 6.5
1. Harry feeds his cats with different mixtures of three types of food, namely X, Y and Z, as given below.
Food X : 10 mg protein, 5 mg carbohydrates, 15 mg vitamins
Food Y : 15 mg protein, 10 mg carbohydrates, 5 mg vitamins
Food Z : 15 mg protein, 5 mg carbohydrates, 15 mg vitamins
Assume that the cats require 190 mg of protein, 95 mg of carbohydrates and 160 mg of vitamins, how
many mg of each food should Harry feed his cats daily to satisfy their nutrient requirements?

2. A shop sells three different sizes of a product: small, medium and large. The price of a large product
is equal to the total price of a small and a medium product. Two different supermarkets get their
supplies from this shop. Supermarket A buys 100 small size, 50 medium size and 50 large size and
pays $1300. Supermarket B buys 50 small size, 50 medium size and 25 large size and pays $825.
(a) From the information above, form a system of linear equations.
(b) Write the above linear equations in the form of a matrix equation. Hence, find the price of a small,
a medium and a large size of the product using the inverse matrix method.

3. A housewife makes three types of cookies, A, B and C. The ingredients for each type of cookies are
as follows:
A : 10 g flour, 10 g chocolate chips, 5 g of skimmed milk
B : 7 g flour, 5 g chocolate chips, 4 g of skimmed milk
C : 4 g flour, 2 g chocolate chips, 3 g of skimmed milk
The total amount of ingredients available are 2.64 kg of flour, 2 kg of chocolate chips and
1.58 kg of skimmed milk.
(a) Obtain a system of linear equations to represent the above information.
(b) Express the system of linear equations in (a) as a matrix equation.
(c) Solve the matrix equation in (b). Hence, determine the number of each type of cookies made if
all the ingredients available are used.

4. Melissa won $20 000 in a competition. She put all the money in three investments, A, B and C.
The amount she put in investment C was $5000 more than that in investment B. After one year,
she received a profit totaling $850 from the three investments. Investment A paid 3.5% annually,
investment B paid 4.5% annually and investment C paid 4.5% annually.
(a) By denoting x, y and z as the amount of money invested in investments A, B and C respectively,
form a system of linear equations based on the above information.
(b) Write the system of linear equations in the form of matrix equation.
(c) Find the amount of money invested in each investment.

187

Pre-Calculus and Calculus.indb 187 13/7/2017 5:57:09 PM


Equations and Inequalities
Chapter 7:

Inequalities

Forest Conservation
Forest conservation is very important for every nation. It preserves the quality of water and
air, which are key to the existence of life. It also helps in maintaining the firmness of soil,
preventing soil erosion and floods. Forest trees play a big role in absorbing carbon dioxide. The
forest is a source of wood and wood products such as paper and furniture. The presence and
health of our forests affect the climate and global temperatures. Thus, forest conservation should
remain a global concern.
Changes in temperature and precipitation are expected to change forest composition,
productivity and existence. A low level of precipitation may signal drought, while high level of
precipitation can reduce air space in the soil that may cause trees to die. Very low and very high
temperatures can also kill trees. Every species of tree has unique requirements for temperature
and precipitation.
Constraints in temperature and precipitation can be written using linear inequalities. A certain
rainforest in North America has the following temperature and precipitation requirements:
Temperature (in °F ): 39  T  54
Precipitation (in cm): 200  P  400

Pre-Calculus and Calculus.indb 188 13/7/2017 5:57:09 PM


Chapter 7 • Inequalities

7 Inequalities
7.1 Intervals on the Real Number Line
7.2 Properties of Inequalities
7.3 Linear Inequalities
7.4 Quadratic Inequalities
7.5 Applications of Inequalities

7.1 Intervals on the Real Number Line


7.1.1 The Real Number Line
7.1.2 Sets of Real Numbers
7.1.3 Intervals of Real Numbers
7.1.4 Intersection and Union of Sets of Real Numbers

7.1.1 The Real Number Line


As discussed in Chapter 1, each real number corresponds to one __ point on
the real number line. For example, the positions of numbers –​​√ 2 ​​,  –5, � and
7.25 are shown in the real number line below.

–5 0 7.25
negative positive
real number real number

The numbers on the real number line increase as we move towards the
right (positive). If a and b are real numbers, then only one of the following
is true:
a < b (a is less than b)
or a = b (a is equal to b)
or a > b (a is greater than b)

Hence,
(i) a < b can also be written as b > a, and on the real number line, a lies to
the left of b.
(ii) The inequality a  b means a < b or a = b.
(iii) The inequality a < b < c means a < b and (at the same time) b < c.

7.1.2 Sets of Real Numbers


A set of positive integers less than 10 can be written as {1, 2, 3, 4, 5, 6, 7, 8,
9}. 1, 2, 3, …, 9 are elements of the set. The elements of the set are listed
inside curly brackets. When it is impossible to list all the elements of a
set, we use dots to indicate after some of the elements of the set have been
listed. For example, the set of all odd integers is written as {1, 3, 5, 7, 9,
11, …}. Alternatively, instead of listing the elements, we can describe the
sets, for example,
{1, 3, 5, 7, 9, 11, …} can be written as {x : x ∈ , x is odd},
and {1, 2, 3, 4, 5, 6, 7, 8, 9} can be written as {x : x ∈ , x < 10}.
189

Pre-Calculus and Calculus.indb 189 13/7/2017 5:57:09 PM


Chapter 7 • Inequalities

7.1.3 Intervals of Real Numbers


Now, we look at the intervals of real numbers. If a set X consists of real
numbers between 3 and 6, then it is not possible to list all the real numbers
of set X. The set can be written in terms of a interval such as
X = x : 3 < x < 6, x  
Note
It is very important to note that the number of elements in X is infinite, and
The number of elements in consists of all the natural numbers, the integers, the rational and irrational
X = {x: a < x < b, x  }, is
numbers between 3 and 6.
infinite.
3 < x < 6 is called the inequality notation of set X. We can also write it in
interval notation as (3, 6), and we may also picture the set geometrically
on the real number line as

3 6

The table below gives the sets of real numbers in inequality notations,
interval notations and their representations on the real number line.

Inequality Interval Representation on a


notation notation real number line

a<x<b (a, b) x
a b

[a, b] x
axb a b

[a, b) x
ax<b a b

a < x < ∞ or x > a (a, ∞) x


a

[a, ∞) x
a  x < ∞ or x  a a

–∞ < x < b or x < b (–∞, b) x


Note b

(–∞, ∞) is the set of all real x


numbers, . –∞ < x  b or x  b (–∞, b] b

–∞ < x < ∞ (–∞, ∞) x

190

Pre-Calculus and Calculus.indb 190 13/7/2017 5:57:10 PM


Chapter 7 • Inequalities

It is useful to note the following:


(i) (a, b) is called an open interval. Both the end points a and b are not
included as elements of the set, and on the real number line, they are
represented by empty circles, °.
(ii) [a, b] is called a closed interval. Both end points a and b are also
elements of the set. On the number line, a and b are represented by
dense circles, •.
(iii) [a, b) is called a half-open or half-closed interval. a is an element of
the set but b is not.
(a, b] is also called a half-open or half-closed interval. b is an element
of the set but a is not.
(iv) The symbol ∞ is not a real number. It is just a notation used to
indicate unboundedness in the positive direction. In the same way,
–∞ denotes unboundedness in the negative direction.
Example 1
List all elements of the following sets.
(a) A = {x: –6 < x  5, x  }
(b) B = {y: 2  y < 7, x  }
(c) C = {t: –5 < t < –1, x  }
(d) D = {x: 1 < x < 2, x  }

SOLUTION
(a) A = {–5, –4, –3, –2, –1, 0, 1, 2, 3, 4, 5}
(b) B = {2, 3, 4, 5, 6}
(c) C = {–4, –3, –2}
(d) D = { } or D = ϕ, the null set.
(The null set does not contain any element.)

Example 2
Rewrite each of the following inequalities using interval notation and
show them on a real number line.
(a) 3 < x < 7 (d) –∞ < y < 3
(b) –2.5  t  5 (e) –5  x < –2
(c) x  1.5

SOLUTION
(a) (3, 7) x
3 7

(b) [–2.5, 5] t


–2.5 5

(c) [1.5, ∞) x
1.5

(d) (–∞, 3) y
3

(e) [–5, –2) x


–5 –2 191

Pre-Calculus and Calculus.indb 191 13/7/2017 5:57:10 PM


Chapter 7 • Inequalities

7.1.4 Intersection and Union of Sets of Real


Numbers
If X and Y are two given sets, then
(i) the intersection set of X and Y, denoted by X  Y, consists of
common elements that exist in set X and set Y. We may define the
intersection set as X  Y = {x: x  X and x  Y}. If there are no
common elements, the sets are said to be disjoint. Disjoint sets are
implied by the statement X  Y = ϕ.
(ii) the union set of X and Y, denoted by X  Y, consists of elements that
exist in X or Y (including those in X and Y). We define the union set
as X  Y = {x: x  X or x  Y}.
Now we look at examples which show the intersection and the union of
sets of real numbers.

Example 3
Given that A = {x: x  , –3  x < 5} and B = {0, 2, 4, 6, 8, 10},
find (a) A  B,
(b) A  B.

SOLUTION
A = {–3, –2, –1, 0, 1, 2, 3, 4} and B = {0, 2, 4, 6, 8, 10}.
(a) A  B = {0, 2, 4}
(b) A  B = {–3, –2, –1, 0, 1, 2, 3, 4, 6, 8, 10}

Example 4
Show each of the following on a real number line and find the interval
notations of the solutions.
(a) (–3, 4)  (0, 1) (c) (–∞, –1)  (1, ∞)

2 ​ , 3)​​   [2, 4)


(d) ​​ (–​ __
(b) (2, 3)  (2, 4] 1

SOLUTION
(a) (0, 1)
 x
–3 0 1 4

(b) (2, 4]
 x
2 3 4

(c) ϕ
x
–1 1


2 ​ , 4)​​  
(d) ​​ (–​ __
1

 x
1
–​​ __ 2 3 4
2 ​​ 

192

Pre-Calculus and Calculus.indb 192 13/7/2017 5:57:10 PM


Chapter 7 • Inequalities

Example 5
Given that sets X = {–6.5, –3, –2.5, –1, 0.5, 1}, Y = [–9, 3) and
Z = (–4, 7), find
(a) X  Z,
(b) (X  Z)  Y,
(c) X  Y  Z.

SOLUTION
(a) –6.5 –3 –1 1
X :
–2.5 0.5

Z :
–4 7
X  Z = {–3, –2.5, –1, 0.5, 1}

(b) –3 –1 1
X  Z : –2.5 0.5

Y :
–9 3
(X  Z)  Y =
 {–3, –2.5, –1, 0.5, 1}  [–9, 3)
= [–9, 3)

(c) X  Y  Z = (X  Y)  Z or X  (Y  Z)
–6.5 –3 –1 1
X  Y = X :
–2.5 0.5

Z  :
–4 7
(X  Y)  Z = {–3, –2.5, –1, 0.5, 1}

193

Pre-Calculus and Calculus.indb 193 13/7/2017 5:57:10 PM


Chapter 7 • Inequalities

Exercise 7.1

1. Show each of the following intervals on a real number line.


(a) (–∞, 5] (f) (–1, ∞)
(b) (–∞, ∞) (g) (–5, 0)  [2, ∞)
(c) [–7, 0) (h) [–2, 4]  (4, 8]
(d) [–3, –1] (i) [0, 9]  (5, 9]
(e) (6, 10) (j) (–7, 8)  (3, 11)

2. Rewrite the following sets in interval notations and show them on a real number line.
(a) –∞ < x  4 (f) p  –4
(b) –3  t < 8 (g) 3  x  7
(c) 0 < u < ∞ (h) –2 < t < 9
(d) y  –5 (i) 0 < x  3
(e) x < 6

3. Sets A, B and C are given as A = (–5, 5), B = [–2, 10) and C = {x: –3 < x  8, x  }.
Find the sets of the following operations.
(a) A  B
(b) B  C
(c) A  C
(d) (A  C)  B
(e) A  B  C

4. Given that U = [–7, 9), V = [9, 18] and W = (0, ∞), find the sets obtained from the following operations.
(a) U  V
(b) U  V
(c) U  W
(d) V  W
(e) (V  W )  U

194

Pre-Calculus and Calculus.indb 194 13/7/2017 5:57:10 PM


Chapter 7 • Inequalities

7.2 Properties of Inequalities


An inequality in one variable is a statement containing the variable with
one of the inequality symbols <, , > or .
The symbols are read as follows:
< – less than
 – less than or equal to
> – greater than
 – greater than or equal to
Examples of inequalities in one variable are:
y
x + 2 < 5,   a2 – 2  2a,  ​​ _____
y – 2 ​​ > 7,   t  5t – 3
2

Before we solve problems involving inequalities, we first discuss the


properties of inequalities, as summarized in the following theorem.

Theorem 7.1

1. If a and b are arbitrary real numbers, then only one of the following
is true:
(i) a < b,
(ii) a = b,
(iii) a > b.
2. For any real number a, a2  0.
3. Let a, b and c be real numbers.
If a < b and b < c, then a < c.
If a > b and b > c, then a > c.
4. Let a, b and c be real numbers.
If a < b, then a + c < b + c.
If a > b, then a + c > b + c.
5. Let a, b and p be real numbers.
a b
If a > b and p > 0, then ap > bp and __
​​  p ​​ > __
​​  p ​​  .
a b
If a > b and p < 0, then ap < bp and __
​​  p ​​ < __
​​  p ​​  .
a b
If a < b and p > 0, then ap < bp and __
​​  p ​​ < __
​​  p ​​  .
a b
If a < b and p < 0, then ap > bp and __
​​  p ​​ > __
​​  p ​​  .
6. If a and b are positive __
real numbers and a > b, then
__
a > b and √ 
2 2
​​ a ​​ > √ ​​  b ​​. 
7. Let a and b be positive real numbers.
If a > b, then __ ​​  1a ​​ < __​​  1 ​​  .
b
__ 1 __
If a < b, then ​​  a ​​ > ​​   ​​  .1
b

195

Pre-Calculus and Calculus.indb 195 13/7/2017 5:57:10 PM


Chapter 7 • Inequalities

Solutions of an inequality in one variable are sets of real numbers which


satisfy the inequality. These solutions are the range of the variable involved.
To solve, the inequality is replaced by equivalent inequalities until the
solution is obtained. We first look at linear inequalities in one variable.

7.3 Linear Inequalities


A given inequality can be replaced by an equivalent inequality by applying
operations of addition, subtraction, multiplication, division, and using the
properties of inequalities that we have discussed.
For example,
(i) x + 5 < 9 is equivalent to x + 5 – 5 < 9 – 5 Adding –5 to each side

Note (ii) –3x  30 is equivalent to (–3x)(–1)  30(–1) Multiply each side by –1


5x 45
When multiplying or (iii) 5x  45 is equivalent to ___
​​ 5 ​​  ___
​​  5 ​​  Divide each side by 5
dividing by a negative
–2x 18
number, we change the (iv) –2x > 18 is equivalent to ___ ​​ –2 ​​ < ___ ​​  –2 ​​  Divide each side by –2
inequality sign.
The following examples illustrate how we solve linear inequalities.

Example 6
Find the solution of the following linear inequalities.
x _____ x–3
(a) 3(x – 2) > 4(2x + 1) (b) ​​ __
6  ​​  ​​  5 ​​ 

SOLUTION
(a) 3(x – 2) > 4(2x + 1)
3x – 6 > 8x + 4
3x – 6 + (–8x + 6) > 8x + 4 + (–8x + 6) Add –8x + 6 to each side
–5x > 10
–5x ___ 10 Divide each side by –5
​​ ____  ​​
–5 –5  < ​​   
 ​​ Note that ‘>’ is changed to ‘<’.
x < –2
The solution x < –2 can also be written as {x: –∞ < x < –2}, or
in interval notation as x  (–∞, –2), or it can be shown on a real
number line as
x
–2
x _____ x–3
(b) ​​ __
6  ​​  ​​  5 ​​ 

x x–3
​​ __ _____
6  ​​(30)  ​​  5 ​​(30)

  Multiply each side by 30

5x  6x – 18
5x – 6x  6x – 18 – 6x Subtract 6x from each side
–x  –18
(–1)(–x)  (–1)(–18) Multiply each side by –1
x  18
The solution is {x: 18  x < ∞}.

196

Pre-Calculus and Calculus.indb 196 13/7/2017 5:57:10 PM


Chapter 7 • Inequalities

Example 7
Find the solution of the following inequalities.
x – 3 ______2x + 1
(a) 2 + _____
​​  4 ​​ 
  ​​ 
5 ​​  

(b) –6  4 – 7x < 3
Show the solution set on a real number line.

SOLUTION
x – 3 ______ 2x + 1
(a) 2 + ​​ _____
4 ​​ 
  ​​ 
5 ​​  
40 + 5(x – 3)  (4)(2x + 1) Multiply each side by 20
25 + 5x  8x + 4
Subtract 25 + 8x from
25 + 5x – (25 + 8x)  8x +4 – (25 + 8x) each side
–3x  –21
–3x ____ –21
​​ ____ –3 ​​  ​​  –3 ​​  Divide each side by –3
x7
The solution is {x: –∞ < x  7}.
x
7
(b) –6  4 – 7x < 3
This is a combined inequality. This inequality has three parts: the
left, the middle and the right. We can solve it by two methods.
Method 1:
To solve this inequality, we break it up into two inequalities.
–6  4 – 7x AND 4 – 7x < 3
Then, solve each inequality separately.
–6  4 – 7x AND 4 – 7x < 3
–6 + 6 + 7x  4 – 7x + 6 + 7x  AND 4 – 7x – 4 < 3 – 4
7x  10 AND –7x < –1
10
___
x  ​​  7 ​​   AND ​​  71 ​​ 
x > __
10 1
We obtained two solutions, x  ​​ ___ __
7 ​​ and x > ​​  7  ​​. These two solutions
10 1
occur simultaneously, that is, x  ​​ ___ __
7 ​​ and at the same time, x > ​​  7 ​​ .
10
​​ 71 ​​  < x  ___
Hence, the solution is __ ​​  7 ​​ .
The solution can also be obtained by taking the intersection of the
10 1
two sets of solutions, x  ___
​​  7 ​​ and x > ​​ __
7 ​​ , as shown below.
x
1
__
​​  7 ​​ 

x
10
___
​​  7 ​​  
The solution is { ​  7 ​ }​​. 
10
​ 17  ​ < x  ___
​​  x: __
x
1
__ 10
​​  7 ​​  ​​ ___
7 ​​  

197

Pre-Calculus and Calculus.indb 197 13/7/2017 5:57:10 PM


Chapter 7 • Inequalities

Method 2:
Since the inequalities are linear, we can solve both inequalities
concurrently. We apply the same operations to all three parts until
the solution is obtained (with x in the center).
–6  4 – 7x < 3
–6 – 4  4 – 7x – 4 < 3 – 4 Subtract 4 from each part
–10  –7x < –1
Divide each part by –7
–10 –1 ​​ 
​​ ____
–7  ​​  x > ___
​​  –7 Note that both inequality
signs have changed.
1
__ ___ 10
​​  7  ​​ < x  ​​  7 ​​ 

The solution is { ​  7 ​ }​​. 


10
​ 17  ​ < x  ___
​​  x: __

Exercise 7.3
1. Solve the following linear inequalities.
5
(a) ​​ __
2 ​​ x + 7(1 – x) > 1
(b) –4 < 3x + 5 < 4
x+4
(c) –x < _____ ​​  5 ​​ 
  2

2. Solve the following inequalities.


(a) –2x + 12 > –8x
(b) 9x – 12 < x + 20
(c) 5(x + 4) > 9x – 8
(d) –2x + 7  2(2x – 1)
(e) –3(–2 + 4x)  2(x – 7)
(f) 7(–3x + 4)  5(–4x – 6)
(g) 4(x – 2) – (2x + 1)  12
(h) –2(3x + 8)  –(–8x – 5)

3. Solve the following inequalities.


(a) 0.15x – 0.25(6 – 5x)  0.5(2x – 9) + 0.75
(b) 0.5(x – 6) – 0.1x < 0.4(2x – 5) + 0.2
3 4
(c) ​​ __ __
4 ​​ x + 8  –​​  3 ​​ x – 9
2 ​​ x + 7 > __
(d) –​​ __ ​​ 42  ​​x
3
(e) –10 < –5x – 8 < 18
x
(f) –5  __ ​​  2  ​​ + 9  3

198

Pre-Calculus and Calculus.indb 198 13/7/2017 5:57:10 PM


Chapter 7 • Inequalities

7.4 Quadratic Inequalities


To solve quadratic inequalities, always put zero on the right side and the
rest on the left. Factor the left side into two linear factors and solve the
inequality. In the following example, we discuss four methods of solving
the quadratic inequality. The first three methods are analytical methods
and the fourth is a graphical method.

Example 8
Solve the quadratic inequality x2 – 10 > 3x.

SOLUTION
x2 – 10 > 3x
x2 – 3x – 10 > 0
(x – 5)(x + 2) > 0 Factor the left side
Method 1:
Product of two real numbers is positive when both are positive OR
when both are negative.
Thus, (x – 5)(x + 2) > 0 when
(x – 5) > 0  AND  (x + 2) > 0 OR (x – 5) < 0  AND  (x + 2) < 0

That is,
x > 5  AND  x > –2 OR x < 5  AND  x < –2

The first set, and at the same time, x > – 2, gives the solution set,
x > 5.
The second set, x < 5 and at the same time, x < –2, gives the solution
set, x < – 2.
The solution is the union of solutions of the first and second set.
Therefore, the solution set is x > 5 OR x < –2.
We represent the set as {x: (x < –2)  (x > 5)}, or
{x: x  {(–∞, –2)  (5, ∞)}}. The solution can also be represented on
the number line as
x
–2 5

Method 2:
(x – 5)(x + 2) > 0
This method uses a sign chart. In this method, the real number line is
separated into intervals by numbers which we obtained when letting
(x – 5)(x + 2) equals 0.
When (x – 5)(x + 2) = 0,
x = 5 or x = –2

199

Pre-Calculus and Calculus.indb 199 13/7/2017 5:57:10 PM


Chapter 7 • Inequalities

These two values separate the real number line into three intervals: (–∞,
–2), (–2, 5) and (5, ∞).
(–∞, –2) (–2, 5) (5, ∞)
x
–2 5

We find that when x takes any value in interval (–∞, 2), for example,
x = –4, the values of both (x – 5) and (x + 2) are negative. Therefore, the
value of the product (x – 5)(x + 2) is positive. The number x = –4, which
we have chosen, is called a test number.
When x takes any value in interval (–2, 5), for example, x = 0, the values
of (x – 5) is negative and the value of (x + 2) is positive, Therefore, the
value of the product (x – 5)(x + 2) is negative.
When x takes any value in interval (5, ∞), for example, x = 7, both (x – 5)
and (x + 2) are positive. Therefore, the value of (x – 5)(x + 2) is positive.
We want the solution for (x – 5)(x + 2) > 0, that is, the values of x such
that the product (x – 5)(x + 2) is positive. Therefore, the solution is
(–∞, –2) OR (5, ∞). We represent the solution set as the union of the two
sets, that is, {x: x  {(–∞, –2)  (5, ∞)}}.
To shorten the above lengthy argument, we use the sign chart shown below.

– – +
– + +
+ – +
x
–2 5

From the chart, since we want (x – 5)(x + 2) > 0, the values of x in


the first and third columns satisfy the inequality. The solution set is
{x: x  {(–∞, –2)  (5, ∞)}}.

Method 3:
Method 2 can be simplified as follows. Instead of choosing a test
number, the sign in each interval can be obtained by determining the
values of x such that (x – 5) > 0 and (x + 2) > 0.
(x – 5) > 0     x > 5
(x + 2) > 0     x > –2
Since (x – 5) > 0 when x > 5, the sign is therefore positive on the right
side of x = 5 (and negative on the left side of x = 5). We present this
on the sign chart by an arrow to the right of x = 5. Note that the empty
circle at the beginning of the arrow indicates that the value x = 5 is
not inclusive. Similarly, the sign of (x + 2) is positive on the right side
of x = –2 (and negative on the left side). Complete the sign chart by
determining the sign of the product (x – 5)(x + 2) in each column. We
use symbol (+) for positive and (–) for negative for the product on the
chart.

200

Pre-Calculus and Calculus.indb 200 13/7/2017 5:57:10 PM


Chapter 7 • Inequalities

– – +

– + +

x
–2 5

We find that the product is positive when –∞ < x < –2 or 5 < x < ∞.
Hence, the solution of (x – 5)(x + 2) > 0 is {x: x  {(–∞, –2)  (5, ∞)}}.
Method 4:
The inequality can also be solved using a sketch of the graph of the
associated function y = (x – 5)(x + 2). The graph of y = (x – 5)(x + 2) is
shown below.
y

x
–2 5

The graph cuts the x-axis at x = –2 and x = 5. At these points, the value
of (x – 5)(x + 2) is zero. The graph is below the x-axis when –2 < x < 5.
Therefore, the value of y = (x – 5)(x + 2) is negative in that interval. The
graph is above the x-axis when –∞ < x < –2 or 5 < x < ∞. Therefore, the
value of y = (x – 5)(x + 2) is positive in the two intervals. Hence, the
solution of y = (x – 5)(x + 2) > 0 is {x: x  {(–∞, –2)  (5, ∞)}}.

In the following example, we will solve the given inequalities using one
of the above methods.

Example 9
Solve the quadratic inequality t 2 – 4t – 4 < 0.

SOLUTION
We find that it is difficult to factor the left side by the cross method.
Hence, let t 2 – 4t – 4 = 0 and find the solution of the equation using the
quadratic formula.
_______
​  b2 – 4ac  
–b ± √
_____________ ​
t = ​​    2a  _____________
​​ 
–(–4) ± √ ​  (–4)
   2
– 4(1)(–4) ​
= _____________________
  
​​      ​​
___
2(1)
4±√
_______ ​  32 ​ 
= ​​  2 ​​   
__
4 ± 4​√ 2 ​ 
= ​​ _______2 ​​  

__
= 2 ± 2​​√ 2 ​​ 
__ __
The solutions of t 2 – 4t – 4 = 0 are t = 2 + 2​​√ 2 ​​ and t = 2 –__2​​√ 2 ​​.  Hence, __the
 ​ ) ​].​​  
left side of the inequality can be factored as [​​  t – (​ 2 + 2​√ 2 ​) ​] [​​   t – (​ 2 – 2​√ 2  201

Pre-Calculus and Calculus.indb 201 13/7/2017 5:57:11 PM


Chapter 7 • Inequalities

__ __
Now, we are going to solve [​​  t – (​ 2 + 2​√ 2 ​) ​] [​​   t – (​ 2 – 2​√ 2  ​ ) ​]​​  < 0 __
using the
__ sign chart __ method. __We have three intervals (​​ –∞, 2 – 2​√ 2 ​) ​​, 
(​​  2 – 2​√ 2 ​ , 2 + 2​√ 2 ​) ​​  and (​​  2 + 2​√ 2 ​ , ∞) ​​. Pick a test number in each interval
and check the sign of each factor. Then, check the sign of the product of
the factors in each interval.
__
 ​ ) ​]:​​ 
[​​  t – (​ 2 – 2​√ 2 – + +
__
 ​ ) ​]:​​ 
[​​  t – (​ 2 + 2​√ 2 – – +
__ __
 ​ ) ​][​​​​   t – (​ 2 + 2​√ 2 ​ ) ​]:​​ 
[​​  t – (​ 2 – 2​√ 2 + – +
__ __ x
​2 – 2​√ 2 
 ​​ ​2 + 2​√ 2 
 ​​
From the sign chart, __ __
the solution of t 2 – 4t – < 0 is {t: 2 – 2​​√ 2 ​​ < t < 2 + 2​​√ 2 ​​ }.

Example 10
Find the range of values of x that satisfy (13 – 4x)(x – 2)  0.
SOLUTION
(13 – 4x)(x – 2)  0
By considering each of the two factors as positive, we get
13
13 – 4x  0    x  ​​ ___
4 ​​ 
x – 2  0   x  2
Representing the above ranges on the real number line,
+ + –

– + +

2 13
4
13
We find that the product is negative when –∞ < x  2 or ​​ ___
4 ​​  x < ∞.
Therefore, the solution of (13 – 4x)(x – 2)  0
is {   .​​  
​  4 ​,  ∞)​}
​​  x: x  (–∞, 2]  [​  ___
13

Example 11
Solve the quadratic inequality 21  10y – y2, and
(i) show the solution set on a real number line,
(ii) sketch the graph of f (x) = y2 – 10y + 21.

SOLUTION
21  10y – y2
0  10y – y2 – 21
0  y2 – 10y + 21
y2 – 10y + 21  0
(y – 3)(y – 7)  0 Factor the left side
When (y – 3)(y – 7) = 0,
y = 3 or y = 7
202

Pre-Calculus and Calculus.indb 202 13/7/2017 5:57:11 PM


Chapter 7 • Inequalities

Therefore, the intervals to be considered are (–∞, 3), (3, 7) and (7, ∞).
Pick a test number in each interval. Check the sign of each factor and
the product of the factors in each interval. The result is shown in the
following sign chart.

(y – 3): – + +
(y – 3): – – +
(y – 3)(y – 7): + – +
3 7
y Note
The solutions include the end
The solution of 21  10y – y2 or (y – 3)(y – 7)  0 is
points 3 and 7.
{y: y  {(–∞, 3]  [7, ∞)}}.
At these points, the value of
(i) On the real number line the solution is represented as (y – 3)(y – 7) is zero.

y
3 7

(ii)

y
0 3 7

The next example shows a combined quadratic inequality. To solve this


inequality, we rewrite it in two inequalities and solve each inequality
separately. We have used this method to solve a combined linear inequality
in Example 7(b).

Example 12
Find the range of values of x which satisfy 3 < x2 – 2x  24. Note
SOLUTION ‘AND’ implies the
two inequalities occur
3 < x2 – 2x  24 simultaneously.
Write it in two separate inequalities,
3 < x2 – 2x  AND  x2 – 2x  24
Solving 3 < x2 – 2x,
3 < x2 – 2x
x – 2x – 3 > 0
2

(x – 3)(x + 1) > 0
When (x – 3)(x + 1) = 0,
x = 3 or x = –1
Thus, the intervals to be considered are (–∞, –1), (–1, 3) and (3, ∞).
Pick a test number in each interval, check the sign for factors (x – 3),
(x + 1) and product (x – 3)(x + 1), and complete the sign chart.
203

Pre-Calculus and Calculus.indb 203 13/7/2017 5:57:11 PM


Chapter 7 • Inequalities

(x – 3): – – +
(x + 1): – + +
(x – 3)(x + 1): + – +
x
–1 3
The solution of 3 < x2 – 2x or (x – 3)(x + 1) > 0
is {x: x  {(–∞, –1)  (3, ∞)}}.
Solving x2 – 2x  24,
x2 – 2x  24
x2 – 2x – 24  0
(x – 6)(x + 4)  0
When (x – 6)(x + 4) = 0,
x = 6 or x = –4
Thus, the intervals to be considered are (–∞, –4), (–4, 6) and (6, ∞).
Pick a test number in each interval, check the sign for factors (x – 6),
(x + 4) and product (x – 6)(x + 4), and complete the sign chart.

(x – 6): – – +
(x + 4): – + +
(x – 6)(x + 4): + – +
x
–4 6
The solution of x2 – 2x  24 or (x – 6)(x + 4)  0 is {x: x  [–4, 6]}.
The solution of 3 < x2 – 2x  24 is the intersection of both solutions,
that is,
{x: x  {(–∞, –1)  (3, ∞)}}  {x: x  [–4, 6]}

x  [–4, 6]: x
–4 6
x  {(–∞, –1)  (3, ∞)}: –1 3 x

Hence, the range of values of x which satisfy 3 < x2 – 2x  24 is


{x: x  {[–4, –1)  (3, 6]}}.

Exercise 7.4
1. Find the range of values of x which satisfy the following inequalities.
(a) 3x2 > 5 – 14x (c) 4 < x2 – 3x  6 – 4x
(b) (2x + 1)(x – 1)  3x + 5 (d) 5x(x + 3) > 2(2x – 1)

2. Solve the following inequalities.


(a) x2 + x – 6 < 0 (e) 4x2 – 8x  0
(b) x2 – 3x – 4  0 (f) x2 + 6x + 9  0
(c) x2 – 4 > 0 (g) (x + 4)2 > 10x + 31
(d) x2 – 16 < 0 (h) –(x – 4)2 + 3 > 0

204

Pre-Calculus and Calculus.indb 204 13/7/2017 5:57:11 PM


Chapter 7 • Inequalities

7.5 Applications of Inequalities


Example 13
Eunice scored 88, 80 and 83 in Mathematics during the first three
quarters. What should her Mathematics grade be in the last quarter so
as to obtain an average grade of at least 85 in Mathematics?

SOLUTION
Step 1: Define variables.
Let x be Eunice’s Mathematics grade in the fourth quarter.
Step 2: Translate into an inequality.
Her average grade in Mathematics must be at least 85.
average grade  85
88 + 80 + 83 + x
​​ ______________    4 ​​   85

Step 3: Solve the inequality.


88 + 80 + 83 + x
​​ ______________
   4 ​​   85 Note
251 + x Average grade
​​ _______ 4 ​​   85
sum of grades
= ________________
)​​   4(85)
  
​​     ​​
4(
251 + x number of quarters
​​  ________
​  4 ​   
251 + x  340
x  340 – 251
x  89
Hence, Eunice must score at least 89 in Mathematics during the
last quarter.

Exercise 7.5
1. Justin can clean the house in 4 hours alone. His friend, Bernie can clean the house in 5 hours alone.
3
How long will they take if they work together to clean at least __
​​ 4 ​​  of the entire house?

6x2 – 7 __
2. (a) Find the largest integer value of x which satisfies the inequality, ________
​​     ​​  2 ​​ .
 ​​ 
(3x + 5)2 3
(b) Mrs. Williams wishes to buy a branded watch that costs $9850.
She is given a payment option as shown below:

20% cash payment when purchase


+
12 monthly instalments thereafter at y%

(i) How much is the cash payment?


(ii) Mrs. Williams will only buy the watch if the monthly instalment does not exceed $690. Find
the greatest possible value of y, correct to the nearest whole number, so that Mrs. Williams
will buy the watch.

205

Pre-Calculus and Calculus.indb 205 13/7/2017 5:57:11 PM


Chapter 7 • Inequalities

3. A test consists of two papers to be taken separately. Paper 1 has 40 multiple choice questions each
carrying 2 marks for a correct answer. Paper 2 has 12 structured questions each carrying 10 marks.
For a pass, a student is required to score at least 50% of the total marks. A student took Paper 1 first
and got 29 questions correct. What is the least average mark he should get for each Paper 2 question
to pass the text? (Correct to the nearest integer.)

4. The table shows the commission scheme given to a dealer.


Total sales Commission
Below $10 000 –
$10 000 – $50 000 5%
Above $50 000 6.5%
Calculate
(a) the commission earned if the total sales is $38 000.
(b) the minimum sales if the dealer wants to earn at least $4800. (Correct to the nearest dollar.)

5. It is given that 4.2 cm  length  5.8 cm and 6.9 cm  breadth  8.4 cm. Estimate, correct to the
nearest whole number,
(a) the greatest possible perimeter of the rectangle,
(b) the least possible area of the rectangle.

6. In triangle PQR, PQ = 4x cm, QR = (x + 1) cm and ∠PQR = 90º.


Q 4x P

x+1

R
If PR is shorter than 28 cm, find the possible length of QR. (Correct to 2 decimal places.)

7. Thomas buys a book at $16. How much must he sell in order to earn at least a profit of 25%?

8. A man invested some money at a simple interest of 4% per annum for 6 years. How much must he
invest if he wants to earn at least an interest of $100 at the end of 6 years? (Correct to the nearest
whole number.)

9. A woman invest $300 at a simple interest of 4% per annum. At least how long must she invest to earn
an interest of $87? (Correct to the nearest whole number.)

10. The exchange rate of Singapore Dollars against Malaysia Ringgit is S$1 = RM 3.09. How much
Singapore Dollars should Marcus sell if he wants to receive at least $5000? (Correct to the nearest
dollar.)

206

Pre-Calculus and Calculus.indb 206 13/7/2017 5:57:11 PM


Analytic Geometry and Trigonometry
Chapter 8:

Graphs of Quadratic Equations

Civil Engineering

Some engineers, architects and master builders used parabolas to strengthen the foundation
and structures of buildings, bridges and roads. The San Juanico bridge in the San
Juanico Strait, the longest bridge in the Philippines with a total length of 2200 meters
(7200 feet) is an example. It spans from Samar to Leyte and is important to the trade
and commerce between the two provinces. It is also a popular tourist attraction in the
South. As you can see in the photo, the middle portion of the San Juanico bridge forms
a curve that resembles a parabola. The curve can be modeled with the quadratic function
y = ax2 + bx + c where a < 0 since the parabola opens downwards.

Pre-Calculus and Calculus.indb 207 13/7/2017 5:57:12 PM


Chapter 8 • Graphs of Quadratic Equations

8 Graphs of Quadratic Equations


8.1 Parabolas
8.2 Circles
8.3 Ellipses
8.4 Hyperbolas

8.1 Parabolas
8.1.1 Definition of a Parabola
8.1.2 Equation of a Parabola
8.1.3 Parametric Equations of a Parabola

axis of symmetry 8.1.1 Definition of a Parabola


P
Definition 8.1

A parabola is the set of all points in a plane, which are the same distance
F from a fixed point F as they are from a fixed line D. The fixed point F
V is called the focus of the parabola, and the fixed line is the directrix.
directrix, D
Figure 8a shows a parabola. The point P is on the parabola, F is the
focus and V is called the vertex. The distance from point P to the focus
is the same as the perpendicular distance from P to the directrix, that is
Figure 8a . The line through the focus and perpendicular to the directrix is
called the axis of symmetry of the parabola. The distance from the vertex
V to focus F is the same as the distance from V to the directrix.

8.1.2 Equation of a Parabola


Now, we are going to derive the equation of a parabola. We will look at
parabolas where the focus is on the x-axis or on the y-axis only.

Equation of a parabola with vertex at (0, 0) and focus (0, p)


The focus, with coordinates (0, p), of this parabola is on the y-axis. If p is
positive, then the parabola opens upwards, as shown in Figure 8b.

y From Figure 8b, the distance from F to V is p. The distance from V to the
directrix is also p, thus the equation of the directrix is y = – p.

Let P (x, y) be any point on the parabola, then


P (x, y)
F (0, p)
p x
V p
D y = –p
Squaring both sides,
Figure 8b

208
The equation of the parabola is

Pre-Calculus and Calculus.indb 208 13/7/2017 5:57:12 PM


Chapter 8 • Graphs of Quadratic Equations

In Figure 8c, the coordinates of the focus is (0, – p) and the graph of the y
parabola opens downwards. The directrix is the line y = p. Using the same
y= p
method used for p > 0, it can be shown that the equation of this parabola V p
is x2 = –4py. x
p D

Example 1 F(0, –p)


P (x, y)
Find the focus, directrix and vertex of each of the following parabolas,
and sketch its graph.
(a) x­2 = 9y (b) x2 = –6y
Figure 8c
SOLUTION
(a) x2 = 9y y
x2 = 9y  4p = 9
9
p = __ ​​  4 ​​ 
​  4 ​)​​  
4 ​ )​​  
F(
Focus = (​​  0, ​ __
9
​​  0, __ 9

x 9
0 Directrix, y = –​​ __ 4  ​​
y = –​__​  9 ​​ Vertex = (0, 0)
4

(b) x2 = –6y
y
3
x2 = –6y  4p = –6
y = __
​​  2 ​​ 3
p = –​​ __ 2  ​​

2 ​ )​​  
x
Focus = (​​  0, –​ __
0 3

F​​ (0, –​ __
2 )​ ​​  
3
3
Directrix, y = ​​ __2  ​​ y
Vertex = (0, 0) P (x, y)

Equation of a parabola with vertex at (0, 0) and focus (p, 0) p p


x
The focus of this parabola is on the x-axis. If p is positive, then the parabola V F (p, 0)
opens to the right, as shown in Figure 8d.
x = –p
Referring to Figure 8d, the distance from F to V is p, thus the coordinates of
the focus is (p, 0), p > 0. The equation of the directrix is x = – p. Figure 8d
y
Let P(x, y) be any point on the parabola, then P (x, y)
|PF| = |PD| D
__________
​​√ (x – p) + y2 ​​  2
= |x + p|
Squaring both sides, x – 2px + p + y = (x + p)2
2 2 2 V
x
F (–p, 0) p p
x2 + y2 – 2px + p2 = x2 + 2px + p2
x=p
The equation of the parabola is y2 = 4px
In Figure 8e, the coordinates of the focus is (–p, 0) and the graph of the
parabola opens to the left. The equation of the directrix is x = p. It can be Figure 8e
shown that the equation of this parabola is y2 = –4px. 209

Pre-Calculus and Calculus.indb 209 13/7/2017 5:57:12 PM


Chapter 8 • Graphs of Quadratic Equations

Example 2
Find the focus, directrix and vertex of each of the following parabolas,
and sketch its graph.
(a) y2 = 4x (b) y2 = –3x

SOLUTION
(a) y
y2 = 4x  p = 1
Focus = (1, 0)
x = –1 Directrix, x = –1
Vertex = (0, 0)
x
V(0, 0) F(1, 0)

(b) y2 = –3x  4p = –3
y
3
p = __ ​​  4 ​​ 

4 ​ , 0)​​  
Focus = (​​  –​ __
3
x 3
V(0, 0) Directrix, x = __ ​​  4 ​​ 
Vertex = (0, 0)

Example 3
Find the equations of the following parabolas that have its vertex at the
origin and satisfies the given condition.
(a) Focus F (0, 2)
(b) Focus F (–8, 0)

SOLUTION
(a) V (0, 0), F (0, 2), p = 2
y



The equation of the parabola is x2 = 8y.

F(0, 2)
x
0

(b) V (0, 0), F (–8, 0), p = –8


y

x
0
F(–8, 0)
The equation of the parabola is y2 = –32x.

210

Pre-Calculus and Calculus.indb 210 13/7/2017 5:57:12 PM


Chapter 8 • Graphs of Quadratic Equations

Equation of a parabola with vertex (h, k) and focus (h, k + p) y

If a parabola with vertex at (0, 0) and focus (0, p) is shifted horizontally x=h
h units and vertically k units, we have a parabola with a vertex at (h, k)
and axis of symmetry parallel to the y-axis. The axis of symmetry of this
P(x, y)
parabola is the line x = h. For convenience, let us take h > 0, k > 0. First, F(h, k + p)
we look at the parabola which opens upwards as shown in Figure 8f.
p
k
The directrix is the line y = k – p, and the coordinates of point D on the p V(h, k) y =k –p
D (x, k – p)
directrix is (x, k – p). Since |PF| = |PD|, we have x

|PF| = |PD| Figure 8f


___________________ y
​​√ (x
   – h)2 + [(y – (k + p)]2 ​​= |y – (k – p)|
(x – h)2 + (y – k – p)2 = (y – k + p)2 D (x, k + p) y = k +p
p
Expand and simplify it, we obtain the equation of the parabola k V(h, k)
p
(x – h)2 = 4p(y – k) F(h, k – p)
P (x, y)
Figure 8g shows a parabola which opens downwards. The focus is x
F (h, k – p) and directrix is y = k + p. It can be easily shown that the
equation of the parabola is (x – h)2 = –4p(y – k).

Example 4 Figure 8g

Find the vertex, focus and directrix of each of the following parabolas,
and sketch its graph.
(a) (x – 3)2 = 8(y + 1)
(b) (x + 5)2 = –16(y + 2)

SOLUTION
y
(a) (x – 3)2 = 8(y + 1)
(x – 3)2 = 8[y – (–1)]
Comparing with the general form
(x – h)2 = 4p(y – k), we find that
h = 3, k = –1 and p = 2. F(3, 1)
x
Hence, the vertex V (h, k) = (3, –1)
V(3, –1)
and the focus F (h, k + p) = (3, 1).
The directrix is the line y = k – p y = –3
y = –3
(b) (x + 5)2 = –16(y + 2)
[x – (–5)]2 = –16[y – (–2)] y
Comparing with the general form y=2
(x – h)2 = –4p(y – k), we find that x
V (–5, –2)
h = –5, k = –2 and p = 4.
Hence, the vertex V(h, k) = (–5, –2)
and the focus F(h, k – p) = (–5, –6). F(–5, –6)
The directrix is the line y = k + p
y=2

211

Pre-Calculus and Calculus.indb 211 13/7/2017 5:57:12 PM


Chapter 8 • Graphs of Quadratic Equations

y Equation of a parabola with vertex (h, k) and focus (h + p, k)


If a parabola with vertex at (0, 0) and focus (p, 0) is shifted horizontally
h units and vertically k units, we have a parabola with a vertex at (h, k)
D (h – p, y) P (x, y) and axis of symmetry parallel to the x-axis. The axis of symmetry of this
parabola is the line y = k. The graph of parabola which opens to the right
p p F (h + p, k) is as shown in Figure 8h.
V (h, k) y=k
The directrix is the line x = h – p, and the coordinates of point D on the
x
h directrix is (h – p, y). Since |PF| = |PD|, we have
___________________ |PF| = |PD|
x = h –p ​​√ [x
   – (h + p)]2 + (y – k)2 ​​ = |x – (h – p)|
Figure 8h (x – h – p)2 + (y – k)2 = (x – h + p)2
y
Expand and simplify it, we obtain the equation of the parabola
(y – k)2 = 4p(x – h)
P (x, y)
D (h + p, y) The parabola which opens to the left is shown in Figure 8i. The focus is
F(h – p, k) and the directrix is . The equation of the parabola is
p p
(y – k) = –4p(x – h).
2

F (h – p, k) V (h, k) y=k

x Example 5
h
Find the vertex, focus and directrix of each of the following parabolas,
x = h +p and sketch its graph.
(a) (y – 3)2 = 8(x – 2)
Figure 8i (b) (y + 2)2 = 4(x + 1)
(c) (y – 1)2 = –4(x + 2)

SOLUTION
(a) Comparing (y – 3)2 = 8(x – 2) with the general form
(y – k)2 = 4p(x – h), we find that k = 3, h = 2 and p = 2.
Hence, the vertex V (h, k) = V (2, 3) and the focus
F (h + p, k) = F (4, 3).
y
The directrix is x = h – p
x=0

V (2, 3) F (4, 3)

x
x=0
(b) (y + 2)2 = 4(x + 1)
[y – (–2)]2 = 4[x – (–1)]
Comparing with the general form (y – k)2 = 4p(x – h), we find that
k = –2, h = –1 and p = 1.
Hence, the vertex V (h, k) = V (–1, –2) and the focus
F (h + p, k) = F (0, –2).
The directrix is x = h – p
x = –2
212

03 Chapter 8.indd 212 18/7/2017 9:56:08 AM


Chapter 8 • Graphs of Quadratic Equations

x
V (–1, –2) F (0, –2)

x = –2

(c) (y – 1)2 = –4(x + 2)


(y – 1)2 = –4[x – (–2)]
Comparing with the general form (y – k)2 = –4p(x – h), we find that
k = 1, h = –2 and p = 1.
Hence, the vertex V (h, k) = V (–2, 1) and the focus
F (h – p, k) = F (–3, 1).
The directrix is x = h + p
x = –1
y
x = –1

F (–3, 1) V (–2, 1)

Note
Example 6
The axis of symmetry
Find the equation of the parabola with vertex at (4, –2) and focus at of parabola
(6, –2). Also, state the equation of the directrix. (y – k)2 = 4p(x – h) is parallel
to the x-axis and the axis of
SOLUTION symmetry of
The vertex and focus both lie on the line y = –2, which is the axis (x – h)2 = 4p(y – k) is parallel
to the y-axis.
of symmetry. The focus lies to the right of the vertex, therefore, the
parabola opens to the right. The distance between vertex and focus is
p = 6 – 4 = 2. Thus, the general form of the equation of parabola is
(y – k)2 = 4p(x – h), where h = 4, k = –2, p = 2.
The equation is
(y + 2)2 = 4(2)(x – 4)
(y + 2)2 = 8(x – 4)
The directrix is x = h – p
x=2

Determining the vertex and focus by completing the square


If the equation of the parabola is given as ax2 + bx + cy + d = 0 or
ay2 + by + cx + d = 0, instead of the standard forms (y – k)2 = 4p(x – h) or
(x – h)2 = 4p(y – k), then we use completing the square method to determine
the vertex and focus of the parabola.
213

Pre-Calculus and Calculus.indb 213 13/7/2017 5:57:12 PM


Chapter 8 • Graphs of Quadratic Equations

Example 7
Find the vertex, focus and directrix of each of the following parabolas,
and sketch its graph.
(a) x2 – 6x – 8y + 1 = 0
(b) y2 – 2y = 4x – 9
(c) y2 = 16x – 8

SOLUTION
(a) x2 – 6x – 8y + 1 = 0
Complete the square as follows:
(x2 – 6x) – 8y + 1 = 0
​​ [​x​​  2​– 6x + (​​  –​ __ )  ]   ( 2 ​​​  ​​ – 8y + 1 = 0
)
2 2
6 __ 6
2  ​
  ​​​ ​ ​​ – ​​​  –​   ​

[x – 6x + (–3) ] – (–3)2 – 8y + 1 = 0
2 2

(x – 3)2 – 8y – 8 = 0
(x – 3)2 = 8(y + 1)
(x – 3)2 = 8[y – (–1)]
The parabola opens upwards.
Comparing with the standard form (x – h)2 = 4p(y – k), we have
h = 3, k = –1 and p = 2.
Hence, the vertex V (h, k) = (3, –1) and the focus F (h, k + p) = (3, 1).
Note The directrix is y = k – p
y
y = –3
The value of p is the distance
from the vertex to the focus
or to the directrix. F (3, 1)

V (3, –1)

y = –3

(b) y2 – 2y = 4x – 9
(y – 1)2 – 1 = 4x – 9
(y – 1)2 = 4x – 8
(y – 1)2 = 4(x – 2)
The parabola opens to the right.
Comparing with the standard form (y – k)2 = 4p(x – h), we have
h = 2, k = 1 and p = 1.
Hence, the vertex V (h, k) = (2, 1) and the focus F (h + p, k) = (3, 1).
The directrix is x = h – p y x=1
x=1

V (2, 1) F (3, 1)

 x

214

Pre-Calculus and Calculus.indb 214 13/7/2017 5:57:12 PM


Chapter 8 • Graphs of Quadratic Equations

(c) y2 = 16x – 8
​ 12  ​)​​  
y2 = 16​​ (x – __
The parabola opens to the right.
Comparing with (y – k)2 = 4p(x – h), we have k = 0, h = ​​ __ 1
2 ​​ and p = 4.
​  21 ​ , 0)​​ and the focus F (h + p, k) = (​​  4​ __
Hence, the vertex V(h, k) = (​​  __ 2 ​ , 0).​​  
1
The directrix is x = h – p
1  ​​
x = –3​​ __
2
​  21 ​​
x = –3​__ y

x
1 ​, 0 ​​  
​  21 ,​ 0)​​  
V (​​  __ F (​​  4​ __
2 )

8.1.3 Parametric Equations of a Parabola


In the Cartesian coordinate system, we represent a function in terms of x
and y. We may also represent a function using other variable, such as t.
This other variable is called a parameter.

Definition 8.2

An equation can be expressed in the form of two equations, x = f(t) and


y = g(t), called parametric equations, with t as the parameter.

The parametric equations of a parabola are stated in the following theorem.

Theorem 8.1

If P (x, y) is a point on a parabola,


(i) x = at2 and y = 2at are the parametric equations of parabola y2 = 4ax,
(ii) x = 2at and y = at2 are the parametric equations of parabola x2 = 4ay.

Usually, the parametric equations are given and we are required to find the
Cartesian equation of the curve. To do this, we need to eliminate the parameter.

215

Pre-Calculus and Calculus.indb 215 13/7/2017 5:57:13 PM


Chapter 8 • Graphs of Quadratic Equations

Example 8
Find the Cartesian equation of the curve represented by the parametric
equations, x = 2t2 and y = 4t.

SOLUTION
x = 2t2 —— (1)
y = 4t —— (2)
To obtain the equation of the curve, we need to eliminate t.
y
From equation (2), t = ​​ __ 4  ​​  .
​  4  ​)​​​  ​​
y y 2
​​  4  ​​into equation (1), x = 2​​​ (__
Substituting t = __
16x = 2y2
y2 = 8x
The Cartesian equation is y = 8x and the curve is a parabola.
2

Exercise 8.1
1. Find the vertex, focus, axis of symmetry and directrix for the following parabolas.
(a) y2 = 8x
(b) x2 = 3y
(c) x2 = –6y
(d) y2 = – x
(e) y2 = 4 – 6x
(f) y2 = 4x – 8
(g) y2 + 2y + 12x + 25 = 0
(h) x2 + 8x + 4y + 12 = 0

2. Sketch the following parabolas.


(a) y2 – 4y – 2x – 4 = 0
(b) y = x­2 + x + 1
(c) x = y2 + y + 1

3. Find the equations of the following parabolas that satisfy the given conditions.
(a) Vertex V (0, 2), focus F (5, 2)
(b) Vertex V (1, –2), focus F (1, 0)
(c) Focus (4, 3), directrix y = –1

4. Sketch the curve represented by the parametric equations, x = 4t2 and y = 8t.

216

Pre-Calculus and Calculus.indb 216 13/7/2017 5:57:13 PM


Chapter 8 • Graphs of Quadratic Equations

5. Find the focus, directrix and vertex of the following parabolas and sketch their graphs.
(a) x2 = 4y
(b) y = 5x2
​​  12 ​​ y2
(c) x = __
(d) y2 = –6x
(e) x2 = –2y
(f) 3y2 = 5x

6. Find the equations of the following parabolas that have its vertex at the origin and satisfy the given
conditions.
(a) Focus, F (0, 2)
(b) Directrix, x = 2
(c) Directrix, y = 1
(d) Focus, F (3, 0)

7. Find the vertex and focus of each of the following parabolas and sketch its graph.
(a) y2 = 4x – 12
(b) (y + 4)2 = 2x – 2
(c) y2 – 6y = x + 3
(d) y2 = 8x – 8
(e) x2 = 8y – 24
(f) x2 –4x = 12y + 8
(g) (x + 3)2 = –4y + 4
(h) y2 + 2y = –16x – 9

8. Find the equation of the parabola whose axis is parallel to the x-axis, vertex is (–4, –2) and passes
through the point (–8, 1). Hence, sketch the parabola.

217

Pre-Calculus and Calculus.indb 217 13/7/2017 5:57:13 PM


Chapter 8 • Graphs of Quadratic Equations

8.2 Circles
8.2.1 Definition of a Circle
8.2.2 Equation of a Circle
8.2.3 Family of Circles
8.2.4 Points of Intersection of a Straight Line and a Circle
8.2.5 Equation of a Tangent to a Circle
8.2.6 Length of a Tangent from a Point to a Circle
8.2.7 Parametric Equations of a Circle

y 8.2.1 Definition of a Circle


P(x, y) Definition 8.3
r
x –h
A circle is the set of all points in a plane, which are at a fixed distance
Q
C(h, k) y – k from a fixed point. The fixed point is the center of the circle and the
fixed distance is the radius of the circle.
x
Figure 8j shows a circle with center C(h, k) and radius r.
Figure 8j
8.2.2 Equation of a Circle
From the definition of the circle, a point P(x, y) lies on the circle if and
______________
only if , that is, √ (x – h)2 + (y – k)2 ​​ = r.
​​    
Squaring both sides, we have (x – h)2 + (y – k)2 = r2  ––– (1)
Equation (1) is the equation of a circle with center at point (h, k) and radius
of length r units. It is called the center-radius equation of the circle.
If the center is the origin, then (h, k) = (0, 0) and the equation of the circle
is x2 + y2 = r2.

Example 9
State the coordinates of the center and the radius of the circle
(x – 2)2 + (y + 4)2 = 64.

SOLUTION
The equation can be written as (x – 2)2 + [y – (–4)]2 = 82.
Hence, the center (h, k) = (2, –4) and the radius r = 8 units.

Example 10
Write the equation of a circle which has center (–5, –12) and the length
of its radius is 3 units.

SOLUTION
The equation is [x – (–5)]2 + [y – (–12)]2 = 32
(x + 5)2 + (y + 12)2 = 9

218

Pre-Calculus and Calculus.indb 218 13/7/2017 5:57:13 PM


Chapter 8 • Graphs of Quadratic Equations

If we expand equation (1), we have x2 + y2 – 2xh – 2yk + h2 + k2 – r­2 = 0.


Substituting g = –h, f = –k and c = h2 + k2 – r2, we have
x2 + y2 + 2gx + 2fy + c = 0  ––– (2)
Equation (2) is called the general equation of a circle. An equation of a
circle in center-radius form can be written in general form and vice versa.
If a circle is given in the general form, the radius and the center of the
circle can be determined by completing the square on equation (2).
(x2 + 2gx) + (y2 + 2fy) + c = 0
(x2 + 2gx + g2) + (y2 + 2fy + f 2) + c = g2 + f 2
(x + g)2 + (y + f )2 = g2 + f 2 – c
which is in center-radius form. Thus, the center is (–g, –f ) and the radius is​​
_________
√ g
  2
+ f 2
– c ​​. 

Example 11
Find the center and radius of each circle and sketch its graph.
(a) x2 + y2 – 2x + 4y – 20 = 0
(b) x2 + y2 + 2x = 4
(c) 2x2 + 2y2 – 2x + 6y – 13 = 0

SOLUTION
(a) x2 + y2 – 2x + 4y – 20 = 0
(x2 – 2x) + (y2 + 4y) = 20
By completing the square,

​​ [​x​​  2​– 2x + (​​  ___2 ​ )​​​  ​]​​  + [​​  y + 4y + (​​  ​  2 ​ )​​​  ​]​​  = 20 + (​​​  ​  2 ​ )​​​  ​​ + (​​​  ​  2 ​ )​​​  ​​
2 2 2 2
​  –2 2 __ 4 –2
___ __ 4

(x – 1)2 + (y + 2)2 = 25
The center (h, k) = (1, –2) and the radius r = 5 units.
Alternative method:
Comparing the given equation, x2 + y2 – 2x + 4y – 20 = 0 with the
general form x2 + y2 + 2gx + 2fy + c = 0, we have g = –1, f = 2 and
c = –20.
the center (–g,___–f ) = (1, –2) and the radius
Thus,_________
r = ​​√ g2 + f 2 – c  
​​ = √
​​  25 ​​ = 5 units.
y

x
0
5
(1, –2)

219

Pre-Calculus and Calculus.indb 219 13/7/2017 5:57:13 PM


Chapter 8 • Graphs of Quadratic Equations

(b) x2 + y2 + 2x = 4
(x2 + 2x) + y2 = 4
(x + 1)2 + y2 = 5 __
The center (h, k) = (–1, 0) and the radius r = √
​​  5 ​​  units.
y

x
(–1, 0)

(c) 2x2 + 2y2 – 2x + 6y – 13 = 0


13
x2 + y2 – x + 3y – ___ ​​  2 ​​ = 0
1 3 13
Comparing with the general form, we have g = –​​ __ __ ___
2 ​​ ,   f  = ​​  2 ​​ and c = –​​  2 ​​ .
2  ​)​​  and r = √
_________
 C(–g, –f ) = (​​  __
3
​  21 ​ , –​ __ g2 + f 2 – c  
​​  __________ ​​
= ​​  __ √
​  41  ​ + __
9 13
​  4 ​  + ___
​  2 ​ ​​  
= 3 units
y

3
x

Example 12
Find the equations of the following circles that satisfy the given conditions.
(a) Center (2, –1) and radius 3 units
(b) Center at the origin and passes through (4, 7)
(c) Endpoints of diameter are P (–1, 3) and Q (7, –5)
(d) Center (7, –3) and touches the x-axis

SOLUTION
(a) h = 2, k = –1, r = 3
(x – h)2 + (y – k)2 = r2
(x – 2)2 + [y – (–1)]2 = 32
(x – 2)2 + (y + 1)2 = 9
k = 0, center passes through (4, 7)
(b) h = 0,_______________
r = ​​√ ___(4
   – 0)2 + (7 – 0)2 ​​
=√​​  65 ​​ units ___ 2
(x – 0)2 + (y – 0)2 = (​​​  √ ​  65 ​) ​​​  ​​
x2 + y2 = 65
220

Pre-Calculus and Calculus.indb 220 13/7/2017 5:57:13 PM


Chapter 8 • Graphs of Quadratic Equations

(c) C = (​​  ______ )​​  


3 + (–5)
–1 + 7 _______
​  2 ​,   ​  2 ​   
= (3, –1)
h= 3, k = –1
________________
r = ​​√ _______
  
(3 + 1)2 + (–1 – 3)2 ​​
=√ ​​  ___
16 + 16 ​​ 
= ​​√ 32 ​​  ___
(x – 3)2 + (y + 1)2 = (​​√ 32 ​​) 2
(x – 3)2 + (y + 1)2 = 32
(d) h = 7, k = –3
y

7
x
0

–3
(7, –3)


r=3
(x – 7)2 + (y + 3)2 = 32
(x – 7)2 + (y + 3)2 = 9

Now, let us look at how the equation of a circle can be formed when two
points on the circle and the diameter of the circle is given.
Example 13
Find the equation of the circle that passes through the points A (0, 3)
and B (4, –1) and having its center on the line, 3x + 4y + 1 = 0.

SOLUTION
A (0, 3)

C
B (4, –1)

3x + 4y + 1 = 0
The perpendicular bisector of the chord joining points A and B passes
through the center of the circle.
Midpoint of AB = (​​  _____ )​​  
3 + (–1)
0 + 4 _______
​  2 ​, 
  ​ 
2 ​   

= (2, 1)
3 – (–1)
Gradient of AB = _______
​​  0 – 4 ​​   

= ​​ ___ 4
–4  ​​ 
=–1
∴ The gradient of the perpendicular bisector of AB is 1. 221

Pre-Calculus and Calculus.indb 221 13/7/2017 5:57:13 PM


Chapter 8 • Graphs of Quadratic Equations

Equation of the perpendicular bisector of AB:


y – 1 = 1(x – 2)
y–x+1=0
To find the center of the circle, we solve the equations 3x + 4y + 1 = 0
and y – x + 1 = 0 simultaneously.
3x + 4y + 1 = 0 –––(1)
–x + y + 1 = 0 –––(2)
Equation (2) × 3: –3x + 3y + 3 = 0 –––(3)
Equation (1) + equation (3): 7y + 4 = 0
4  ​​
y = –​​ __
7
Substituting into equation (2), –x – ​​ __ 4
7 ​​  + 1 = 0
3
x = __
​​  7 ​​ 
∴ The center of the circle is (​​  __ 7  ​).​​  
3 4
​  7 ​ , –​ __
_________________

Radius of the circle = ​​ ( ​  7 ​  – 0)​​​  ​ + (​​  –​ __


√ 7 ​  – 3)​​​  ​ ​​
2 2
__ 3 4
 ​​   
________

√ = ​​  ___
9 625
​  49  ​ + ____
____
​  49 ​ ​​  

√ = ​​  ____
634
​  49 ​ ​​  units
∴ The equation of the circle is
4 ​  ​​​  ​ ​​  = ____
​ 7  ​)​​​  ​​ + [​​  y – (​​  –​ __
​​​ (x – __ 7 ) ] ​​  49 ​​ 
2 2
3 634

6 9 8 16 ____ 634
x2 – __​​  7 ​​ x + ​​ ___ __ ___
49  ​​ + y + ​​  7 ​​ y + ​​  49 ​​ = ​​  49 ​​ 
2

49x2 – 49y2 – 42x + 56y + 9 + 1+ = 634


49x2 + 49y2 – 42x + 56y – 609 = 0

If three points on a circle are given, the equation of the circle can be
obtained by substituting the points into the general equation of the circle.
The next example shows how to find the equation.

Example 14
Find the equation of the circle that passes through points A (2, 3),
B (4, –1) and C (2, –1).

SOLUTION
The general equation of the circle is x2 + y2 + 2gx + 2fy + c = 0.
At point A(2, 3), 4 + 9 + 4g + 6f + c = 0
4g + 6f + c = –13 ––– (1)
At point B(4, –1), 16 + 1 + 8g – 2f + c = 0
8g – 2f + c = –17 ––– (2)
At point C(2, –1), 4 + 1 + 4g – 2f + c = 0
4g – 2f + c = –5 ––– (3)

222

Pre-Calculus and Calculus.indb 222 13/7/2017 5:57:13 PM


Chapter 8 • Graphs of Quadratic Equations

Equation (1) – equation (3): 8f = – 8


f = –1
Equation (2) – equation (3): 4g = –12
g = –3
Substituting f = –1 and g = –3 into equation (1),
–12 – 6 + c = –13
c = 5
Hence, the equation of the circle is x2 + y2 – 6x – 2y + 5 = 0.
Alternative method:
The perpendicular bisectors of the chords AB and BC meet at the center
of the circle.
A (2, 3)

C (2, –1) B (4, –1)


3 – (–1) ___ 4  ​​ = –2
mAB = _______
​​  2 – 4 ​​ 

= ​​  –2

​​  21 ​​ 
m AB = __
Midpoint of AB = (​​  _____ )​​  
3 + (–1)
​  2 + 4   _______
2 ​,  ​  2 ​   
= (3, 1)
Equation of the perpendicular bisector of AB:
​​ 21 ​​ (x – 3)
y – 1 = __
2y – 2 = x – 3
2y – x = –1  ––– (1)
–1 – (–1)
mBC = ________
​​  2 – 4 ​​  
= 0
m BC = 0
Midpoint of BC = (​​  _____  ​​  
)
–1 + (–1)
​  2 + 4   ________
2 ​,  ​  2 ​  
= (3, –1)
Equation of the perpendicular bisector of BC: x = 3  ––– (2)
Substituting equation (2) into equation (1),
2y – 3 = –1
2y = 2
y = 1
∴ The center of the circle is (3, 1).
_________________
Radius of the circle = OC = √ ​​  __
(3
   – 2)2 + [1 – (–1)]2 ​​
= ​​√ 5 ​​  units
∴The equation of the circle is (x – 3)2 + (y – 1)2 = 5
x2 – 6x + 9 + y2 –2y + 1 – 5 = 0
x2 + y2 – 6x – 2y + 5 = 0
223

Pre-Calculus and Calculus.indb 223 13/7/2017 5:57:13 PM


Chapter 8 • Graphs of Quadratic Equations

8.2.3 Family of Circles


Let us consider two circles C1 and C2, where C1 = x2 + y2 + 2g1x + 2f1 y + c1,
and C2 = x2 + y2 + 2g2x + 2f2 y + c2.
We are going to look at cases where the two circles intersect at two points,
and where the two circles intersect at one point.
When two circles intersect, the equation obtained is C1 + kC2 = 0  ––– (1)
where k is an arbitrary constant.
Case 1: C1 and C2 intersect at two points
When k ≠ –1, equation (1) represents a family of circles which pass through
the two intersection points. Figure 8k(i) shows the two circles, C1 and C2
and the family of circles are shown in dotted lines.

C2

C1
common chord

(i) k ≠ –1 (ii) k = –1
Figure 8k
When k = –1, equation (1) becomes a linear equation C1 – C2 = 0. This is
the equation of the line passing through the two intersection points. This
line is the common chord, as seen in Figure 8k(ii).

Case 2: C1 and C2 intersect at one point


If the circles intersect at one point only, it means the circles touch each
other. There are two ways the circles touch each other, either externally or
internally as in Figure 8l.

C1 C2 C1

C2

(i) C1 and C2 touch externally (ii) C1 and C2 touch internally

Figure 8l

224

Pre-Calculus and Calculus.indb 224 13/7/2017 5:57:13 PM


Chapter 8 • Graphs of Quadratic Equations

When the circles touch and , equation (1) represents the family of
circles (in dotted lines) as shown in Figure 8m(i) and 8m(ii). When k = –1,
equation (1) becomes a linear equation and it is the tangent line to both
circles, as shown in Figure 8m(iii) and 8m(iv).

(i) k ≠ –1 (ii) k ≠ –1

(iii) k = –1 (iv) k = –1
Figure 8m

Example 15
Find the points of intersection of the following two circles:
x2 + y2 – 4x + 4y – 12 = 0 and x2 + y2 – 5x + 3y – 4 = 0.
SOLUTION

common chord

x2 + y2 – 4x + 6y – 12 = 0  ––– (1)
x2 + y2 – 5x + 3y – 4 = 0  ––– (2)
Equation (1) – equation (2): x + 3y – 8 = 0  ––– (3)
This is the equation of the common chord.
From equation (3), x = 8 – 3y.
Substituting x = 8 – 3y into equation (1),
(8 – 3y)2 + y2 – 4(8 – 3y) + 6y – 12 = 0
64 – 48y + 9y2 + y2 – 32 + 12y + 6y – 12 = 0
10y2 – 30y + 20 = 0
y2 – 3y + 2 = 0
(y – 1)(y – 2) = 0
y = 1 or 2
When y = 1, x = 8 – 3 = 5.
When y = 2, x = 8 – 6 = 2.
Hence, the points of intersection are (5, 1) and (2, 2).
225

Pre-Calculus and Calculus.indb 225 13/7/2017 5:57:13 PM


Chapter 8 • Graphs of Quadratic Equations

Example 16
Write down the equation which represents the family of circles
passing through the intersection points of two circles C1 and C2,
where C1 = x2 + y2 – 4x + 6y – 12 and C2 = x2 + y2 – 5x + 3y – 4.
Find the equations of the circles when k = –2 and k = 1. Sketch C1, C2
and the circles obtained when k = –2 and k = 1.
SOLUTION
The family of circles are represented by C1 + kC2 = 0.
That is, x2 + y2 – 4x + 6y – 12 + k(x2 + y2 – 5x + 3y – 4) = 0.
When k = –2,
x2 + y2 – 4x + 6y – 12 – 2(x2 + y2 – 5x + 3y – 4) = 0
–x2 – y2 + 6x – 4 = 0
x2 + y2 – 6x + 4 = 0 —— (1)
The equation of the circle is x2 + y2 – 6x + 4 = 0. _________
This is a circle with center (–g, –f ) = (3, 0) and radius r = ​​√ __ g2 + f 2 – c   ​​
= ​​√ 5 ​​  units
When k = 1,
x2 + y2 – 4x + 6y – 12 + (x2 + y2 – 5x + 3y – 4) = 0
2x2 + 2y2 – 9x + 9y – 16 = 0
9 9
x2 + y2 – __ ​​  2 ​​ x + ​​ __ 2  ​​y – 8 = 0 —— (2)
9 9
The equation of the circle is x2 + y2 – __ ​​  2 ​​ x + ​​ __
2  ​​y – 8 = 0.

_________
This is a circle with center (–g, –f ) = (​​  __ )
9 9
​  4 ​ , –​ __
4  ​
  ​​ and radius r = ​​
√ g2 + f 2 – c ​​  

____

_________ √
= ​​  ____
145
​  8 ​ ​​   units
C1 has center (–g, –f ) = (2, –3) and radius r = √ ​​  g2 + f 2 – c  
​​= 5___ units.

2  ​)​​  and radius r = ​​√ g + f – c ​​   √ 


_________
C2 has center (–g, –f ) = (​​  __
5 3 25
​  2 ​ , –​ __ 2 2
= ​​ ___​  2 ​ ​​  units.
From Example 15, the points of intersection of C1 and C2 are (5, 1) and
(2, 2).
Based on the radius, the sizes of the circles are:
equation (1) < C2 < equation (2) < C1.
The sketches of C1, C2 and the circles obtained when k = –2 and k = 1
are shown below.
y

(2, 2) (5, 1)
x

k = –2
C2

k =1
C1

226

Pre-Calculus and Calculus.indb 226 13/7/2017 5:57:13 PM


Chapter 8 • Graphs of Quadratic Equations

8.2.4 Points of Intersection of a Straight Line


and a Circle
Consider a line with equation y = mx + c′ and a circle C with equation
x2 + y2 + 2gx + 2fy + c = 0. To find the points of intersection of the line and
the circle, we substitute y = mx + c′ into C. We will then obtain a quadratic
equation in x.
Let the quadratic equation be y = a1x2 + b1x + c1.
If the discriminant D = b12 – 4a1c1 > 0, the line cuts the circle at two points.
D = b12 – 4a1c1 = 0, the line touches the circle (the line
is a tangent to the circle).
D = b12 – 4a1c1 < 0, the line and the circle never meet
(neither touch nor intersect).

Example 17
Given a line, y – 7x = 2 and a circle, x2 + y2 + 8x + 2y – 8 = 0, determine
whether the line and the circle touch, intersect at two points or never
meet. If they intersect, find the points of intersection.

SOLUTION
Substituting y = 7x + 2 into x2 + y2 + 8x + 2y – 8 = 0,
x2 + (7x + 2)2 + 8x + 2(7x + 2) – 8 = 0
x2 + 49x2 + 28x + 4 + 8x + 14x + 4 – 8 = 0
50x2 + 50x = 0
a = 50, b = 50, c = 0
D = b2 – 4ac = 502 – 4(50)(0)
= 2500 > 0
Therefore, the line and the circle intersect at two points.
Solving 50x2 + 50x = 0, x2 + x = 0
x(x + 1) = 0
x = 0 or –1
When x = 0, y = 2.
When x = –1, y = –7 + 2 = –5.
∴ The points of intersection are (0, 2) and (–1, –5).

8.2.5 Equation of a Tangent to a Circle


Let P (x1, y1) be a point on a circle. A tangent to the circle at point P (x1, y1)
is the line which touches the circle at this point. The equation of the
tangent is obtained by finding the gradient and then forming the equation.
The gradient is usually found using differentiation. In this section, we will
not use differentiation. Instead, we will use the general method to find
the gradient. A normal to a circle at point P (x1, y1) is the line that is
perpendicular to the tangent line at that point. If the gradient of the tangent
is mT and the gradient of the normal is mN , then the product of and
is –1. 227

Pre-Calculus and Calculus.indb 227 13/7/2017 5:57:14 PM


Chapter 8 • Graphs of Quadratic Equations

Let us look at the following example.


Example 18
Find the equation of the tangent and normal to the circle,
x2 + y2 – 6x + 3y – 5 = 0 at the point (−1, −2).

SOLUTION
2 ​ )​​. 
Center of the circle (–g, –f ) = (​​  3, –​ __
3

(  3 ​)​​  
​​  3, –​__
2

(–1, –2)

normal
tangent

2 ​ )​  ___
–2 – (​  –​ __
3 1
_________
–​ __
12 ​ 
= ​​  –4 ​​ = ​​ __
Gradient of normal = ​​  –1 – 3 ​​ 
  8 ​​ 
Note
Gradient of tangent = –8
mT × mN = –1 Equation of tangent:
y – (–2) = –8[x – (–1)]
y + 2 = –8x – 8
y + 8x + 10 = 0
Equation of normal:
1  ​​[x – (–1)]
y – (–2) = ​​ __
8
y + 2 = ​​ 18 ​​ (x + 1)
__

8y + 16 = x + 1
8y – x + 15 = 0

The equation of the tangent can also be obtained using the following
theorem.
Theorem 8.2

(i) If the equation of the circle is x2 + y2 + 2gx + 2 f y + c = 0, then the


tangent to the circle at point P (x1, y1) is
xx1 + yy1 + (g)(x + x1) + ( f )(y + y1) + c = 0
(ii) If the equation of the circle is x2 + y2 = r2, then the tangent to the
circle at point P (x1, y1) is xx1 + yy1 = r2.

228

Pre-Calculus and Calculus.indb 228 13/7/2017 5:57:14 PM


Chapter 8 • Graphs of Quadratic Equations

Let us use Theorem 8.2 for the next example.


Example 19
Find the equations of the tangent and normal to the following circles at
the given points.
(a) x2 + y2 = 18, (–3, 3)
(b) x2 + y2 – 18x + 6y + 25 = 0, (1, –2)

SOLUTION
(a) Using Theorem 8.2, the equation of the tangent:
x(–3) + y(3) = 18
3y = 3x + 18
y=x+6
Gradient of tangent = 1
Gradient of normal = –1
y–3
Equation of normal: ​​ _____
x + 3   ​​ = –1
y – 3 = –x – 3
y = –x
(b) g = –9, f = 3, c = 25, x1 = 1, y1 = –2
Using Theorem 8.2, the equation of the tangent:
xx1 + yy1 + (g)(x + x1) + ( f )(y + y1) + c = 0
x(1) + y(–2) + (–9)(x + 1) + 3(y – 2) + 25 = 0
x – 2y – 9x – 9 + 3y – 6 + 25 = 0
y = 8x – 10
Gradient of tangent = 8
Gradient of normal = –​​ __ 1
8 ​​ 
y+2 1
Equation of normal: ​​ _____ __
x – 1 ​​ = –​​  8 ​​ 
8y + 16 = –x + 1
8y = –x – 15

8.2.6 Length of a Tangent from a Point to a Circle


Consider a point Q (a, b) outside a circle, x2 + y2 + 2gx + 2 f y + c = 0, as y
shown in Figure 8n. The length of the tangent from point Q(a, b) to the
circle is given by the following theorem.
P
Theorem 8.3 d
C(–g, –f ) Q(a, b)
The length d of a tangent from point Q (a, b) to the circle,
_________________ x
x2 + y2 + 2gx + 2fy + c = 0 is d = ​√ a  
2
+ b2 + 2ga + 2 f b + c ​​.
Figure 8n

229

Pre-Calculus and Calculus.indb 229 13/7/2017 5:57:14 PM


Chapter 8 • Graphs of Quadratic Equations

PROOF
Referring to Figure 8n,
_________ _______________
CP = r = √ ​​  g + f 2 – c ​​ 
2
​​  (a
and CQ = √    + g)2 + (b + f )2 ​​.
Using Pythagorean theorem on PCQ,
(CQ)2 = (CP)2 + (PQ)2
(PQ)2 = (CQ)2 – (CP)2
(PQ)2 = (a + g)2 + (b + f )2 – (g2 + f 2 – c)
(PQ)2 = a2 + 2ag + g2 + b2 + 2bf + f 2 – g2 – f 2 + c
(PQ)2 = a___________________
2
+ b2 + 2ag + 2bf + c
PQ = √ a2 + b2 + 2ag + 2bf + c ​​ ___________________
​​    
Since PQ is the length d, we get d = √ a2 + b2 + 2ag + 2bf + c ​​.
​​    

Example 20
Find the length of the tangent
(a) from Q(2, 5) to the circle, (x – 7)2 + (y – 1)2 = 16,
(b) from Q(2, –3) to the circle, x2 + y2 – 10x – 4y + 12 = 0.

SOLUTION
(a) (x – 7)2 + (y – 1)2 = 16
x2 + y2 – 14x – 2y + 34 = 0
g = –7, f = –1, c = 34, a = 2,___________________
b=5
Length of the tangent, d = √ a2 + b2 + 2ag + 2bf + c ​​
​​    
__________________
= ​​√ 2  
2
+ 52 – 28 – 10 + 34 ​​
= 5 units
Alternative method:
The center of the circle is (7,1) and the radius is 4 units.
_______________
Distance between (7,1) and (2, 5) = √ ​​  _______
  
(7 – 2)2 + (1 – 5)2 ​​
y = ​​√ ___
25 + 16 ​​ 
Q(2, 5) = ​​√ 41 ​​ 

___ 4
​​  41 ​​ 
√ (7, 1)
x
0

By Pythagorean theorem,__________
___ 2
length of the tangent = √
​​ ( ​​  ​√ 41 ​)  ​​​ ​– 42 ​​ 
___
= ​​√ 25 ​​ 
= 5 units
(b) x2 + y2 – 10x – 4y + 12 = 0
g = –5, f = –2, c = 12, a = 2,___________________
b = –3
Length of the tangent, d = √ a2 + b2 + 2ag + 2bf + c ​​
​​  _____________________
  
= ​​√ ___
2  
2
+ (–3)2 – 20 + 12 + 12 ​​
= ​​√ 17 ​​ units
230

Pre-Calculus and Calculus.indb 230 13/7/2017 5:57:14 PM


Chapter 8 • Graphs of Quadratic Equations

8.2.7 Parametric Equations of a Circle


If we let x = r cos t and y = r sin t, we find that
x2 + y2 = (r sin t)2 + (r cos t)2 = r2(sin2 t + cos2 t).
From trigonometry, sin2 t + cos2 t = 1.
Therefore, x2 + y2 = r2, which is the equation of a circle with center at the
origin and radius r.
Thus, x = r sin t and y = r cos t are parametric equations of the circle,
x2 + y2 = r2.
Theorem 8.4

If P (x, y) is a point on a circle, then


(i) x = r cos t and y = r sin t are the parametric equations of
x2 + y2 = r2, a circle with center at the origin and radius of r units,
(ii) x – h = r cos t and y – k = r sin t are the parametric equations of
(x – h)2 + (y – k)2 = r2 a circle with center at (h, k) and radius of
r units.

Example 21
Find the Cartesian equations of the following curves represented by
following parametric equations.
(a) x = 2 cos t and y = 2 sin t
(b) x = 1 + 4 cos t and y = –2 + 4 sin t

SOLUTION
(a) x = 2 cos t  x2 = 4 cos2 t ––– (1)
y = 2 sin t  y2 = 4 sin2 t ––– (2)
Add equations (1) and (2), we get
x2 + y2 = 4(sin2 t + cos2 t)
x2 + y2 = 4(1) sin2 t + cos2 t = 1
Hence, the curve is x2 + y2 = 4, a circle with center at the origin and
radius of 2 units.
(b) x – 1 = 4 cos t  (x – 1)2 = 16 cos2 t  ––– (1)
y + 2 = 4 sin t  (y + 2)2 = 16 sin2 t  ––– (2)
Add equations (1) and (2), we get
(x – 1)2 + (y + 2)2 = 16(sin2 t + cos2 t)
(x – 1)2 + (y + 2)2 = 16(1) sin2 t + cos2 t = 1
Hence, the curve is (x – 1) + (y + 2)2 = 16, a circle with center at
2

(1, –2) and radius of 4 units.

231

Pre-Calculus and Calculus.indb 231 13/7/2017 5:57:14 PM


Chapter 8 • Graphs of Quadratic Equations

Exercise 8.2
1. Find the center and radii of the following circles.
(a) (x – 3)2 + (x + 5)2 = 16
(b) x2 + y2 – 4x + 6y – 12 = 0
(c) x2 + y2 + x = 0
(d) 2x2 + 2y2 + 12x – 2y – 3 = 0

2. Find the equations of the following circles in general form, for the given conditions.
(a) Center C = (–3, 4) and radius r = 3 units
(b) Center C = (0, –6) and radius r = 5 __units
(c) Center C = (–1, 0) and radius r = √ 
​​ 6 ​​ units

3. Find the equation of each of the following circles which has its center at point (2, 3) and
(a) touches the y-axis,
(b) touches the x-axis,
(c) passes through the origin.

4. Find the equations of the circles which have the following points as the endpoints of their diameters.
(a) (1, 0) and (–3, 4)
(b) (x1, y1) and (x2, y2)

5. Find the equations of the circles which pass through the following three points.
(a) (–1, 5), (–2, –2) and (5, 5)
(b) (1, 1), (1, –1) and (2, 0)

6. Show that the circle x2 + y2 – 2cx + c2 = 0, where c > 0, lies completely in the first quadrant and touches
both coordinate axes.

7. The center of a circle lies on the line, y = 2x – 2 and this circle cuts the x-axis at points (1, 0) and
(3, 0). Find the equation of the circle.

8. Find the equations of the tangent and normal to the given circle at the given point on the circle in each
of the following cases.
(a) x2 + y2 – 6x – 16y + 48 = 0, (6, 12)
(b) x2 + y2 – 6x + 4y + 3 = 0, (0, –1)

9. Find the length of the tangents


(a) from point (8, 4) to the circle, x2 + y2 – 6x + 5 = 0,
(b) from point (6, 7) to the circle, x2 + y2 – 2x – 8y = 8,
(c) from point (10, 9) to the circle, (x – 2)2 + (y – 3)2 = 9.

10. Find an equation in terms of x and y, given x = cos θ – 2 and y = sin θ + 3.

232

Pre-Calculus and Calculus.indb 232 13/7/2017 5:57:14 PM


Chapter 8 • Graphs of Quadratic Equations

11. Find the equation of each of the following circles for the given conditions.
(a) Center (1, 2) and radius 5 units
(b) Center (3, 4) and circle passing through the origin
(c) Center (1, –2) and circle touching the line, x + y – 4 = 0

12. Show that each of the following equations represents a circle and find the center and radius of the
circle.
(a) x2 + y2 + 4y – 2x + 1 = 0
(b) x2 + y2 – 2x – 2y = 2
(c) x2 + y2 + 6y = 2
(d) 16x2 + 16y2 + 8x + 32y + 1 = 0
(e) x2 + y2 – 16x + 12y – 200 = 0

13. Find the equations of the following circles in general form, which pass through the given points.
(a) (1, 2), (4, 5), (6, –4)
(b) (–2, 5), (2, 1), (–3, 2)
(c) (1, 2), (1, 5), (3, 6)

14. Find the equation of the circle that passes through the points (3, 3) and (4, 2) and having center on the
line, 3x – 4y –2 = 0.

15. Find the equation of the circle which has the points (1, 1) and (9, 7) as the ends of its diameter.

16. Find the equation of the circle whose center is at the point (6, 8) and passes through the point (10, 11).

17. A circle with center (5, 7) touches the x-axis. Find the equation of this circle.

18. The straight line, y + x –11 = 0 cuts the circle, x2 – y2 – 8y – 9 = 0 at two points. Find the coordinates
of these two points.

19. Find the coordinates of the points of intersection of each of the following pairs of circle.
(a) (x – 2)2 + (y + 3)2 = 25 and x2 + y2 – 5x + 3y – 4 = 0
​ 2 ​ )​​​  ​​ = ___
(b) 4(x – 2)2 + (2y + 3)2 = 5 and (x – 3)2 + (​​​  y + __
2
5 25
​​  4 ​​ 

20. Find the length of the tangents from the given point to the given circle in each of the following cases.
(a) (4, 6 ), x2 + y2 – 4x + 2y = 6
(b) (6, 7 ), (x – 1)2 + (y – 4)2 = 25

21. Find the equations of the tangent and normal to the given circle at the given point on the circle in each
of the following cases.
(a) x2 + y2 = 26, (1, 5 )
(b) x2 + y2 – 6x – 10y – 82 = 0, (–1, –5 )
233

Pre-Calculus and Calculus.indb 233 13/7/2017 5:57:14 PM


Chapter 8 • Graphs of Quadratic Equations

22. It is given that circle C1 has equation x2 + y2 – 6x – 6y = 0 and circle C2 has center at (1, 1). Circle C1
is inside circle C2 and both circles touch at the point (0, 0).
(a) Find the center and radius of circle C1.
(b) Show that the origin and the centers of both circles are collinear.
(c) Find the equation of the common tangent of C1 and C2.

23. A straight line, y – 2x = 0 intersects a circle, 2x2 + 2y2 + 4x – 7y – 20 = 0 at points A and B. Find the
coordinates of A and B. Hence, find the equation of the circle in general form which passes through
points A and B and the point (4, 0).

234

Pre-Calculus and Calculus.indb 234 13/7/2017 5:57:14 PM


Chapter 8 • Graphs of Quadratic Equations

8.3 Ellipses
8.3.1 Definition of an Ellipse
8.3.2 Equation of an Ellipse
8.3.3 Parametric Equations of an Ellipse

8.3.1 Definition of an Ellipse


Definition 8.4

An ellipse is the set of all points in a plane, the sum of whose distances
from two fixed points in the plane is a constant. The two fixed points
are the foci of the ellipse.

Figure 8o shows an ellipse with two foci, F1 and F2 and a point P (x, y) on
the ellipse. From the definition of ellipse, |PF1| + |PF2| = a constant. The
midpoint of segment F1F2 is called the center of the ellipse. Points V1 and
V2 are the vertices of the ellipse. Segment V1V2 (which contains the foci)
is called the major axis. The segment M1M2 is the minor axis. Segment
L1L2 and L3L4 are called the latus rectum.
P (x, y) L1
M1

L3
V1
F1
C
F2
V2 L2

L4 M2

Figure 8o
Now, we will derive equation of an ellipse.

8.3.2 Equation of an Ellipse


Equation of an ellipse with center (0, 0) and foci (±c, 0)
This ellipse has its foci on the x-axis, that is, the major axis is along the
x-axis. Its center is at the origin, as shown in Figure 8p. If we let the
coordinates of F1 be (c, 0), where c > 0, then the coordinates of F2 are
(–c, 0) and |F1F2| = 2c.
y

M2 (0, b) P(x, y)

x
V2 (–a, 0) F2 (–c, 0) C F1 (c, 0) V1(a, 0)

M1 (0, –b)

Figure 8p
235

Pre-Calculus and Calculus.indb 235 13/7/2017 5:57:14 PM


Chapter 8 • Graphs of Quadratic Equations

From the definition of ellipse,


|PF1| + |PF2| = a constant
Let the constant sum be 2a.
Then, |PF1| + |PF2| = 2a
__________ __________
​​√ (x – c) + y  ​​ 
2 2
+√ (x + c)2 + y2 ​​ 
​​  __________ = 2a __________
​​√ (x – c) + y  ​​  2 2
= 2a – ​​√ (x + c)2 + y2 ​​ 
Squaring both sides, __________
​(x – c)2 + y2​= 4a2 – 4a​√ (x + c)2 + y2 ​​ 
​  __________ + (x + c)2 + y2
x – 2cx + c + y = 4a –__________
2 2 2 2
4a​​√ (x + c) + y  ​ 
2 2
+ x2​+ 2cx + c2 + y2
–4cx – 4a2 = –4a​​√ (x + c)2 + y2 ​​ 
__________
​  (x + c)2 + y2 ​​ 
a​√ = a2 + cx
Squaring both sides again,
a2[(x + c)2 + y2] = a4 + 2a2cx + c2x2
a2(x2 + 2cx + c2 + y2) = a4 + 2a2­cx + c2x2
a2x2 + 2a2cx + a2c2 + a2y2­ = a4 + 2a2cx + c2x2
a2x2 – c2x2 + a2y2 = a4 – a2c2
x2(a2 – c2) + a2y2 = a2(a2 – c2)
Dividing both sides by a2(a2 – c2),
x2 y2
​​ __2  ​​ + _______ ​​  2   2 ​​ 
=1
a a –c
Note that P (x, y) is a point on the ellipse. Thus, PF1F2 is a triangle and
since the sum of two sides of a triangle is always greater than the third
side, we have |PF1| + |PF2| > |F1F2|.
This is, 2a > 2c
a>c
Since a > c,
a2 > c2
______
a2 – c2 > 0
​​  a2 – c2 ​​ 
If we let √ =b
a – c = b2,
2 2

x2 y
2
the equation of the parabola becomes __ ​​  2  ​​ + __
​​    ​​ = 1.
a b2
Since c > 0 and a2 – c2 = b2, then a2 > b2
a>b
The coordinates of the vertices can be obtained by letting y = 0 and hence,
we have . Thus, V1 = (a, 0) and V2 = (–a, 0). The length of the major
axis is 2a. If we let x = 0, we have y = b, thus the coordinates of the
endpoints of the minor axis are M1 = (0, b) and M2 = (0, –b). The length of
the minor axis is 2b.

236

Pre-Calculus and Calculus.indb 236 13/7/2017 5:57:14 PM


Chapter 8 • Graphs of Quadratic Equations

x2 y
2
Substituting x = c into __ ​​  2  ​​ + __
​​  2  ​​ = 1,
a b
__ c2 __ y2
​​  2  ​​ + ​​  2  ​​ = 1
a b
y2 c2
​​ __2  ​​ = 1 – __ ​​  2  ​​
b a
y2 = b2​(​  1 – __ ​  2  ​)​​  
​c​​  2​

a
b2
y2 = __
​​  2 ​​ (a2 – c2)
a
______
b 2 2
y = ±​​ __
a a – c  ​​ 

​​ ​​√ 
______
b2
Since √ = b​, y = ± ​​ __
​​  a2 – c2 ​  a ​​ .
2b2
Therefore, the length of the latus rectum is ___
​​  a   ​​.

Summary

The ellipse with horizontal major axis and center (0, 0) has the following:
x2 y
2
Equation: __
​​  2  ​​ + __
​​  2  ​​= 1, where a > b > 0
a b
Vertices: (a, 0)
Major axis: Horizontal, length = 2a
Minor axis: Vertical, length = 2b
______
Foci: (c, 0), where c = √ ​​  a – b2 ​​ 
2

2b2
Latus rectum: Vertical, length = ___ ​​  a   ​​

Equation of an ellipse with center (0, 0) and foci (0, ±c)


This ellipse has its foci on the y-axis, that is, the major axis is along the
y-axis. Its center is at the origin, as shown in Figure 8q. Let the foci be F1
(0, c) and F2 (0, –c), then |F1F2| = 2c.
y
V1 (0, b)

F1 (0, c)
P (x, y)
x
M2 (–a, 0) C M1 (a, 0)

F2 (0, –c)

V2 (0, –b)
Figure 8q

237

Pre-Calculus and Calculus.indb 237 13/7/2017 5:57:14 PM


Chapter 8 • Graphs of Quadratic Equations

For this ellipse, we let the constant sum be 2b.


Then,
__________
|PF | + |PF2| = 2b
__________
1
​​√ x2 + (y – c)2 ​​  +√​​  x__________
2
+ (y + c)2 ​​ 
= 2b __________
​​√ x2 + (y – c)2 ​​  = 2b – ​​√ x2 + (y + c)2 ​​ 
Squaring both sides, __________
x2 + (y – c)2 = 4b2 – 4b​√ x2 + (y + c)2 ​​ 
​  __________ + x2 + (y + c)2
x2 + y2 – 2cy + c2 = 4b2 –__________ 4b​√ ​  x2 + (y + c)2 ​​ 
+ x2 + y2 + 2cy + c2
–4cy __________
– 4b2 = –4b​√ ​  x2 + (y + c)2 ​​ 
​  x2 + (y + c)2 ​​ 
b​√ = b2 + cy
Squaring both sides again,
b2[x2 + (y + c)2] = b4 + 2b2cy + c2y2
b2(x2 + y2 + 2cy + c2) = b4 + 2b2cy + c2y2
b2x2 + b2y2 + 2b2cy + b2c2 = b4 + 2b2cy + c2y2
b2x2 + b2y2 – c2y2 = b4 – b2c2
b2x2 + y2(b2 – c2) = b2(b2 – c2)
Dividing both sides by b­2(b2 – c2),
x2 y2
​​ ______    ​​ + __
​​   ​​ = 1
b2 – c2 b2
Since |PF1| + |PF2| > |F1F2|,
we have 2b > 2c
b>c
Thus, b – c2 > 0
2

b2 > c2
______
b – c2 > 0
2

Let √ ​​  b2 – c2 ​​ 


=a
b2 – c2 = a2,
x2 y
2
the equation of the parabola becomes __ ​​  2  ​​ + __​​  2  ​​ = 1.
a b
The coordinates of the vertices, endpoints of the minor axis and the lengths
of the latus rectum can be obtained using the same procedures we have
discussed previously.

Summary
Note
The ellipse with vertical major axis and center (0, 0) has the following:
x2
__ y
__
2
x2 y
2
For the ellipse ​ ​ 2 ​​ + ​ ​ 2 ​​ = 1,
a b Equation: __
​​  2  ​​ + __
​​  2  ​​= 1, where b > a > 0
a b
(i) if a > b, the major axis is
Vertices: (0, b)
on the x-axis.
(ii) if b > a, the major axis is Major axis: Vertical, length = 2b
on the y-axis. Minor axis: Horizontal, length ______
= 2a
Foci: (0, c), where c = √ ​​  b2 – a2 ​​  2
2a
Latus rectum: Horizontal, length = ___ ​​     ​​
b

238

Pre-Calculus and Calculus.indb 238 13/7/2017 5:57:14 PM


Chapter 8 • Graphs of Quadratic Equations

Example 22
Find the foci, vertices and lengths of the latus rectum, major and minor
x2 y
2
axes for the ellipse, __
​​ 9 ​​  + __
​​  4 ​​  = 1. Hence, sketch its graph.

SOLUTION
x2
Since the denominator of ​​ __9 ​​  is larger, the ellipse has a horizontal major
axis.
a2 = 9  a = 3
b2 = 4______
 b = 2 __
c=√ ​​  b2 – a2 ​​ 
=__√
​​  5 
 ​​
y
 Foci = (​​√ 5 ​​,  0)
Vertices = (3, 0) 2
2b 2
Length of latus rectum = ___ ​​  a   ​​
2(2)2 x
= ​​ _____ –3 __ __ 3
 ​​   –​ 5 

0 ​ 5 

3 √
​ ​​
  √ ​​

8
__
= ​​  3 ​​  units
–2
Length of major axis = 2(3) = 6 units
Length of minor axis = 2(2) = 4 units

Example 23
The vertices of an ellipse are (±4, 0) and their foci are at (±1, 0). Find
its equation and sketch the graph.

SOLUTION
Vertices = (4, 0)  a = 4
Foci = (1, 0)  c = 1
c2 = a2 – b2 y
___
1 = 42 – b2 ​​  15 ​​  

b2 = 15
Hence, the equation of the ellipse is
x2 y
2
x
​​ __2  ​​ + __ ​​  2  ​​ = 1 –4 –1 0 1 4
a b
x2 ___ y2
​​ ___
16 15  ​​ + ​​    ​​ = 1 ___
–​​√ 15 ​​  

239

Pre-Calculus and Calculus.indb 239 13/7/2017 5:57:14 PM


Chapter 8 • Graphs of Quadratic Equations

Example 24
Find the foci of the ellipse, 16x2 + 9y2 = 144 and sketch its graph.

SOLUTION y

Dividing 16x2 + 9y2 = 144 by 144, 4


x2 y
2
__
​​ 9 ​​  + ___
​​  16  ​​ = 1
Thus, the major axis is______ vertical.
a = 3, b = 4 and c = √ ​​  __
42 – 32 ​​  –3 0 3
x
__ = ​​√ 7  
 ​​
Focus = (0, ​​√ 7 ​​)  –

–4

Equation of an ellipse with center (h, k) and foci (h ± c, k)


If an ellipse, with center at the origin and its horizontal major axis is shifted
h units horizontally and k units vertically, we will get an ellipse with its
center at (h, k). If the foci were originally at (±c, 0), the new foci will be at
(h ± c, k). This can be seen in Figure 8r.
(x – h)2 ______(y – k)2
The equation of this ellipse is ______
​​  2 ​​  + ​​   ​​  = 1, where a > b > 0.
a b2

Summary

The ellipse with horizontal major axis and center (h, k) has the following:
(x – h)2 ______(y – k)2
Equation: ______
​​  2 ​​  + ​​   ​​  = 1, where a > b > 0
a b2
Vertices: (h  a, k)
Major axis: Horizontal, length = 2a
Minor axis: Vertical, length ______ = 2b
Foci: (h  c, k), where c = √  ​​ a2 – b2 ​​ 
2b2
Latus rectum: Vertical, length = ___ ​​  a ​​ 
y
M2 (h, k + b)

V2(h – a, k) F2(h – c, k) F1 (h + c, k) V1 (h + a, k)
C(h, k)
0 x

M1 (h, k – b)

Figure 8r

Equation of an ellipse with centre (h, k) and foci (h, k ±c)


If an ellipse, with center at the origin and its vertical major axis is shifted
h units horizontally and k units vertically, we will get an ellipse with center
at (h, k) and foci at (h, k  c), as seen in Figure 8s.
(x – h)2 ______(y – k)2
The equation of this ellipse is ______
​​  2 ​​  + ​​   ​​  = 1, where b > a > 0.
240 a b2

Pre-Calculus and Calculus.indb 240 13/7/2017 5:57:15 PM


Chapter 8 • Graphs of Quadratic Equations

Summary

The ellipse with vertical major axis and center (h, k) has the following:
(x – h)2 ______(y – k)2
Equation: ______
​​  2 ​​  + ​​   ​​   = 1, where b > a > 0
a b2
Vertices: (h, k  b)
Major axis: Vertical, length = 2b
Minor axis: Horizontal, length = 2a
______
Foci: (h, k  c), where c = √ ​​  b – a2 ​​ 
2

2a2
Latus rectum: Horizontal, length = ___ ​​     ​​
b
y
V1 (h, k + b)

F1 (h, k + c)

M2 (h – a, k) M1 (h + a, k)
C (h, k)
0 x

F2 (h, k – c)

V2 (h, k – b)

Figure 8s

Example 25

Sketch the graph of the ellipse, and determine the


coordinates of the foci.

SOLUTION
y
Center = (–1, 2)  h = –1, k = 2 (–1, 5)
a2 = 4  a = 2
b2 = 9  b = 3
Since______ b > a, the major axis is vertical. (–3, 2) (–1, 2) (1, 2)
c=√ b2 – a2 ​​ 
​​  _____
= ​​√ __9 – 4 ​​ 
= ​​√ 5  ​​ 0
x
Foci = (h, k  c) __
= (–1, 2  √ ​​  5 ​​)  (–1, –1)
__ __
 The foci are (–1, 2 + √ ​​  5 ​​)  and (–1, 2 – √
​​  5 ​​)  .

Determining the center and foci by completing the square


(x – h)2 ______(y – k)2
If the equation of the ellipse is not given in the form ______
​​  2 ​​  + ​​   ​​ 
 = 1,
a b2
we use completing the square method to determine the center and foci.

241

Pre-Calculus and Calculus.indb 241 13/7/2017 5:57:15 PM


Chapter 8 • Graphs of Quadratic Equations

Example 26
Show that each of the following equations represents an ellipse and
sketch the graph.
(a) 9x2 – 72x + 16y2 + 32y = –16
(b) y2 + 9x2 + 18x + 6y = 0

SOLUTION
(a) 9x2 – 72x + 16y2 + 32y = –16
9(x2 – 8x) + 16(y2 + 2y) = –16
9[(x – 4)2 – 16] + 16[(y + 1)2 – 1] = –16
9(x – 4)2 – 144 + 16(y + 1)2 – 16 = –16
9(x – 4)2 + 16(y + 1)2 = 144
(x – 4)2 _______(y + 1)2
​​ ______ 16  ​​  
+ ​​  9 ​​   = 1
This is the equation of an ellipse with center (4, –1).
h = 4, k = –1, a = 4, b = 3
Since______ a2 > b2, the major axis is horizontal.
c = √ a2 – b2 ​​ 
​​  ______
= ​​√ __ 16 – 9 ​​ 
=√ ​​  7  ​​
Vertices = (h  a, k)
= (4  4, –1)
= (0, –1) and (8, –1)
Foci = (h  c,__k)
= (4  √  7 ​​,  –1)
​​ __ __
= (4 – ​​√ 7 ​​,  –1) and (4 + √ ​​  7 ​​,  –1)
M = (h, k  b)
= (4, –1  3)
= (4, –4) and (4, 2)
y

(4, 2)

4 x
0 C (4, –1)
__ __
(0, –1) (4 – ​​√  
7 ​​, –1) 7 ​​, –1) (8, –1)
(4 + ​​√  

(4, –4)

(b) y2 + 9x2 + 18x + 6y = 0


9x2 + 18x + y2 + 6y = 0
9(x2 + 2x) + y2 + 6y = 0
9[(x + 1)2 – 1] + (y + 3)2 – 9 = 0
9(x + 1)2 – 9 + (y + 3)2 – 9 = 0
9(x + 1)2 + (y + 3)2 = 18
(y + 3)2
(x + 1)2 _______
​​ _______ 2  ​​  
+ ​​  18 ​​  = 1
242

Pre-Calculus and Calculus.indb 242 13/7/2017 5:57:15 PM


Chapter 8 • Graphs of Quadratic Equations

This is the equation of __ an ellipse ___ with center (–1, –3).


h = –1, k = –3, a = √  ​​ 2 ​​,  b = √ ​​ 18 ​​ 
Since______ b2 > a2, the major axis is vertical.
c = √ b2 – a2 ​​ 
​​  ______
= ​​√ ___
18 – 2 ​​ 
= ​​√ 16 ​​ 
=4
Vertices = (h, k  b) ___
= (–1, –3  √  18 ​​) 
​​ ___ ___
= (–1, –3 – ​​√ 18 ​​)  and (–1, 3 + √ ​​  18 ​​) 
Foci = (h, k  c)
= (–1, –3  4)
= (–1, –7) and (–1, 1)
M = (h  a, k) __
= (–1  √  2 ​​,  –3)
​​ __ __
= (–1 – ​​√ 2 ​​,  –3) and (–1 + √ ​​  2 ​​,  –3)
y

(–1, 1)
x
0

C(–1, –3)

(–1, 7)

8.3.3 Parametric Equations of an Ellipse

Theorem 8.5

If P(x, y) is a point on an ellipse, then x = a cos t and y = b sin t are the


x2 y
2
parametric equations of the ellipse, __​​  2  ​​ + __
​​  2  ​​ = 1.
a b

Example 27
Verify that x = a cos t and y = b sin t are the parametric equations of an
x2 y
2
ellipse, ​​ __2  ​​ + __
​​  2  ​​ = 1.
a b
SOLUTION
x = a cos t  x2 = a2 cos2 t
y = b sin t  y2 = b2 sin­2 t
x2 y _______
2
a2 cos2 t _______
b2 sin2 t
​​ __2  ​​ + __
​​  2  ​​ = ​​  2 ​​ 
 + ​​   ​​   
a b a b2
= cos2 t + sin2 t
=1 243

Pre-Calculus and Calculus.indb 243 13/7/2017 5:57:15 PM


Chapter 8 • Graphs of Quadratic Equations

Example 28
Find the parametric equations of an ellipse, x2 + 4y­2 = 64.
SOLUTION
x2 + 4y2 = 64
x2 ___ y2
​​ ___
64 16  ​​ = 1

 ​​ + ​​ 
 a = 8, b = 4
From Theorem 8.5, the parametric equations are x = 8 cos t and
y = 4 sin t.
Exercise 8.3
1. Find the equations of the ellipses which satisfy 5. Find the equations for the ellipses that statisfy
the following conditions. the given conditions.
(a) The vertices are at (±3, 0) and the ellipse (a) Foci = (±3, 0) and vertices = (±4, 0)
passes through the points (0, ±1). (b) Foci = (0, ±4) and vertices = (0, ±5)
(b) The distance between the focal points is (c) Foci = (0, ±2) and length of major
16 units and the length of the major axis is axis = 12 units
34 units.
(c) The foci are at (±2, 0) and the ellipse 6. Find the center, foci and vertices of the
passes through (2, –3). following ellipses, and determine the lengths
of the major and minor axes. Hence, sketch the
2. Sketch the graph of the following ellipses. graph.
x2 ___ y2 (y – 1)2
(a) ​​ ___ 
 ​​ + ​​    ​​ = 1 (x – 2)2 ______
(a) ​​ ______
 ​​  + ​​ 
25 16 9 4 ​​   = 1
(b) 4x2 + y2 = 4
(b) (x – 3)2 + 16(y + 3)2 = 16
(x – 1)2 ___ y2
(c) ​​ ______
25  ​​   + ​​    ​​ = 1
16 (c) 3x2 + 4y2 – 6x – 24y + 38 = 0
(x – 3) 2
(y – 1)2 (d) x2 + 4y2 + 20x – 40y + 100 = 0
(d) ​​ ______ 9  ​​   + ​​ 
______
25 ​​  = 1 (x + 2)2
(e) ​​ _______
4 ​​   + y2 = 1
(e) x2 + 2y2 – 2x – 4y – 1 = 0
(f) 9x2 + 4y2 –18x + 16y – 11 = 0 (x – 3)2 ______ (y – 1)2
(f) ​​ ______
9  ​​  + ​​ 
25 ​​  = 1

3. Sketch the curve with parametric equations, (g) 4x2 + y2 + 8x – 4y – 8 = 0


x = 2 cos t and y = 5 sin t.
7. Find the center, lengths of the major and minor
4. Find the vertices, foci and state the lengths axes for the ellipse, 6x2 + 24y2 + 48x = 0.
of the latus rectum, major and minor axes for Hence, sketch the ellipse.
each ellipse.
x2 __ y2
(a) ​​ __
4 9 ​​  = 1
 ​​
  + ​​ 
x2 __ y2
(b) ​​ ___
16 4  
 ​​ + ​​   ​​  = 1

(c) x2 + 4y2 = 4
(d) 2x2­ + __ ​​  12 ​​ y2 = 8
(e) y2 + 4x2 = 1

244

Pre-Calculus and Calculus.indb 244 13/7/2017 5:57:15 PM


Chapter 8 • Graphs of Quadratic Equations

8.4 Hyperbolas
8.4.1 Definition of a Hyperbola
8.4.2 Equation of a Hyperbola
8.4.3 Parametric Equations of a Hyperbola

8.4.1 Definition of a Hyperbola

Definition 8.5

A hyperbola is the set of all points in a plane, the difference of whose


distances from two fixed points is a constant. The two fixed points are
the foci of the hyperbola.

Figure 8t shows a hyperbola with two foci, F1 and F2. The midpoint of
segment F1 F2 is called the center of the hyperbola. Point P(x, y ) is on the
hyperbola. From the definition, we have |PF1| – |PF2| = a constant.
Points V1 and V2 are the vertices of the hyperbola. The segment V1V2 is
called the transverse axis and the segment W1W2 is called the conjugate
axis. Note that points W1 and W2 are not on the hyperbola. The lines A1
and A2 are asymptotes of the hyperbola. The segments L1 L2 and L3 L4 are
called the latus rectum.
A1

A2 P(x, y)
W1 L1
L3
V1 F1
V2 C L2
F2
L4 W2

Figure 8t
We will only be discussing hyperbolas with horizontal and vertical
transverse axis.

8.4.2 Equation of a Hyperbola


Equation of a hyperbola with center (0, 0) and foci (±c, 0)
Figure 8u shows the simplest form of hyperbola. Its foci are on the
x-axis. Let F1 (c, 0) and F2 (–c, 0) be the foci, and P(x, y) be a point on the
hyperbola but not on the x-axis.
y

P(x, y)

x
F2 (–c, 0) V2 V1 F1(c, 0)

Figure 8u 245

Pre-Calculus and Calculus.indb 245 13/7/2017 5:57:15 PM


Chapter 8 • Graphs of Quadratic Equations

From the definition of hyperbola,


|PF1| – |PF2| = a constant
Let the constant difference be 2a.
Then,
__________
|PF | – |PF2| = 2a
__________
1
​​√ (x – c) + y  ​​ 
2 2
– ​​√ (x + c)2 + y2 ​​ 
__________
= 2a __________
​​√ (x – c) + y  ​​  2 2
​​  (x + c)2 + y2 ​​ 
= 2a + √
Following the procedures that we have done for ellipse, we can simplify
x2 y2
the equation as __
​​  2  ​​ – ______
​​  2   2 ​​  = 1.
a c –a
______
Letting ​​√ c2 – a2 ​​  =b
c – a = b2,
2 2

x2 y
2
we have __ ​​  2  ​​ – __
​​  2  ​​ = 1.
a b
Figure 8v shows the complete hyperbola. Letting y = 0 gives the
x-intercept. Thus, we find that the vertices are V1 (a, 0) and ___ V2 (–a, 0).
This hyperbola has no y-intercept (letting x = 0 gives y = ​​√ –b2 ​​, some complex
values). The endpoints of the conjugate axis are W1 (0, b) and W2 (0, –b).
Plot the points V1 (a, 0) and V2 (–a, 0), W1 (0, b) and W2 (0, –b) and draw
vertical and horizontal lines connecting these points. The diagonals of the
rectangle drawn will be the asymptotes.
b
The gradient of the asymptotes are ± ​__ ​  a ​​, hence the equations of the asymptotes
b
(which pass through the origin) are y = ± __ ​​  a ​​x. The two branches of the
hyperbola are sketched using the asymptotes as guides.
x2 y
2
Substituting x = c into __ ​​  2  ​​ – __
​​  2  ​​ = 1,
a b
y2 c2
we have ​​ __2  ​​ = __ ​​    ​​ – 1
b a2
c2 – a2 2
y2 = ______
​​  2 ​​b­  ­

a
Since c2 – a2 = b2,
b4
y­2 = ___ ​​  ­2 ​​­  
a 2
b
y = ± ​__ ​  a ​​ 
2b2
Therefore, the length of the latus rectum is ___ ​​  a   ​​.
y
b
__ b
y = –​​  a  ​​x y = ​​ __
a ​​x

W1(0, b)
latus rectum latus rectum
F1(c, 0)
x
F2 (–c, 0) V2(–a, 0) V1(a, 0)

W2(0, –b)

Figure 8v
246

Pre-Calculus and Calculus.indb 246 13/7/2017 5:57:15 PM


Chapter 8 • Graphs of Quadratic Equations

Summary

The hyperbola with center (0, 0) and foci (± c, 0) has the following:
x2 y
2
Equation: __
​​  2  ​​ – __
​​  2  ​​= 1, where a > 0 and b > 0
a b
Vertices: (a, 0)
Transverse axis: Horizontal, length = 2a
Conjugate axis: Vertical, length = 2b
b
Asymptotes: y = ​​ __ a x​​ ______
Foci: (c, 0), where c = √  ​​ a2 + b2 ​​ 
2b2
Latus rectum: Vertical, length = ___ ​​  a   ​​

Equation of a hyperbola with center (0, 0) and foci (0, ±c)


The foci are on the y-axis with coordinates (0, ± c) as shown in Figure 8w.
y

F1(0, c) P(x, y)

F2(0, –c)

Figure 8w
From the definition of hyperbola,
|PF1| – |PF2| = a constant
Let the constant difference be 2b.
Then, __________ |PF | – |PF2| = 2b
__________
1
​​√ x2 + (y – c)2 ​​  – ​​√ x__________
2
+ (y + c)2 ​​ 
= 2b __________
​​√ x2 + (y – c)2 ​​  = 2b – ​​√ x2 + (y + c)2 ​​ 
y2 x2
Simplifying, we get __ ​​  2  ​​ – ______
​​  2   2 ​​ =1
______ b c –b
Letting ​​√ c2 – b2 ​​  =a
c2 – b2 = a2,
y2 x2
we have ___​​  2  ​​ – __
​​    ​​ = 1.
b a2
Figure 8x shows the complete hyperbola. Letting x = 0 gives the
y-intercept. Thus, we find that the vertices are V1(0, b) and V2 (0, –b ).
___
This hyperbola has no x-intercept (letting y = 0 gives x = ​​√ –a2 ​​,  some
complex values). The endpoints of the conjugate axis are W1 (a, 0) and
b
W2 (–a, 0). The gradient of the asymptotes are ​__
​  a ​​, hence the equations of
b
the asymptotes are y = ​__
​  a ​​x. 247

Pre-Calculus and Calculus.indb 247 13/7/2017 5:57:15 PM


Chapter 8 • Graphs of Quadratic Equations

y2 x2
Substituting y = c into __
​​  2  ​​ – __
​​    ​​ = 1,
b a2
__ x2 c2
we have ​​  2  ​​ = __ ​​    ​​ – 1
a b2
c2 – b2 2
x2 = ______ ​​  2 ​​a  

b
a4
Since c2 – b2 = a2, we have x2 = __ ​​  2 ​​ 
b
a2
x = ​__ ​   ​​ 
b
2a2
Therefore, the length of the latus rectum is ___ ​​     ​​.
y b

y=– x y= x
F1(0, c)
latus rectum
V1(0, b)

x
W2(–a, 0) W1(a, 0)

V2(0, –b)
latus rectum F2(0, –c)

Figure 8x
Summary

The hyperbola with center (0, 0) and foci (0, ± c) has the following:
y2 x2
Equation: __
​​  2  ​​ – __
​​    ​​= 1, where a > 0 and b > 0
b a2
Vertices: (0, b)
Transverse axis: Vertical, length = 2b
Conjugate axis: Horizontal, length = 2a
b
Asymptotes: y = ​​ __ a x​​ ______
Foci: (0, c), where c = √  ​​ a2 + b2 ​​ 
2a2
Latus rectum: Horizontal, length = ___ ​​     ​​
b
Example 29
Find the vertices, foci and asymptotes of the following hyperbolas and
sketch their graphs. Also, find the length of their latus rectum.
x2 ___ y2 y2 ___ x2
(a) ​​ __ ​​
9 16  – ​​   
 ​​ = 1 (b) ​​ __
1 25  ​​ = 1
 ​​
  – ​​ 
SOLUTION
x2 ___ y2
(a) ​​ __
9 16  ​​ = 1
 ​​
  – ​​ 
a2 = 9  a = 3
b2 = 16 ______ b=4
c=√  a2 + b2 ​​ 
​​ ______
=√ ​​  9 + 16 ​​ 
248 =5

Pre-Calculus and Calculus.indb 248 13/7/2017 5:57:15 PM


Chapter 8 • Graphs of Quadratic Equations

Center = (0, 0)
Vertices = (–3, 0) and (3, 0)
Foci = (–5, 0) and (5, 0)
Asymptotes: y = ​__​  34 ​​ x
2b2
Length of latus rectum = ___ ​​  a   ​​
2(42)
= ​​ _____ 3 ​​   
32
= ___​​  3 ​​ units
y
y=– x y= x
4

x
–5 –3 3 5

–4

y2 ___ x2
(b) ​​ __
1 25  ​​ = 1
 ​​
  – ​​ 
a2 = 25  a = 5
b=1
b2 = 1______
c=√ a + b2 ​​ 
​​  ______
2

= ​​√ ___
25 + 1 ​​ 
= ​​√ 26 ​​ 
Center = (0, 0)
Vertices = (0,___ –1) and (0, 1)___
Foci = (0, –​​√ 26 ​​, ) and (0, √
​​  26 ​​) 
1
__
Asymptotes: y = ​​  5 ​​ x
2a2 2(25)
Length of latus rectum = ___ ​​     ​​ = ​​ _____
1 ​​ 
 = 50 units
b
y
___
​​√ 26 ​​  

1 y= x

x
–5 0 5
–1 y=– x
___
–​​√ 26 ​​  

249

Pre-Calculus and Calculus.indb 249 13/7/2017 5:57:15 PM


Chapter 8 • Graphs of Quadratic Equations

Example 30
Find the equations for the following hyperbolas that satisfy the given
conditions.
(a) Foci = (±5, 0) and vertices = (±3, 0)
(b) Foci = (0, ±2) and vertices = (0, ±1)
(c) Asymptotes y = ± x and hyperbola passes through (5, 3)

SOLUTION y
(a) a = 3 4
c=5
c2 = a2 + b2
25 = 9 + b2 –3 3 x
–5 0 5
16 = b2
b=4
x2 y
2
–4
Equation of the hyperbola is __ ​​  2  ​​ – __ ​​  2  ​​ = 1
a b
x2 y
2
​​  9 ​​  – ___
__
​​  16  ​​ = 1 y

(b) b = 1 2
c=2 1
c2 = a2 + b2
4 = a2 + 1 x
0
a2 = 3 __
–1
a=√ ​​  3 
 ​​
y2 x2 –2
Equation of the hyperbola is __
​​  2  ​​ – __
​​    ​​ = 1
b a2
__ y2 x2
​​  1 ​​  – __ ​​  3 ​​  = 1
x2
y2 – __ ​​  3 ​​  = 1

b
(c) Since the asymptotes are y = ± x, __ ​​  a ​​ = 1
a=b
x2 y
2
Equation of hyperbola is either __ ​​  2  ​​ – __ ​​  2  ​​= 1 —— (1)
a a
y2 x2
or __
​​  2  ​​ – __ ​​    ​​= 1 —— (2)
a a2
Since the hyperbola passes through (5, 3),
52 32
substituting x = 5 and y = 3 into (1), __ ​​  2 ​​ – __ ​​   ​​ = 1
a a2
25 – 9 = a2
a2 = 16
a = ±4
32 __
__ 52
substituting x = 5 and y = 3 into (2), ​​  2 ​​ – ​​  2 ​​ = 1
a a
9 – 25 = a2
a2 = –16 (no solution)
x2 y
2
Hence, the equation of the hyperbola is ___ ​​ 16  ​​ – ___
​​  16  ​​ = 1.

250

Pre-Calculus and Calculus.indb 250 13/7/2017 5:57:16 PM


Chapter 8 • Graphs of Quadratic Equations

Equation of a hyperbola with center (h, k) and foci (h ± c, k)


If a hyperbola, with center at the origin and its horizontal transverse axis
is shifted h units horizontally and k units vertically, we will get a hyperbola
with center at (h, k) and foci at (h  c, k), as seen in Figure 8y.
(x – h)2 ______(y – k)2
The equation of this hyperbola is ______
​​  2 ​​  – ​​   ​​ 
 = 1.
a b2
y
b
__ b
y – k = –​​  a  ​​(x – h) y – k = __
​​  a  ​​(x – h)

V2 (h – a, k) V1 (h + a, k)
F1 (h – c, k) C (h, k) F2 (h + c, k)

x
0

Figure 8y
Summary

The hyperbola with center (h, k) and foci (h  c, k) has the following:
(x – h)2 ______(y – k)2
Equation: ______
​​  2 ​​  – ​​   ​​  = 1, where a > 0 and b > 0
a b2
Vertices: (h  a, k)
Transverse axis: Horizontal, length = 2a
Conjugate axis: Vertical, length = 2b
b
Asymptotes: y – k = ​​ __ a (​​ x – ______
h)
Foci: (h  c, k), where c = √  ​​ a2 + b2 ​​ 
2b2
Latus rectum: Vertical, length = ___ ​​  a   ​​

Equation of an ellipse with center (h, k) and foci (h, k ± c)


If a hyperbola, with center at the origin and its vertical transverse axis is
shifted h units horizontally and k units vertically, we will get a hyperbola
with center at (h, k) and foci at (h, k ± c), as seen in Figure 8z.
(y – k)2 ______(x – h)2
The equation of this hyperbola is ______
​​  2 ​​  – ​​   ​​ 
 = 1.
b a2
Summary

The hyperbola with center (h, k) and foci (h  c, k) has the following:
(y – k)2 ______(x – h)2
Equation: ______
​​  2 ​​  – ​​   ​​  = 1, where b > 0 and a > 0
b a2
Vertices: (h, k  b)
Transverse axis: Vertical, length = 2b
Conjugate axis: Horizontal, length = 2a
b
Asymptotes: y – k = ​__ ​  a (​​ x – ______
h)
Foci: (h, k  c), where c = √  ​​ a2 + b2 ​​ 
2a2
Latus rectum: Horizontal, length = ___ ​​     ​​
b
251

Pre-Calculus and Calculus.indb 251 13/7/2017 5:57:16 PM


Chapter 8 • Graphs of Quadratic Equations

F1 (h, k + c)
y – k = (x – h)

V1 (h, k + b)

C (h, k)

V2 (h, k – b) y – k = – (x – h)
F2 (h, k – c)

x
0
Figure 8z

Example 31
(y + 2)2 (x – 1)2
Sketch the graph of the hyperbola ​​ _______
1  ​​  
– ​​  ______
9 ​​   = 1, showing the
vertices, center, foci and asymptotes.

SOLUTION
The transverse axis ______is parallel to y-axis.
a = 3, b = 1, c = ​​√ _____ a + b2 ​​ 
2

= ​​√ ___
9 + 1 ​​ 
= ​​√ 10 ​​ 
Center = (1, –2)  h = 1, k = –2
Vertices = (1, –3)___ and (1, –1) ___
Foci = (1, –2 – √ ​​  10 ​​)  and (1, –2 + √ ​​  10 ​​) 
b
Asymptotes: y – k = ​​ __ a (​​ x – h)
y + 2 = ​__ ​  31 ​​ (x – 1)
y + 2 = __ ​​ 13  ​​x – ​​ __
1  ​​  and  y + 2 = –​​ __
3
1 1
__
3 ​​ x + ​​  3  ​​
7 5
​​  31 ​​ x – ​​ __
y = __ 3  ​​
1 __
y = –​​ __
3 ​​ x – ​​  3  ​​
y

(1, –2 + )
x
(1, –1)

C (1, –2)
(1, –3) y=– x–

(1, –2 – )

252

Pre-Calculus and Calculus.indb 252 13/7/2017 5:57:16 PM


Chapter 8 • Graphs of Quadratic Equations

Determining the center and foci by completing the square


If the equation of the hyperbola is not given in the form
(x – h)2 _______
______ (y – k)2 (y – k)2 ______
______ (x – h)2
​​   ​​ 
 – ​​   ​​  
= 1 or ​​   ​​  
– ​​   ​​ 
 = 1, we use completing the
a2 b2 b2 a2
square method to determine the center and foci.
Example 32
Show that the equation represents a hyperbola, 9x2 – 72x – 16y2 – 32y = 16.

SOLUTION
9x2 – 72x – 16y2 – 32y = 16
9(x2 – 8x) – 16(y2 + 2y) = 16
9[(x – 4)2 – 16] – 16[(y + 1)2 – 1] = 16
9(x – 4)­2 – 144 – 16(y + 1)2 + 16 = 16
9(x – 4)2 – 16(y + 1)2 = 144
(x – 4)2 _______(y + 1)2
​​ ______ 16   ​​
   
– ​ 
​ 9    ​​ = 1
This is the equation of a hyperbola with horizontal transverse axis.

Example 33
Sketch the graph with equation 9x2 – y2 + 18x + 6y = 4.

SOLUTION
9x2 – y2 + 18x + 6y = 4
9(x2 + 2x) – y2 + 6y = 4
9[(x + 1)2 – 1] – [(y – 3)2 – 9] = 4
9(x + 1)2 – 9 – (y – 3)2 + 9 = 4
9(x + 1)2 – (y – 3)2 = 4
9(x + 1)2 ______ (y – 3)2
​​ ________ 4  ​​  
 – ​​  4 ​​   = 1
(x + 1)2 ______ (y – 3)2
​​ _______  ​​  
 – ​​  4 ​​   = 1
​  94 ​ 
__

a2= __​​  94 ​​   a = __ ​​  32 ​​ 


b2 = 4  b = 2 _____ ___ ___
______
​  22 ​ 
√ 4
√ 22
__ ___ √
____
c=√ ​​  a + b  ​​ 
2 2
= ​​  ​  9 ​  + 2 ​​ = ​​  ​  9 ​ ​​  = ​​  3 ​​   
Center = (–1, 3)  h = –1, k = 3
Vertices = (h  a, k)
= (​​  –1  __ ​  32 ​ , 3)​​  
= (​​  –​ __ 3 ​ , 3)​​  and (​​  ​  3 ​ , 3)​​  
5 1
__

Foci = (h  c, k) ___
= (​​  –1 ± ​  3 ​,    3)​​  
____​√ 22 ​ 
___ ___
= (​​  –1 – ​  3 ​,    3)​​  and (   3)​​  
​  22 ​ 

____ ​  22 ​ 

____
​​  –1 + ​  3 ​, 

253

Pre-Calculus and Calculus.indb 253 13/7/2017 5:57:16 PM


Chapter 8 • Graphs of Quadratic Equations

b
Asymptotes: y – k = ± ​​ __
a (​​ x – h)
y – 3 = ± ​​ __ 2  ​​(x + 1)
​  32 ​ 
__

y – 3 = 3(x + 1)  and  y – 3 = –3(x + 1)


y – 3 = 3x + 3 y – 3 = –3x – 3
y = 3x + 6 y = –3x
y
y = –3x
y = 3x + 6

 5 ​, 3)​​  
( ( 3 ​ , 3)​​  
​​  –​__ 1
​​  –​ __
3
___ ___

(​​  –1 – ​  3 ​,   3)​​   (​​  –1 + ​  3 ​,   3)​​  


C (–1, 3)
​  22 ​ 

____ ​  22 ​ 

____

x
–1 0

Example 34
Find the center, vertices, foci and asymptotes of the hyperbola,
9y2 + 18y = x2 + 6x + 18.

SOLUTION
9y2 + 18y – x2 – 6x = 18
9(y2 + 2y) – (x2 + 6x) = 18
9[(y + 1)2 – 1] – [(x + 3)2 – 9] = 18
9(y + 1)2 – 9 – (x + 3)2 + 9 = 18
9(y + 1)2 – (x + 3)2 = 18
(y + 1)2 _______ (x + 3)2
​​ _______ 2___ ​​  
 – ​​  18 ​​ 
 = 1
a2 = 18  a = √ ​​__ 18 ​​ 
2
b=√
b = 2______ ​​  2 
 ​​___
c=√ ​​  18 + 2 ​​  =√ ​​  20 ​​ 
Center = (–3, –1)  h = –3, k = –1
Vertices = (h, k ± b) __
= (​​  –3, –1 ± √ ​ __  2 ​ )​​  
__
= (​​  –3, –1 – √ ​  2 ​ )​​  and (​​  –3, –1 + √ ​  2 ​) ​​  
Foci = (h, k ± c) __
= (​​  –3, –1 ± √  20 ​)  ​​
​ __ __
= (​​  –3, –1 – √ ​  20 ​)  ​​ and (​​  –3, –1 + √ ​  20 ​)  ​​

254

Pre-Calculus and Calculus.indb 254 13/7/2017 5:57:16 PM


Chapter 8 • Graphs of Quadratic Equations

b
Asymptotes: y – k = ± ​​ __
a (​​ x – h) __
 2 
​ ___

____  ​
y + 1 = ± ​​    ​​  (x + 3)
​√ 18 ​ 
y + 1 = __ 1  ​​(x + 3)
​​ 31 ​​ (x + 3)  and  y + 1 = –​​ __
3
​​ 13  ​​x + 1
y + 1 = __ 1  ​​x – 1
y + 1 = –​​ __
3
y = ​​ __1 1  ​​x – 2
y = –​​ __
3 ​​ x 3

8.4.3 Parametric Equations of a Hyperbola


x2 y
2
The equations, x = a sec t and y = b tan t satisfy __ ​​  2  ​​ – __
​​  2  ​​= 1, since
a b
(a sec t)2 ________
________ (b tan t)2 ________
a2 sec­2 t _______
b2 tan2 t
​​   ​​ 
 – ​​   ​​ 
 = ​​   ​​  – ​​   ​​  
= sec­ t – tan2 t = 1.
  2
a2 b2 a2 b­2
x2 y ­
2
Thus, x = a sec t and y = b tan t are the parametric equations of  __ ​​  2  ​​ – __
​​  2  ​​ = 1.
a b
Theorem 8.6

If P (x, y) is a point on a hyperbola,


(i) x = a sec t and y = b tan t are the parametric equations of the
x2 y
2
hyperbola, __​​  2  ​​ – __
​​  2  ​​ = 1.
a b
x = a tan t and y = b sec t are the parametric equations of the
(ii) 
y2 x2
hyperbola, __​​  2  ​​ – __
​​    ​​ = 1.
b a2

Example 35
Find the curve with parametric equations, x = 2 sec t and y = 4 tan t.
SOLUTION
x = 2 sec t  and y = 4 tan t
x2 = 4 sec2 t y2 = 16 tan2 t
x2 y2
​​ __
4  ​​
  = sec2
t ​​  ___
16  ​​ = tan t
2

x2 ___ y2
Since sec2 t – tan2 t = 1, we have ​​ __
4 16  ​​ = 1, a hyperbola with horizontal
 ​​
  – ​​ 
transverse axis.

255

Pre-Calculus and Calculus.indb 255 13/7/2017 5:57:16 PM


Chapter 8 • Graphs of Quadratic Equations

Exercise 8.4
1. For each of the following hyperbolas, find the vertices, foci and asymptotes. Hence, sketch the
hyperbolas.
x2 __ y2
(a) ​​ __
9 4 ​​  = 1
 ​​
  – ​​  (d) y2 – 9x2 = 25
y2 __ x2
(b) ​​ __
9 ​​  – ​​  4 ​​  = 1 (e) x2 – 9y2 – 2x – 36y – 26 = 0
(x + 3)2 _______(y + 1)2
(c) ​​ _______
25  ​​ –
  ​​  144 ​​ =1

2. Find the equations of hyperbolas which satisfy the following conditions.


(a) Foci = (0, ±1) and vertices = (​​  0, ± ​ __ 1 ​  ​​  
3)
(b) Foci = (±9, 0)___and vertices = (±4, 0 )
(c) Foci = (​​  0, ±​√ 10 ​) ​​ and the asymptotes are y = ±3x
(d) Vertices = (±4, 0) and the hyperbolas passes through (8, 2)

3. Find the equation of parabola that has center at the origin, horizontal transverse axis of length 8 units
and conjugate axis of length 4 units.
y2 x2
4. Verify hat x = a tan t and y = b sec2 t are parametric equations of the hyperbola, __
​​  2  ​​ – __
​​    ​​ = 1.
b a2
5. Find the center, foci, vertices, asymptotes and the length of the latus rectum of the following hyperbolas
and sketch its graph.
x2
(a) y2 – ___ ​​  25  ​​ = 1
y2 ___ x2
(b) ​​ __
9 16  ​​ = 1
 ​​
  – ​​ 
x2 ___ y2
(c) ​​ __
4 36  ​​ = 1
 ​​
  – ​​ 

6. Find the equations for the following hyperbolas that satisfy the given conditions.
(a) Foci = (±2, 0) and vertices = (±1, 0)
(b) Foci = (0, ±5) and vertices = (0, ±3)
(c) Vertices = (±2, 0) and asymptotes y = ±6x

7. Find the center, foci, vertices and asymptotes of the following hyperbolas. Hence, sketch its graph.
(a) 16(x + 1)2 – 4(y – 3)2 = 16
(b) x2 – 16x – y2 – 12y = –24
(c) 4y2 – (x + 1)2 = 4
(d) (y – 1)2 – 25(x + 3)2 = 100
(e) 16x2 – 9y2 – 96x = 0

8. It is given, 5x2 – 12y2 – 10x + 24y – 67 = 0.


(a) Show that the equation represents a hyperbola.
(b) Determine the coordinates of the center and vertices of the hyperbola.
(c) Find the gradients of the asymptotes.
(d) Sketch the hyperbola and label its asymptotes.
256

Pre-Calculus and Calculus.indb 256 13/7/2017 5:57:16 PM


Analytic Geometry and Trigonometry
Chapter 9:

Trigonometry

Navigation and other Branches of


Mathematics
One of the major applications of trigonometry is finding the heights of towers and mountains.
Navigation is among the earliest applications of trigonometry, where we generally find the
distance of a shore from any point in the sea. Trigonometry plays a vital role for scientists in
giving weather reports as trigonometry is used in oceanography for calculating the intensity
and height of tides in oceans. Other applications of trigonometry include finding the
distance between different planets and other celestial bodies. One common application of
trigonometry is in the field of architecture, where architects use trigonometry for calculating
different aspects like roof slopes, light angles, structural load, ground surfaces, and so on.
Other applications of trigonometry are in scientific areas such as cartography, crystallography,
land surveying, geophysics, electronics, electrical engineering, pharmacology, seismology,
visual perception, number theory, statistics, machining, geodesy, medical imaging, optics,
and acoustics. You should also know that trigonometry is used throughout mathematics as
it is linked with many other fields like physical, natural, and social sciences. Trigonometric
functions like sine and cosine are used everywhere for different purposes. Trigonometry
also has its uses in calculus and linear algebra.

Pre-Calculus and Calculus.indb 257 13/7/2017 5:57:17 PM


Chapter 9 • Trigonometry

9 Trigonometry
9.1 Angles
9.2 Pythagorean Theorem
9.3 Trigonometric Ratios
9.4 Fundamental Trigonometric Identities
9.5 Applications of Trigonometry

9.1 Angles
9.1.1 Definition of Angles
9.1.2 Degree Measure
9.1.3 Radian Measure
9.1.4 Relationship of Degree Measure and Radian Measure

9.1.1 Definition of Angles

e
Trigonometry is derived from the Greek word, trigonon, which means
id triangle and metron, or measure. It is a branch of mathematics that deals
als
in with triangles and the relationships between the side lengths and the angles
m
ter
of the triangle.
vertex In geometry, an angle is the union of two rays. These rays are called sides
initia and they have a common endpoint called the vertex.
l side
In trigonometry, an angle is constructed by rotating a ray about its endpoint.
Figure 9a The starting position of the ray is called the initial side of the angle, and
y
the position of the ray after the rotation is called the terminal side of the
angle. The endpoint of the ray represents the vertex of the angle. This is
terminal side shown in Figure 9a.

You can place an angle in a coordinate system as shown in Figure 9b. The
angle is said to be in standard position if and only if its vertex is located
at the origin and its initial side is lying on the positive side of the x-axis.
x
0 initial side
Consider the following angle in standard position.
y
Figure 9b

x
O

The figure above shows ∠AOB with OA as the initial side. The other side
OB is the terminal side of the angle.
If the rotation is anti-clockwise, the angle measure is positive.
If the rotation is clockwise, the angle measure is negative.
Angles can be represented by capital letters or Greek letters, such as
258
α (alpha), β (beta), γ (gamma) and θ (theta).

Pre-Calculus and Calculus.indb 258 13/7/2017 5:57:17 PM


Chapter 9 • Trigonometry

9.1.2 Degree Measure


The most common unit of angle measure is the degree, denoted by º. One
degree of angle represents ____1   ​​ of a full rotation or a complete revolution
​​ 360
about the vertex.

A full anti-clockwise revolution corresponds to 360º, a half revolution to


180º and a quarter revolution to 90º.

Angles that measure between 0º and 90º are acute, while obtuse angles
measure between 90º and 180º. Figure 9c shows the different angle
measures and the location of their terminal side in a coordinate system.

θ = 90º
y

Quadrant II Quadrant I
90º < θ < 180º 0º < θ < 90º
x
θ = 180º θ = 0º

Quadrant III Quadrant IV


180º < θ < 270º 270º < θ < 360º

θ = 270º
Figure 9c
The angle is said to be in the quadrant where its terminal side lies. If the
terminal side of an angle in standard position lies on an axis, the angle is
said to be quadrantal.

9.1.3 Radian Measure


Another way to measure angles is in radians. Radian measure is defined y
using the central angle of a circle. The central angle refers to the angle radius
whose vertex is the center of the circle. arc
central angle
Definition 9.1 x

One radian is the measure of a central angle θ that intercepts an arc s


and the length of the radius r of the circle (where s = r).
That is, θ = 1 when s = r.
s Figure 9d
In general, θ = _​​  r ​​ where θ is measured in radians.
y
This is illustrated in Figures 9d and 9e.
r s
Recall that the circumference of a circle is equal to 2πr units.
θ
Thus, one full anti-clockwise revolution is equivalent to the length of an r x
arc s, that is, s = 2πr.
Based on the definition of radian, an angle θ of one complete anti-clockwise
s 2πr
revolution is equal to θ = _​​  r ​​ = ___
​​  r   ​​ = 2π. Figure 9e

259

04 Chapter 9.indd 259 18/7/2017 11:28:32 AM


Chapter 9 • Trigonometry

Therefore, a half revolution corresponds to π and a quarter revolution


π
corresponds to __
​​  2 ​​ .
y y

π __ π
​​   ​​ 
2
x x

A radian measure is a real number. It does not have a unit since taking
the ratio of two quantities, s and r that have the same unit, will cancel the
indicated unit.

9.1.4 Relationship of Degree Measure and Radian


Measure
Note Examine the table below.
Throughout the rest of the
chapter, we write ‘rad’ to
Revolution of Corresponding Corresponding
express radians. Angle about the Vertex Degree Measure Radian Measure

Full revolution 360º 2π rad

Half revolution 180º π rad

π
__
Quarter revolution 90º ​​  2 ​​  rad

π
__
One-sixth of a revolution 60º ​​  3 ​​  rad

π
__
One-eighth of a revolution 45º ​​  4 ​​  rad

π
__
One-twelfth of a revolution 30º ​​  6 ​​  rad

Based on the table above, the relationship between the two angle measures
is:
180º = π radians
​​ 180   ​​ rad and 1 rad = (​​  ____   ​​  .
 )
π 180 º
It follows that 1º = ____ ​  π ​

Summary
π
(i) To convert degrees to radians, multiply the given degrees by ____
​​ 180º
   ​​ 
.
180º
(ii) To convert radians to degrees, multiply the given radians by ____
​​  π ​​. 

260

Pre-Calculus and Calculus.indb 260 13/7/2017 5:57:17 PM


Chapter 9 • Trigonometry

Example 1
Express the following angles in radians.
(a) 90º (d) 150º
(b) 180º (e) 135º
(c) 270º (f) 45º

SOLUTION
π π
(a) 90º = 90º × ____ ​​ 180º    ​​   (d) 150º = 150º × ​​ ____    ​​ 
180º
π 5π
= __
​​  2 ​​  rad = ___
​​  6 ​​ rad
= 1.571 rad = 2.618 rad Note
____π π
(c) 180º = 180º × ​​ 180º    ​​   (e) 135º = 135º × ​​ ____    ​​ 
180º Answers can be given
3π in terms of π or numeric
= π rad = ___​​  4 ​​ rad numbers. Most of the time,
= 3.142 rad = 2.356 rad it is not necessary to write
π π the unit radian (rad).
(c) 270º = 270º × ____    ​​  
​​ 180º (f) 45º = 45º × ​​ ____    ​​ 
180º
3π π
= ___
​​  2 ​​ rad = __
​​  4 ​​  rad
= 4.712 rad = 0.785 rad

Example 2
Express the following angles in degrees.

(a) 1 rad (d) ​​ ___
2 ​​ rad
(b) 10 rad (e) π rad

(c) 3π rad (f) ​​ ___
5 ​​ rad
SOLUTION
180º 5π 5π ____ 180º
(a) 1 rad = 1 × ​​ ____
π ​​  
  (d) ​​ ___ ___
2 ​​ rad = ​​  2 ​​ × ​​  π ​​ 

= 57.30º = 450º
180º 180º
(b) 10 rad = 10 × ​​ ____
π ​​    (e) π rad = π × ____ ​​  π ​​ 

= 572.96º = 180º
180º
____ 3π 3π 180º
(c) 3π rad = 3π × ​​  π ​​    (f) ​​  5 ​​ rad = ___
___
​​  5 ​​ × ​​ ____
π ​​ 

= 540º = 108º

261

Pre-Calculus and Calculus.indb 261 13/7/2017 5:57:17 PM


Chapter 9 • Trigonometry

Exercise 9.1
1. C
 onvert the following angle measures from degrees to radians.
(a) 135º (f) 315º
(b) 140º (g) –60º
(c) –315º (h) 225º
(d) 3000º (i) –250º
(e) 450º (j) –215º

2. Convert the following angle measures from radians to degrees.


3π 5π
(a) ​​ ___
4 ​​   (f) –​​ ___
6 ​​ 

(b) –​​ ___
3 ​​   (g) –3π
12π 12π
(c) ​​ ____
7 ​​    (h) –​​ ____
7 ​​  
11π 11π
(d) ​​ ____
6 ​​    (i) ​​ ____
3 ​​  
7π 21π
(e) –​​ ___
4 ​​   (j) ​​ ____
5 ​​  

3. Express the following angles in radians.


(a) 40º (c) 220º (e) 11.125º
(b) 120º (d) 15º (f) 20º

4. Express the following angles in degrees.


π
(a) 2.5 rad (c) ​​ ___
12  ​​ rad (e) 1.5π rad
3π 3π
(b) 20 rad (d) ​​ ___
7 ​​ rad (f) ​​ ___
8 ​​ rad

5. F
 ind the radian measure of the following angles with the given degree measure.
(a) 35° (c) –150° (e) –112°
(b) 72° (d) 415° (f) –13°

6. Find the degree measure of the following angles with the given radian measure.
3π 1  ​​ rad
(a) –​​ ___
4 ​​   (d) ​​ __
3
π
(b) 1.5 rad (e) –​​ __
9 ​​ 
π
(c) –​​ __
5 ​​  

7. Find the acute angle in radians for the following angles.


15π 7π 9π
(a) ​​ ____
4 ​​    (b) –​​ ___
3 ​​   (c) ​​ ___
4 ​​ 

8. Find the acute angle in degrees for the following angles.


(a) –700º (b) 833º (c) 1200º

262

Pre-Calculus and Calculus.indb 262 13/7/2017 5:57:17 PM


Chapter 9 • Trigonometry

9.2 Pythagorean Theorem

Theorem 9.1

The Pythagorean theorem states that the sum of the squares of the legs
of a right triangle is equal to the square of the length of the hypotenuse.
A
hy
po
ten
leg use

C leg B

(AC)2 + (BC)2 = (AB)2

Given that ABC is a right triangle with hypotenuse of length c and legs
of lengths a and b, let us prove that a2 + b2 = c2.
A

c
b

C B
a
PROOF:
Draw altitude CD to hypotenuse AB such that the length of the projection
of AC is x and the length of the projection of BC is y.
A
x
D c
a y

C b B
AB AC
Since ACB and ADC are similar, we obtain ​​  AC  ​​ = ______
​​  AD  ​​
__ c a
​ ​  a ​​ = __
​​  x ​​
AB  ​​ = ___ CB
Since ACB and CDB are similar, we obtain ​___
​  CB ​​  DB  ​​
c b
​ ​ __ ​​ = __
​​   ​​
b y
From which, we obtain the relations a = cx and b = cy.
2 2

Adding the corresponding sides of these equations, we obtain


a2 + b2 = cx + cy
= c(x + y)
Note that c = x + y.
Hence, we obtain the desired equation,
a2 + b2 = c(x + y)
= c(c)
= c2
The Pythagorean theorem is a simple yet powerful theorem. It becomes
the fundamental logic on which many of the mathematical formulas are
based. It has impacted various scientific and engineering concepts and 263
developments.

Pre-Calculus and Calculus.indb 263 13/7/2017 5:57:17 PM


Chapter 9 • Trigonometry

Example 3
The given figure shows a right triangle.
A

c
b

C B
a

(a) Find a when b = 21 cm and c = 29 cm.


(b) Find b when a = 3 cm and c = 5 cm.
(c) Find c when b = 7.1 cm and a = 4.1 cm, correct your answers to
2 decimal places.

SOLUTION
Apply the Pythagorean theorem to find the lengths of the unknown sides.
(a) a2 + b2 = c2
a2 + 212 = 292
a2 + 441 = 841
Note
____ a2 = 400 ____
–​​√ 400 ​​ is rejected as the
a = √ ​​ 400 ​​ 
length is always positive.
a = 20 cm
(b) a + b = c2
2 2

32 + b2 = 52
9 + b2 = 25
b2 = 16___
b = √ 
​​ 16 ​​ 
b = 4 cm
(c) a + b2 = c2
2

c2 = (4.1)2 + (7.1)2
c2 = 16.81 + 50.41
c2 = 67.22
_____
c2 = √ 
​​ 67.22 ​​ 
c ≈ 8.20 cm

264

Pre-Calculus and Calculus.indb 264 13/7/2017 5:57:17 PM


Chapter 9 • Trigonometry

Definition 9.2

A Pythagorean triple is a set of three natural numbers, a, b and c that


satisfy the equation, a2 + b2 = c2.

For example, the numbers 3, 4 and 5 form a Pythagorean triple because


32 + 42 = 52. The numbers 20, 21 and 29 also form a Pythagorean triple.
A Pythagorean triple forms the sides of a right triangle with the largest
number being the length of the hypotenuse. The Pythagorean triple can be
used to test whether a triangle with all three sides known is a right triangle.
We apply the converse of the Pythagorean theorem, a2 + b2 = c2 to determine
whether a given triangle is a right triangle.

Example 4
The sides of PQR measure 30 cm, 40 cm and 50 cm. Determine
whether PQR is a right triangle.

SOLUTION
The length of the longest side is 50 cm.
502 = 2500
Sum of the squares of the other two sides
= 302 + 402
= 900 + 1600
= 2500
Hence, PQR is a right triangle.

265

Pre-Calculus and Calculus.indb 265 13/7/2017 5:57:17 PM


Chapter 9 • Trigonometry

Exercise 9.2
1. Find the value of x in each of the following figures.
(a) (c)
x
3
x 10

x
(b) (d)

6
x
16 3x

10

2.

15 17

s
6 25
r
q

26 p

Study the figure above and find the values of


(a) r,
(b) q,
(c) p + q,
(d) r + s + q.

3. Determine whether each of the following sets of lengths can form a right triangle.
(a) 20, 90, 101
(b) 3, 10, 13
(c) 65, 72, 97

266

Pre-Calculus and Calculus.indb 266 13/7/2017 5:57:17 PM


Chapter 9 • Trigonometry

9.3 Trigonometric Ratios


Trigonometric functions are usually remembered based on a right triangle.

hypotenuse
opposite

θ
adjacent
Six basic trigonometric functions are defined as
opposite
sin θ = __________
​​    ​​ 
hypotenuse
adjacent
cos θ = __________
​​    ​​ 
hypotenuse
opposite
tan θ = _______
​​   ​​  Note
adjacent
​​  1   ​​ 
For easy remembrance:
cosec θ = ____
sin θ O
__
TOA  T = ​​   ​​
A
​​  1   ​​ 
sec θ = _____ A
__
cos θ CAH  C = ​​    ​​
H
​​  1   ​​ 
cot θ = _____ O
__
SOH  S = ​​   ​​
tan θ H

Example 5
4
Given that sin θ = ​​ __
5 ​​ , find the values of cos θ, tan θ, cosec θ, sec θ and
cot θ.

SOLUTION
A right triangle is obtained below.

θ 5
3

4
3 4 5 5 3
Hence, cos θ = ​​  5 ​​ , tan θ = ​​  3  ​​, cosec θ = ​​ __
__ __ __ __
4  ​​, sec θ = ​​  3  ​​ and cot θ = ​​  4 ​​ .

sin θ
Note that tan θ can also be given by tan θ = _____
​​    ​​. 
cos θ
Based on a right triangle, if one acute angle is θ, then the other angle is
90º – θ.

hypotenuse 90° – θ
opposite

θ
adjacent

267

Pre-Calculus and Calculus.indb 267 13/7/2017 5:57:17 PM


Chapter 9 • Trigonometry

Thus, we have
adjacent
sin (90º – θ) = __________
​​     ​​  = cos θ
hypotenuse
opposite
cos (90º – θ) = __________
​​    = sin θ
 ​​ 
hypotenuse
adjacent
tan (90º – θ) = _______
​​    ​​ = cot θ
opposite

Example 6
4  ​​, show that
Given that sin θ = ​​ __
5
(a) sin (90º – θ) = cos θ,
(b) cos (90º – θ) = sin θ,
(c) tan (90º – θ) = cot θ.

SOLUTION
θ
5
3

90º – θ
4
3
(a) sin (90º – θ) = ​​ __
5 ​​  = cos θ
(b) cos (90º – θ) = __ ​​  54 ​​  = sin θ
3
(c) tan (90º – θ) = __ ​​  4 ​​  = cot θ

Values of the trigonometric ratios can be obtained from statistical tables


or calculators. However, values for special angles can be described using
the triangles below.

45º 30º
2
1

45º 60º
1 1

θº θ rad sin θ cos θ tan θ


0º 0 rad 0 1 0
__
π ​√ 3 ​ 
30º __
​​  6 ​​  rad ​​  1 ​​ 
__
2
___
​​  2 ​​  1__
___
​​     ​​ 
​  3 ​ 

π
__ 1__
___ 1__
___
45º ​​  4 ​​  rad ​​     ​​  ​​     ​​  1
​  2 
√  ​ ​  2 
√  ​
__
__
π ​√ 3 ​ 
60º __
​​  3 ​​  rad ___
​​   ​​ 
2 ​​  1 ​​ 
__
2 ​​√ 3 ​​ 

π
__
90º ​​  2 ​​  rad 1 0 ∞

268

Pre-Calculus and Calculus.indb 268 13/7/2017 5:57:17 PM


Chapter 9 • Trigonometry

Example 7
__
2(sin 30º + cos 45º)
________________ ​  2 ​ +__ 2

______
Show that   
​​  tan
   45º + cot 45º ​​ = ​​   ​​. 

2​√ 2 ​ 

SOLUTION
2​ (__ ​  1__   ​ )​ 
​  21 ​  + ___
2(sin 30º + cos 45º)
________________ _________ ​√ 2 
 ​
​​   
  
tan 45º + cot 45º ​​ = ​​  1 + 1 ​​ 

 ​ 
__
2​ (______
2​√ 2 ​  )
​  2 ​ +__ 2

​   ​ 
_________
= ​​  2  ​​   
__
​  2 ​ +__ 2

= ______
​​   ​​ 

2​√ 2 ​ 

Trigonometric functions can be evaluated for angles of any sizes including


obtuse angles and negative angles. This can be done in two steps:
Step 1: Identify the quadrants where θ is situated as illustrated in Figure 9f.
If an angle θ (measured from the positive x-axis) is in the first
quadrant, then sin θ, cos θ and tan θ take positive values.
If the angle θ is in the second quadrant, then sin θ takes positive
value while cos θ and tan θ take negative values.
If the angle θ is in the third quadrant, then tan θ takes positive
value while sin θ and cos θ take negative values.
If the angle θ is in the fourth quadrant, then cos θ takes positive
value while sin θ and tan θ take negative values.
y Note
Second Quadrant First Quadrant For easy remembrance of
Sine positive All positive ASTC:
x • All Silly Tom Cats
Third Quadrant Fourth Quadrant • Add Sugar To Coffee
Tangent positive Cosine positive • All Science Teachers Crazy

Figure 9f
Step 2: Determine the basic angle, θA.
Basic angle, θA, for an angle θ is the acute angle measured from
the x-axis to terminal line of angle θ, as shown in Figure 9g.
y y
terminal line
terminal line
θ = θA θA θ
x x

(i) (ii)
y y

θ θ
x x
θA θA
terminal line
terminal line
(iii) (iv)
269
Figure 9g

04 Chapter 9.indd 269 18/7/2017 10:05:35 AM


Chapter 9 • Trigonometry

The calculation of basic angle, θA, for θ  0 is given in Table 9h.


Quadrant Basic angle, θA
I θA = θ
II θA = 180º – θ
III θA = θ – 180º
IV θA = 360º – θ
Table 9h
For example, θ lies in the third quadrant as illustrated in Figure
9g(iii). Then, the basic angle θA is given by θA = θ – 180º, which
is measured from the x-axis to the terminal line of angle θ.
However, the formula given in Table 9h will have to be adjusted
when angle is negative. For example, when θ = –150º, then the
angle is in the third quadrant.
y

x
θA
θ = –150º

Hence, θA is given by 180° – 150° = 30°.

Example 8
Determine the basic angles of the following.
(a) 37º (d) –125º
(b) 350º (e) –220º
(c) 230º (f) 520º

SOLUTION
(a) θ is in the first quadrant. (c) θ is in the third quadrant.
Hence, θA = 37º Hence, θA = 230º – 180º = 50º
y
y
θ = 230º
θ = θA = 37º x
x θA = 50º

(b) θ is in the fourth quadrant. (d) θ is in the third quadrant.
Hence, θA = 360º – 350º = 10º Hence, θA = 180º –125º = 55º
y
y
θ = 350º
x x
θA = 10º θA = 55º
θ = –125º

270

04 Chapter 9.indd 270 18/7/2017 10:03:22 AM


Chapter 9 • Trigonometry

(e) θ is in the second quadrant. (f) θ is in the second quadrant.


Hence, θA = 220º – 180º = 40º Hence, θA = (360º + 180º) – 520º
= 20º
y y

θA = 40º θA = 20º
x x
θ = –220º θ = 520º

Example 9
Find the values of the following trigonometric functions.
(a) sin 120º (e) tan (–330º)
(b) cos 330º (f) cos (–210º)
(c) tan 225º (g) cot 235º
(d) sin (–45º) (h) sec 145º

SOLUTION
(a) Quadrant: II ⇒ positive (e) Quadrant: I ⇒ positive
θA: 180º – 120º = 60º θA: 360º – 330º = 30º
Hence, sin 120º = + __sin 60º Hence, tan (–330º) = + tan 30º
​  3 
√  ​ 1__   ​​ 
= ___
​​  2 ​​   = ​​ ___
​  3 ​ 

(b) Quadrant: IV ⇒ positive (f ) Quadrant: II ⇒ negative
θA: 360º – 330º = 30º θA: 210º – 180º = 30º
Hence, cos 330º = + __cos 30º Hence, cos (–210º) = –cos __
30º
​  3 
√  ​ ​  3 ​ 

= ___ ​​  2 ​​   = –​​ ___
2 ​​ 
(c) Quadrant: III ⇒ positive ​​ tan 1235º
(g) cot 235º = _______    ​​ 
θA: 225º – 180º = 45º Consider tan 235º:
Hence, tan 225º = + tan 45º Quadrant: III ⇒ positive
= 1 θA: 235º – 180º = 55º
Hence,
tan 235º = + tan 55º = 1.428
cot 235º = ​​ _____ 1   ​​ 
= 0.700
1.428
(d) Quadrant: IV ⇒ negative 1   ​​ 
(h) sec 145º = ​​ _______
cos 145º
θA : 45º Consider cos 145º:
Hence, sin (–45º) = –sin 45º Quadrant: II ⇒ negative
1__   ​​
= –​​ ___   θA: 180º – 145º = 35º
​√ 2 ​ 
Hence,
cos 145º = –cos 35º = –0.819
sec 145º = –​​ _____ 1   ​​ = –1.221
0.819

271

Pre-Calculus and Calculus.indb 271 13/7/2017 5:57:18 PM


Chapter 9 • Trigonometry

Exercise 9.3
1. Determine the base angle for the following angles.
(a) 60º (f) –130º
(b) 120º (g) –500º
(c) 320º (h) 20º
(d) 132º (i) 333º
(e) 199º

2. D
 etermine the quadrant (I, II, III or IV) and base angle for each of the following angles. Determine the
sign (+/–) associated to the trigonometric functions.
Quadrant Base angle Sign
sin 30º
cos 120º
tan 190º
tan 350º
cos (–220º)
cos 330º

3. Find the values of the following trigonometric functions by completing the table below.

Base angle Sign Value


sin 120º
cos 240º
tan 350º
sec 220º

4. Find the values of the following trigonometric functions.


(a) sin 110º (d) cos (–300º)
(b) cos 300º (e) cosec 123º
(c) cos 40º

5. Find the values of sin θ, cos θ, tan θ, sec θ, cosec θ and cot θ for the following triangles.
(a)
θ
5

12
(b) 4
3
θ

(c)
θ
7

24

272

Pre-Calculus and Calculus.indb 272 13/7/2017 5:57:18 PM


Chapter 9 • Trigonometry

6. Find the lengths of sides labelled x. Correct your answers to four decimal places.
(a)
x
20°
10
(b)
x

45°
12
(c)
62° 25
x

7. Find the values of x, correct to four decimal places.


(a)

70

30° 50°
x
x
(b)
46° 20°

65

(c)
25°

x
(d)
x

100 70°
60°

8. Find the exact values (or in surd form) of the following trigonometric functions.
(a) sin 150º (d) cos (–225º)
(b) cos 225º (e) cosec (–630º)
(c) tan 330º (f) sec (–60º)

273

Pre-Calculus and Calculus.indb 273 13/7/2017 5:57:18 PM


Chapter 9 • Trigonometry

9.4 Fundamental Trigonometric Identities


y
A trigonometric function is a function of angles based on a unit circle as
B (x, y) illustrated in Figure 9i. Let θ be the angle from line OA until line OB. Let
(0, 1)
= B (cos t, sin t)the arc length AB, say t, equals rθ or θ (as r =1). Here, θ is in degrees while
t t is in radians. Trigonometric functions, sine, cosine, tangent, cotangent,
θ
x secant and cosecant, are defined as: y
(–1, 0) O A (1, 0)
sin t = y cos t = x tan t = __ ​​  x ​​, where x ≠ 0
x
(0, –1)
x2 + y2 = 1 cot t = __
​​  y ​​,where y ≠ 0 ​​  1x ​​, where x ≠ 0 cosec t = __
sec t = __ ​​  1y ​​, where y ≠ 0
where (x, y) is a coordinate point on a unit circle, x2 + y2 = 1.
Figure 9i
Furthermore, it is defined that coordinate points B(x, y) = B(cos θ, sin θ)
are on the circle.
Thus, substituting x = cos θ, y = sin θ in x2 + y2 = 1 gives
sin2 θ + cos2 θ = 1 ––– (1)
Divide equation (1) by cos2 θ:
tan2 θ + 1 = sec2 θ ––– (2)
Divide equation (1) by sin2 θ:
1 + cot2 θ = cosec2 θ ––– (3)
Definition 9.3

The basic trigonometric identities are given by


sin2 θ + cos2 θ = 1
1 + tan2 θ = sec2 θ
1 + cot2 θ = cosec2 θ

These identities are also called the Phytagorean identities. They are very
useful in solving trigonometric problems.

Example 10
Prove the following identities.
(a) sin θ cot θ = cos θ
tan x
(b) ​​ _______
cosec   x  
​​ = sec x – cos x
cos x _______ sin x
(c) ​​ _____
sec x  
​​ + ​​  cosec  x  
​​ = 1
(d) (1 – cos β)(1 + cos β) = _______ ​​  1 2    ​​
cosec β

SOLUTION
(a) sin θ cot θ = sin θ  ​​ _____ 1    
​​
tan θ
= sin θ  _____ ​​  1   ​​ 
sin θ
_____
​   
 ​ 
cos θ
cos θ
= sin θ  _____
​​   ​​ 
sin θ
= cos θ

274

04 Chapter 9.indd 274 18/7/2017 10:04:16 AM


Chapter 9 • Trigonometry

tan x tan x
(b) ​​ _______
cosec   ​​ = _____
x   ​​  ____  
​  1   
sin x
 ​​= tan x sin x
 ​ _____ sin x
= ​​  cos x    
​​  sin x
sin x
2
= _____
​​  cos x ​​ 
1 – cos2 x
= ​​ ________ cos x    ​​ 
= sec x – cos x
cos x _______ sin x cos x _____ sin x
(c) ​​ _____
sec x  
​​ + ​​  cosec   x   ​​ = _____
​​  _____ 1  
 ​​ + ​​  ____   ​​ 
​ ​  1    
​  cos x   ​
sin x
= cos2 x + sin2 x
=1
(d) (1 – cos β)(1 + cos β) = 12 – cos2 β
= sin2 β
​​  1 2   
= _______ ​​
cosec β

Example 11
Prove the following identities.
1 – cos α ________ sin α
(a) ​​ ________   
​​ = ​​  1 + cos 

α   ​​
sin α
(sin x + cos x)2
____________
(b) ​​      ​​ 
= 2 + sec x cosec x
sin x cos x

SOLUTION
1 – cos α ________ 1 – cos α ________ 1 + cos α
(a) ​​ ________   
​​ = ​​ 
     ​​  ​​  1 + cos α 
   ​​
sin α sin α
1 – cos2 α
= _____________
​​        ​​
sin α(1 + cos α)
sin2 α
= _____________
​​        ​​
sin α(1 + cos α)
sin α
= ________
​​  1 + cos  α   ​​
(sin x + cos x)2 ________________________
____________ sin2 x + cos2 x + 2 sin x cos x
(b) ​​    = ​​    
  ​​       ​​
sin x cos x sin x cos x
1 + 2 sin x cos x
______________
= ​​         ​​
sin x cos x
1     2 sin x cos x
= _________
​​  ​​ + __________
​​    ​​
sin x cos x sin x cos x
= 2 + sec x cosec x

275

Pre-Calculus and Calculus.indb 275 13/7/2017 5:57:18 PM


Chapter 9 • Trigonometry

Exercise 9.4
1. Prove the following identities.
1 + tan2 x ____________
1   ​​
(a) ​​ ________   
​​ = ​​    
1 – tan2 x cos2 x – sin2 x
(b) cot2 x – tan2 x = cosec2 x – sec2 x

1   ​​ 
(c) ​​ _______ 1   ​​ 
– ________
​​  = 2 tan x sec x
1 – sin x 1 + sin x
cosec x – sin x
____________
(d) ​​    sec x  = cos3 x
​​ 

(e) sin θ cot θ = cos θ


(f) tan θ cos θ = sin θ
​​  1 2   
(g) (1 – cos α)(1 + cos α) = _______ ​​
cosec α
sec x – cos x
(h) ​​ ___________
sec x    = sin2 x
​​ 

1 + sin x ____________
1 
(i) ​​ ________  
​​ = ​​      ​​
1 – sin x (sec x – tan x)2

1   ​​ 
(j) ​​ ________ = 1 + tan2 x
1 – sin2 x
1 – cos β ________ sin β
(k) ​​ ________   
​​ 
= ​​    ​​ 
sin β 1 + cos β
1 + sin θ ___________ 1   ​​ 
(l) ​​ ________   
​​ 
= ​​ 
cos θ sec θ + tan θ
1 + sec θ ________
1 + cos θ
(m) ​​ ________ ​​ 
= ​​    
​​
sec θ – 1 1 – cos θ
cot θ + 1 ________
1 + tan θ
(n) ​​ ________ ​​ 
= ​​    
​​
cot θ – 1 1 – tan θ
sin3 x + cos3 x sin 2x
(o) ​​ ____________
   ​​ = ​1 – _____
   ​  2 ​​   
sin x + cos x
(p) sec4 x – tan4 x = sec2 x + tan2 x
1 + sin x
(q) ​​ ________  ​​ = (tan x + sec x)2
1 – sin x
tan A + tan B
(r) ​​ ___________
  
cot A + cot B ​​ = tan A tan B

1
(s) ​​ ___________ ​​  sec x 1– 
+ __________
sec x + tan x ​​  tan x ​​ 
= 2 sec x

(t) sec t – sin t tan t = cos t

276

Pre-Calculus and Calculus.indb 276 13/7/2017 5:57:18 PM


Chapter 9 • Trigonometry

9.5 Applications of Trigonometry


One of the major applications of trigonometry is finding the heights of
towers and mountains. In order to do these applications, we need to learn
the concept on angles of elevation and depression.

Angles of Elevation and Depression


A line segment from observation point O to point B being observed is
called the line of sight of B. The angle having its vertex at O, made by a
horizontal ray and the line of sight is called the angle of elevation of B or
the angle of depression of B, depending whether as B is above or below
the point O.
The angles of elevation and depression are always measured with respect
to the horizontal ray. The angle of elevation is formed when one looks up
at point B.
B

ht
f sig
eo
lin
angle of elevation
O horizontal ray

The angle of depression is formed when one looks down at point B.


horizontal ray
O
angle of elevation
lin
eo
f si
ght

B
Take a look at the illustration below. The angle of elevation is denoted by
a, and the angle of depression is represented by β.

As you can see in the illustration above, the angles of elevation and
depression form alternate interior angles. In this case, we can use
properties of parallel lines to solve problems involving angles of elevation
and depression.
277

Pre-Calculus and Calculus.indb 277 13/7/2017 5:57:18 PM


Chapter 9 • Trigonometry

Example 12
A tower casts a shadow of 100 meters long. The angle of elevation of
the point A on the ground to the top of the tower is 50°. What is the
height of the tower?
C

50º
A 100 m B

SOLUTION
Find BC using the tangent function of 50º.
BC
tan 50º = ___
​​ AB  ​​
BC
tan 50º = ____
​​ 100  ​​ 
(100)(tan 50º) = BC
BC ≈ 119.18 m
Hence, the height of the tower is approximately 119.18 meters.

Example 13
Karen needs to know the height of a tree. From a given point A on the
ground, he finds that the angle of elevation to the top of the tree is 35.6º.
He then moves back 50 meters. From the second point B, the angle of
elevation to the top of the tree is 24.4º. Find the height of the tree.
C

24.4º 35.6º
B 50 m A D

SOLUTION
There are two right triangles in the diagram. Line segment CD represents
the height of the tree. First, apply the tangent function of 35.6º on the
right triangle ACD.
CD
tan 35.6º = ___
​​ AD  ​​
(AD)(tan 35.6º) = CD
CD = (AD)(tan 35.6º)

278

Pre-Calculus and Calculus.indb 278 13/7/2017 5:57:18 PM


Chapter 9 • Trigonometry

Next, apply the tangent function of 24.4º on the right triangle BCD.
CD
tan 24.4º = ___     
​​ BA + AD ​​
CD
tan 24.4º = ____    ​​ 
​​ 50 + AD
(50 + AD)(tan 24.4º) = CD
CD = (50 + AD)(tan 24.4º)
Since each expression is equal to CD, we equate both expressions
together.
(AD)(tan 35.6º) = (50 + AD)(tan 24.4º)
(AD)(tan 35.6º) = (50)(tan 24.4º) + (AD)(tan 24.4º)
(AD)(tan 35.6º) – (AD)(tan 24.4º) = (50)(tan 24.4º)
(AD)(tan 35.6º – tan 24.4º) = (50)(tan 24.4º)
(50)(tan 24.4º)
AD = ___________________
​​  tan
      35.6º – tan 24.4º ​​
AD ≈ 86.46656
Find the value of CD by substituting the value of AD in any of the
equations.
CD
tan 35.6º = ​​ ___
AD  ​​
CD
0.7159 = _______    ​​ 
​​  86.46656
CD ≈ 61.90
The height of the tree is approximately 61.90 meters.

Example 14
A bus is parked 200 m away from a building. The angle of depression Note
of the bus (at point C) from the top of the building (at point B) is 55º. The horizontal ray and the
Find the height of the building. ground (AC) form a pair of
B D parallel lines. The angle of
depression from point B
55º to point C and the angle
of elevation from point C
to point B form pairs of
adjacent angles.
Hence, the angles of
depression and elevation
are equal.
A 200 m C

279

Pre-Calculus and Calculus.indb 279 13/7/2017 5:57:19 PM


Chapter 9 • Trigonometry

SOLUTION
The angles of depression and elevation are equal.
∠ ACB = ∠ DBC
∠ ACB = 55º
Apply the tangent function of 55º to find AB.
AB  ​​
tan 55º = ___
​​ AC

AB  ​​ 
tan 55º = ____
​​ 200
(200)(tan 55º) = AB
AB ≈ 285.63
The height of the building is approximately 285.63 meters.

Exercise 9.5
1. Mr. Raymond spotted something shiny on the ground from the balcony of his house. The balcony is
32 meters above the ground. Suppose the angle of depression is 23º. How far is the shiny object from
the ground?

2. A tree casts a shadow 70 meters long at an angle of elevation of 30º. How tall is the tree?

3. Liza took her son to the tower where she works. Amazed at what he saw, he asked his mother how tall
it was. Liza gave his son an estimate. Suppose she and his son were standing 100 meters away from
the tower. The angle of elevation from the point where they were standing to the top of the tower was
75º. How tall is the tower?

4. An observer notices a poster on a wall of a building 100 meters away. Suppose the angle of elevation
from the ground to the poster is 22º, and the angle of elevation from the ground to the top of the
building is 43º. How far is the poster from the top of the building?

5. From a point 200 meters away from the base of a building on a flat ground, the top of the building is at
an angle of elevation of 60º. What would be the angle of elevation for a point at half the height of the
building as seen from the same location?

280

Pre-Calculus and Calculus.indb 280 13/7/2017 5:57:19 PM


Functions and Graphs
Chapter 10:

Functions

Physiology

Determining a target training heart rate is important to many people, especially athletes
and patients recovering from heart attacks. One of the most commonly used methods to
determine a target training heart rate when exercising is the Karvonen method. This
method uses a linear function which relates the target heart rate in beats per minute
(bpm) to the resting and reserve heart rates.
The target heart rate (HRtarg)is computed as
HRtarg = p(HRrsv) + HRrest
where p is the training percentage,
HRrest is the resting heart rate in beats per minute (bpm), and
HRrsv is the reserve heart rate.
The reserve heart rate is computed as HRrsv = HRmax – HRrest where HRmax is the
maximum heart rate. The maximum heart rate is equal to HRmax = 220 – A where A
is the age of the person training. Depending on the training targets, p is commonly
between 50% and 85% (0.50 – 0.85) when exercising.

Pre-Calculus and Calculus.indb 281 13/7/2017 5:57:19 PM


Chapter 10 • Functions

10  Functions
10.1 Definition of a Relation
10.2 Definition of a Function
10.3 Domain and Range of a Function
10.4 Composite Functions
10.5 Inverse Functions

10.1  Definition of a Relation


It is common to see around us ‘relation’ between a group of individuals
with other groups of individuals. This ‘relation’ can be due to family
relationships, similarities or even differences. This ‘relation’ can be
expressed mathematically.

Definition 10.1

A relation between two sets, A and B, is the matching between the


elements of set A to the corresponding elements of set B under a set of
rule called the law of correspondence.

Consider the following statements.


Carlos owns an apartment.
Andres owns a house.
Maria owns a farm.
Laura owns an apartment.

The statements show a kind of relation between four individuals and the
types of property. The set of individuals is A = {Carlos, Andres, Maria,
Laura} and the set of the types of property is B = {apartment, house, farm}.
The law of correspondence that defines the relation between these two sets
is  f  = “own”. This relation can be described by the following diagram.

A B
f = “own”
Carlos apartment
Andres
house
Maria
Laura farm

Figure 10a

282

Pre-Calculus and Calculus.indb 282 13/7/2017 5:57:19 PM


Chapter 10 • Functions

The relation can also be stated as ordered pair:


{(Carlos, apartment), (Andres, house), (Maria, farm), (Laura, apartment)}
or by ‘’ symbol:
Carlos  apartment
Andres  house
Maria  farm
Laura  apartment
or by a graph:
Name
Carlos
Andres
Maria
Laura
Types of property
apartment farm house

Relation defined by Figure 10a is a many-to-one relation since both Carlos


and Laura own an apartment.
There are four types of relation altogether: one-to-one, one-to-many, many-
to-one and many-to-many. Below are some examples of the respective
relations.
Relation in Figure 10b is a one-to-one relation because each person in set
A only owns one type of property in set B.

A B
f = “own”
Andres apartment
Diego house
Paula farm
Diana shop

Figure 10b

Relation in Figure 10c is a one-to-many relation because at least one


person in set A owns two or more types of property in set B.

A f = “own” B

apartment
Juan
house
Sofia
land
Luisa
farm
shop
Figure 10c

283

Pre-Calculus and Calculus.indb 283 13/7/2017 5:57:19 PM


Chapter 10 • Functions

Relation in Figure 10d is a many-to-one relation because more than one


person in set A owns the same type of property in set B.
A f = “own” B

apartment
Juan
house
David
farm
Camilo
shop

Figure 10d

Relation in Figure 10e is a many-to-many relation because at least one


person owns more than a type of property and at least one person has the
same type of property.
A f = “own” B

Mike apartment
Miguel house
Valentina farm
Andres shop

Figure 10e

Example 1
Given a relation ‘a factor of’ from set A = {2, 3, 4} to
set B = {1, 2, 3, 4, 5, 6, 7, 8}, describe the relation in a diagram.
Determine the type of this relation.

SOLUTION
A B
f = “a factor of”
1
2 2
3
4
3
5
6
4 7
8

2 in set A is a factor of 2, 4, 6 and 8 in set B.


3 in set A is a factor of 3 and 6 in set B.
4 in set A is a factor of 4 and 8 in set B.
Hence, this relation is a many-to-many relation.

284

Pre-Calculus and Calculus.indb 284 13/7/2017 5:57:19 PM


Chapter 10 • Functions

Example 2
Given a relation, y = x2 from set X = {x: 0  x  3, x  } to set
Y = {y: 0  y  9, y  }, describe the relation in a graph. Determine the
type of this relation.

SOLUTION
y

x
0 2 4
This relation is a one-to-one relation because each possible element x in
set X is related to only one element y in set Y.

Definition 10.2

Consider a relation from set A to set B. Set A is called the domain of


this relation. Elements in set B which are matched with elements in
set A is called the range of relation. The whole of set B is called the
codomain of the relation.

Example 3
Consider the given relations. Determine the domain, range and
codomain of the relation.
A B
f = “own”
Andres apartment
Diego house
Paula farm
Diana shop

SOLUTION
Domain = {Andres, Diego, Paula, Diana}
Range = {apartment, house, farm, shop}
Codomain = Range = {apartment, house, farm, shop}

285

Pre-Calculus and Calculus.indb 285 13/7/2017 5:57:19 PM


Chapter 10 • Functions

Example 4
Consider the given relations. Determine the domain, range and
codomain of the relation.
f = “a factor of ”
A B

1
2 2
3
4
3
5
6
4 7
8

SOLUTION
Domain = {2, 3, 4}
Range = {2, 3, 4, 6, 8}
Codomain = set B ={1, 2, 3, 4, 5, 6, 7, 8}

Example 5
Consider the given relations. Determine the domain, range and
codomain of the relation.
y
domain

6
range
4

x
0 2 4

SOLUTION
Domain = {x: 0  x  3, x  }
Range = {y: 0  y  9, y  }
Codomain = Range = {y: 0  y  9, y  }

286

Pre-Calculus and Calculus.indb 286 13/7/2017 5:57:19 PM


Chapter 10 • Functions

10.2  Definition of a Function

Definition 10.3

A function is a relation from set A to set B such that every element of


set A is matched with only one element of set B.

Thus, out of the four types of relations, only one-to-one and many-to-one
relations satisfy this condition and are considered to be functions.
The vertical line test is a graphical way of determining whether a relation is
a function.

Definition 10.4

The vertical line test states that, if any vertical line intersects the graph
of a function in at most one point, then it is a graph of a function.

Vertical lines are drawn to see if the lines intersect the graph at more than
one point. An alternative way, instead of drawing vertical lines, is to use a
pen or a ruler and to move it across the graph.
Consider the graph of a relation below.
move the pencil in this direction

2
x
-4 -2 0 2 4
-2

-4

Notice that the pencil intersects the graph at only one point. Therefore, the
relation is a function.

287

Pre-Calculus and Calculus.indb 287 13/7/2017 5:57:21 PM


Chapter 10 • Functions

Example 6
Given the following graphs, determine whether the relation defined by
each graph is a function.
(a) y (b) y

4
Note 3
Only one-to-one and 2
2
many-to-one relations
x
are functions. 0 2 4 1
–2
x
–8 –4 4 8
–4

(c) y

x
–8 –4 4 8
–2

SOLUTION
(a) y
4

2
Move line A along the x-axis. At each
x position, line A cuts the curve once.
0 2 4 Hence, the relation is a function.
–2

–4
line A

(b) y

3 Move line B along the x-axis.


At each position, line B cuts the
2
curve once. Hence, the relation is
1 a function.

x
–8 –4 4 8
line B

288

Pre-Calculus and Calculus.indb 288 13/7/2017 5:57:21 PM


Chapter 10 • Functions

(c) y

Move line C along the x-axis.


2 Line C cuts the curve twice
at most values of x. Hence,
x
–8 –4 4 8 this relation is a one-to-many
–2
relation. Hence, it is not a
function.
line C

Numerous notations have been used to represent function. The most popular
and useful one is the  f (x) notation, read ‘f  of x’.  f (x) denotes the number
matched to x by the function. The value for  f (x) is obtained by replacing
every x in the  f (x) assigned to x as illustrated in the following examples.

Example 7
Consider the function  f (x) = 3 + 2x for x = 1, 2 and 3. Find f(1), f (2)
and f (3).

SOLUTION
For x = 1,  f (1) = 3 + 2(1) = 5.
For x = 2,  f (2) = 3 + 2(2) = 7.
For x = 3,  f (3) = 3 + 2(3) = 9.

Example 8
4 +__2x
Given that  f (x) = ​​ ______ ​  
​√ x   ​​ for x > 0, find  f (4).

SOLUTION
Substituting x = 4 into  f (x),
4 + 2(4) 12 ​​ = 6
f (4) = ​​ _______
__ ​​ = ​​ ___

2
​  4 ​ 

Functions can be represented using other letters besides f, for instance, g,


h, F or G. Certain functions have been given special names such as ‘lg’ for
logarithm of base 10 or ‘cos’ for cosine function.

289

Pre-Calculus and Calculus.indb 289 13/7/2017 5:57:21 PM


Chapter 10 • Functions

Exercise 10.2
1. For each of the relations H, identify its type of relation and determine whether it is a function.
(a) A H B (d) A H B

(b) A H B (e) A H B

(c) A H B (f) A H B

2. D
 etermine whether the following graphs are functions.
(a) y (b) y

290

Pre-Calculus and Calculus.indb 290 13/7/2017 5:57:22 PM


Chapter 10 • Functions

(c) y (e) y

x x

(d) y (f) y

x x

3. Determine whether each of the relations H is a function. Explain your answer.


(a) A = {a: 0 < a ≤ 5, a ∈ }
B = {2, 3, 8, 10, 18, 30, 32, 40, 50}
H = {(a, b): b = 2a2}
(b) H = {(1, 3), (2, 5), (3, 5), (4, 1), (7, 2)}
(c) P H Q

1 0
2 –1
3 –2
4 –3
5 –5

4. Determine whether the following graphs are functions.


(a) (b) y
y

3 2

2
1
1
x
1 2 3
x
1 2 3

291

Pre-Calculus and Calculus.indb 291 13/7/2017 5:57:22 PM


Chapter 10 • Functions

10.3  Domain and Range of a Function


The definition of a function is accompanied with the statements on its
domain and range. The same meaning of domain and range used in
relations applies here.

Definition 10.4

The domain of f (x) is the set of all values taken by x denoted by Df ,
while the range is all the corresponding values taken by f (x) for each
x denoted by Rf .
If  f (x1) is contained in , we also say that  f (x) is defined at x = x1.
Otherwise,  f (x) is undefined at x = x1.

Example 9
Determine the domains and ranges of the following functions.
(a)  f (x) = 7 + x
2x + 1
(b) h(x) = ______
​​  x + 3 ​​ 
__
(c) g(x) = √  ​​ x  ​​

SOLUTION
(a)  f (x) = 7 + x
The domain consists of all real numbers x for which f (x) = 7 + x are
defined.
Hence, Df = {x: x  } or Df = (–∞, +∞) or simply Df = .
The range consists of values taken by f (x) for each x in Df . You may
plot the graph of f (x) = 7 + x. It is clear that f (x) can take values
from –∞ to +∞. That is, the range consists of all real numbers as
well.
Hence, Rf = {x: x  } or Rf = (–∞, +∞) or simply Rf = .
y

4
Range, Rf
x
–8 –4 4 8
–4

Domain, Df

292

Pre-Calculus and Calculus.indb 292 13/7/2017 5:57:22 PM


Chapter 10 • Functions

2x + 1
(b) h(x) = ______
​​  x + 3  
 ​​ The function h (x) is defined and takes real numbers for any real
number x except x = –3 (division of zero is not possible).
Hence, Dh = {x: x  , x ≠ 3} or Dh = {x: x  \{–3}}.
y

4
Range, Rf
x
–8 –4 4 8
–4

Domain, Df

By the graph of h(x), it is clear that h(x) takes all values in real
number set except h(x) = 2.
Hence, Rh = {y: y  , y ≠ 2} or Rh = {y: y  \{2}}.
__
(c) g(x) = √ 
​​ x  
 ​​ The function is defined if x takes positive numbers or zero (square
root of a negative number cannot give a real number), thus g(x) can
only take positive numbers or zero. Hence, the domain and range
of this function are given by Dg = Rg = [0, +∞).
y

2 Range, Rf

x
4 8
Domain, Df

293

Pre-Calculus and Calculus.indb 293 13/7/2017 5:57:22 PM


Chapter 10 • Functions

Example 10
Determine the domains and ranges of the following piecewise functions.
2, 0 < x  3
(a)  f (x) = –1, 3 < x  4
–8, 4  x  7
x + 7, –6 < x  –3
(b) g(x) =
1 – x, –3 < x  4

SOLUTION
(a) y

6
4
2
x
–2 2 4 6 8
–4
–6
–8

Df = (0, 7]
Rf = {–8, –1, 2}
(b) y
6

x
–6 –4 –2 2 4
–2

–4

Dg = (–6, 4]
From the graph of g(x), Rg = [–3, 4].

Definition 10.5

The function f is an onto function from X to Y if each object in a


domain X can be mapped to an image in the range Y under the function
f.

Definition 10.6

The function f is a one-to-one function from X to Y if every object in


a domain X can be mapped to exactly one image in the range Y under
the function f.

294

Pre-Calculus and Calculus.indb 294 13/7/2017 5:57:22 PM


Chapter 10 • Functions

Example 11
Determine whether the following diagrams describe an onto function or
a one-to-one function from P to Q.
(a) P f Q (b)
P f Q

SOLUTION
(a) It is a one-to-one function from P to Q because every element in P
has exactly one corresponding element in Q.
(b) It is an onto function from P to Q because every object in P is
mapped to an image in Q.

Definition 10.7

The function f is an even function if  f (x) =  f (–x).


Geometrically, the graph of an even function is symmetric with respect
to the y-axis.

Definition 10.8

The function  f  is an odd function if – f (x) = f (–x).


Geometrically, the graph of an odd function is symmetric with respect
to the origin.

Example 12
Show that  f (x) = x2 + 8 is an even function.
SOLUTION
We need to show  f (x) =  f (–x).
 (–x)2 + 8
 f (–x) =
= x2 + 8
=  f (x)
Hence,  f (x) = x2 + 8 is an even function.

Example 13
Show that  f (x) = 5x3 – x is an odd function.
SOLUTION
We need to show – f (x) =  f (–x).
 5(–x)3 – (–x)
 f (–x) =
= –5x3 + x
= –(5x3 + x)
= – f (x)
Hence,  f (x) = 5x3 – x is an odd function.
295

Pre-Calculus and Calculus.indb 295 13/7/2017 5:57:22 PM


Chapter 10 • Functions

Exercise 10.3
1. Find the domains and ranges of the following functions defined by the law of correspondence.
_____
(a) f (x) = √ 
​​ x – 3 ​​   ​​  2 1   ​​ 
(i) f (x) = _____
x –1
_____
(b) f (x) = √ 
​​ 3 – x ​​   (j) f (x) = ______ 1   ​​ 
​​  _____
​√ x – 1 ​ 
2
_____

√ 
______
x–1
(c) f (x) = √
​​  2x + 1 ​​   (k) f (x) = ​​ _____
​  x + 1 
 ​ ​​ 
__
x2 – 1
(d) f (x) = √ 
​​ x2 ​​   (l) f (x) = _____
​​  x + 1 ​​ 

​​  1x ​​
(e) f (x) = __ (m) f (x) = _______ 1   ​​ 
​​  ______
​  2x + 1 ​ 

(f ) f (x) = _____ 1   ​​  


​​  x + (n) f (x) = |x|
2

3
(g) f (x) = ______
​​  2x – 2 ​​   (o) f (x) = |x + 2|

1   ​​   |x|
(h) f (x) = ______
​​  _____ (p) f (x) = _____
​​  x +  1 ​​ 
​  x + 4 ​ 

2. Calculate   f (2),  f (–1) and  f (a + h) for each of the following functions, if they exist.
|x|
​​  2x 2+ 2 ​​  
(a) f (x) = ______ (c) f (x) = _____
​​  x –  1 ​​ 
_____

√ 
x+2
(b) f (x) = ​​ _____
​  x – 2 ​ ​​  
  (d) f (x) = x2 + 2x – 3

3. Determine whether the following plots are functions. If yes, find the domains and ranges of the
functions.
y y
(a) (c)
5

x
x
4

(b) y (d) y

2 x
x

296

Pre-Calculus and Calculus.indb 296 13/7/2017 5:57:22 PM


Chapter 10 • Functions

4. Find the domain and range of the following functions.


(a) y = (x + 1)3 – 2
(b) y = 2ex
(c) y = ex + 2 – 1
_____
(d) y = –​​√ x + 1 ​​ + 1
(e) y = ln (x + 1)
(f) y = ln (x – 3) + 2
​​  x –1 1 ​​ + 2
(g) y = _____
(h) y = (x – 2)2 + 2
_____
(i) y = √ 
​​ x – 1 ​​ + 2
___
(j) y = √ 
​​ –x ​​ 
(k) y = –e–x
(l) y = |x – 1| + 2
(m) y = –ln (x + 2) – 2

(n) y = (x – 2)2, x  2
2, x < 2

(o) y = ln x, x  1
x3, x < 1

5. Determine whether the following functions are odd or even. Explain your answers.
(a) y = x5 + 3x3
(b) y = (2x2 + 1)2
(c) y = 4x6 – 6
(d) y = 2x3 – 3

297

Pre-Calculus and Calculus.indb 297 13/7/2017 5:57:22 PM


Chapter 10 • Functions

10.4  Composite Functions


New functions can be constructed from given functions in many ways.

Definition 10.9

Let f (x) and g(x) be two functions with domain Df and Dg respectively.
The sum, difference, product and quotient of the functions f (x) and g(x)
can be defined analytically as follows:
(a) (f + g)(x) = f (x) + g(x) in which Df + g = Df  Dg
(b) (f – g)(x) = f (x) – g(x) in which Df – g = Df  Dg
(c) (f  g)(x) = f (x)  g(x) in which Df  g = Df  Dg
​  g  ​)​​( x) = ____
(d) ​​ (__
f f (x)
​​    ​​ in which ​​D​ __​ gf  ​​​​​​ = Df  Dg – {x: g(x) = 0}
g(x)

Example 14
Let f (x) = 2x + 3 and g(x) = 4x + 1 be two functions with domain Df = 
and Dg =  respectively. Hence, find the following and their domains.
(a) (f + g)(x) (c) (f  g)(x)
(d) ​​ (__ ​ ​​( x)
​  g  )
f
(b) (f – g)(x)

SOLUTION
(a) (f + g)(x) = f (x) + g(x) = 6x + 4 with Df + g = Df  Dg = 
(b) (f – g)(x) = f (x) – g(x) = 2 – 2x with Df – g = Df  Dg = 
(c) (f  g)(x) = f (x)  g(x) = (2x + 3)(4x + 1) with Df  g = Df  Dg = 
(d) ​​ (__ ​ ​​( x) = ____
​  g  )  ​​ with ​​D​ __​ gf  ​​​​ = { 1 ​  .​​  
4 ​ }}
f f (x) ______
​​  x: x   \ ​ {–​ __
2x + 3
​​    ​​ = ​​   
g(x) 4x + 1

Example 15
Let f (x) = 2x + 1. Find f (x2), f ( f (x)) and [f (x)]2.

SOLUTION
f (x2) = 2(x2) + 1 = 2x2 + 1
f ( f (x)) = f (2x + 1) = 2(2x + 1) + 1 = 4x + 3
[f (x)]2 = (2x + 1)2

In some cases, we need to evaluate __


a function of x. For
a function of_______
​​  |–8| + 2 ​​. 
example, we want to calculate e​​ ​​ ​√ 8 ​ ​​or sin (ln 2) or √
__
The calculation of e​​ ​​ ​√ 8 ​ ​​can be__
accomplished in two steps:
Step 1: Find the value of √  ​​ 8 ​​ ≈ 2.8284.
__
Step 2: Evaluate e​​ ​​ ​√ 8 ​​​  = e2.8284 ≈ 16.918.
__ __
If we let u = g(x) = √  ​​ x ​​  and f (u) = eu, then finding ​​e​​  ​√ 8 ​​​  is basically the
same as solving f (g(8)). Function defined by f (u) = f (g(x)) is called the
298 composition of f with g.

Pre-Calculus and Calculus.indb 298 13/7/2017 5:57:22 PM


Chapter 10 • Functions

Definition 10.10

Given functions f and g,  f  g exists if Rg  Df ≠ ϕ. This is called the


composite function of  f and g and is defined as
( f  g)(x) = f (g(x))
The domain of f  g is the set of all real numbers x in the domain of g,
which in turn, g(x) is in the domain of f.

Example 16
If ( f  g)(x) = 9x2 + 6x and g(x) = 3x + 1, find f (x).

SOLUTION
Given (f  g)(x) = 9x2 + 6x
f  (g(x)) = 9x2 + 6x
f (3x + 1) = 9x2 + 6x  ––– (1)
Let y = 3x + 1
y–1
x = _____
​​  3 ​​ 

y–1
Substituting x = ​​ _____
3 ​​ 
 into equation (1),

f (y) = 9​​​ (​  3 ​  )​​​   ​​ + 6​​ (​   ​  )​​  


2
y–1
_____ y–1
_____
  3  
= 9​​ (_________  ​​  + 2(y – 1)
)
y​ ​​  2​– 2y + 1
​  9 ​  
= y2 – 2y + 1 + 2y – 2
= y2 – 1
Hence, f (x) = x2 – 1.

Observe that for f (g(x)) to exist, x must be in the domain of function g and
g(x) must be in the domain of function f. Otherwise, f (g(x)) does not exist.

The domain and range of can be identified easily by using the arrow
diagram based on the five cases as shown below.
Case 1: Rg = Df  Df  g = Dg
Rf  g = Rf g f

Dg Rg Df Rf

Case 2: Rg  Df = Rg  Df  g = Dg
Rf  g  Rf
g f

Rg
Dg Df Rf

299

Pre-Calculus and Calculus.indb 299 13/7/2017 5:57:22 PM


Chapter 10 • Functions

Case 3: Rg  Df = Df  Df  g  Dg
Rf  g = Rf
g f

Df
Dg Rg Rf

Case 4: Rg  Df ≠ Rg ≠ Df ≠ ϕ Rg

 D f  g  Dg
Df  g g
 Rf  g  Rf
f Rf  g
Dg

Df Rf
Case 5: Rg  Df ≠ ϕ  f  g does not exist
By determining the intersection of Rg and Df , we can decide which case
should be considered. It can be observed that Df  g is always a subset of Dg
and Rf  g is always a subset of Rf .

Example 17 _____
Let f (x) = x2 – 2 and g(x) = √ 
​​ x – 1 ​​ 
.
(a) Determine Dg, Rg, Df and Rf .
(b) Draw the arrow diagram representing the composite function f  g.
Comment on the existence of f  g. Find f  g.
(c) Find Df  g and Rf  g.
(d) Given that h(x) = 2x, find f  g  h and comment on the existence of
f  g  h.

SOLUTION
– 2: Df = (–∞, ∞) and Rf = [–2, ∞)
(a) f (x) = x2_____
g(x) = √  ​​ x – 1 ​​ 
: Dg = [1, ∞) and Rg = (0, ∞)
(b) Rg  Df = Rg

g f

Rg
Dg Df Rf

It can be seen that Rg is contained in Df . Hence, f  g exists.
f  g = f (g(x))
_____
= f (√ )
​​  x – 1 ​​ 
_____ 2
= (​​​  ​√ x – 1 ​) ​​​   ​​ – 2
=x–1–2
=x–3

300

Pre-Calculus and Calculus.indb 300 13/7/2017 5:57:23 PM


Chapter 10 • Functions

(c) Since Rg  Df = Rg, then Df  g = Dg = [1, ∞). On the other hand, Rf  g


is determined by taking Rg as the new domain of f.
y

–2
Hence, Rf  g = [–2, ∞). New domain = [0, ∞)
(d) (f  g  h)(x) = ( f  g)(h(x))
= ( f  g)(2x)
= (2x) – 3
= 2x – 3
f  g  h exists if Rh  Df  g ≠ ϕ
Rh =  and Df  g = [1, ∞)
Hence, f  g  h exists.

Example 18
1
Given that f (x) = ​​ _____
x –  1 ​​ and g(x) = x , find the domain and range of f  g.
3

SOLUTION
Df = (–∞, 1)  (1, ∞)
Rf = (–∞, 0)  (0, ∞)
Dg = 
Rg = 
Observed that Rg  Df = Df . Hence, we have the following diagram.
Rg
Dg

g f

Df
Rf =
y
Hence, Rf  g = Rf = \{0}.
On the other hand, Df  g is
determined by taking
Df as the new range of g.
New range =
x
1

Hence, Df  g = \{1}.
301

Pre-Calculus and Calculus.indb 301 13/7/2017 5:57:23 PM


Chapter 10 • Functions

Exercise 10.4
1. Find the composite functions, f  g and g  f of each of the following and determine their respective
domains and ranges, if they exist.
​​  12 ​​ x + 7
(a) f (x) = x + 2, g(x) = __ ​​  2 1   ​​, g(x) = x2 + 3
(d) f (x) = _____
x +1
(b) f (x) = x + 2 , g(x) = 3x + 7
2 _____ 1   ​​, g(x) = |x|
(e) f (x) = ​​  x + 1
_____
(c) f (x) = x2 + 1, g(x) = √ 
​​ x + 2 ​​ 

2. Identify the possible functions of f (x) and g(x) given the following f  g.
x2 + 1
(a) ​​ _____  ​​  
  (c) x2 – 10x + 25
x2
______ |x + 2|
(d) ​​ _____
3
(b) ​​√   2x + 1 ​​  
x + 3 ​​ 
_____
3. Given that f (x) = 3x + 2 and g(x) = √ 
​​ x + 1 ​​ 
, find ( f  g)(3) and (g  f )(–1).

4. Given that f (g(x)) = 3x2 – 1 and g(x) = 3x2 – 1, find f (x).

​​  1x ​​, find the following.


5. Given that f (x) = x – 2, g(x) = x2 and h(x) = __
(a) (  f  + g)(x) and its domain
(b) (  f  – g)(x) and its domain
(c) (g  h)(x) and its domain
​   ​)​​( x) and its domain
g
(d) ​​ (__
h
​​  2x ​​, find the following.
6. Given that f (x) = 1 – x, g(x) = x2 – 3 and h(x) = __
(a) f  g (b) g  f (c) f  h (d) h  h (e) f  g  h

7. If h(x) = x + 1 and (k  h)(x) = x2 + 2x + 1, find k(x).
__
8. Given that f (x) = ​​√ x ​​,  g(x) = x and h(x) = ex, find the composite function f  g  h and determine its
domain.

9. Functions f, g and h are defined as follows:


f (x) = |x + 3|
g(x) = x2 + 1
h(x) = __​​  1x ​​
(a) Find f  g  h and determine its domain.
(b) Find all the asymptotes of f  g  h.
(c) Sketch the graph of f  g  h.

​​  12  ​​,  find h(x) such that (f  g  h)(x) = _____


10. Given that f (x) = x and g(x) = __ ​​  1  2 ​​, and determine the domain
x (3 – x)
of f  g  h.

302

Pre-Calculus and Calculus.indb 302 13/7/2017 5:57:23 PM


Chapter 10 • Functions

10.5  Inverse Functions


 f  : x → y is a function that maps x to y, as described by the arrow diagram
in Figure 10f(i). Clearly, f is a one-to-one function. If we reverse the
relationship between the two quantities, we have a relationship described
by Figure 10f(ii) and denoted by . Clearly, is also a one-to-one
function with . The new function is called the inverse of
the original function.
f f –1
2 4 4 2
3 9 9 3
4 16 16 4
5 25 25 5

x y y x
(i) (ii)
Figure 10f

Consider the relationship described in Figure 10g(i). Since it is a


many-to-one relation, g is a function. However, when we reverse the
original function as shown in Figure 10f(ii), the new relation is one-to-
many relation. One-to-many relation is not a function. Hence, the inverse
of g does not exist.
g g –1
2 4 4 2
3 9 9 3
4 16 16 4
5 25 25 5

x y y x
(i) (ii)
Figure 10g

In general, a function f only has an inverse if f is a one-to-one function.

Given the graph of a function, the horizontal line test can be used to
determine whether the function is a one-to-one function.

Definition 10.11

The horizontal line test states that, if a horizontal line moving along the
y-axis intersects the graph of a function in at most one point, then the
function f is a one-to-one function.

303

Pre-Calculus and Calculus.indb 303 13/7/2017 5:57:23 PM


Chapter 10 • Functions

From Figure 10h(i), since a horizontal line moving along y-axis cuts at
only one point, the function f is one-to-one. However, function g defined
in Figure 10h(ii) is not one-to-one because the horizontal line cuts the
curves at more than one point.
y y

4 4
horizontal horizontal
2 line 2 line

x x
–2 2 –2 2
–2 –2

–4 –4

(i) (ii)
Figure 10h

Definition 10.12

If  f : x → y is a one-to-one function that maps x to y, then the inverse


function, written as  f –1, is a function that maps y back to x. It follows
that the domain of  f –1 is the range of  f  and the range of  f –1 is the
domain of f. Each f and  f –1 reverses the effect of the other.

Without the graph, we can still determine one-to-one function algebraically


as shown in Examples 16 and 17.

Example 19
Determine whether the following functions have inverse functions. If
yes, find the domain and range of their inverses.
_____
(a) f (x) = √ 
​​ x + 2 ​​   (b) g(x) = x2

SOLUTION_____
​​ x + 2 ​​ 
(a) f (x) = √ 
Assuming  f (a) = f (b), where a and b  Df ,
_____ _____
​​√ a + 2 ​​ = √ 
​​ b + 2 ​​ 
a+2=b+2
a=b
 f (a) =  f (b) is one-to-one and hence, it has an inverse function.
For  f (x), Df = [–2, ∞) and Rf = [0, ∞). Hence, the domain and range
of the inverse of  f  is
​​D​ f ​​​ = Rf = [0, ∞) and ​​R​ f ​​​ = Df = [–2, ∞).
–1 –1

(b) g(x) = x2
Since g(2) = 4 and g(–2) = 4, we have a many-to-one function.
Thus, g is not a one-to-one function. Hence, g does not have an
inverse function.

304

Pre-Calculus and Calculus.indb 304 13/7/2017 5:57:23 PM


Chapter 10 • Functions

Example 20
x+1
Show that the function h(x) = ______
​​  2x + 3  is one-to-one.
 ​​ 
SOLUTION
Suppose that h(x1) = h(x2).
x1 + 1 ______ x +1
Then, ​​ ______ 2x1 + 3  = ​​  2x2 + 3 
 ​​   ​​ 
2
(x1 + 1)(2x2 + 3) = (x2 + 1)(2x1 + 3)
2x1x2 + 3x1 + 2x2 + 3 = 2x1x2 + 3x2 + 2x1 + 3
x1 = x2
Since x1 = x2 whenever h(x1) = h(x2), then h(x) is one-to-one.

One of the property of the inverse function of f is that D​ 


​​ f  ​​​ = Rf  and –1

​​ f ​  ​​​ = Df . That means, point


R –1 of  f (x) is changed to (b, a) of y =  f –1(x).
This is achieved by reflecting the points about y = x. Consequently, the
graph of y =  f –1(x) is obtained by reflecting the graph of y =  f (x) about the
line y = x.
Example 21
Consider graph of f (x) = x3 – 8 for x  (0, 2) as shown below. Sketch
the graph of f –1(x).
y

x
4 8
–4

–8

SOLUTION
Identify several important points on the curve of f (x) = x3 – 8, namely
the endpoints A(2, 0) and B(0, –8) and any point in between such as
C(1, –7).
Under the reflection about , A(2, 0) becomes A'(0, 2),
B(0, –8) becomes B'(–8, 0),
C(1, –7) becomes C'(–7, 1).
Connecting the points A', B' and C' will give the graph of f –1(x) as
illustrated below.
y

A x
–8 –4 4 8
–4 f

–8 B C
305

Pre-Calculus and Calculus.indb 305 13/7/2017 5:57:23 PM


Chapter 10 • Functions

The inverse function  f –1(x) is obtained by solving y = f (x) for x as a


function of y, x = g(y). Hence, we write f –1(x) = g(x).

Example 22
4x + 1
Given that g(x) = ​​ ______ ​​, find the inverse function of g, that is, g–1(x).
2x + 3 
SOLUTION
4x + 1
Let y = g(x) = ______
​​  2x + 3 
 ​​.
Express x in terms of y.
y(2x + 3) = 4x + 1
2xy + 3y = 4x + 1
2xy – 4x = 1 – 3y
2x(y – 2) = 1 – 3y
1 – 3y
x = ​​ _______  ​​ 
2(y – 2)
1 – 3x
Hence, the inverse of g(x) is g –1(x) = _______
​​    ​​ 
.
2(x – 2)

Example 23
Let f (x) = x3 – 8 for 0  x  2.
(a) Show that f (x) is one-to-one.
(b) Find f –1(x) and ​​D​ f ​​​.–1

SOLUTION
(a) Suppose that f (x1) = f (x2), for 0  x1  2 and 0  x2  2.
Then, x13 – 8 = x23 – 8
x13 – x23 = 0
(x1 – x2)(x12 + x1x2 + x22) = 0
Since x12 + x1x2 + x22 > 0 except when x1 = x2 = 0,
we have x1 – x2 = 0
x1 = x2
Hence, f (x) is one-to-one.
(b) Let y = x3 – 8
y + 8 = x3
_____
x = ​​√   y + 8 ​​ 
3

3
_____
  x + 8 ​​ 
Hence, f –1(x) = ​​√  .
Since Rf = [–8, 0], the domain of inverse function is
​​D​ f ​​​ = Rf = [–8, 0].
–1

306

Pre-Calculus and Calculus.indb 306 13/7/2017 5:57:23 PM


Chapter 10 • Functions

Example 24
x–2
Show that f (x) = 7x + 2 and g(x) = _____
​​  7 ​​ 
 are inverse of each other.

SOLUTION
f  g = f (g(x))

= f (​​  _____  )​​  


x–2
​  7 ​

= 7​​ (_____  )​​  + 2
x–2
​  7 ​

=x
Hence, f (x) and g(x) are inverses of each other.

Exercise 10.5
1. D
 etermine whether the following functions are one-to-one. If yes, find the inverse functions. Determine
the domain and range of the inverse functions.
(a) ​​  1x ​​ + x
f (x) = __
(b) f (x) = |x| – 2
(c) f (x) = x4 – x2 + 1
(d) f (x) = 3ex + 2
(e) f (x) = 1 – x
(f) f (x) = –x3

2. Show that f (x) and g(x) are inverses of each other for each of the following.
(a) f (x) = x + 2, g(x) = x – 2
__
(b) f (x) = x2 for x  0, g(x) = √  ​​ x  ​​
(c) f (x) = ex – 1, g(x) = ln (1 + x)
1–x 1–x
(d) f (x) = _____  ​​, g(x) = _____
​​  1 + x  ​​  1 + x 
 ​​

3. For each of the following functions, determine whether it is one-to one. If yes, find its inverse and
domain of the inverse function. Sketch, on the same axes, the function and its inverse.
(a) f (x) = 2x + 3
​​  1x ​​
(b) f (x) = __
_____
(c) f (x) = √ 
​​ 5 – x   ​​
(d) f (x) = x 3

4. It is given that f (x) = ln (3x + 2).


(a) Show that f (x) is one-to-one.
(b) Find the inverse function  f –1(x).
(c) State the domains and ranges of f (x) and f –1(x).
(d) Sketch the graphs of f (x) and f –1(x) on the same coordinate axes.

307

Pre-Calculus and Calculus.indb 307 13/7/2017 5:57:23 PM


Chapter 10 • Functions

2x + 3
5. It is given that f (x) = ______
​​  x – 1 ​​, where x ≠ 1.
(a) Find f –1(x).
​​  12 ​​ , find c.
(b) If g(x) = 5 – x and (g  f –1)(c) = __

6. F
 ind the inverse of f (x) = 1 + ex + 2. State the domain and the range of the inverse function f –1(x). Sketch
the graphs of f (x) and f –1(x) on the same coordinate axes.

2x
7. It is given that f​ (x) = _____
​ x –  1 ​​. 
(a) Determine the composite function f  f. State the values of x such that f  f is undefined.
(b) Find the inverse function, f –1(x) and state its domain.
8x2 – 4x
(c) Find g(x) such that (g  f )(x) = _______
​​   ​​ 
.
(x – 1)2

8. It is given that f (x) = 2 – x2 and g(x) = e2x.


(a) Determine whether f is a one-to-one function. Explain your answer.
(b) Find g–1(x) and state its domain.
(c) Find the composite function g­–1  f.

9. Functions f and g are defined as follows:


f (x) = x_____
2
+2
g(x) = √ 
​​ x + 2 ​​ 
(a) State the domains and ranges of f and g.
(b) Find the composite functions g  f and g  f  g.
(c) Sketch the graphs of f and g on the same coordinate axes.
(d) State the domain of f that needs to be defined so that f is the inverse of g.

10. The functions f and g are given as follows:


f (x) = |x + 1|
g(x) = 2x + 1
(a) State the domains and ranges of f and g.
(b) Find the composite function f  g and sketch its graph.
(c) State whether f  g is a one-to-one function. Explain your answer.
(d) State the possible domains of f  g so that it has inverse.
(e) Sketch the possible graphs of (f  g)–1.

308

Pre-Calculus and Calculus.indb 308 13/7/2017 5:57:23 PM


Functions and Graphs
Chapter 11:

Graphs

Polynomial functions play an important role in mathematics, business, science,


engineering and other fields. Polynomials are used to model sales of products for a given
period of time. In business, polynomials are used to present revenue, cost, and profit. In
geometry and construction, polynomial functions are used to obtain the dimensions of
the tools or materials used. Engineers use polynomial functions to represent and model
buildings, highways, bridges and other structures. Also, they can measure complicated
figures with the use of polynomial functions. In this chapter, we will discuss graphs of
polynomials and other functions.

Pre-Calculus and Calculus.indb 309 13/7/2017 5:57:23 PM


Chapter 11 • Graphs

11 Graphs
11.1 Transformation of Graphs
11.2 Linear Functions
11.3 Quadratic Functions
11.4 Absolute Functions
11.5 Rational Functions
11.6 Radical Functions
11.7 Exponential and Logarithmic Functions
11.8 Trigonometric Functions
11.9 Inverse Trigonometric Functions
11.10 Hyperbolic Functions

11.1  Transformation of Graphs


The table below shows the transformation of the graph of f (x).
f (x) + k Translate the graph of f (x) vertically k units
upward if k is positive and k units downward if
k is negative.
Alternative method: Translate the x-axis of the
graph of f (x) k units downward if k is positive
and k units upward if k is negative. Change the
labeling of the y-axis accordingly.
(Warning: This method will also transform all
the other graphs on the same axes. The same
warning applies to all the following alternative
methods.)
f (x + k) Translate the graph of f (x) horizontally k units
to the left if k is positive and k units to the right
if k is negative.
Alternative method: Translate the y-axis of the
graph of f (x) k units to the right if k is positive
and k units to the left if k is negative. Change
the labeling of the x-axis accordingly.
–f (x) Reflect the graph of f (x) in the x-axis.
f (–x) Reflect the graph of f (x) in the y-axis.
f –1(x) Reflect the graph of f (x) in the line y = x.
kf (x) Stretch the graph of f (x) vertically. The scale
factor is k and the x-axis remains unchanged.
If k is negative, the transformation includes a
reflection in the x-axis.
Alternative method: If k is positive, multiply
the labels on the original y-axis by k.

310

Pre-Calculus and Calculus.indb 310 13/7/2017 5:57:24 PM


Chapter 11 • Graphs

f (kx)  tretch the graph of f (x) horizontally. The scale


S
1
__
factor is ​ k ​ and the y-axis remains unchanged.
If k is negative, the transformation includes a
reflection in the y-axis.
Alternative method: If k is positive, divide the
labels on the original x-axis by k.
|f (x)| Reflect the part of the graph of f (x) which is
below the x-axis in the x-axis, and retain the part
of the graph of f (x) which is above the x-axis.
f (|x|)  eplace the part of the graph of f (x) to the left
R
of the y-axis with the reflection of the right of
the graph in the y-axis.
1
___
​​     ​​  The graph f (x) and that of its reciprocal
f (x)
​ ​ 1   ​​  are related in the following ways:
function ___
f (x)
(i) For the same value of x, they have the
same sign.
(ii) Both functions equal 1 simultaneously.
(iii) Both functions cannot be zero for any
value of x.
(iv) When one function approaches zero, the
other approaches infinity.
(v) When one is decreasing, the other is
increasing and vice versa.
f (k – x) or f [–(x – k)]  ranslate the graph k units to the left if k is
T
positive and k units to the right if is negative.
Reflect the new graph in the y-axis.
Alternative method: Reflect the graph of f (x)
k
__
in the vertical axis, x = ​ 2 ​.

f (kx + b) Translate the graph of f (x) horizontally b units


to the left if b is positive and b units to the right if b
is negative. Stretch the new graph horizontally
1
__
with the y-axis unchanged and scale factor of ​ k ​.
If k is negative, the stretch includes a reflection
in the y-axis.

We will discuss more about the specific functions and their transformation
in the subsequent sections.

311

Pre-Calculus and Calculus.indb 311 13/7/2017 5:57:24 PM


Chapter 11 • Graphs

11.2  Linear Functions


In Chapter 10, we show that the determination of domain and range of a
function is made easier by looking at its graph. In this chapter, we look at
the steps used in obtaining the sketches of the graphs based on the basic
graph of the function. In doing so, it is important to have some reference
points or lines for the function such as the origin, y-intercept, x-intercept
and asymptotes.
A linear function is defined as f (x) = a + bx, where a and b are constants.
We can easily plot this line by determining any two points on the line, for
instance, (0, a) and (​​  –​ __ ​ , 0)​​,  and draw a line connecting the points.
a
b
Alternatively, we can obtain the sketch by referring to the basic linear
function,  f (x) = x with the reference point (0, 0). We may then obtain the
sketches of other linear functions from the basic plot using transformation
as described in the table below.
Function Effect Graph
f (x)

Basic function, The reference point is the


x
f (x) = x origin (0, 0). (0, 0)

0 ​ )​​,  that
​  xy ​)​​  = (​​       
​  –2
f1(x) = x + 2 Translation (​​     
f (x)

is, moving the line of f (x) = x


to the left by 2 units.
The reference point (0, 0) is x
(–2, 0)
moved to (–2, 0).
f2(x) = x – 2 Translation (​​      ​  20 ​ )​​,  that is,
​  xy ​)​​  = (​​      f2(x)

moving the line of f (x) = x to x


the right by 2 units. (2, 0)
The reference point (0, 0) is
moved to (2, 0).
f3(x) = –x Reflect the basic line about the f3(x)
x-axis.
Note The reference point (0, 0) does
(0, 0)
• Positive gradient:
not change. x

increasing slope
• Negative gradient:
decreasing slope f4(x) = 2x Values of f 4(x) = 2x are twice f4(x)
the values of f (x) = x, that is, 2x
Note the slope of f4(x) is steeper 4 x
than the slope of f (x). 2
The greater the gradient, the x
The reference point (0, 0) does (0, 0) –2
steeper the slope. not change.

In all cases, it would be helpful in the sketching of the graphs if we identify


312
the y-intercept as well.

Pre-Calculus and Calculus.indb 312 13/7/2017 5:57:24 PM


Chapter 11 • Graphs

Example 1
Sketch the following graphs. State the domain and range of the functions.
(a) f1(x) = x – 3 (c) f3(x) = 1 + 2x
x
(b) f2(x) = __
​​  2  ​​ (d) f4(x) = 5

SOLUTION
(a) f1(x) = x – 3
To obtain the plot of f1(x) = x – 3 from f (x) = x, we do the translation
( ​  yx )
​​      ​  03 ​ )​​  on f (x) = x, that is, moving the line of f (x) = x to the right
​ ​​  = (​​     
by 3 units. The reference point (0, 0) becomes (3, 0).
y-intercept of f1(x): When x = 0, y = –3.
f (x)
x

x–3

(0, 0) x
(3, 0)

–3

The domain is (–∞, ∞) and the range is (–∞, ∞).


x
(b) f2(x) = __
​​  2  ​​
x
To obtain the plot of f2(x) = __
​​  2  ​​ from f (x) = x, values of f (x) = x are
x
halved to give values of f2(x) = ​​ __ 2  ​​, that is , the slope of f2(x) is gentler
than the slope of f (x). The reference point (0, 0) does not change. Note
f (x) The smaller the gradient, the
x gentler the slope.
1– x
2

(0, 0)
x
–2
–1
–2

The domain is (–∞, ∞) and the range is (–∞, ∞).

313

Pre-Calculus and Calculus.indb 313 13/7/2017 5:57:24 PM


Chapter 11 • Graphs

(c) f3(x) = 1 + 2x
First, rewrite f3(x) = 1 + 2x as f3(x) = 2​​ (x + __ ​ 12  ​).​​  
Then, the plot of f3(x) = 2​​ (x + __ ​ 21 ​ )​​ can be obtained from f (x) = x in
two steps:
Step 1: Move the line of f (x) = x to the left by __ ​​ 21 ​​  unit to give
​​  21 ​​ . The reference point (0, 0) is moved to (​​  –​ __ 2 ​ , 0)​​. 
f31(x) = x + __ 1

Step 2: The values of f31(x) = x + __ ​​  21 ​​  are now doubled to give
f3(x) = 2​​ (x + __ ​ 12  ​)​​  or f3(x) = 2x + 1, that is, the slope of f3(x)
is steeper than the slope of f31(x). The reference point

( 2 ​ , 0)​​ does not change.


1
​​  –​ __
y-intercept of  f3(x): When x = 0, y = 1.
​​  21 ​​
f (x) –2x + __

​ 21 ​
x + __
1
x
1 ​​ 
​​  __
( 2 ​, 0)​​  
1
​​  –​ __ 2
x
(0, 0)

The domain is (–∞, ∞) and the range is (–∞, ∞).


(d)  f4(x) = 5
Note
This is a horizontal line.
A horizontal line is a constant f (x)
function. A constant function
is a linear function for which
the range does not change for 5
every value in the domain.
x

Exercise 11.2
1. Sketch the graphs of the following functions.
x x
(a)  f (x) = __
​​  2  ​​ + 3 (g)  f (x) = 16 – __
​​ 2  ​​
(b)  f (x) = 3x – 1 (h)  f (x) = 3x
(c)  f (x) = 2 – x (i)
g(x) = 4x + 3
(d)  f (x) = 2x – 10 (j) h(x) = 2 – 2x
x 6
(e)  f (x) = –​​ __
3  ​​ + 15 (k) p(x) = __​​  7 ​​ x – 2
x
(f)  f (x) = 2 – __ ​​ 5  ​​ (l)   f (x) = 3

314

Pre-Calculus and Calculus.indb 314 13/7/2017 5:57:24 PM


Chapter 11 • Graphs

11.3  Quadratic Functions


A quadratic function is of the form ax2 + bx + c, where a, b and c are
constants. It can also be written in the form a​(​​  x + ___   ​​​   ​ + d​, where
​ 2a  ​)
b 2
4ac – b2
_______
d = ​​  4a    ​​. The basic quadratic function is f(x) = x with the reference
2

point (0, 0). Sketches of the other quadratic functions can be obtained
using transformation as described in the table below.

Function Effect Graph


Basic The reference point is the f (x)
function, origin (0, 0).
f (x) = x2
x
(0, 0)

Translation (​​      0 ​ )​​,  that


​  xy ​)​​  = (​​       
​  –2
f1(x) = (x + 2)2 f1(x)

is, moving the curve of


f (x) = x2 to the left by 2 units.
The reference point (0, 0) is x
moved to (–2, 0). (–2, 0)

f2(x) = (x – 2)2 Translation (​​      ​  02 ​ )​​,  that is,


​  xy ​)​​  = (​​      f2(x)
moving the curve of  f (x) = x2
to the right by 2 units.
The reference point (0, 0) is x
(2, 0)
moved to (2, 0).
f3(x) = x2 + 2 Translation (​​      ​  02 ​ )​​,  that
​  xy ​)​​  = (​​      f3(x)

is, moving the curve of


f (x) = x2 upward by 2 units.
The reference point, (0, 0) is (0, 2)
x
moved to (0, 2).

​  xy ​)​​  = (​​      0  ​  ​​,  that


f4(x) = x2 – 2 Translation (​​      ​  –2 ) f4(x)

is, moving the curve of


f (x) = x2 downward by 2 units.
x
The reference point (0, 0) is
moved to (0, –2). (0, –2)

f5(x) = –x2 Reflect the basic curve about f5(x)


the x-axis. (0, 0)
x
The reference point (0, 0)
does not change.

In all cases, it would be helpful in the sketching of the graphs if we identify


the x-intercept or y-intercept as well. 315

Pre-Calculus and Calculus.indb 315 13/7/2017 5:57:24 PM


Chapter 11 • Graphs

Example 2
Sketch the following graphs. State the domain and range of the
functions.
(a) f1(x) = (x – 2)2 + 3
(b) f2(x) = 2x2 – 1
(c) f3(x) = 1 – (x + 1)2

SOLUTION
(a) f1(x) = (x – 2)2 + 3
To obtain the plot of f 1(x) = (x – 2)2 + 3 from f (x) = x2, we do it in
two steps:
Step 1: Move the curve of f (x) = x2 to the right by 2 units to give
f11(x) = (x – 2)2. The reference point (0, 0) is moved to
(2, 0).
Step 2: Move the curve of f11(x) = (x – 2)2 upward by 3 units to give
f1(x) = (x – 2)2 + 3. The reference point (2, 0) is moved to
(2, 3).
f (x)
(x – 2)2 + 3

(x – 2)2
x2

(2, 3)
x
(0, 0) (2, 0)

The domain is (–∞, ∞) and the range is (3, ∞).


(b) f2(x) = 2x2 – 1
To obtain the plot of f2(x) = 2x2 – 1 from f (x) = x2, we do it in two
steps:
Step 1: The values of f (x) = x2 are doubled to give f21(x) = 2x2, that
is, the curve of f21(x) = 2x2 is narrower than the curve of
f (x) = x2. The reference point (0, 0) does not change.
Step 2: Move the curve f21(x) = 2x2 downward by 1 unit to give
f2(x) = 2x2 – 1. The reference point (0, 0) is moved to
(0, –1).
f (x)
2x2
x2

x
(0, 0)
2x – 1
2

(0, –1)

The domain is (–∞, ∞) and the range is (–1, ∞).


316

Pre-Calculus and Calculus.indb 316 13/7/2017 5:57:24 PM


Chapter 11 • Graphs

(c) f3(x) = 1 – (x + 1)2


To obtain the plot of f3(x) = 1 – (x + 1)2 from f (x) = x2, we do it in
three steps:
Step 1: Move the curve f (x) = x2 to the left by 1 units to give
f31(x) = (x + 1)2. The reference point (0, 0) is moved to
(–1, 0).
Step 2: Reflect curve of f31(x) = (x + 1)2 about the x-axis to give
f32(x) = –(x + 1)2. The reference point (–1, 0) does not
change.
Step 3: Move the curve f 32(x) = –(x + 1)2 upward by 1 unit to give
f (x) = 1 – (x + 1)2. The reference point (–1, 0) is moved to
(–1, 1).
f (x)

(x + 1)2 x2

(–1, 1)

x
(–1, 0)
1 – (x + 1)2

–(x + 1)2

The domain is (–∞, ∞) and the range is (–∞, 1).

Exercise 11.3
1. Sketch the graphs of the following functions.
​ 12  ​)​​​   ​​
(a)  f (x) = 1 + (​​​  x – __
2

(b)  f (x) = 2(1 – x2) – 3


(c)  f (x) = 1 – (x – 1)2
(d)  f (x) = x2 + 4x + 4

2. Sketch the graph of  f (x) = 2x2 – 6x + 5 = 2(x – 1.5)2 + 0.5.

3. Sketch, on the same pair of axes, the graphs of


(a) y = –x2,
(b) y = –3x2,
(c) y = –9x2.

4. Sketch, on the same pair of axes, the graphs of


(a) y = –3(x – 2)2,
(b) y = –(3x – 2)2.

317

Pre-Calculus and Calculus.indb 317 13/7/2017 5:57:24 PM


Chapter 11 • Graphs

11.4  Absolute Functions


x, x  0
The definition of an absolute value of x is |x| = –x, x < 0 .
y

y = |x|

A function y = f (x) has a specific output value for an input value. For any
input value, an output value would be evaluated which could be any real
number based on the function rule.
The absolute value of a function f (x) is | f (x)|.
Here, f (x) is evaluated as usual and the output is made into a positive value
number if it is a negative value.
The absolute values of functions are necessary when calculating quantities
like speed and distance traveled.
The definition of an absolute value of a function is
f (x),  f (x)  0
| f (x)| =
–f (x),  f (x) < 0
For instance,  f (x) = x2 – 9, | f (x)| = |x2 – 9|.
When x = 2,  f (2) = –5
| f (2)| = |–5| = 5
Consider the graph of f(x) = x2 – 3 as shown below.
f (x)

x2 – 3

__ __ x
–​√3 ​  ​ 3 ​ 

–3

The graph of | f (x)| is obtained by reflecting about the x-axis the part of the
graph below the x-axis and retaining the part which is above the x-axis.
The graph of –| f (x)| is the reflection of | f (x)| about the x-axis.
Since | f (x)| is 0 or positive, –| f (x)| is 0 or negative.
f (x)

|x2 – 3|

3
__ __ x
–​√3 ​  ​ 3 
√  ​
318

Pre-Calculus and Calculus.indb 318 13/7/2017 5:57:24 PM


Chapter 11 • Graphs

Exercise 11.4
1. Given the graph of y = x, sketch the following functions on the same axes.
y
y=x

(a) y = –x
(b) y = |x|
(c) y = |–x| + 2
(d) y = 2|x| + 1

2. It is given that a function  f (x) = |3x – 7|.


(a) Sketch y = 3x – 7 and the function  f (x).
(b) What is the equation of the line of symmetry for  f (x)?

3. It is given that a function F(x) = |2x2 + 13x – 7| for all real values of x.
(a) Sketch the function F(x).
(b) What is the equation of the line of symmetry?

4. Sketch the following functions on the same graph.


(a) y = –2x + 3
(b) y = |–2x + 3|
(c) y = |–2x + 3| + 2
__
1
(d) y = ​   ​ |–2x + 3|
2

5. Sketch the graph of the function  f (x) = 2x2 – 9x for –7  x  12.

6. Sketch the graphs of y = 2x + 5 and y = |x + 3| on the same graph. Then, solve the equation
|x + 3| = 2x + 5.

7. Sketch the graph of y = |3 – 2x| – 4 and y = x + 4. Then, solve the equation |3 – 2x| – 4 = x.

319

Pre-Calculus and Calculus.indb 319 13/7/2017 5:57:24 PM


Chapter 11 • Graphs

11.5  Rational Functions


a
We now look at sketching functions of the form ​​ _____     
​​ + c, where a, b and
x–b
1
__
c are constants. The basic function is f (x) = ​​  x ​​. The function does not have
a specific reference point but has two asymptote lines, x = 0 and y = 0 as
the reference lines. Sketches of the other functions of this form can be
obtained using transformation as described in the table below.

Function Effect Graph

Basic function, The reference lines are f (x)


​​  1x ​​
f (x) = __ x = 0 and y = 0.

0 ​ )​​, 
​  xy ​)​​  = (​​       
1    
f1(x) = _____
​ ​x + 2 ​​ Translation (​​      ​  –2 f1(x)
that is, moving the curve
​​  1x ​​ to the left by
of f (x) = __
2 units. x
The reference line, x = 0 –2
is moved to x = –2.
The other reference line,
y = 0 does not change. x = –2

​  x –1 2 ​​ 
​f2(x) = _____ Translation (​​      ​  20 ​ )​​, 
​  xy ​)​​  = (​​      f2(x)

that is moving the curve


​​  1x ​​to the right by
of f (x) = __
2 units. x
2
The reference line, x = 0
is moved to x = 2.
The other reference line,
y = 0 does not change. x=2

​ ​1x ​​  + 2
f3(x) = __ Translation (​​      ​  02 ​ )​​, 
​  xy ​)​​  = (​​      f3(x)

that is, moving the curve


​​  1x ​​ upward by 2
of f (x) = __
units.
The reference line, y = 0 2 y=2
x
is moved to y = 2.
The other reference line,
x = 0 does not change.

320

Pre-Calculus and Calculus.indb 320 13/7/2017 5:57:25 PM


Chapter 11 • Graphs

0  ​  ​​, 
​  xy ​)​​  = (​​       
​ ​1x ​​  – 2
f4(x) = __ Translation (​​      ​  –2 ) f4(x)
that is, moving the curve
​​  1x ​​ downward by
of f (x) = __
2 units. x
The reference point, y = 0 –2 y = –2
is moved to y = –2.
The other reference line,
x = 0 does not change.
​  1x ​​ 
f5(x) = –​__ Reflect the basic curve f5(x)
about the x-axis.
The reference lines, x = 0
and y = 0 do not change.
x

In all cases, it would be helpful in the sketching of the graphs if we find the
x-intercept or y-intercept as well.
Example 3
Sketch the following graphs. State the domain and range of the
functions.
x
​​  x –1 2 ​​ + 2
(a) f1(x) = _____ ​​  2x ​ – 1​
(b) f2(x) = __ (c) f3(x) = _____
​​  x – 1 ​​ 

SOLUTION
​​  x –1 2 ​​ + 2
(a) f1(x) = _____
To obtain the plot of  f1(x) = _____ ​​  x –1 2 ​​ + 2 from f (x) = __ ​​  1x ,​​ we do it in two
steps:
Step 1: M  ove the curve f (x) = __ ​​  1x ​​to the right by 2 units to give
 f11(x) = _____ ​​  x –1 2 ​​ 
. The reference line, x = 0 is moved to x = 2.
The other reference line, y = 0 does not change.
Step 2: M  ove the curve f11(x) = _____ ​​  x –1 2 ​​ upward by 2 units to give
​​  x –1 2 ​​ + 2. The reference line, y = 0 is moved to y = 2.
 f1(x) = _____
3
y-intercept of  f1(x): When x = 0, f1(x) = __ ​​  2 ​​ .
f1(x)
3
__
x-intercept of  f1(x): When  f1(x) = 0, x = ​​  2 ​​ .

2
3
––
2
x
3 2
––
2

The domain is {x: x  , x ≠ 2} and the range is {x: x  , x ≠ 2}.


321

Pre-Calculus and Calculus.indb 321 13/7/2017 5:57:25 PM


Chapter 11 • Graphs

​​  2x ​​ – 1
(b)  f2(x) = __
To obtain the plot of  f2(x) = ​​ __ 2 ​​– 1 from  f (x) = __ ​​  1x ,​​ we do it in two
x
steps:
Step 1: The values of  f (x) = __ ​​  1x ​​are doubled to give  f21(x) = __ ​​  2x .​​
The reference lines, x = 0 and y = 0 do not change.
Step 2: M ove the curve  f21(x) = __ ​​  2x ​​downward by 1 unit to give
​​  2x ​​ – 1. The reference line, y = 0 is moved to y = –1.
 f2(x) = __
The other reference line, x = 0 does not change.
x-intercept of  f2(x): When  f2(x) = 0, x = __ ​​  21 ​​ .
f 2(x)

​​  1 ​​
__
2
x

–1

The domain is {x: x  , x ≠ 0} and the range is {x: x  , x ≠ –1}.


x
(c)  f3(x) = _____
​​  x – 1 ​​ 
x x–1+1
Note that f3(x) = _____
​​  x – 1 ​​ can be written as f3(x) = ________
​​  x – 1 ​​ 

1
= ​​ _____
x – 1 ​​ + 1
Hence, to obtain the plot of  f3(x) = ​​ _____ 1 1
__
x – 1 ​​ from   f (x) = ​​  x ,​​ we do it in
two steps:
 ove the curve f (x) = __
Step 1: M ​​  1x ​​to the right by 1 unit to give
​​  x –1 1 ​​ 
 f31(x) = _____ . The reference line, x = 0 is moved to x = 1.
The other reference line, y = 0 does not change.
 ove the curve  f31(x) = _____
Step 2: M ​​  x –1 1 ​​ upward by 1 unit to give
​​ x –1 1 ​​ + 1. The reference line, y = 0 is moved to y = 1.
 f3(x) = _____
x-intercept of  f3(x): When x = 0, f3(x) = 0.
f 3(x)

1
x
0 1

The domain is {x: x  , x ≠ 1} and the range is {x: x  , x ≠ 1}.

322

Pre-Calculus and Calculus.indb 322 13/7/2017 5:57:25 PM


Chapter 11 • Graphs

Exercise 11.5
1. Sketch the graphs of the following functions.

(a)   ​​  2x1– 1  


f (x) = ______

 ​​ (b)   ​​  x –1 1 ​​ – 1


f (x) = _____

x+1
f (x) = _____
(c)   ​​  x – 1 ​​ 

(d)   ​​  x –2 1 ​​ 


f (x) = _____

(e)   1 ​​ + 1
f (x) = –​​ __
x

323

Pre-Calculus and Calculus.indb 323 13/7/2017 5:57:25 PM


Chapter 11 • Graphs

11.6  Radical Functions


_____
We now look at sketching functions of the form a​​√ x__ + b  ​​ + c, where a,
b and c are constants. The basic function is f (x) = ​√ x ​​.  The function has
a reference point (0, 0) and no reference lines. Sketches of the other
functions of this form can be obtained using transformation as described
in the table below.
Function Effect Graph
Basic function, The reference point is (0, 0). f (x)
__
 f (x) = ​√ x ​​ 

x
(0, 0)

0 ​ )​​, 
_____
f1(x) = ​√ x + 2 ​​  Translation (​​      ​  xy ​)​​  = (​​       
​  –2
f1(x)
that is, moving the curve of
__
 f (x) = √
​  x ​​ to the left by
2 units.
x
The reference point (0, 0) is (–2, 0)
moved to (–2, 0).
​  20 ​ )​​,  that is,
_____
f2(x) = ​√ x – 2 ​​  Translation (​​      ​  xy ) ​ ​​  = (​​      f2(x)
__
moving the curve of  f (x) = ​​√ x ​​  
to the right by 2 units.
x
The reference point (0, 0) is (2, 0)
moved to (2, 0).
​  xy ​)​​  = (​​      ​  02 ​ )​​,  that
__
f3(x) = ​√ x ​​ + 2 Translation (​​      f3(x)

is, moving the curve of


__
 f (x) = ​√ x ​​  upward by 2 units. (0, 2)
The reference point (0, 0) is x
moved to (0, 2).
​  xy ​)​​  = (​​      ​  02 ​ )​​,  that
__
f4(x) = ​​√ x ​​ – 2 Translation (​​      f4(x)
is, moving the curve of
__
 f (x) = √ 
​​ x  ​​ downward by
x
2 units.
The reference point (0, 0) is (0, –2)
moved to (0, –2).
__
f5(x) = –​​√ x ​​  Reflect the basic curve about f5(x)
the x-axis. x
The reference point (0, 0) (0, 0)
does not change.
___
f6(x) = √ 
​​ –x ​​  Reflect the basic curve about f6(x)
the y-axis.
The reference point (0, 0)
does not change. x
(0, 0)

In all cases, it would be helpful in the sketching of the graphs if we find the
324 x-intercept or y-intercept as well.

Pre-Calculus and Calculus.indb 324 13/7/2017 5:57:25 PM


Chapter 11 • Graphs

Example 4
Sketch the following graphs. State the domain and range of the
functions. _____
(a)  f1(x) = √  x + 1 ​​ + 1
​​ _____
​​ 2 – x  
(b)  f2(x) = √  ​​
__
(c)  f3(x) = 2​​√ x  ​​ – 2

SOLUTION_____
​​  x + 1 ​​ + 1
(a)  f1(x) = √ _____ __
To obtain the plot of f1(x) = √  ​​ x + 1 ​​ + 1 from  f (x) = √ 
​​ x  ,​​ we do it in
two steps:
__
Step 1: Move the_____ curve f (x) = √ ​  x ​​ to the left by 1 unit to give
f11(x) = √ ​  x + 1  
.​​ The reference point (0, 0) is moved to (–1, 0).
_____
Step 2: Move the curve f11(x) = √
_____ ​  x + 1  
​​ upward by 1 unit to give
f1(x) = √​  x + 1  
​​ + 1. The reference point (–1, 0) is moved
to (–1, 1).
y-intercept of  f1(x): When x = 0,  f1(x) = 2.
f1(x)

(–1, 1)
x

The domain is {x: x  –1} and the range is {x: x  1}.


_____
(b) f2(x) = √ 
​​ 2 – x  ​​ _____ _______
Write the function  f2(x) = √  x  
​​ as  f2(x) = √
​​ 2 –_______ ​​  –(x – 2) ​​ 
.
__
To obtain the plot of  f2(x) = √ ​​  –(x – 2) ​​  from  f (x) = √  ​​ x  ,​​ we do it in
two steps:
__
Step 1: M  ove the_____ curve f (x) = √  ​​ x  ​​to the right by 2 units to give
 f21(x) = √ ​​ x – 2 ​​ 
. The reference point (0, 0) is moved to
(2, 0). _____
Step 2: R  eflect the curve  f21(x) = √ 
_______ ​​ x – 2 ​​ about the _____ y-axis to give
 f2(x) = √​​  –(x – 2) ​​ 
, or equivalently, f2(x) = √  ​​ 2 – x   .​​ The
reference point (2, 0) does not change. __
y-intercept of  f2(x): When x = 0,  f2(x) = √  ​​ 2 ​​ .
x-intercept of  f2(x): When  f2(x) = 0, x = 2.
f2(x)

√2

x
(2, 0)

The domain is {x: x  2} and the range is {x: x  0}.
325

Pre-Calculus and Calculus.indb 325 13/7/2017 5:57:25 PM


Chapter 11 • Graphs

__
(c)  f3(x) = 2​​√ x  ​​ – 2
__ __
To obtain the plot of  f3(x) = 2​​√ x  ​​– 2 from  f (x) = ​​√ x  ,​​ we do it in two
steps:
__ __
Step 1: The values of  f (x) = ​√ x ​​ are doubled to give  f31(x) = 2​​√ x  .​​
The reference point (0, 0) does not change.
__
Step 2: M ove the curve  f31(x) = 2​​√ x  ​​downward by 2 units to give
__
​  x ​ – 2​. The reference point (0, 0) is moved to
 f3(x) = 2​√
(0, –2).
y-intercept of  f3(x): When x = 0,  f3(x) = –2.
x-intercept of  f3(x): When  f3(x) = 0, x = 1.
f3(x)

x
1
(0, –2)

The domain is {x: x  0} and the range is {x: x  –2}.

Exercise 11.6
1. Sketch the graphs of the following functions.
______
f (x) = √ 
(a)   ​​ 2x – 1 ​​ 

_____
​ f (x) = 3 + √ 
(b)   ​​ 1 – x ​​ 
_____
f (x) = –​​√ 1 – x ​​ 
(c)  
_____
f (x) = √ 
(e)   ​​ x – 3 ​​ 
__
f (x) = 1 – √ 
(f)   ​​ x ​​ 

326

Pre-Calculus and Calculus.indb 326 13/7/2017 5:57:25 PM


Chapter 11 • Graphs

11.7 
Exponential and Logarithmic
Functions
11.7.1 Definition of Exponential and Logarithmic Functions
11.7.2 Relationship between Exponential and Logarithmic Functions
11.7.3 Exponential Function with Base e
11.7.4 Graphs of Exponential and Logarithmic Functions

11.7.1 D
 efinition of Exponential and Logarithmic
Functions
Consider a function of the form  f (x) = ax, where x   and a  +\{1}.
Function of this form is called an exponential function, where a is the
base and x is the exponent. Different graphs of  f (x) = ax will be observed
depending on the values of a as given in Figure 11a.
y y
y = ax y = ax

Range, Rf Range, Rf
a a
1 1
x x
1 –1
Domain, Df Domain, Df

(i) a > 1 (ii) 0 < a < 1


Figure 11a
It can be seen that:
(i) The domain of  f (x) = ax is (–∞, ∞) and the range is (0, ∞). That is,
y cannot be negative or zero.
(ii) The graph of  f (x) = ax is continuous.
(iii) The x-axis is the horizontal asymptote.
(iv) For a > 1, the graph cuts the y-axis at y = 1 and passes through the
point (1, a).
For 0 < a < 1, the graph cuts the y-axis at and passes through the
point (–1, a).
(v) The function  f (x) = ax is one-to-one.
(vi) If a > 1, then  f (x) = ax is an increasing function.
If 0 < a < 1, then  f (x) = ax is a decreasing function.

Since  f (x) = ax is one-to-one, then its inverse exists. The inverse function
of the exponential functions is the logarithmic function given by

f –1(x) = loga x

The graph of y = loga x is the reflection of the graph y = ax about the line
y = x as illustrated in Figure 11b. The domain of f –1(x) = loga x is (0, ∞)
and the range is (–∞, ∞).

327

Pre-Calculus and Calculus.indb 327 13/7/2017 5:57:25 PM


Chapter 11 • Graphs

y
y = loga x

x Range, Rf a x Range, Rf
1 a 1
–1
y = loga x

Domain, Df Domain, Df

(i) a > 1 (ii) 0 < a < 1


Figure 11b
It can be seen that:
(i) The domain of y = loga x is (0, ∞) and the range is (–∞, ∞). That is,
x cannot be negative or zero.
(ii) The graph of y = loga x is continuous.
(iii) The y-axis is the vertical asymptote.
(iv) For a > 1, the graph cuts the x-axis at x = 1 and passes through the
point (a, 1).
For 0 < a < 1, the graph cuts the x-axis at x = 1 and passes through the
point (a, –1).
(v) The function y = loga x is one-to-one.
(vi) If a > 1, then y = loga x is an increasing function.
If 0 < a < 1, then y = loga x is a decreasing function.

11.7.2 R
 elationship between Exponential and
Logarithmic Functions
The first important relationship between an exponential function and a
logarithmic function has been described above. That is, the inverse of
 f (x) = ax is  f –1(x) = loga x, or the inverse of  f (x) = loga x is  f –1(x) = ax.

The second relationship is that any exponential function has its equivalent
logarithmic form, for example,
y = ax is equivalent to x = loga y
This result uses several algebraic results involving logarithmic expression:
y = ax
loga y = x loga a Take logarithm of base a on both sides

loga y = x loga a Power law of logarithms

loga y = x Since loga a = 1

Similarly, any logarithmic function has its equivalent exponential form,


for example,
y = loga x is equivalent to x = ay
This result uses several algebraic results involving exponential expression:
ay = a​​ 
​​ log x​​
a Take exponent of base a on both sides

328 ay = x Since ​​a​​  log ​​ = x


a
x

Pre-Calculus and Calculus.indb 328 13/7/2017 5:57:25 PM


Chapter 11 • Graphs

Example 5
It is given that  f (x) = log2 x and g(x) = 22x.
(a) Is  f (x) the inverse of g(x)? Note
(b) Find f  g, if it exists. Determine its domain and range. f  (x) is the inverse function of
g(x) if f (g(x)) = x.
SOLUTION
(a) Need to show fg(x) = x.
  fg(x) =  f (22x) = log222x = 2x log2 2 = 2x ≠ x
Hence, f (x) is not the inverse of g(x).
(b) ( f  g)(x) = f (g(x)) = 2x as found in (a).
Rg = Df = (0, ∞)

g f

Rg = Df
Dg = Df  g Rf = Rf  g
Hence, Df  g =  and Rf  g = .

11.7.3 Exponential Function with Base e


1  ​ ​​​    ​​.
m
Consider the expression, (​​​  1 + __
​ m )
1  ​ ​​​    ​​
m

Figure 11c gives the values taken by the expression above for increasing m (
​​​  1 + __
​ m )

values of m. As m approaches infinity, (​​​  1 + __1  ​ ​​​  m  ​​ approaches 2.7183. This


​ m ) . .
special number is called e. Exponential function with base e has been . .
m approaches infinity

. .

approaches 2.7183
used extensively in calculus as it is able to reduce a complicated problem
100,000 2.7183
into a simpler form. 10,000 2.7181
The graph of an exponential function with base e,  f (x) = ex, is given in 1,000 2.7169
100 2.7048
Figure 11d(i). Almost all properties of  f (x) = ax listed before are also
10 2.594
possessed by  f (x) = ex except that it will pass through the point (1, e).
Figure 11c
y y

ex
ln x
1
e
1 x
1 e
x
1
(i) (ii)
Figure 11d
The function  f (x) = ex also has an inverse which is the logarithmic function
given by f –1(x) = loge x, or simply f –1(x) = ln x. Its graph is given by
Figure 11d(ii). Function in the form ln x is also known as natural
logarithmic function.
329

Pre-Calculus and Calculus.indb 329 13/7/2017 5:57:25 PM


Chapter 11 • Graphs

Example 6
Given that  f (x) = ln (3x + 1), find  f –1  f, if it exists.

SOLUTION
Let y = ln (3x + 1)
ey = 3x + 1
e –1
y
​​ _____  = x
3 ​​ 
ex – 1
Hence, f –1(x) = _____
​​  3 ​​.  

(f –1  f )(x) = f –1( f (x))
= f –1(ln (3x + 1))
e​ ​​  ln (3x + 1)​– 1
= ​​ __________ 3 ​​   
3x + 1 – 1
= _________
​​  3 ​​   

=x
ex – 1
This confirms that _____
​​  3 ​​ 
 is the inverse of ln (3x + 1).

11.7.4 G
 raphs of Exponential and Logarithmic
Functions
Sketches of functions of the forms  f (x) = a g(x) and  f (x) = loga g(x) can be
obtained from the basic curves of  f (x) = a x and  f (x) = loga x respectively.
The table below summarizes the steps to obtain the sketches of  f (x) = e g(x)
from  f (x) = e x.

Function Effect Graph


Basic function, The reference point is f (x)
f (x) = ex (0, 1) and the reference
line is y = 0.
(0, 1)
x
0

Translation (​​      0 ​ )​​, 


​  yx ​)​​  = (​​       
​  –2
f1(x) = e x + 2 f1(x)
that is, moving the curve
of  f (x) = ex to the left by
2 units. (–2, 1)
The reference point (0, 1)
is moved to (–2, 1). x
0
The reference line, y = 0
does not change.

330

Pre-Calculus and Calculus.indb 330 13/7/2017 5:57:25 PM


Chapter 11 • Graphs

f2(x) = ex + 2 Translation (​​      ​  20 ​ )​​, 


​  xy ​)​​  = (​​     
that is, moving the curve f2(x)
of  f (x) = ex upward by
2 units. (0, 3)
The reference point (0, 1) y=2
moved to (0, 3). 2
x
The reference line, y = 0
is moved to y = 2.
1  ​  ​​, 
​  xy ​)​​  = (​​       
f3(x) = ex – 1 – 2 Translation (​​      ​  –2 )
that is, moving the curve
f3(x)
of  f (x) = ex to the right by
1 unit and downward by
2 units. x
The reference point (0, 1) (1, –1)
is moved to (1, 1), then to 2 y = –2
(1, –1).
The reference line, y = 0
is moved to y = –2.
f4(x) = e2x The graph of  f (x) = ex is f4(x)
e2x
tilted upward.
The reference point (0, 1)
does not change.
1
x

f5(x) = 2ex Values of  f5(x) = 2ex are f5(x)


twice the values of
 f (x) = ex.
The reference point (0, 1) (0, 2)
is moved to (0, 2).
(0, 1)
x

Likewise, the sketches of  f (x) = ln (g(x)) can also be obtained from


 f (x) = ln x.

331

Pre-Calculus and Calculus.indb 331 13/7/2017 5:57:25 PM


Chapter 11 • Graphs

Example 7
Sketch the following graphs. State the domain and range of the functions.
(a)  f1(x) = ex – 3
(b)  f2(x) = ex + 3 – 2
(c)  f3(x) = 2e2x
(d)  f4(x) = e3x + 1

SOLUTION
(a)  f1(x) = ex – 3
To obtain the plot of  f1(x) = ex – 3 from  f (x) = ex, we do the translation

( ​  yx )
​​      ​  03 ​ )​​  on   f (x) = ex, that is, moving the curve of  f (x) = ex to the
​ ​​  = (​​     
right by 3 units. The reference point (0, 1) is moved to (3, 1). The
reference line, y = 0 does not change.
f1(x)

(3, 1)
x

The domain is (–∞, ∞) and the range is (0, ∞).


(b)  f2(x) = ex + 3 – 2
To obtain the plot of  f2(x) = ex + 3 – 2 from  f (x) = ex, we do the
–3 ​  ​​  on  f (x) = ex, that is, moving the curve
​ ​​  = (​​       
​  yx )
translation (​​      ​  –2 )
of  f (x) = e to the left by 3 units and downward by 2 units.
x

The reference point (0, 1) is moved to (–3, 1), then to (–3, –2).
The reference line, y = 0 is moved to y = –2.
f2(x)

x
(–3, –2)
y = –2
–2

The domain is (–∞, ∞) and the range is (–2, ∞).


(c)  f3(x) = 2e2x
To obtain the plot of  f3(x) = 2e­2x from  f (x) = ex, the curve of
 f (x) = ex is tilted upward to give  f31(x) = e2x. The reference point
(0, 1) does not change. Then, the values of  f31(x) = e2x are doubled
to give  f3(x) = 2e2x. The reference line, y = 0 does not change. The
reference point (0, 1) is moved to (0, 2).

332

Pre-Calculus and Calculus.indb 332 13/7/2017 5:57:26 PM


Chapter 11 • Graphs

f (x)

(0, 2)

(0, 1)
x

The domain is (–∞, ∞) and the range is (0, ∞).


(d)  f4(x) = e3x + 1
To obtain the plot of  f4(x) = e3x + 1 from  f (x) = ex, the curve
of  f (x) = ex is tilted upward to give  f41(x) = e3x. The reference point
(0, 1) does not change. Then, we do the translation (​​      ​  yx ) ​  10 ​ )​​  
​ ​​   = (​​     
on  f41(x) = e3x, that is, moving the curve of  f41(x) = e3x upward by
1 unit. The reference point (0, 1) is moved to (0, 2). The reference
line, y = 0 is moved to y = 1.
f (x)

(0, 2)
y =1
(0, 1)
x

The domain is (–∞, ∞) and the range is (1, ∞).

Example 8
Sketch the following graphs. State the domain and range of the functions.
(a)  f1(x) = ln (x + 1)
(b)  f2(x) = ln (x – 1) + 2
(c)  f3(x) = 2 ln (2x)
(d)  f4(x) = ln (x + 1)2
SOLUTION
(a)  f1(x) = ln (x + 1)
To obtain the plot of  f1(x) = ln (x + 1) from  f (x) = ln x, we do the
​  yx )
translation (​​      0 ​ )​​  on   f (x) = ln x, that is, moving the curve of
​ ​​  = (​​       
​  –1
 f (x) = ln x to the left by 1 unit. The reference point (1, 0) is moved
to (0, 0). The reference line, x = 0 is moved to x = –1.
f1(x)
x = –1
ln (x + 1)

x
–1 (0, 0)

The domain is (–1, ∞) and the range is (–∞, ∞).


333

Pre-Calculus and Calculus.indb 333 13/7/2017 5:57:26 PM


Chapter 11 • Graphs

(b)  f2(x) = ln (x – 1) + 2
To obtain the plot of  f2(x) = ln (x – 1) + 2 from  f (x) = ln x, we do
​  yx )
the translation (​​      ​  21 ​ )​​  on f (x) = ln x, that is, moving the curve of
​ ​​  = (​​     
 f (x) = ln x to the right by 1 unit and upward by 2 units. The reference
point (1, 0) is moved to (2, 0), then to (2, 2). The reference line,
x = 0 is moved to x = 1.
f2(x)
x =1

x
1

The domain is (1, ∞) and the range is (–∞, ∞).


(c)  f3(x) = 2 ln (2x)
First, rewrite  f3(x) = 2(ln 2 + ln x). Then, the plot of  f3(x) = 2(ln 2 + ln x)
can be obtained from  f (x) = ln x in two steps:
Step 1: M  ove the curve  f (x) = ln x upward by ln 2 units to give
 f31(x) = ln x + ln 2. The reference point (1, 0) is moved to
(1, ln 2). The reference line, x = 0 does not change.
Step 2: The values of  f31(x) = ln x + ln 2 are now doubled to give
 f3(x) = 2(ln x + ln 2). The slope of  f3(x) = 2(ln x + ln 2)
is steeper than  f31(x) = ln x + ln 2. The reference point
(1, ln 2) is moved to (1, 2 ln 2). The reference line, x = 0
does not change.
x-intercept: When y = 0, 2 ln (2x) = 0
ln (2x) = 0
2x = e0
1    since e0 = 1
x = ​​ __
2 ​​ 
f3(x)
2 ln 2x
ln 2x
(1, 2 ln 2)
ln x
(1, ln 2)

x
​​  1 ​​ 
__
2
(1, 0)

The domain is (0, ∞) and the range is (–∞, ∞).

334

Pre-Calculus and Calculus.indb 334 13/7/2017 5:57:26 PM


Chapter 11 • Graphs

(d)  f4(x) = ln (x + 1)2


We may start by considering  f41(x) = ln x2. Note that  f41(x) = ln x2
is still defined when x takes negative number. Hence, the plot of
y = ln x2 will have curves on both sides of y-axis as shown
below and the values of  f41(x) are twice the values of  f (x) = ln x
(since ln x2 = 2 ln x). There is a vertical asymptote, that is, the y-axis.
f41(x) vertical
asymptote

x
–1 1

To obtain the plot of f4(x) = ln (x + 1)2 from f41(x) = ln x2, we do


​  yx )
the translation (​​      0 ​ )​​   on f41(x) = ln x , that is, moving the
​ ​​   = (​​       
​  –1 2

curve of f41 = ln x2 to the left by 1 unit. Hence, the reference points


(–1, 0) and (1, 0) is moved to (–2, 0) and (0, 0). The reference line,
x = 0 is moved to x = –1.
f4(x)
x = –1

x
–2 –1

The domain is (–∞, –1)  (–1, ∞) and the range is (–∞, ∞).

335

Pre-Calculus and Calculus.indb 335 13/7/2017 5:57:26 PM


Chapter 11 • Graphs

Exercise 11.7
1. Find the logarithmic form of the following exponential functions.
(a) y = 3x
(b) y = 32x
(c) y = 2e5x + 1

2. Find the exponential form of the following logarithmic functions.


(a) y = loga 2x
(b) y = 2 loga x

3. Sketch the following functions. State the domain and range of the functions.
(a) y = ex – 4
(b) y = ex + 1 – 2
e2x
(c) y = ___
​​  2 ​​ 

(d) y = 2ex – 2 + 3
(e) y = ln (x + 2)
(f) y = 3 + ln (x – 2)
(g) y = – ln (x + 2)
(h) y = 2 – ln (x + 2)

4. Find the composition function f  g and g  f and determine their respective domain and range for
 f (x) = e2x and g(x) = ln x.

5. Identify possible functions of f (x) and g(x) given that


(a) ( f  g)(x) = e2x – 1,
(b) (g  f )(x) = ln (2x + 2).

336

Pre-Calculus and Calculus.indb 336 13/7/2017 5:57:26 PM


Chapter 11 • Graphs

11.8  Trigonometric Functions


11.8.1 Graphs of sin x, cos x, tan x, cot x, sec x and cosec x
11.8.2 Graphs of a sin kx, a cos kx and a tan kx

11.8.1 Graphs of sin x, cos x, tan x, cot x, sec x


and cosec x
By selecting the appropriate points, the plots of each trigonometric
function can be obtained. Figures 11e and 11f give the plots of sine and
cosine functions.
y y

1 1

x x
π 0 π π 0 π
3π –π
–2π –​​ __ –​​ __
2 ​​  ​​ __
2 ​​ 
π 3π
__ 2π 3π –π
–2π –​​ __ –​​ __
2 ​​  ​​ __
2 ​​ 
π 3π
​​ __ 2π
2 ​​  –1
​​  2 ​​  2 ​​  –1 2 ​​ 

Figure 11e Graph of y = sin x Figure 11f Graph of y = cos x

Note that sine and cosine functions are periodic with period 2π. This
should be true as a complete rotation for a cycle takes an angle of 2π.
Mathematically, a function is a periodic function with period 2π if
 f (t + 2π) = f (t) for all t in the domain. Thus, in interval [–2π, 2π], we
observe two complete cycles of the graph. Further, the amplitude of the
function is 1. That is, the range of these functions is [–1, 1].

Observe the plots of sine and cosine functions. If the plot of sine function
π
is shifted 90° or ​​ __
2 ​​  to the right, then it will become a cosine function.
π
Similarly, if the plot of cosine function is shifted 90° or __
​​  2 ​​ to the right, then
it will become a sine function. That means,
sin (90° – x) = cos x,
cos (90° – x) = sin x.
Note
On the other hand, it can also be observed that f (x) is odd if
sin (–x) = –sin x, f (–x) = –f (x) for all x in Df .

cos (–x) = cos x.


Note
That is, sine function is an odd function while cosine function is an even
f (x) is even if
function. f (x) = f (–x) for all x in Df .

337

Pre-Calculus and Calculus.indb 337 13/7/2017 5:57:26 PM


Chapter 11 • Graphs

Figure 11g gives the plot of tangent function. It can be seen that tangent
function is periodic with period π. Mathematically, a function is a periodic
function with period π if f (t + π) = f (t) for all t in the domain. Observe
π ___ 3π
the plot of tangent function, there are vertical asymptotes at x = ± ​​ __
2 ​​ , ± ​​  2 ​​ ,

± ​​ ___
2 ​​ , ... and tan (–x) = – tan x.
y

x
π 0 π

–​​ __ –π –​​ __
2 ​​ 
__
​​  2 ​​  π 3π
__
2 ​​   ​​  2 ​​  

Figure 11g Graph of y = tan x

The plots of cotangent, secant and cosecant functions can be obtained by


taking the reciprocal of tangent, cosine and sine functions respectively.
The plots are given in Figures 11h, 11i and 11j. It can be seen that secant
and cosecant functions are periodic with period 2π while cotangent
function is periodic with period π.
y

x
π 0 π

–​​ __ –π –​​ __
2 ​​  ​​ __
2 ​​ 
π 3π
__
2 ​​   ​​  2 ​​  

Figure 11h Graph of y = cot x

1
π π x

–​​ __ –π –​​ __
2 ​​ 
0 ​​ __
2 ​​ 
π 3π
__
2 ​​   ​​  2 ​​  
–1

Figure 11i Graph of y = sec x

338

Pre-Calculus and Calculus.indb 338 13/7/2017 5:57:26 PM


Chapter 11 • Graphs

1
x
π 0 π
3π –π
–​​ __ –​​ __
2 ​​  ​​ __
2 ​​ 
π 3π
__
2 ​​   ​​  2 ​​  
–1

Figure 11j Graph of y = cosec x

11.8.2 Graphs of a sin kx, a cos kx and a tan kx


Graphs of y = a sin kx, y = a cos kx and y = a tan kx, where k is a positive
real number, take similar shape as the original graphs of y = sin x, y = cos
x and y = tan x respectively. We call the original graphs as the reference
graph. Further, we need to identify
(i) the period of the new function:
period = ___ ​ ​2π ​​ (for sine and cosine functions)
k
__ π
period = ​​   ​​(for tangent function)
k
(ii) amplitude of the new function: a (for sine and cosine functions only)
Period will determine whether the graph is compressed (k > 1) or expanded
(0 < k <1) along the x-axis about the y-axis if compared to the reference
graphs of y = sin x, y = cos x and y = tan x. Amplitude will compress or
expand the reference graphs of y = sin x, y = cos x and y = tan x along the
y-axis about the x-axis.

The effects of having different periods and amplitudes for sine function
are illustrated below.
Figure 11k gives the plot of y = sin 2x, where k = 2 and a = 1. The period

is ​​ ___
2 ​​  = π, which means one complete cycle is observed in [0, π]. The
graph starts at (0, 0) and the curve follows the pattern of y = sin x. Thus,
we have a more compressed plot compared to y = sin x.
y

x
–2π –π 0 π 2π 3π

–1

Figure 11k Graph of y = sin 2x

339

Pre-Calculus and Calculus.indb 339 13/7/2017 5:57:26 PM


Chapter 11 • Graphs

x 2π
Figure 11l gives the plot of y = sin __
​​ 2  ​​. Now, the period is​​ ___  ​​ = 4π, which
​  12 ​ 
__

means a complete cycle is observed in [0, 4π]. The graph of y = sin x is


x
expanded to give the graph of y = sin __ ​​ 2  ​​.
y

x
–3π –2π –π 0 π 2π 3π 4π

–1
x
Figure 11l Graph of y = sin __
​​ 2  ​​
x
Figure 11m gives the plot of y = 2 sin __
​​  2  ​​. The period is 4π, which means
one complete cycle is observed in [0, 4π]. The amplitude is 2, which means
the minimum occurs at –2 while the maximum occurs at 2. That means the
x
curve of y = sin __
​​  2  ​​ is expanded along the y-axis about the x-axis giving the
x
graph of y = 2 sin __ ​​  2  ​​.
y
2

x
–3π –2π –π 0 π 2π 3π 4π

–1

–2

x
Figure 11m Graph of y = 2 sin __
​​ 2  ​​

Similarly, the effects are also observed for cosine function.


​​ 12 ​​ x and
Figure 11n gives the overlapping graphs of y = cos x, y = cos __
​​ 12 ​​ x.
y = 2 cos __
y cos x
1 ​​ x
cos ​​ __
2 2
2 cos ​​ __ 1 ​​ x
2
1

x
–4π –3π –2π –π 0 π 2π 3π 4π

–1

–2

Figure 11n Graphs of cosine functions


340

Pre-Calculus and Calculus.indb 340 13/7/2017 5:57:26 PM


Chapter 11 • Graphs

x
Figure 11o gives graphs of y = tan x and y = tan ​​ __ 2  ​​on the same plot. For
π __ π
y = tan x, the period is π and a complete cycle is observed in (​​  –​ __ 2 ​ , ​  2 ​ ). ​​
x
For y = tan __
​​ 2  ​​, the period is 2π and a complete cycle is observed in (–π, π).
y

tan x
​​  x  ​​
tan __
2

π π x
–π –​​ __
2 ​​ 
0 __
​​  2 ​​  π 3π
__
​​  2 ​​ 

x
Figure 11o Graphs of y = tan x and y = tan __
​​ 2  ​​
For y = 2 tan x, a complete cycle remains the same as for y = tan x. However,
the graph is pulled up or down about the y-axis from the graph of y = tan x
as illustrated in Figure 11p.
y

tan x
2 tan x

x
π 0 π
–π –​​ __
2 ​​  ​​ __
2 ​​ 
π 3π
__
​​  2 ​​ 

Figure 11p Graphs of y = tan x and y = 2 tan x


In order to determine the exact value of a in y = a tan x, we need to know at
least one point on the graph. For example, it is given that one of the curves
​  4 ​ , 2)​​.  Hence, 2 = a tan __
π π
passes through (​​  __ ​​  4 ​​  giving a = 2.

341

Pre-Calculus and Calculus.indb 341 13/7/2017 5:57:26 PM


Chapter 11 • Graphs

Example 9
Identify the trigonometric functions for each of the following graphs.
(a) y
1

x
–2π –π 0 π 2π


–1
(b) y
2

x

___ 2π 0 2π 4π
–​​  3 ​​  –​​ ___ ​​ 
3
___
​​   ​​ 
3
___
​​   ​​ 
3
–2

(c) y

( ​  6 ​,  3)​​  

​​  ___
x
–π π 0 π π 5π
–​​ ___
3 ​​  ​​ ___
3 ​​  ​​ ___
3 ​​ 

(d) y
​​  1 ​​ 
__
2

x
–4π –2π 0 2π 4π

1 ​​ 
–​​ __
2
(e) y

x
–6π –3π 0 3π 6π

–1

(f) y

​​  1__   ​​ 


___
​√ 3 ​ 
x
–3π 0 π 3π 6π 9π

342

Pre-Calculus and Calculus.indb 342 13/7/2017 5:57:26 PM


Chapter 11 • Graphs

SOLUTION
(a) Amplitude = 1 ​​ 12  ​​
(d) Amplitude = __
Period = π Period = 4π
2π 2π
​​ ___ ​​ = π ​​ ___ ​​ = 4π
k k
k = 2 k = __​​  21 ​​ 
x
∴ y = sin 2x ∴ y = __ ​​  21 ​​  sin __
​​  2  ​​

(b) Amplitude = 2 (e) Amplitude = 1



Period = ___  
​​  3 ​​ Period = 6π
2π 2π 2π
​​ ___ ​​ = ___  
​​  3 ​​ ​​  ___
 ​​ = 6π
k k
k = 3 k = __​​  31 ​​ 
x
∴ y = 2 cos 3x ∴ y = cos __ ​​ 3  ​​


(c) Period = ___
​​  3 ​​   (f) Period = 6π

π 2π π
​​ __ ​​ = ___  
​​   ​​ ​​ __
 ​​= 6π
k 3 k
3 1
k = ​​ __
2 ​​   k = ​​ __
6 ​​ 
5π 1__   ​​ ,
When x = ​​ ___
6 ​​ , y = 3, When x = π, y = ​​ ___
​  3 ​ 

3x x
y = a tan ___
​​  2 ​​   y = a tan ​​ __
6  ​​

​​  2(  
 ​​​​   ​  6 ​ )​​ ​​ 
3 ___ 5π 1__   ​​ = a tan __ π
3 = a tan __ ___
​​  6 ​​ 
​  3 ​ 


3 = a tan ___
​​  4 ​​   a=1
x
a = 3 ∴ y = tan __
​​ 6  ​​
3x
∴ y = 3 tan ___
​​ 2 ​​  

343

Pre-Calculus and Calculus.indb 343 13/7/2017 5:57:27 PM


Chapter 11 • Graphs

Exercise 11.8
1. Identify the trigonometric functions for each of the following graphs.
(a) y (d)
y
3
1

x x
–2π –π 0 π 2π –π 0 π

–1
–3
(b) y (e) y
3 1

0 x x
π 0 π
–π –​​ __
2 ​​  ​​  π ​​ 
__ π –π
2

–3 –1

(c) y

2
π __ π x

__ π
__ 0 __
​​  8 ​​  ​​  4 ​​  3π
–​​  4 ​​  –​​  4 ​​  ​​ __
4 ​​ 

2. Plot the graphs of the following trigonometric functions.


x
y = 2 sin __
(a) y = sin 3x (d) ​​ 3  ​​

(b) y = 2 cos (e)


y = cos 3x
x
(c) y = 3 tan __
​​ 4  ​​ (f) y = sec 3x

344

Pre-Calculus and Calculus.indb 344 13/7/2017 5:57:27 PM


Chapter 11 • Graphs

11.9  Inverse Trigonometric Functions


Carrying the same notation of inverse functions, the corresponding
inverse trigonometric functions can be written as sin–1 x, cos­–1 x and
tan–1 x, however they may sometimes be confused with the reciprocals. To
avoid this confusion, these inverse functions can also be written as arcsin x,
arccos x and arctan x.
Without constraining the domains of trigonometric functions, they are
periodic and hence, many-to-one functions, which means there exist
infinite number of x with the same function value.
Figure 11q demonstrates how the graph of the function y = sin x intersects
with the line y = 0.5. Observe that each point of intersection shares the
same function value of 0.5.
y

y = sin x
1
y = 0.5

x
–π 0 π 2π 3π 4π

–1

Figure 11q Graph of y = sin x for x  [–π, 4π]


Hence, in order for trigonometric functions to have inverses, the domains
must be restricted. We can obtain one-to-one sine function by constraining
​   ​, ​   ​]​​,  as seen in Figure 11r.
the domain of the function y = sin x to [​ ​ –__π __ π
2 2
y
y = sin x
1
π ​​ 
–​​  __
2
x
0 π ​​ 
​​  __
2

–1

2 ​ , ​  2 ​ ]​​  
π __ π
Figure 11r Graph of y = sin x for x  [​​  –​ __
Hence, the graph of corresponding inverse function can be shown by
reflecting the graph of y = sin x over the line y = x, as illustrated in Figure
11s.
y
y = sin–1 x
y=x
1 y = sin x
π ​​ 
–​​  __
2 x
0 π ​​ 
​​  __
2
–1

Figure 11s Graph of y = sin–1 x 345

Pre-Calculus and Calculus.indb 345 13/7/2017 5:57:27 PM


Chapter 11 • Graphs

The corresponding ranges of inverse functions are usually within their


principal values. The principal values of the inverse functions are shown
Note below.
For the inverse function
Inverse Functions Domain Range (within principal values)
y = tan–1 x, the range given in
π π
π
the table is –​​ __
π
__ y = sin–1 x –1  x  1 –​​ __ __
2 ​​   y  ​​  2 ​​ 
2 ​​  < y < ​​  2 ​​ ,
π π
instead of –​​ __ __
2 ​​   y  ​​  2 ​​ . This y = cos–1 x –1  x  1 0yπ
π π
is because y = tan x is
y = tan –1 x x –​​ __ __
2 ​​  < y < ​​  2 ​​ 
π
undefined when x = __ ​​  2 ​​  or
π
x = –​​ __
2 ​​ . Graphs of trigonometric functions together with their inverses are
illustrated below, within the principal values respectively.
y y y
y = sin x y = tan x
π ​​ 
​​  __
2 y=x y=x
π
π ​​ 
y = ​​  __
y = sin–1 x y=x 2
1 y = cos–1 x
y = tan–1 x
π ​​ 
–​​  __
2 –1 x
π ​​  x 0
0 1 ​​  __
2
π
x
–1 0 y = cos x
π
–1 y = –​​ __
2 ​​ 
–1
π ​​  π ​​ 
x = ​​  __
–​​  __ π
__ 2
2 x = –​​  2 ​​ 

Figure 11t Figure 11u Figure 11v


Graph of y = sin–1 x Graph of y = cos–1 x Graph of y = tan–1 x

Example 10
Evaluate the following expressions, giving your answer in radians
within principal range.
(a) y = sin–1(–1)
(b) y = tan–1(–40)

SOLUTION
π π π
(a) y = sin–1(–1) = –​​ __
__ __
2 ​​ 
–​​   ​​   y  ​​   ​​ 
2 2
__ π __ π
(b) y = tan–1(–40) = –1.55 –​​   ​​  < y < ​​   ​​ 
2 2

346

Pre-Calculus and Calculus.indb 346 13/7/2017 5:57:27 PM


Chapter 11 • Graphs

Example 11
Find the value of y in radians within principal range for each of the
following.
​  5 ​ )​] ​​  
π
(a) y = sin–1 [​​  sin (​  __
(b) y = cos [sin–1 (1)]

SOLUTION
​  5 ​ )​​  = x.
π
(a) Let sin (​​  __
π
By the definition of inverse function, sin–1 x = __ ​​  5 ​​ .
​  5 ​ )​] ​​  = sin–1 x = __
π π
Hence, y = sin–1 [​​  sin (​  __ ​​  5 ​​ .
(b) Let sin–1(1) = x.
By the definition of inverse function, sin x = 1
π
x = __ ​​  2 ​​ 
​  2 ​ )​​  = 0.
π
Hence, y = cos [sin–1 (1)] = cos x = cos (​​  __

Exercise 11.9
In the following questions, the angles are in radians and all inverse trigonometric functions are in principal
ranges.

​  12 ​ )​​. 
1. Evaluate θ = cos–1 (​​  __

2. Evaluate the expression tan–1 (sin 0.3).

​ 13 ​ )​​  = __
3. Solve the equation cos–1 (​​  2x + __ ​​  13 ​​ .
__ __
​   ​ )​​  + (​​  cos–1 ___
4. Find the value of sin (​​  sin ___ ​   ​ )​​. 
​√ 3 ​  –1 ​√ 3 ​ 
2 2

( 3 ​ )​​  = –​​  4 ​​ .


1 ​  ​​  + tan–1 ​​  –​ __
5. Show that tan–1 (​​  –​ __
2)
1 ___3π

347

Pre-Calculus and Calculus.indb 347 13/7/2017 5:57:27 PM


Chapter 11 • Graphs

11.10  Hyperbolic Functions


Hyperbolic function is a class of functions which is defined in terms of
exponential function. It has interesting properties in the manner that is
analogous to trigonometric function.

Definition 11.1

For any real number x, the hyperbolic sine of x, denoted by sinh (x), is
1 ​​ (ex – e–x) and the hyperbolic cosine of x, denoted
defined by sinh (x) = ​​ __
2
by cosh(x), is defined by cosh (x) = __ ​​  12 ​​ (ex + e–x).

The other types of hyperbolic functions also exist and defined as follows.
sinh (x) ______ ex – e–x
tanh (x) = _______
​​    = ​​  ex + e–x 
 ​​   ​​
cosh (x)
cosh (x) ______ ex + e–x
​​  1   ​​ 
coth (x) = _______ = _______
​​  = ​​  ex – e–x  
 ​​  ​​
tanh (x) sinh (x)

​​  1   ​​ 
sech (x) = _______ ​​  2   ​​ 
= ______
cosh (x) ex + e–x

​​  1   ​​ 
cosech (x) = _______ ​​  2   ​​ 
= ______
sinh (x) ex – e–x

Example 12
Evaluate the following, giving the answers in three decimal places.
(a) sinh (1)
(b) cosh (2)
(c) tanh (2)

SOLUTION

​​ 21 ​​ (e1 – e–1) = 1.175


(a) sinh (1) = __

​​ 21 ​​ (e2 + e2) = 3.762


(b) cosh (2) = __

sinh (2) ______ e2 – e–2


(c) tanh (2) = _______
​​   
 ​​ 
= ​​  2  
 ​​ = 0.964
cosh (2) e + e–2

Example 13
Express the following expressions in terms of exponential functions.
(a) cosh (x) + sinh (x)
(b) cosh (x) – sinh (x)

SOLUTION
​​  21 ​​ (ex + e–x) + __
(a) cosh (x) + sinh (x) = __ ​​  21 ​​ (ex – e–x) = ex

348 ​​  21 ​​ (ex + e–x) – ​​ __


(b) cosh (x) – sinh (x) = __ 1 x –x
2 ​​ (e – e ) = e
–x

Pre-Calculus and Calculus.indb 348 13/7/2017 5:57:27 PM


Chapter 11 • Graphs

Example 14
Show the following.
(a) sinh (x – y) + cosh (x – y) = exe–y
(b) sinh (x – y) – cosh (x – y) = –eye–x
(c) cosh (x + y) + sinh (x + y) = exey
(d) cosh (x + y) – sinh (x + y) = e–xe–y

SOLUTION
1  ​​(ex – y – e–(x – y))
Note that sinh (x – y) = ​​ __
2
1 x–y
cosh (x – y) = ​​ __
2 ​​ (e + e )
–(x – y)

(a) sinh (x – y) + cosh (x – y) = ex – y = exe–y


(b) sinh (x – y) – cosh (x – y) = –ey – x = –eye–x

​​  21 ​​ (ex + y – e–(x + y))


Similarly, sinh (x + y) = __

​​  21 ​​ (ex + y + e–(x + y))


cosh (x + y) = __
(c) cosh (x + y) + sinh (x + y) = ex + y = exey
(d) cosh (x + y) – sinh (x + y) = e–(x + y) = e–xe–y

Based on the definition of sinh x and cosh x, we can show that


​​  12 ​​ (e–x – ex) = –​​ __
sinh (–x) = __ 1 ​​ (ex – e–x) = –sinh (–x)
2
cosh (–x) = __ ​​  12 ​​ (e–x – ex) = cosh (x)

Note that  f (x) is an even function if  f (–x) =  f (x) for all x in the domain
and  f (x) is an odd function if  f (–x) = –f (x) for all x in the domain. Hence,
sinh (x) is an odd function while cosh (x) is an even function. These
properties can be observed from the plots of sinh x and cosh x given in
Figure 11w. Observe that, the functions are not periodic.
y

y = cosh (x)

1 y = sinh (x)
1 ​​ex
y = ​​ __
2
x

Figure 11w Graphs of y = sinh (x) and y = cosh (x)

349

Pre-Calculus and Calculus.indb 349 13/7/2017 5:57:27 PM


Chapter 11 • Graphs

Example 15
Show that sech x is an even function.

SOLUTION
​​  –x 2 –(–x) ​​ 
sech (–x) = ________ ​​  ex +2 e–x  
= ______ ​​= sech (x)
e +e
Hence, sech x is an even function.

Exercise 11.10
1. Find the values of the following hyperbolic functions.
(a) sech (–1)
(b) coth (0.5)
(c) cosh 0
(d) sinh (–3)
(e) 1 – sinh2 (1)
(f) cosech (–3)

​​  12 ​​ , find the value of x.


2. Given that the value of tanh = __

3. Show that tanh (x) is an odd function. Sketch the graph of y = tanh (x). (Hint: e–x → 0 as x → 0)

4. Using the definition of hyperbolic functions, show the following.


(a) cosh (2x) cosh (2y) = __ ​​  12 ​​ [cosh (2x + 2y) + cosh (2x – 2y)]
(b) cosh (x – 2y) + sinh (x – 2y) = ex – 2y

350

Pre-Calculus and Calculus.indb 350 13/7/2017 5:57:27 PM


Limits and Derivatives
Chapter 12:

Limits and Continuity

We see practical applications of calculus every day. Limits help us make certain confident
predictions that otherwise, cannot be easily measured. A civil engineer derives results
from data to solve problems relating to water movement, volume and distribution, leading
to practical storm drain design. A structural engineer calculates forces of structural
elements, their strength and bearing capacity. A mechanical engineer computes complex
areas to determine frictional forces.
Measuring a chemical reaction or temperature over time are common everyday examples.
The amount of new compound formed or new temperature attained would be the limit of
a function, as time approaches infinity.

Pre-Calculus and Calculus.indb 351 13/7/2017 5:57:28 PM


Chapter 12 • Limits and Continuity

12 Limits and Continuity


12.1 Introduction to Limits
12.2 Properties of Limits
12.3 Limits at Infinity
12.4 Horizontal and Vertical Asymptotes
12.5 Continuity

12.1 Introduction to Limits


A limit of a function  f (x) at a number a looks at the behaviour of the
function f (x) as x gets closer and closer to a, as illustrated in Figure 12a.
In other words, we are looking at the value taken by  f (x) as x approaches
a.

from right side Notice that x approaches a from two directions. If x approaches a from
the right side, then the limit of  f (x) as x approaches a from the right side
from left side is written as
x
a
​​  lim​+ ​​​   f (x)
x → ​a​​  ​
Figure 12a
If f (x) approaches a number L as x approaches a from the right side, then
we write

​​  lim​+ ​​​   f (x) = L


x → ​a​​  ​

On the other hand, if x approaches a from the left side, then the limit of 
f (x) as x approaches a from the left side is written as

​​ xlim​  ​​​   f (x)


→ ​a​​ –​

If f (x) approaches a number M as x approaches a from the left side, then


we write

​​ xlim​  ​​​   f (x) = M


→ ​a​​ –​

Consequently, if ​​  lim​+ ​​​   f (x) = ​​ xlim​  ​​​   f (x), then we say that the limit of   f (x) as
→ a​ ​​ –​
x → a​ ​​  ​
x approaches a exists and is denoted by

​​ lim​
x→a
 ​​​   f (x)

That is, if L = M, then x​​  lim​
→a
 ​​​   f (x) = L. Otherwise, we say that x​​ lim​
→a
 ​​​   f (x) does
not exist.

352

Pre-Calculus and Calculus.indb 352 13/7/2017 5:57:28 PM


Chapter 12 • Limits and Continuity

Example 1
Evaluate ​​  lim​ ​​​ (x + 2).
x→2

SOLUTION
Note that we are only interested in the behaviour of f (x) = x + 2 as x
approaches 2. For intuitive understanding, we consider values close to
2 as given below.
x f (x) = x + 2
2.2 4.2
2.1 4.1
x approaches 2
2.01 4.01 f (x) approaches 4
(from the right
side) 2.001 4.001
2.0001 4.0001
2 4
1.9999 3.9999
1.999 3.999
x approaches 2
(from the left
1.99 3.99
f (x) approaches 4
side) 1.9 3.9
1.8 3.8

From the right side,  f (x) = x + 2 approaches 4.


Hence, ​​  lim​+ ​​​ (x + 2) = 4.
x→2

From the left side,  f (x) = x + 2 also approaches 4.


Hence, ​​  lim​– ​​​ (x + 2) = 4. 4 right
x→2

In conclusion, the limit of  f (x) = x + 2 as x approaches 2 exists because


left
​​  lim​+ ​​​    f (x) = ​​  lim​– ​​​    f (x) and hence, ​​ lim​ ​​​ (x + 2) = 4. 2
x→2 x→2 x→2

This is the same value when we substitute x = 2 directly into the function 
f (x) = x + 2. However, this is only true if the function  f (x) is continuous x
–2 2
at x = a. In this example,  f (x) = x + 2 is continuous at x = 2 as shown in
the plot of  f (x) = x + 2 in Figure 12b. Figure 12b
The concept of continuity will be covered later in this chapter.

353

Pre-Calculus and Calculus.indb 353 13/7/2017 5:57:28 PM


Chapter 12 • Limits and Continuity

Example 2
x3 – 1
Evaluate ​​  lim​ ​​​ _____
​​  x – 1 ​​ 
.
x→1

SOLUTION
x3 – 1 x3 – 1
Note that  f (x) = ​​ _____
x–1  
 ​​is undefined at x = 1. However,  
​​​
lim​
​​  _____
​​  x – 1 ​​ can
x→1
still be found because our interest is on the neighbourhood of x = 1,
not on the point x = 1. Hence, we obtain the following table.
x3 – 1
x f (x) = _____
​​  x – 1 ​​ 
1.2 3.6400
1.1 3.3100
x approaches 1
(from the right 1.01 3.0300 f (x) approaches 3
side) 1.001 3.0030
1.0001 3.0003
1 undefined
0.9999 2.9997
0.999 2.9880
x approaches 1 0.99 2.9701
(from the left f (x) approaches 3
side) 0.9 2.701
0.8 2.44

x3 – 1
From the right side,  f (x) = _____
​​  x – 1 ​​ approaches 3.
x3 – 1
Hence, ​​  lim​+ ​​​ _____
​​  x – 1 ​​ = 3.
x→1

x3 – 1
From the left side,  f (x) = _____
​​  x – 1 ​​ also approaches 3.
x3 – 1
Hence, ​​  lim​– ​​​ _____
​​  x – 1 ​​ = 3.
x→1

x3 – 1
right In conclusion, the limit of  f (x) = _____
​​  x – 1 ​​ as x approaches x = 1 exists
3
because the values of the one-sided limits are equal.
x3 – 1
Hence, ​​  lim​ ​​​ _____
​​  x – 1 ​​ = 3.
left x→1
1
x3 – 1
x The behaviour of  f (x) = ​​  _____
x – 1 ​​ when x approaches 1 can be observed
–1 1
clearly from Figure 12c.
Figure 12c
x3 – 1
It is clear that as x approaches 1,  f (x) = _____
​​  x – 1 ​​ approaches 3.

354

Pre-Calculus and Calculus.indb 354 13/7/2017 5:57:28 PM


Chapter 12 • Limits and Continuity

Example 3
|x|
Evaluate ​​  lim​ ​​​ __
​​  x ​​ .
x→0

SOLUTION
Let us look at this problem intuitively. We obtain the following table.
|x|
x f (x) = __
​​  x ​​ 
0.2 1
0.1 1
x approaches 0 0.01 1
(from the right side)
0.001 1
0.0001 1
0 undefined
–0.0001 –1
–0.001 –1
x approaches 0 –0.01 –1
(from the left side) –0.1 –1
–0.2 –1

|x|
From the right side,  f (x) = __
​​  x ​​ approaches 1. right
1
|x|
__
Hence, ​​  lim​+ ​​​ ​​  x ​​ = 1.
x→0
x
|x|
__
From the left side,  f (x) = ​​  x ​​ approaches –1. left
–1
|x|
__
Hence, ​​  lim​– ​​​ ​​  x ​​ = –1.
x→0

The values of the one-sided limits are not equal. Hence, we conclude Figure 12d
|x|
that the limit of  f (x) = __
​​  x ​​ as x approaches 0 does not exist.
This can be observed clearly from Figure 12d.

 ​​​ =   f (x) by looking at several values around x = a


It is tedious to find x​​ lim​
→a
as what we have done so far. Instead, we will be using a list of basic
properties of limits which have been proven to hold. The properties are
listed in the next section together with examples.

355

Pre-Calculus and Calculus.indb 355 13/7/2017 5:57:28 PM


Chapter 12 • Limits and Continuity

12.2 Properties of Limits


The table below shows the basic properties of limits, for any real numbers
a.

Properties Examples

(i)  ​​​ c = c, where c is a constant ​​ xlim​


​​ xlim​
→a →a
 ​​​8 = 8

(ii)  ​​​ xn = an, where n is any


​​ xlim​
→a
​​  lim​ ​​​ xn = 5n
x→5
positive integer

(iii)  ​​​ [  f (x) ± g(x)]


​​ xlim​
→a
​​  lim​ ​​​ (3x + x2) = ​​  lim​ ​​​ 3x + ​​  lim​ ​​​ x2
x→2 x→2 x→2

= x​​  lim​
→a
 ​​​   f (x) ± x​​  lim​
→a
 ​​​ g(x) =6+4
= 10

(iv)  ​​​ [  f (x)  g(x)]


​​ xlim​ ​​  lim​ ​​​ [x3(x + 2)]
→a x→2

= x​​  lim​  ​​​   f (x)  ​​ xlim​


 ​​​ g(x) = ​​  lim​ ​​​ x3  ​​  lim​ ​​​ (x + 2)
→a →a x→2 x→2

=8×4
= 32

f (x) ________ ​ xlim​  ​​   f (x) ​  lim​ ​​ 2x


2x
 ​​​ ____ ​​  lim​ ​​​ _____ = __________
→a x→1
(v) ​​ xlim​
→ a g(x)
​​   ​​ = ​​   ​​ 
, ​​  2    ​​  ​​    ​​ 
​ xlim​  ​​ g(x) x→1 x + 3 ​  lim​ ​​ (x2 + 3)
→a x→1
where x​​  lim​  ​​​ g(x) ≠ 0 2
__
→a = ​​  4 ​​ 
​​  12 ​​ 
= __

(vi)  ​​​ [cf (x)] = c​​ xlim​


​​ xlim​
→a →a
 ​​​   f (x), ​​  lim​ ​​​ [5(3x2 + 1)]
x→9
where c is a constant = 5 ​​  lim​ ​​​ (3x2 + 1)
x→9

= 5(244)
= 1220
____ _______ 3
__________
n n
 ​​​ ​​    f (x) ​​ = ​​√   ​ xlim​
(vii) ​​ xlim​
→a √ →a
 ​​  f (x) ​​   
, ​​  lim​ ​​​ ​​√ 
  2x + 7x + 5 ​​ 
2
x → 2_______________
where n is a positive integer = 3​​    ​   
and x​​  lim​ →a
 ​​​   f (x)  0 3
√_____________
x→2
lim​ ​​ (2x2 + 7x + 5) ​​
= ​​√     
2(2)2 + 7(2) + 5 ​​
=3

356

Pre-Calculus and Calculus.indb 356 13/7/2017 5:57:28 PM


Chapter 12 • Limits and Continuity

Example 4
Evaluate the following limits.
4x + 2
(a) ​​  lim​ ​​​ (x2 + 3x) (c) ​​ lim​ ​​​ ______
​​   ​​ 
x → –2 x→7 x + 2
______
(b) ​​ lim​ ​​​ ​​√ 3x + 1 ​​   (d) ​​ lim​ ​​​ [2x5(3x2 – 4)]
x→0 x→2

SOLUTION
(a) ​​  lim​ ​​​ (x2 + 3x) = ​​  lim​ ​​​ x2 + ​​  lim​ ​​​ 3x Property (iii)
x → –2 x → –2 x → –2

= ​​  lim​ ​​​ x2 + 3​​  lim​ ​​​ x Property (vi)


x → –2 x → –2

= (–2) + 3(–2)
2
Property (ii)
= –2
______ ___________
(b) ​​ lim​ ​​​ ​​√ 3x + 1 ​​ 
= ​​  ​  lim​ ​​ (3x + 1) ​​ 
x→0 √____________
x→0
Property (vii)

= ​​  3​ 
   lim​ ​​ x + ​ lim​ ​​ 1 ​​
√_______
x→0 x→0
Properties (iii) and (vi)

=√
​​  3(0) + 1 ​​  Properties (i) and (ii)
=1
​  lim​ ​​ (4x + 2)
4x + 2 __________
(c) ​​ lim​ ​​​ ______
x→7
​​  x + 2 ​​ 
= ​​   ​​  Property (v)
x→7 ​  lim​ ​​ (x + 2)
x→7

4 ​  lim​ ​​ x + ​ lim​ ​​ 2
= _____________
x→7 x→7
  
​​     ​​ Properties (iii) and (vi)
​  lim​ ​​ x + ​ lim​ ​​ 2
x→7 x→7

4(7) + 2
= ​​ _______
7 + 2 ​​ 
  Properties (i) and (ii)

30
= ​​ ___
9 ​​ 
10
= ​​ ___
3 ​​ 
(d) ​​ lim​ ​​​ [2x5(3x2 – 4)] = ​​ lim​ ​​​ 2x5  ​​  lim​ ​​​ (3x2 – 4) Property (iv)
x→2 x→2 x→2

= 2 ​​  lim​ ​​​ x5  [​​  3 ​  lim​ ​​ x2 – ​ lim​ ​​ 4]​​   Property (iii) and (vi)
x→2 x→2 x→2

= 2(2)5[3(2)2 – 4] Properties (i) and (ii)


= 512

f (x) ________
____ ​ xlim​→a
 ​​   f (x)
Property (v) states that x​​ lim​ 
​​​ ​​ 
→ a g(x)
 ​​
  = ​​     ​​​ g(x) ≠ 0. What
 ​​ only if ​​ xlim​
​ xlim​
→a
 ​​ g(x) →a

happens when ​​ xlim​ →a


 ​​​ g(x) = _____ 0? One possibility is that the limit does not exist.
​ x + 4 ​ 

​​  1   ​​ and ​​  lim​ ​​​ ______

Both ​​  lim​ ​​​ _____ ​​  x    ​​ do not exist.
x→2 x – 2 x→0

357

Pre-Calculus and Calculus.indb 357 13/7/2017 5:57:28 PM


Chapter 12 • Limits and Continuity

f (x)
However, the other possibility is that the limit can still exist because ____
​​     ​​
g(x)
is only undefined at x = a but both one-sided limits from the left and right
sides exist and are equal (as seen in Example 2). This can be obtained by
f (x)
simplifying ____
​​    ​​using one of the following methods:
g(x)
Method 1:
f (x)
Factor  f (x) or g(x) and then simplify ____
​​     ​​.
g(x)
E.g.  If  f (x) = x2 – 4, then x2 – 4 = (x – 2)(x + 2).
If  f (x) = x3 – 1, then x3 – 1 = (x – 1)(x2 + x + 1).
Method 2:
____f (x)
​​ 
Multiply g(x)   ​​ with the conjugate of  f (x) or g(x) and then simplify.
_____ _____
E.g.  If  f (x) = ​​√ x + 4 ​​ – 2, then  f (x) = ​​√ x + 4 ​​ + 2 is the conjugate of  f (x).

This is illustrated in the following examples.


Example 5
Evaluate the following. _____
x–2 ​  x + 4 ​ – 2

(a) ​​ lim​ ​​​ _____
​​  2   ​​   (b) ​​ lim​ ​​​ ​​  _________
x   

​​
x→2 x – 4 x→0

SOLUTION
f (x) x–2
(a) ​​ lim​ ​​​ ____
​​    ​​ = ​​  lim​ ​​​ _____
​​  2    ​​ 
x → 2 g(x) x→2 x – 4

Note that g(x) = x2 – 4 and this can be factored as


x2 – 4 = (x –2)(x + 2).
x–2 x–2
Hence, ​​  lim​ ​​​ _____ ​​  2   = ​​  lim​ ​​​ _____
 ​​  ​​        ​​
x→2 x – 4 x → 2 (x – 2)(x + 2)

​​  1   ​​ 
= ​​  lim​ ​​​ _____
x → 2 x + 2

​​  41 ​​ 
= __
_____
____ f (x) ​  x + 4 ​ – 2

_________
(b) ​​ lim​ ​​​ ​​ 
x → 0 g(x)
  ​​ = ​​  lim​ ​​​ ​​ 
x→0 x   

​​
_____ _____
Note that  f (x) = √ ​​  x + 4 ​​ – 2 and its conjugate is √
_____
​​  x + 4 ​​ + 2.
f (x) x + 4 ​ + 2
​  _____

Now, multiplying ____ ​​    ​​ with ​​ _________  
 ​​does not change the
g(x) ​
√  x + 4 ​ + 2
_____
​  x + 4 ​ + 2

function as _________
​​  _____  
 ​​ = 1.
​  x + 4 ​ + 2

_____ _____ _____
​​ = ​​  lim​ ​​​ (​​  ​   ​ ​​  
​√ x + 4 ​ + 2 )
​  x + 4 ​ – 2

_________ ​  x + 4 ​ – 2

_________ √ x + 4 ​ + 2
​  _____
_________
Hence, ​​  lim​ ​​​ ​​ 
x→0 x   

x→0 x   

​ × ​   
x_____
+4–4
= ​​  lim​ ​​​ ___________
​​        ​​
x → 0 x ​ (√ ​  x + 4 ​ + 2) ​
= ​​  lim​ ​​​ _________
​​  _____ 1   ​​ 
x→0 √ ​  x + 4 ​ + 2
= __ ​​  41 ​​ 

358

Pre-Calculus and Calculus.indb 358 13/7/2017 5:57:28 PM


Chapter 12 • Limits and Continuity

Example 6
3x2 – 9
Evaluate ​​  lim​ ​​​ ______
​​   ​​ 
.
x → 4 |x – 2|

SOLUTION
–(x – 2), x < 2
Note that |x – 2| =
x – 2, x  2
Since the limit approaches 4 which is in the interval [2, ∞), we use
|x – 2| = x – 2.
3x2 – 9 3x2 – 9
Hence, ​​  lim​ ​​​ ______
​​  = ​​  lim​ ​​​ ______
 ​​  ​​  x – 2 ​​ 
x → 4 |x – 2| x→4

3(42) – 9
= ________
​​  4 – 2 ​​  
39
= ___
​​  2 ​​ 

Exercise 12.2

1. Find the following limits.


x–3
(a) ​​  lim​ ​​​ (3x + 7) (h) ​​ lim​ ​​​ _____
​​  2    ​​ 
x → –2 x→3 x – 9

x–5
(b) ​​ lim​
 ​​​ (7x + 3) (i) ​​ lim​ ​​​ __________
​​  2    ​​ 
x→c x → 5 2x – 9x + 5

x2 – 9
(c) ​​ lim​ ​​​ [(x + 2)(x – 1)] (j) ​​ lim​ ​​​ ______
​​  3   ​​ 
x→1 x → 3 x – 27
_____
x+2 1–√ ​  x + 1 ​ 
(d) ​​ lim​ ​​​ _____
​​  x – 1 ​​   (k) ​​ lim​ ​​​ _________
​​  x   
​​ 
x→0 x→0
__ _____
_____
​√ 3 ​ – ​√ x – 4 ​ 
__________
(e) ​​ lim​ ​​​ ​​√ x + 2 ​​   (l) ​​ lim​ ​​​ ​​   ​​ 

x→5 x→7 7x – x2
1
__
​   ​ 
(f) ​​ lim​ ​​​ (4x + ​​7)​​  3​​ (m) ​​  lim​+ ​​​ (3x2 + 2)
x→5 x→3

x
​​  2 1   ​​  
(g) ​​ lim​ ​​​ _____ (n) ​​  lim​– ​​​ _____
​​  5 – x ​​ 
x→3 x – 9 x→3

7x + 2, x  2
2. Given that  f (x) =  , find ​​ lim​– ​​​   f (x), ​​  lim​+ ​​​   f (x) and ​​  lim​ ​​​   f (x).
x + 3, x < 2 x→2 x→2 x→2

ex, x < 0
3. Given that  f (x) =  , find ​​ lim​ –​​​   f (x), ​​  lim​ +​​​   f (x) and ​​  lim​ ​​​   f (x).
x , x  0 x → –2 x → –2 x → –2

|x|, x ≠ 0
4. Given that  f (x) =  , find ​​ lim​– ​​​   f (x), ​​  lim​+ ​​​   f (x) and ​​  lim​ ​​​   f (x).
7, x = 0 x→0 x→0 x→0

1, x < 0
5. Given that  f (x) = 2, 0  x  4 , find ​​ lim​+ ​​​   f (x), ​​  lim​+ ​​​   f (x) and ​​  lim​ ​​​   f (x).
x→4 x→0 x→3
3, x > 4

359

Pre-Calculus and Calculus.indb 359 13/7/2017 5:57:28 PM


Chapter 12 • Limits and Continuity

6. From the graph below, state the values of ​​ lim​ ​​​   f (x), ​​  lim​ ​​​   f (x) and ​​  lim​ ​​​   f (x), if they exist.
x → –2 x→0 x→2

x
–4 –2 0 2 4

–2

7. Evaluate the following limits, if they exist.


_____
​  x + 5 ​ – 3

(a) ​​ lim​ ​​​ _________
​​  x – 4 ​​   
x→4

|4 – x|
(b) ​​ lim​ ​​​ _______
​​  2  ​​ 

x → 4 2x – 3x
_____
​√ x + 5 ​ 
______
(c) ​​ lim​ ​​​ ​​  x – 1 ​​ 
x→1
_____
​√ x + 5 ​ – 2
_________
(d) ​​  lim​ ​​​ ​​  x + 1 ​​ 

x → –1

e2x – 1
(e) ​​ lim​ ​​​ ​​ _____
ex + 1   ​​ 
x→0

e2x – 1
(f) ​​ lim​ ​​​ _____
​​  ex – 1   ​​ 
x→0

360

Pre-Calculus and Calculus.indb 360 13/7/2017 5:57:28 PM


Chapter 12 • Limits and Continuity

12.3 Limits at Infinity


In the previous section, the limit of a function  f (x) takes a real number L as
x approaches a. However, there is a possibility that as x approaches a,  f (x)
becomes unbounded, that is,  f (x) becomes larger and larger towards
infinity. If the function  f (x) increases without limit when x approaches a,
then we write
 ​​​   f (x) = +∞
​​ lim​
x→a

The above limit takes this form if both the left and right sides of the limit a x
approach +∞. That is, ​​  lim​ ​​​   f (x) = +∞ and x​​  lim​
→a
 ​​​   f (x) = +∞. This is shown

x → a+
graphically in Figure 12e.
Figure 12e
If the function  f (x) decreases without limit when x approaches a, then we
write
 ​​​   f (x) = –∞.
​​ lim​
x→a

Likewise, the above limit takes this form if ​​ lim​ ​​​   f (x) = –∞ and
x → a+
​​ xlim​ ​​​   f (x) = –∞. This is shown graphically in Figure 12f. x
→a
– a

 ​​​   f (x) = +∞ or x​​  lim​


Note that when we write ​​ xlim​
→a →a
 ​​​   f (x) = –∞, that does not
mean that the limit exists. This is because or is not a real number.
Instead, it only points out that  f (x) will take a very large value when x
approaches a. Figure 12f
Let us consider the following example.

Example 7
Evaluate ​​  lim​ ​​​ _____ 1   ​​ 
​​  x +
x → –1 1.
SOLUTION
Consider the following table.

x f (x) = _____ 1   ​​ 


​​  x + 1
–0.8 5
x approaches –1 –0.9 10 f (x) approaches
(from the right –0.99 100 positive infinity
side)
–0.999 1000
–0.9999 10000
–1 Underfined
–1.0001 –10000
–1.001 –1000
x approaches –1 f (x) approaches
(from the left –1.01 –100
negative infinity
side) –1.1 –10
–1.2 –5

361

Pre-Calculus and Calculus.indb 361 13/7/2017 5:57:29 PM


Chapter 12 • Limits and Continuity

1  ​​
It can be seen that as x approaches –1 from the right side,  f (x) = ​​ _____
x + 1 
becomes larger and larger approaching infinity. On the other hand, as
f (x) x approaches –1 from the left side,  f (x) = ​​ _____ 1  ​​becomes smaller and
x + 1 
+∞
smaller approaching negative infinity as shown in Figure 12g.
Hence, we can write both limits as
1
​​  1   ​​ = +∞ and ​​ lim​ –​​​ _____
​​  lim​ +​​​ _____ ​​  1   ​​ = –∞.
x x → –1 x + 1 x → –1 x + 1
–1
​​  x 1+  1  ​​
As the one-sided limits have different signs for infinity, ​​ lim​ ​​​ _____
x → –1
–∞ cannot be equal to +∞ or –∞.

Figure 12g In the cases above, we only consider the limits as x approaches a real
number. It is possible to look at the limits as x approaches a very large
value, usually denoted by symbols, –∞ or +∞. These limits are written as
​​ xlim
→ +∞
​ ​​​   f (x) and x​​ l→ ​​​​   f (x).
im–∞ 
If the function  f (x) approaches L when x increases without limit to positive
infinity, then we can write it as
​​ xlim
→ +∞
​ ​​​   f (x) = L.
If the function  f (x) approaches M when x decreases without limit to
negative infinity, then we can write it as
​​ xlim
→ –∞
​ ​​​   f (x) = M.
These limits can be understood intuitively as well and are illustrated in the
following examples.
Example 8
Evaluate ​​ xlim
→ +∞ x
​​  1 .​​
​ ​​​ __

SOLUTION
im+∞ 
Consider the following table to determine x​​ l→ ​​  1x .​​
​​​​ __

x ​​  1x ​​
f (x) = __
... ...
100000 0.00001
10000 0.0001
To positive f (x) approaches
infinity
1000 0.001 zero
100 0.01
10 0.1
... ...
x
Note that as x goes to positive infinity, f (x) = ​​  1x ​​gets closer and closer
__
x → +∞
to 0.
Hence, we write the limit as
Figure 12h ​​ xlim ​​  1 ​​ = 0.
​ ​​​ __
→ +∞ x

362 This is illustrated in Figure 12h.

Pre-Calculus and Calculus.indb 362 13/7/2017 5:57:29 PM


Chapter 12 • Limits and Continuity

Example 9
Evaluate ​​ xlim
→ –∞ x
​​  1 .​​
​ ​​​ __

SOLUTION
im–∞ 
Consider the following table to determine x​​ l→ ​​  1x .​​
​​​​ __

x ​​  1x ​​
f (x) = __
... ...
–10 –0.1
–100 –0.01
To negative f (x) approaches
infinity
–1000 –0.001 zero
–10000 –0.0001
–100000 –0.00001 x
... ...
Note that as x goes to negative infinity, f (x) = ​​  1x ​​gets closer and closer
__
to 0.
Hence, we write the limit as
​​ xlim
→ –∞ x
​​  1 ​​ = 0.
​ ​​​ __ Figure 12i

This is illustrated in Figure 12i.

We can also find the limits involving infinity using the basic properties
of limits as given in Section 12.2. Let us look at the following examples.
Example 10
Evaluate the following limits.
(a) ​​ xlim ​ ​​​ [x2(x + 1)] (b) ​​ xl→
im–∞​ ​​​ [3(x + 1)2] (c) ​​ xlim ​ ​​​ _______ 1   ​​ 
​​  ______
→ –∞ → +∞ √ ​  4x + 3 ​ 
SOLUTION
(a) ​​ xlim
→ –∞
​ ​​​ [x2(x + 1)] = x​​ l→im–∞ ​​​​ x2  ​​ xlim
→ –∞
​ ​​​ (x + 1) Property (iv) Note
= x​​  lim
→ –∞
​ ​​​ x  (​​  ​ xlim
2
→ –∞
​ ​​ x + x​ l→ ​​​ 1)​​  
im–∞  Property (iii) Since x2 is always positive
= (+∞)(–∞ + 1) Property (i)
for all values of x,
= (+∞)(–∞) ​​ xlim​ ​​​ x2 = +∞
→ –∞
= –∞
(b) ​​ xlim
→ –∞
​ ​​​ [3(x + 1)2] = 3 x​​ l→ im–∞ ​​​​ (x + 1)2 Property (iv) Note
= 3(+∞) Since (x + 1)2 is always
= +∞ positive for all values of x,
​  lim ​ ​​ 1 ​​ xlim​ ​​​ (x + 1)2 = +∞
(c) ​​ xlim ​   ​​​ _______
​​  1   ​​ 
______ = ___________
​​    
x → +∞ ______   ​​ Property (v) → –∞
→ +∞ √ ​  4x + 3 ​  ​ xlim
→ +∞
​  
​​ ​
√  
4x + 3 ​ 
​  lim ​ ​​ 1
= ________________
_______________
x → +∞
​​        ​​ Properties (iii), (vi) and (vii)
​√ 4  
​ xl→
im+∞  ​​​ x + x​ l→
im+∞ 
​​​ 3 ​

= ​​ __________ 1   ​​ 
_________
​√ 4(+∞) + 3 ​ 
= ___​​  +∞1    ​​
363
=0

Pre-Calculus and Calculus.indb 363 13/7/2017 5:57:29 PM


Chapter 12 • Limits and Continuity

From Example 10, we use standard results involving infinity such that
4(+∞) = +∞ or ‒∞ + 1 = ‒∞ or (+∞)(–∞) = –∞ or ___ 1   ​​ = 0.
​​  +∞
 f (x) __
However, what happens if the answer for ​​ xlim ​​​​ ____

→ +∞ g(x)
​​   ​​ is ​​  ∞
∞ ​​? The alternative
 f (x)  f (x)
approach is to change the form of ____ ​​   ​​ first by multiplying ____ ​​   ​​ with
g(x) g(x)
__ ​  ​x1​​  n  ​​ 
__
​​  __  ​​ , where n is the highest power of x in g(x). This is illustrated in the
​  ​x1​​  n  ​​ 
following examples.

Example 11
2x + 3
 ​​​ ______
Evaluate ​​ xlim​ ​​ 
→ +∞ x2 + 1
 ​​ 
.

SOLUTION
2x + 3 __ ∞ .​​
Direct substitution of in the function gives x​​ lim​  ​​​ ______
​​ 
→ +∞ x2 + 1
 ​​ 
= ​​  ∞
 f (x) 2x + 3
Hence, let us consider x​​ lim​ ​​​ ______
→ +∞ g(x)
​​   ​​​ ______
 ​​ = x​​  lim​ ​​ 
→ +∞ x2 + 1
.
 ​​ 

The term with the highest power in g(x) = x2 + 1 is x2. Hence, we

 f (x) ______
__
​  12  ​ 
______ 2x + 3 __ ​x​​  ​
multiply ​​   ​​ = ​​   ​​ 
with ​​  __   ​​.
g(x) x2 + 1 ​  12  ​ 
​x​​  ​
2x 3 __ 3
 f (x)
__
​  12  ​  ___
​  2 ​ + __ ​  2  ​  ​  2x ​ + __
​  2  ​ 
______ 2x + 3 __
______ ​x​​  ​ _______
x x ______ x
​​   ​​ = ​​   ​​  1   ​​ = ​​  2
 ​​   ​​ 
= ​​   ​​ 
g(x) x2 + 1 __ ​  2  ​  __ x​ ​​  ​ 1 1 + __
​  2 ​ + __​  2  ​  ​ 12  ​ 
​x​​  ​ x x x
3
​  2x ​ + __
__
​  2  ​ 
2x + 3
______ ______ x _____ 0+0
 ​​​ ​​ 
Hence, x​​  lim​
→ +∞ x2 + 1
 ​​ 
= x​​  lim​
 ​​​ ​​ 
→ +∞ __1  ​​  = ​​  1 + 0 
 ​​ = 0.
1 + ​  2  ​ 
x

Example 12
Evaluate the following limits. ______
3x2 + 1
______ ​  3x + 1 ​ 

_______
(a) ​​ xlim​  ​​​ ​​ 
→ –∞ x2 + 1
 ​​   (d) ​​ xlim​
 ​​​ ​​ 
→ +∞ x + 1
 ​​ 

__
5x3 + 1
______ ​  x  ​+ 1
 ​​​ ______

(b) ​​ xlim​   ​​​
→ +∞ x + 1
​​   ​​   (e) ​​ xlim​ ​​ 
→ +∞ x + 1
 ​​ 

3x + 1
(c) ​​ xlim​
→ –∞
 ​​​ _______
​​  _____   ​​ 
​√ x2 + 1 ​ 

364

Pre-Calculus and Calculus.indb 364 13/7/2017 5:57:29 PM


Chapter 12 • Limits and Continuity

SOLUTION
⎞ ⎛
⎜ ⎟
​  12  ​ 
__ Note
 ​​​ ​​  ​   2  ​  1   ​ ​​  
3x2 + 1
______ 3x2 + 1 __
_____ ​x​​  ​
(a) ​​ xlim​  ​​​ ​​ 
→ –∞ x2 + 1
 ​​ 
= x​​  lim​  ​  x 2 is the term with the
x + 1 __
⎝ ​  2  ​ ⎠
→ –∞
​x​​  ​ highest power of x in the
denominator. Hence, we
3x2 1
___
​  2 ​ + __​  2  ​  ​  12  ​ 
__
_______x x f (x) ​x​​  ​
= x​​  lim​ ​​​ ​​   ​​  ____ __
multiply ​​  g(x) ​​ with ​​  __  ​​ .
→ –∞ __ x​ ​​  ​ __
2
1
​  2 ​ + ​  2  ​  ​  12  ​ 
x x ​x​​  ​
3 + __​ 12  ​ 
_____ x
= x​​  lim​ ​​​ ​​    ​​
1 + ​ 12  ​ 
__
→ –∞
x
3+0
= ​​ _____
1 + 0   ​​
=3

1  ​ ​)​​  
​  1x ​
__ Note
x → +∞ ( x + 1
5x3 + 1
______ 5x3 + 1 __
______
(b) ​​ xlim​  ​​​
→ +∞ x + 1
​​   ​​ = ​​  lim​  ​​​ ​​  ​   ​  ​  x is the term with the
​ __ x highest power of x in the
5x3 __
___ 1 denominator. Hence, we
​  x   ​ + ​  x ​
= ​​ xlim​  ​​​ _______
​​  x __  ​​  f (x) ​  1x ​
__
​  x ​ + ​  1x ​ multiply ​​ ____
→ +∞ __ __
g(x)  ​​
  with ​​   ​​ .
​  1x ​
__

5x2 + __
______ ​ 1x ​
= x​​  lim​  ​​​ ​​   ​​ 
→ +∞
1 + __ ​ 1x ​
+∞ + 0
= ______
​​  1 + 0 ​​ 
= +∞

⎜ ⎟
⎛ ⎞
​  1__ 2  ​ 
____
Note
​√ ​x​​  ​ ​ 
​   _____  ​ ​​  
3x + 1 3x + 1 ____
 ​​​ _______  ​​​ ​​  _______
__
(c) ​​ xlim​ ​​  _____   ​​ 
= x​​  lim​    ​   ​  ____  
​  1__ 2  ​ 
→ –∞
​√ x + 1 ​  → –∞

√ x

​ ​​  2
​ + 1 ​   ​​√ x2 ​​ is the term with the
⎝ ​√ ​x​​  ​ ​ ⎠
2
highest power of x in the
3x __
___ 1 denominator. Hence, we
______ x ​ + ​  x ​
​  _____
​  1__ 2  ​ 
____
= ​​ xlim​   ​​​ ​​     ​​  f (x) ​√ ​x​​  ​ ​ 

x​ ​​  2​ 1
→ –∞ ____ ____
​  __ ​  2 ​  + ​ __  2 ​ ​   multiply ​​  g(x) ​​ with ​​  ____  ​​ .
​x​​  ​ x ​  1__ 2  ​ 
__ 1 ​√ ​x​​  ​ ​ 
3 + ​ x ​
= _______
​​  _____  ​​ 
​  1 + __
x √
​ 12  ​ ​  
3_____
______ +0
= ​​    ​​ 
​  1 + 0 ​ 

=3

365

Pre-Calculus and Calculus.indb 365 13/7/2017 5:57:29 PM


Chapter 12 • Limits and Continuity

 ​  ​​  
______ ______ __
​  1x ​
→ +∞ ( x + 1
​  1x ​)
​  3x + 1 ​ 

_______ ​  3x + 1 ​ __

_______
(d) ​​ xlim​  ​​​ ​​ 
→ +∞ x + 1
 ​​ 
   ​​​ ​​  ​ 
= x​​  lim​  ​ 
  ​  __
Note
_______


3x
​  1  ​ ​  
x is the term with the
​  ___
​   ​ + __
highest power of x in the x
2
x 2

denominator. Hence, we = ​​  lim​ ​​​ ________


​​  x  ​​   
x → +∞ __
​  1x ​
​  x ​ + __
f (x)
____ __ ​  1x ​
__
______
multiply ​​  g(x) ​​ with ​​  __  ​​ .

3 1
​  1x ​ ​  __
​  x ​ + __
​  2  ​ ​  
_______ ​x​​  ​
= ​​ xlim​

→ +∞
​​​ ​​  1  ​​ 
__

1 + ​ x ​
_____

√ 0+0
= ​​  _____
​  1 + 0 
 ​ ​​ 
=0

1  ​ )​​  
​  1x ​
__
__ _____

→ +∞ ( x + 1
​  x  ​+ 1
______
√ ​  x ​ + 1
______
√   __
(e) ​​ xlim​  ​​​ ​​   ​​   ​​​ ​​  ​ 
= x​​  lim​  ​ 
 ​  __
Note → +∞ x + 1
​  x ​
x is the term with the ___
x 1
​  __
​  2  ​ ​  + __
highest power of x in the
= x​​  lim​
  ​​​

_______
​​ 
​x​​  ​ x
​   ​
denominator. Hence, we
→ +∞ __ x __
​  x ​ + ​  x ​ 1  ​​ 

f (x) ​  1x ​
__
__
multiply ____
​​  g(x) ​​ with __
​​  __  ​​ . 1 ​ ​ + __ 1
​  1x ​ √
 ​  ​  __
______
 ​​​ ​​ 
= ​​ xlim​
x ​  x ​
 ​​ 
→ +∞
1 + __ ​ 1x ​
0+0
= ​​ _____
1 + 0   ​​
=0

366

Pre-Calculus and Calculus.indb 366 13/7/2017 5:57:29 PM


Chapter 12 • Limits and Continuity

Exercise 12.3
1. Find the following limits, if they exist.
2x x
(a) ​​  lim​+ ​​​ _____
​​  2    ​​    ​​​ _____
(j) ​​ xlim​ ​​     ​​ 
→∞ x + 1
x→2 x – 4
x2
(b) ​​  lim​+ ​​​ [ln (x – 2)] (k) ​​ xlim​ ​​​ ______
​​ 
→ –∞ 2x + 1
   ​​ 
x→2
2x
(c) ​​  lim​– ​​​ _____
​​  x –  3 ​​   (l) ​​ xlim​ ​​  __1   ​​ 
 ​​​ ______
→ ∞ ​  x ​ + 2
x→3 √
_____
2
_______
_____ ​√ x2 + x ​ 
_________ –2
(d) ​​  lim​+ ​​​ ​​  2    ​​   (m) ​​ xlim​  ​​​ ​​  2x + 3 ​​   
x → 3 ​√ ​x​​  ​– 9 ​  → –∞

3x
(e) ​​ lim​ ​​​ (ln x2) (n) ​​ xlim​  ​​​ ______
→ –∞ 2x + 2
​​     ​​ 
x→0
_______
1   ​​ ​√ 16x2 – 9 ​ 
(f) ​​ lim​ ​​​ ___________
​​     (o) ​​ xlim​   ​​​ ________
​​  x + 1 ​​   
x → 2 (x – 2)(x + 1) → +∞

→ ∞ ( x2
​  1x ​)​​  
3x
(g) ​​ xlim​  ​​​ ​​  __ ​  1  ​ + __ (p) ​​  lim​+ ​​​ _____​​     ​​ 
x → 2 |x – 2|
_____
​√ x2 – 9 ​ 
(h) ​​ xlim​
→∞ x + 2
​​  1   ​​  
 ​​​ _____ (q) ​​ xlim​   ​​​
→ +∞ 6x + 1
______
​​   ​​ 
_____
​  x – 9 ​ 

(i) ​​ xlim​
→∞
 ​​​ (3x2 + 1) (r) ​​ xlim​  ​​​ ______
→ +∞ x2 – 1
​​   ​​ 

2. Find the value of c for which x​​ lim​ ______2x    __ 1


  ​​​ ​​   x ​​ = ​​   ​​ .
→ +∞ 2 + c 2
x

367

Pre-Calculus and Calculus.indb 367 13/7/2017 5:57:29 PM


Chapter 12 • Limits and Continuity

12.4 Horizontal and Vertical Asymptotes


In Section 12.3, we have results involving infinite limits such
 ​​​ h(x) = b or ​​  lim​+ ​​​ h(x) = ±∞ or ​​ xlim​
as x​​  lim​
→ ±∞
 ​​​ h(x) = ±∞. The numbers x = a or
→ a–
x→a

y = b that the limits approach form lines called asymptotes. Generally,


there are two types of asymptotes, vertical and horizontal asymptotes.
Vertical asymptote of the graph h(x) is the line x = a when we have one
of the following:
(i) ​​  lim​+ ​​​ h(x) = +∞
x→a

(ii) ​​  lim​+ ​​​ h(x) = –∞


x→a

(iii) ​​ xlim​ ​​​ h(x) = +∞


→ a–

(iv) ​​ xlim​ ​​​ h(x) = –∞


→ a–

Most of the times, a is the value where h(x) is undefined. For instance,
f (x)
when the denominator of the function h(x) = ____
​​  g(x) ​​ equals zero.

On the other hand, horizontal asymptote of the graph h(x) is the line
y = b when we have one of the following:
(i) ​​ xlim​
→ +∞
 ​​​ h(x) = b
(ii) ​​ xlim​
→ –∞
 ​​​ h(x) = b

We look at the following examples on locating the vertical and horizontal


asymptotes of a function h(x).

Example 13
Find the horizontal and vertical asymptotes, if any, of the following
functions. Illustrate the asymptotes for each function in a graph.
(a)  f (x) = _____ ​​  x –2 2 ​​ 
x+1
(b) g(x) = _____
​​  2    ​​ 
x –4
x
(c) h(x) = ​​ _____ x + 3 ​​ 
(d) k(x) = ______
​​  _____ 1   ​​ 
​  x – 3 ​ 

x
(e) m(x) = _____ ​​  2    ​​ 
x +2
SOLUTION
​​  x –2 2 ​​ 
(a)  f (x) = _____
Vertical asymptote:
​​  x –2 2 ​​ 
  f (x) = _____ is undefined when x – 2 = 0. Hence, we should consider
x = 2 as a candidate of vertical asymptote.
368

Pre-Calculus and Calculus.indb 368 13/7/2017 5:57:29 PM


Chapter 12 • Limits and Continuity

​​  x –2 2 ​​  
Evaluate the one-sided limits of  f (x) = _____ :
Substituting x = 1.99999 into the limit,
​​  2   ​​ = –∞.
​​  lim​– ​​​ _____
x→2 x – 2

Substituting x = 2.00001 into the limit,


​​  2   ​​ = +∞.
​​  lim​+ ​​​ _____
x→2 x – 2
​​  x –2 2 ​​ is x = 2.
Hence, the vertical asymptote for  f (x) = _____
Horizontal asymptote:
Consider the infinite limits of this function:

 ​​​ ​​  ​     ​  ​  1  ​  ​​  = x​​  lim​


​  1x ​
__ 2 ​
​ __ ​  2x ​
__

→ +∞ ( x – 2 __
​  x ​)
2
_____ 2
_____ __ _____ x _____ 0
​​  lim​ ​​​ ​​     ​​ = x​​  lim​
x → +∞ x – 2 →
 ​​​ ​​  __x  __
+∞ 2  ​​ = x​​  lim​
→ +∞
 ​​​ ​​   __ 2  ​​ = _____
​​  1 – 0 ​​ = 0
​  x ​ – ​  x ​ 1 – ​ x ​
To be more specific, as x → +∞,  f (x) = _____ ​​  x 2– 2   ​​ → 0 from the right
2 ​
​ __
x
side. For example, substitute x = 2000 into _____ ​​   __  ​​, we have 0.001

1 – ​ 2x ​
​  2x ​
__
which is larger than 0. Hence, _____
​​   __  ​​ → 0 from the right side.
1 – ​ 2x ​

We may write the limit as x​​ lim​ ​​  2   ​​ = 0+.


 ​​​ _____
→ +∞ x – 2

Similarly,

 ​  ​​  = x​​  lim​


​  1x ​
__ 2 ​
​ __ ​  2x ​
__

( )
2
_____ 2
_____ __ _____ x _____ 0
​​ xlim​  ​​​ ​​     ​​ = x​​  lim​
→ –∞ x – 2
 ​​​ ​​  ​  x – 2 ​  ​  __
→ –∞ 1  ​​​ ​​  x  __
→ –∞ __ 2  ​​ = x​​  lim​ ​​​ ​​   __
→ –∞ 2  ​​ = _____
​​  1 – 0 ​​ = 0
​  x ​ ​  x ​ – ​  x ​ 1 – ​ x ​

As x → –∞,  f (x) = _____ ​​  x 2–  2  ​​ → 0 from the left side. For example,

​  2x ​
__
substitute x = –3000 in _____ ​​   __  ​​, we have –0.00067 which is smaller

1 – ​ 2x ​
​  2x ​
__
_____
than 0. Hence, ​​   __  ​​ → 0 from the left side.
1 – ​ 2x ​
We may write the limit as x​​ l→im​ –∞ x – 2
​​  2   ​​ = 0–.
 ​​​ _____

Hence, the horizontal asymptote for  f (x) = _____ ​​  x –2 2 ​​ is y = 0.


The asymptotes are illustrated below.
f (x)
​​  lim​+ ​​​ f (x) = –∞
x→2

 ​​​ f (x) = 0+
​​ xlim​
→ +∞

x
2
​​  lim​ ​​​ f (x) =
x → –∞
0– horizontal
asymptote
vertical
asymptote
​​  lim​ ​​​ f (x) = –∞
x → 2–
369

Pre-Calculus and Calculus.indb 369 13/7/2017 5:57:29 PM


Chapter 12 • Limits and Continuity

x+1
(b) g(x) = _____ ​​  2    ​​  
x –4
Vertical asymptotes:
x+1
 g(x) = ​​ _____   ​​ is undefined when x2 – 4 = 0. After factoring,
x2 – 4
(x – 2)(x + 2) = 0 gives solutions x = ± 2. Hence, we should
consider x = 2 and x = –2 as candidates of vertical asymptotes.
x+1
Evaluate the one-sided limits of g(x) = _____ ​​  2   at x = 2 and x = –2:
 ​​ 
x –4
Substituting x = 1.99999 into the limit,
x + 1
​​  lim​– ​​​ _____
​​  2    ​​  = –∞.
x→2 x – 4

Substituting x = 2.00001 into the limit,


x + 1
​​  lim​+ ​​​ _____
​​  2    ​​  = +∞.
x→2 x – 4

Substituting x = –2.00001 into the limit,


x + 1
​​  lim​ –​​​ _____ ​​  2    ​​  = –∞.
x → –2 x – 4

Substituting x = –1.99999 into the limit,


x + 1
​​  lim​ +​​​ _____ ​​  2   = +∞.
 ​​ 
x → –2 x – 4
x+1
Hence, the vertical asymptotes for g(x) = _____ ​​  are x = 2 and x = –2.
 ​​  

x2 – 4
Horizontal asymptote:
Consider the infinite limits of this function:
⎛ ⎞
⎜  ⎟ 
__
​  12  ​ 
 ​​​ ​​  ​    ​  1   ​ ​​  
_____ x + 1 x + 1 __
_____ ​x​​  ​
​​ xlim​  ​​​ ​​ 
→ +∞ x2 – 4
 ​​ 
  = x​​  lim​
→ +∞ x2 – 4 __
 ​ 

⎝ ​  2  ​ ⎠
​x​​  ​
__ x __ 1
​  2  ​ + ​  2  ​ 
______ x x
= ​​ xlim​   ​​​ ​​   ​​ 
→ +∞ 2
__x __ 4
​  2 ​  – ​  2  ​ 
x x
​  1x ​ + __
__
​  12  ​ 
______ x
= x​​  lim​ ​​​ ​​   ​​ 
→ +∞
1 – ​ 42  ​ 
__
x
0+0
_____
= ​​  1 – 0 ​​ 
=0
x+1
To be more specific, as x → +∞, g(x) = ​​  _____  ​​ → 0 from the right


x2 – 4
x + 1
side. We may write the limit as x​​ lim​  ​​​ _____
→ +∞ x2 – 4
​​   ​​ 
  = 0+.

​  1x ​ + __
__
​  12  ​ 
x + 1
_____ ______ x _____ 0+0
Similarly, x​​  lim​  ​​​ ​​ 
→ –∞ x2 – 4
 ​​ 
  = x​​  lim​
→ –∞
 ​​​ ​​  __ 4  ​​  = ​​  1 – 0 ​​ = 0
1 – ​  2  ​ 
x
x+1
_____
As x → –∞, g(x) = ​​  2   → 0 from the left side. We may write the
 ​​ 
x –4
x + 1
limit as x​​  lim​ ​​​ _____
​​ 
→ –∞ x2 – 4
= 0–.
 ​​ 

x+1
Hence, the horizontal asymptote for g(x) = _____ ​​  2   is y = 0.
 ​​ 
370 x –4

Pre-Calculus and Calculus.indb 370 13/7/2017 5:57:29 PM


Chapter 12 • Limits and Continuity

The asymptotes are illustrated below.


f (x)

​​  lim​ +​​​ g(x) = ∞ ​​  lim​+ ​​​ g(x) = +∞


x→2
x → –2

 ​​​ f (x) = 0+
​​ xlim​
→ +∞

x
–2 2
​​ xlim​ ​​​ g(x) = 0–
→ –∞ horizontal
asymptote
vertical
asymptotes

​​  lim​ –​​​ g(x) = –∞ ​​  lim​ ​​​ g(x) = –∞


x → –2 x → 2–

x
(c) h(x) = _____
​​  x + 3 ​​ 
Vertical asymptote:
x
h(x) = _____
​​  x + 3 ​​ 
is undefined when x + 3 = 0. Hence, we should consider
x = –3 as a candidate of vertical asymptote.
x
Evaluate the one-sided limits of h(x) = _____
​​  x + 3 ​​ 
:
Substituting x = –3.00001 into the limit,
x
​​  lim​ –​​​ _____
​​  x + 3 ​​ = +∞.
x → –3

Substituting x = –2.99999 into the limit,


x
​​  lim​ +​​​ ​​ _____
x + 3 ​​ = –∞.
x → –3
x
Hence, the vertical asymptote for h(x) = _____
​​  x + 3 ​​ is x = –3.
Horizontal asymptote:
Consider the infinite limits of this function:

 ​​​ ​​  ​     ​  ​  1  ​  ​​  


​  1x ​
__

→ +∞ ( x + 3 __
​  x ​)
x
_____ _____ x __
 ​​​ ​​     ​​ = x​​  lim​
​​ xlim​
→ +∞ x + 3

x
_____ ​ __x ​

= x​​  lim​ ​​​
→ +∞ __
​​  x  3  ​​ 
​  x ​ + __ ​  x ​

= ​​ xlim​ ​​  1 3  ​​ 


 ​​​ _____
1 + __
→ +∞
​ x ​

= _____ 1   ​​ 
​​  1 + 0
= 1–

371

Pre-Calculus and Calculus.indb 371 13/7/2017 5:57:30 PM


Chapter 12 • Limits and Continuity

Similarly,
x
​​ xlim​ ​​​ _____
​​   3  ​​ = x​​  lim​ ​​  1 3  ​​ = _____
 ​​​ _____ ​​  1   ​​ = 1+.
→ –∞ __ → +∞ __ 1 + 0
x + ​ x ​ 1 + ​ x ​
 Hence, both results suggest that the horizontal asymptote for
x
h(x) = _____ ​​  x + 3 ​​ is y = 1.
The asymptotes are illustrated below.
f (x)

​​  lim​ –​​​ h(x) = –∞


x → –3

horizontal
asymptote
​​ xlim​ ​​​ h(x) = 1+
→ –∞

1
 ​​​ h(x) = 1–
​​ xlim​
→ +∞
x
–3

vertical
asymptote

​​  lim​ ​​​ h(x) = +∞


x → –3+

(d) k(x) = ______ 1   ​​ 


​​  _____
​  x – 3 ​ 

Vertical asymptote:
The domain of k(x) = ______ 1   ​​ 
​​  _____ is {x: x > 3}. Thus, we should
​  x – 3 ​ 

investigate the limit of k(x) when x approaches x = 3 from the
right only.
​​  lim​+ ​​​ ______
​​  _____ 1   ​​  = +∞
x→3 √ ​  x – 3 ​ 
Hence, the vertical asymptote for k(x) = ______ 1   ​​ 
​​  _____ is x = 3.
​  x – 3 ​ 

Horizontal asymptote:
Consider the infinite limits of this function:
⎛ 1__  ​ ⎞
⎜  ⎟ 
___
​  √  x  ​
= ​​  lim​ ​​​ ​​  ​   _____  ​  1   ​ ​​  
______ 1 ______ 1 ___ ​
​​  lim​ ​​​ ​​  _____    ​​     ​ 
​  x – 3 ​  x → +∞ √ ​  x – 3 ​  ___
⎝ ​  x    ​​ ⎠
x → +∞ √ __
​  √
1__
​ ___
______ ​√ x    ​​ 
_____
 ​​​ ​​ 
= x​​  lim​    ​​ 
x __ 3

→ +∞ __
​  ​  x ​ – ​  x ​ ​ 
___
​  √1__
___________ ​  x    ​​ 
= ​​ xlim​ ​​​ ​​     ​​ 
3

​  1 – __
→ +∞
​ x ​ ​ 
0
= ​​ ______
_____
   ​​ 
​  1 – 0 ​ 

= 0+

372

Pre-Calculus and Calculus.indb 372 13/7/2017 5:57:30 PM


Chapter 12 • Limits and Continuity

Hence, this result suggests that the horizontal asymptote for


k(x) = ______ 1   ​​ 
​​  _____ is y = 0.
​  x – 3 ​ 

The asymptotes are illustrated below.
f (x)

​​  lim​ ​​​ k(x) = +∞


x → 3+

​​  lim​ ​​​ k(x) = 0+


x → +∞

x
3

vertical
asymptote horizontal
asymptote

x
(e) m(x) = _____
​​  2    ​​ 
x +2
Vertical asymptote:
x
Notice that the denominator of m(x) = ​​ _____    
 ​​does not take the
x2 + 2
value 0. Hence, we conclude that this function does not have any
vertical asymptote.
Horizontal asymptote:
Consider the infinite limits of this function:
⎛ ⎞
⎜  ⎟ 
​  12  ​ 
__ 1 ​
​ __
 ​  1   ​ ​​  ​​ xlim​
 ​​​ ​​  ​      ​ 
x
_____ x
_____ __ ​x​​  ​ _____ x 0
 ​​​ ​​     ​​ 
​​ xlim​
→ +∞ x2 + 2
= x​​  lim​
→ +∞ x2 + 2 __
 ​​​ ​​   __
2 = _____
 ​​  ​​  1 + 0 ​​ = 0+.
⎝ ​  2  ​ ⎠ → +∞
1 + ​  2  ​ 
​x​​  ​ x
1 ​
​ __
x
_____ _____ x 0
Similarly, x​​  lim​  
​​​ ​​ 
→ –∞ x2 + 2
   ​​ = ​​  lim​
x → –∞
  ​​​ ​​   __
2 = _____
 ​​  ​​  1 + 0 ​​ = 0–.
1 + ​  2  ​ 
x
Hence, both results suggest that the horizontal asymptote for
x
m(x) = _____
​​  2    ​​  is y = 0.
x +2
The asymptotes are illustrated below.
f (x)

​​  lim​ ​​​ k(x) = 0+


x → +∞

​​ xlim​ ​​​ k(x) = 0–
→ –∞ horizontal
asymptote

373

Pre-Calculus and Calculus.indb 373 13/7/2017 5:57:30 PM


Chapter 12 • Limits and Continuity

Exercise 12.4
1. Find the horizontal and vertical asymptotes, if any, of the following functions.
​​  1  2 ​​ 
f (x) = _____
(a)  
1–x
_____ x
f (x) = ​​  2 – x  
(b)   ​​

​​  1x ​​ + 1
f (x) = __
(c)  

​​  1  2 ​​ 
f (x) = _______
(d)  
(x + 2)
3x
f (x) = _____
(e)   ​​     ​​ 
9 – x2
x
f (x) = _______
(f)   ​​  _____    ​​ 
​√ ​x​​  ​+ 3 ​ 
2
_____
​√ ​x​​  2​+ 9 ​ 
f (x) = _______
(g)   ​​  x    ​​ 
x2
f (x) = _____
(h)   ​​     ​​ 
1 – x2
x –__2
f (x) = _____
(i)   ​​  √​  x  ​​ ​  

x
2. Function h is defined as h(x) = _______
​​  _____     ​​. Find the asymptotes of h.
​√ ​x​​  ​– 9 ​ 
2

x
3. A function f is defined by  f (x) = ________
​​  ______
   ​​. 
​√ 16 – ​x​​  2​ ​ 
(a) State the domain of  f.
(b) Find the vertical asymptotes of  f.
(c) State the horizontal asymptotes of  f, if they exist.
(d) Find  f –1 and determine the range of  f –1, if it exists.

374

Pre-Calculus and Calculus.indb 374 13/7/2017 5:57:30 PM


Chapter 12 • Limits and Continuity

12.4 Continuity
Continuity, in layman terms, means smooth transition from something to
another. Continuity in function means the motion of the function from one
point to another is unbroken. Intuitively, we can imagine drawing such
continuous function using a pen without having to lift the pen. Consider
the following plots.

a x a x

(i) (ii)

b
a x a x

(iii) (iv)
Figure 12j
Consider Figures 12j(i) and 12j(iv). If we move a pen along the curve
from left to right, we do not have to lift the pen at all. That means the
function is continuous at x = a.

However, in Figure 12j(ii), we have to lift the pen at x = a because the


function is undefined at x = a. Similarly, in Figure 12j(iii), we lift the pen
at x = a because the function shifts from  f (x) = b to  f (x) = c. Thus, these
functions are not continuous at x = a.

Definition 12.1

A function  f (x) is said to be continuous at x = a if all the following


conditions are satisfied:
(i)   f (a) is defined (exists),
(ii) ​​ lim​
x→a
 ​​​   f (x) exists, and
 ​​​   f (x) =  f (a).
(iii) ​​ lim​
x→a

375

Pre-Calculus and Calculus.indb 375 13/7/2017 5:57:30 PM


Chapter 12 • Limits and Continuity

Example 14
3x2 + 5
Determine whether  f (x) = ______ is continuous at x = 3.
​​  2x + 1 ​​ 

SOLUTION
We inspect the three conditions:
(a) Is  f (3) defined?
3(3)2 + 5 ___ 32
  f (3) = ________
​​  = ​​  7 ​​ . Hence,   f (x) is defined and the first condition
 ​​ 
2(3) + 1
is satisfied.
(b) Does ​​  lim​ ​​​   f (x) exist?
x→3
3x2 + 5 ___ 32
​​ lim​ ​​​   f (x) = ​​  lim​ ​​​ ______
​​  2x + 1 ​​ 
= ​​  7 ​​ . Hence, the limit exists and the second
x→3 x→3
condition holds.
(c) Does ​​  lim​ ​​​   f (x) =  f (3)?
x→3

From (a) and (b), the third condition also holds.


3x2 + 5
Hence, we conclude that  f (x) = ______ is continuous at x = 3.
​​  2x + 1 ​​ 

Example 15
x2, x  0
Determine whether  f (x) = is continuous at x = 0.
2x, x < 0

SOLUTION
We inspect the three conditions:
(a) Is  f (0) defined?
  f (0) = (0)2 = 0. Hence,  f (0) is defined and the first condition is
satisfied.
(b) Does ​​  lim​ ​​​   f (x) exist?
x→0
Need to consider one-sided limits because different  f (x) are defined
at both sides of x = 0:
​​  lim​+ ​​​   f (x) = ​​  lim​+ ​​​ x2 = 0
x→0 x→0

​​  lim​– ​​​   f (x) = ​​  lim​– ​​​ 2x = 0


x→0 x→0

Since ​​  lim​ +​​​   f (x) = ​​  lim​ –​​​   f (x) = 0, the limit exists and the second
x→0 x→0
x condition holds.
(c) Does ​​  lim​ ​​​   f (x) =  f (0)?
x→0

From (a) and (b), the third condition also holds.


 ence, we conclude that  f (x) is continuous at x = 0. This can be shown
H
Figure 12k from the graph of  f (x) as seen in Figure 12k.

376

Pre-Calculus and Calculus.indb 376 13/7/2017 5:57:30 PM


Chapter 12 • Limits and Continuity

Example 16
x + 2, x  1
Determine whether  f (x) = is continuous at x = 1.
x, x < 1

SOLUTION
We inspect the three conditions:
(a) Is  f (1) defined?
  f (1) = 1 + 2 = 3. Hence,  f (1) is defined and the first condition is
satisfied.
(b) Does ​​  lim​ ​​​   f (x) exist?
x→1

Need to consider one-sided limits:


​​  lim​+ ​​​   f (x) = ​​  lim​+ ​​​ (x + 2) = 3
x→1 x→1 3
​​  lim​– ​​​   f (x) = ​​  lim​– ​​​ x = 1
x→1 x→1
1
Since ​​  lim​ +​​​   f (x) ≠ ​​ lim​ –​​​   f (x), the limit does not exist. Hence, the x
x→1 x→1
second condition does not hold.
Hence, we conclude that  f (x) is not continuous at x = 1. This can be
shown from the graph of  f (x) as seen in Figure 12l. Figure 12l

Example 17
ex, x ≠ ln 2
Determine whether  f (x) = is continuous at x = ln 2.
3, x = ln 2

SOLUTION
We inspect the three conditions:
(a) Is  f (ln 2) defined?
  f (ln 2) = 3. Hence,  f (ln 2) is defined and the first condition is
satisfied.
(b) Does ​​  lim​ ​​​   f (x) exist?
x → ln 2
Need to consider one-sided limits:
​​  lim​ +​​​   f (x) = ​​  lim​ +​​​ ex = eln 2 = 2
x → ln 2 x → ln 2

​​  lim​ –​​​   f (x) = ​​  lim​ –​​​ ex = eln 2 = 2


x → ln 2 x → ln 2

Since ​​  lim​ +​​​   f (x) = ​​  lim​ –​​​   f (x), the limit exists and the second
x → ln 2 x → 2
condition holds.
3
(c) Does ​​  lim​ ​​​   f (x) =  f (ln 2)?
x → ln 2
From (a) and (b), it is clear that ​​  lim​ ​​​   f (x) ≠  f (ln 2). Thus, the third 1
x → ln 2
condition is not satisfied. x
ln 2
 ince the third condition is not satisfied, we conclude that  f (x) is not
S
continuous at x = ln 2. This is true if we observe the graph of  f (x) as Figure 12m
seen in Figure 12m.

377

Pre-Calculus and Calculus.indb 377 13/7/2017 5:57:30 PM


Chapter 12 • Limits and Continuity

Definition 12.2

A function  f (x) is said to be continuous in an open interval (a, b) if the


function  f (x) is continuous at every point between a and b.

Example 18
5x + 2   
Determine whether  f (x) = ______
​  ​is continuous in the intervals
x–2
(–2, 0) and (1, 3).

SOLUTION
The function  f (x) = ​ 5x + 2  
______
​ is defined and continuous for all x  
x–2
except at x = 2, that is, Df  = {x: x  , x ≠ 2} = (–∞, 2)  (2, ∞).
Consider the interval (–2, 0). This interval does not contain x = 2.
Hence, we say that  f (x) is continuous in this interval.
However, the interval (1, 3) contains x = 2 where  f (x) is undefined.
Hence, we say that  f (x) is not continuous in this interval.

Example 19
_____
Determine whether  f (x) = √
​​  x + 2 ​​ is continuous in the interval (–3, 0).
SOLUTION _____
The domain of  f (x) = ​​√ x + 2 ​​ is Df  = [–2, ∞).
However, (–3, 0) ⊄ [–2, ∞). _____
Hence, we conclude that  f (x) = √ ​​  x + 2 ​​ is not continuous in the interval
(–3, 0).

Definition 12.3

A function  f (x) is continuous in a closed interval [a, b] if the following


conditions are satisfied:
(i)   f (x) is continuous in the open interval (a, b),
(ii) ​​  lim​+ ​​​   f (x) =  f (a), and
x→a
(iii) ​​  lim​– ​​​   f (x) =  f (b).
x→b

Example 20
______
​​  x2 – 25 ​​ 
Determine whether  f (x) = √ is continuous in the closed interval
[–10, –5].

SOLUTION ______
​​  x2 – 25 ​​ 
The domain of  f (x) = √ is Df  = (–∞, –5]  [5, ∞).
The interval [–10, –5]  Df .
Now, we investigate whether the three conditions for  f (x) to be
continuous in the closed interval [–10, –5] are satisfied.
378

Pre-Calculus and Calculus.indb 378 13/7/2017 5:57:30 PM


Chapter 12 • Limits and Continuity

______
​​  x2 – 25 ​​ 
(a) Is  f (x) = √ continuous in the______ open interval (–10, –5)?
Let c  (–10, –5), then x​​ lim​ →c
 ​​​   f (x) = √
​​  c2 – 25 ​​ 
=  f (c) for all c between
–10 and –5. Hence, the first condition is satisfied.
(b) Does ​​  lim​ +​​​  f (x) =  f (–10)?
x → –10 ______ ___
We have ​​  lim​ +​​​  f (x) = ​​  lim​ +​​​ ​​√ x2 – 25 ​​ 
= ​​√ 75 ​​ =   f (–10). Hence, the
x → –10 x → –10
second condition is satisfied.
(c) Does ​​  lim​ –​​​  f (x) =  f (–5)?
x → –5 ______
We have ​​  lim​ –​​​  f (x) = ​​ lim​ –​​​ ​​√ x2 – 25 ​​ 
= 0 =  f (5). Hence, the third
x → –5 x → –5
condition is satisfied.
______
Hence, we conclude that  f (x) = √
​​  x2 – 25 ​​
 is continuous in the closed
interval [–10, –5].

Exercise 12.5
1. Determine whether the following functions are continuous at the given points.
f (x) = 2x_____
(a)   3
– 5, at x = 2
f (x) = √
(b)   ​​  x2 – 3 ​​, at x = 4
x–2
f (x) = _____
(c)   , at x = 3
​​  x – 3 ​​ 

x2, x < 3
f (x) =
(d)    , at x = 3
1 – x2, x  3
x2 – 1, x  1
f (x) =
(e)    , at x = 1
1 – x, x < 1
x + 2, x  –1
f (x) = x2, –1 < x < 1 , at x = –1 and 1
(f)  
0, x  1

2. Given that  f (x) = |x + 2|, sketch the graph and discuss the continuity of the function.

x2 + k, x < 1
3. Given that  f (x) = is continuous in the interval (–∞, ∞), find k.
x3, x  1

4. A function f is defined as follows:


1, x  –1
 f (x) = 1 – kx, –1 < x < 1
(x –1), x  1
(a) Given that f is continuous at x = 1, find the value of constant k.
(b) Determine whether f is continuous at x = –1.
(c) Sketch the graph of y =  f (x).

379

Pre-Calculus and Calculus.indb 379 13/7/2017 5:57:30 PM


Chapter 12 • Limits and Continuity

5. A function is defined as follows:


4x + 1
______
​​  x – 2 ​​,  x < 2
 f (x) =
3x + 2, x  2
5
Determine whether  f  is continuous at x = __
​​  3 ​​ .
______
6. (a) Sketch the graph of  f (x) = √
​​  3x – x2 ​​ 
and determine the continuity of  f  in the interval [0, 3].
______
​√ 3x – x2 ​ 
_______
(b) State the set of values of x such that  f (x) = ​​  x + 2 ​​ 

is not continuous.

_____
7. (a) Sketch the graph of  f (x) = √
​​  x2 – 9 ​​ 
and state the interval where f is continuous.
(b) Function g is defined as follows:
x 2
– k, –3 < x < 3
_____
g (x) =
​​√ x2 – 9 ​​, 
others
Given that g is continuous for all values of x, find k.

8. Function f is defined as follows:


ex, x  0
 f (x) =
1 – 4x2, x < 0
(a) Find ​​  lim​ ​​​   f (x).
x→0
(b) Determine whether f is continuous at x = 0. Explain your answer.
(c) Sketch the graph of y =  f (x) for –2  x  2.

9. (a) State the conditions for a function f to be continuous at x = a.


(b) A function f is defined as follows:
x2 – 4
______
, x ≠ 2
​​  x – 2 ​​ 
 f (x) =
3, x=2
(i) Sketch the graph.
(ii) Discuss the continuity of f at x = 2.

10. A function g is defined as follows:


x, x < –2
g(x) = px + q, –2  x  2
x
​​ __
2  ​​ – 1, x > 2
Determine the values of the constants p and q such that g is continuous in the interval (–∞, ∞).

380

Pre-Calculus and Calculus.indb 380 13/7/2017 5:57:30 PM


Limits and Derivatives
Chapter 13:

Differentiation

Historical Note

Both Isaac Newton (1643–1727) and Gottfried Wilhelm von


Leibniz (1646–1716) were instrumental in the discovery and
development of calculus. Newton used mathematics as a tool to
explain the physical world, and developed concepts of differential
and integral calculus from his conceptual understanding of physics
and geometry. Leibniz, who was also a prominent contributor in
the history of philosophy, developed a system of infinitesimal
calculus in the mid 1670s, and his way of writing calculus and
notation is still used today. The calculus controversy refers to
the bitter dispute between Newton and Leibniz, over who first
invented calculus, that lasted to their deaths. Even after Leibniz’s
death, the debate carried on for many years.

Pre-Calculus and Calculus.indb 381 13/7/2017 5:57:31 PM


Chapter 13 • Differentiation

13 Differentiation
13.1 Geometric Interpretation of Derivatives
13.2 Definition of Derivative Functions
13.3 The Derivative as a Function
13.4 Differentiation Rules
13.5 Second and Higher-order Derivatives
13.6 Implicit Differentiation
13.7 Derivatives of Exponential and Logarithmic Functions
13.8 Derivatives of Trigonometric Functions
13.9 Derivatives of Inverse Trigonometric Functions
13.10 Linearization and Differentials
13.11 L’Hôpital’s Rule

13.1 Geometric Interpretation of


Derivatives
In order to understand the meaning and concept of derivative or
differentiation of a function, we first need to look at the geometric
interpretation through a graphical illustration.

Let A (x0,  f (x0)) and B (x1,  f (x)) be two points on a graph of a continuous
function y =  f (x).
y

y = f (x)

f (x1) B

A
f (x0)
x
x0 h x1

If a straight line is drawn passing through A and B, then the gradient of the
line can be written as
f (x) – f (x)
_________
​​  x – x  ​​. 

1 0

If we denote , the gradient of the line can be presented as


f (x0 + h) – f (x0)
______________
​​     
h ​​.

382

Pre-Calculus and Calculus.indb 382 13/7/2017 5:57:31 PM


Chapter 13 • Differentiation

Now, let move closer and closer to , that is, h = x1 – x0 approaching


0, then let B get closer and closer to A along the curve of y =  f (x). If B is
sufficiently close to A, then the straight line AB will approach the line AC.
The line AC is called the tangent line at A or the gradient of the function
y =  f (x) at (x0,  f (x0)).
y

y = f (x)

f (x1) B
C
A
f (x0)
x
x0 h x1

The tangent line or gradient at A can then be obtained by taking


f (x0 + h) – f (x0)
_______________
​​  lim​ ​​​   
​​    ​​. 
h→0 h
The process of getting the tangent line or gradient at the point A on the
graph is called differentiation from first principles or differentiation
using definition.

13.2 Definition of Derivative Functions


Derivative or differentiation is denoted by the notation  f '(x) (read as
dy
‘f  prime x’) or ___
​​    ​​ if given y =  f (x) and the formal definition is given below.
dx
Definition 13.1

The derivative (or differentiation) of a function f (x) at a point x is


defined as f (x + h) –  f (x)
 f '(x) = ​​  lim​ ​​​ _____________
  
 ​​    ​​, 
h→0 h
provided the limit exists.

Using the definition, we can find the derivative of any function  f (x) by the
following steps:
Step 1: Find  f (x + h).
Step 2: Find  f (x + h) –  f (x).
f (x + h) – f (x)
____________
Step 3: Find   
​​    ​​. 
h
f (x + h) – f (x)
____________
Step 4: Find ​​  lim​ ​​​   
​​    ​​. 
h→0 h

383

Pre-Calculus and Calculus.indb 383 13/7/2017 5:57:31 PM


Chapter 13 • Differentiation

If x = a is in the domain of a function f, then the derivative of  f (x) at x = a


can be defined as follows.

Definition 13.2

(a) The derivative (or differentiation) of a function  f (x) at a point x = a,


written as  f '(a) is defined as
f (a + h) – f (a)
 f '(a) = ​​  lim​ ​​​ ____________
  
​​    ​​, 
h→0 h
provided the limit exists.
(b) Alternatively, if we write h = x – a, then
f (x) – f (a)
 f '(a) = ​​  lim​ ​​​ _________
​​  x – a    ​​, 
h→a
provided the limit exists.

Example 1
Find the derivatives of the following functions from first principles.
(a)  f (x) = 5x (b)  f (x) = 3x2 + 2
SOLUTION
(a)  f (x) = 5x
  f (x + h) = 5(x + h)
  f (x + h) –  f (x) = 5(x + h) – 5x
= 5x + 5h – 5x
= 5h
f____________
(x + h) – f (x) ___ 5h
​​      ​​  = ​​   ​​ = 5
h h
f (x + h) – f (x)
____________
Hence,  f '(x) = ​​  lim​ ​​​    ​​    ​​ 
h→0 h
= ​​  lim​ ​​​ 5
h→0
=5
(b)  f (x) = 3x + 22

  f (x + h) = 3(x + h)2 + 2


  f (x + h) –  f (x) = 3(x + h)2 + 2 – (3x2 + 2)
f (x + h) – f (x) ____________________
____________ 3(x + h)2 + 2 – (3x2 + 2)
​​      ​​ 
= ​​       
​​
h h
3(x2 + 2hx + h2) + 2 – 3x2 – 2
= ​​ ________________________
        ​​
h
3x2 + 6hx + 3h2 + 2 – 3x2 – 2
_______________________
=    ​​      ​​
h
6hx + 3h2
= ________
​​      ​​ 
h
h(6x + 3h)
= ​​ _________    ​​

h
= 6x + 3h
Hence,  f '(x) = ​​  lim​ ​​​ (6x + 3h) = 6x. Substitute h = 0
h→0

384

Pre-Calculus and Calculus.indb 384 13/7/2017 5:57:31 PM


Chapter 13 • Differentiation

Example 2
Find the derivatives of the following functions using the definition.
1   ​​ __
(a)  f (x) = ___
​​  2x (b)  f (x) = 3​​√ x  ​​

SOLUTION
1
(a)  f (x) = ​​ ___
2x   ​​
f (x + h) = ___
  ​​  1     ​​
2(x + h)
​  1    
___
​ – ___
​  2x1   ​
2(x + h)
f '(x) = ​​  lim​ ​​​ ​​ ___________
       ​​

h→0 h
x – (x + h)
_________
​   ​ 
2x(x + h)
_________
= ​​  lim​ ​​​ ​​      ​​  Taking the lowest common denominator
h→0 h
–h
= ​​  lim​ ​​​ _________
​​     ​​ 
h → 0 2hx(x + h)

​​  –1   ​​ 


= ​​  lim​ ​​​ ________
h → 0 2x(x + h)

= –​​ ___1   ​​  Substitute h = 0


2x2
__
(b)  f (x) = 3​​√ x  ​​ __
  f (x + h) = 3​​√ x + h  __

​​
__
3​√ x + h 

____________ ​– 3​√ x  ​

  f '(x) = ​​  lim​ ​​​ ​​      ​​ 
h→0 __ h _____

__ __ Multiply by the
= 3 ​​  lim​ (
​  x  ​) ​​√_____
​  x + h  

____________ ​– √ ​  x  ​ __________x + h  
​√ _____ ​+ √ x  ​ conjugate of
​  __

​​​ ​​  ​      
​ × ​    ​ ​​
h→0 h ​√ x + h  
​+ √ __
 x + h   ​​  x  ​​
​​ – √
(x + h) – x
= 3 ​​  lim​ ​​​ _____________
​​     _____  
__  ​​
h → 0 h(​√ x + h   ​+ √ ​  x  )​
h
= 3 ​​  lim​ ​​​ _____________
​​     _____   __  ​​
h → 0 h(​√ x + h   ​+ √ ​  x  )​
= 3 ​​  lim​ ​​​ __________
​​  _____ 1  __ ​​ 
h → 0 ​√ x + h   ​+ √​  x  ​
3__
____
= ​​  2​   x   ​​​  Substitute h = 0

Example 3
Find the derivatives of
_____ f (x + h) – f (x)
____________
(a)  f (x) = √  ​​ x + 1 ​​ at x = 3 using  f '(x) = ​​  lim​ ​​​    ​​    ​​, 
h→0 h
f (a + h) – f (a)
​​  2 2 ​​ at x = 2 using  f '(a) = ​​  lim​ ​​​ _____________
(b)  f (x) = ___ ​​      ​​ 
3x h→0 h
f (x) – f (a)
 ​​​ _________
or  f '(a) = ​​ xlim​
→a
​​  x – a    ​​. 

385

Pre-Calculus and Calculus.indb 385 13/7/2017 5:57:31 PM


Chapter 13 • Differentiation

SOLUTION
_____
(a)  f (x) = √ 
​​ x + 1 ​​ at x = 3
f (x + h) – f (x)
____________
Using  f '(x) = ​​  lim​ ​​​    ​​    ​​, 
h→0
________
h _____
​  x + h + 1 ​ 

________________ –√ ​  x + 1 ​ 
 f '(x)= ​​  lim​ ​​​   
​​    ​​ 
h→0
________
h _____ ________ _____
= ​​  lim​ ​​​ ( × ​    ​)​​  
​  x + h + 1 ​ 

________________ –√ ​  x + 1 ​  ________________
 x + h + 1 ​ 
​√________ –√  x + 1 ​ 
​ _____
​​  ​      ​ 
h→0 h ​√ x + h + 1 ​ ​  x + 1 ​ 
–√
+ h + 1) – (x _____
(x ________
___________________ + 1)
= ​​  lim​ ​​​ ​​       ​​
h → 0 h ​ (​√ x + h + 1 ​  +√ ​  x + 1 ​) ​ 
h
= ​​  lim​ ​​​ ___________________
​​     ________   _____  ​​
h → 0 h ​ (​√ x + h + 1 ​  +√ ​  x + 1 ​) ​ 
= ​​  lim​ ​​​ ​​ ________________
   ________ 1  _____ ​​
h → 0 ​√ x + h + 1 ​  +√ ​  x + 1 ​ 
= ​​ _______ 1   ​​ 
_____
2​√ x + 1 ​ 
Hence,  f '(3) = _______
​​  _____ 1   ​​  ​​  1 ​​ .
= __
2​√ 3 + 1 ​  4

​​  2 2 ​​ at x = 2
(b)  f (x) = ___
3x
f (a + h) – f (a)
Using  f '(a) = ​​  lim​ ​​​ _____________
​​      ,​​ 
h→0 h
f (2 + h) – f (2)
 f '(2) = ​​  lim​ ​​​ _____________
​​      ​​ 
h→0 h
________
​  2   ​  ​  1 ​ 
– __
___________ 3(2 + h)2 6
= ​​  lim​ ​​​ ​​     
​​

h→0 h
4 – (2 + h)2
__________
​   ​ 

__________ 6(2 + h)2
= ​​  lim​ ​​​ ​​      ​​ 
h→0 h
4 – (4 + 4h + h2)
______________
  
​      ​
______________ 6(4 + 4h + h2)
= ​​  lim​ ​​​   
​​    ​​

h→0 h
–h(4 + h)
= ​​  lim​ ​​​ _____________
​​       2  ​​
h → 0 6h(4 + 4h + h )

–(4 + h)
= ​​  lim​ ​​​ ____________
​​       2  ​​
h → 0 6(4 + 4h + h )

4
= –​​ ___
24  ​​ 
= –​​ __1  ​​
6

386

Pre-Calculus and Calculus.indb 386 13/7/2017 5:57:31 PM


Chapter 13 • Differentiation

OR
f (x) – f (a)
 ​​​ _________
Using  f '(a) = x​​  lim​
→a
​​  x – a    ​​, 
f (x) – f (2)
 f '(2) = ​​  lim​ ​​​ _________
​​  x – 2    ​​ 
x→2

​  2 2 ​ – _____
___
​  2   ​ 
3x 3(22)
_________
= ​​  lim​ ​​​ ​​ x–2  ​​   
x→2
8 – 2x2
______
​   ​  

12x2
= ​​  lim​ ​​​ ______
​​  x–2  ​​ 

x→2

2(4 – x2)
= ​​  lim​ ​​​ _________
​​  2   ​​ 
x → 2 12x (x – 2)

2(2 – x)(2 + x)
= ​​  lim​ ​​​ ____________
​​     ​​ 
x→2 12x2(x – 2)
–2(2 + x)
= ​​  lim​ ​​​ ________
​​   ​​ 

x→2 12x2
= –​​ __1
6 ​​ 

Exercise 13.2
1. Find the derivatives of the following functions from first principles.
f (x) = 2x – 5
(a)  
f (x) = x2 + 4
(b)  

2. Differentiate the _____


following functions using the definition of the derivative.
(a)  f (x) = 2​​√ x + 1 ​​ 
(b)  f (x) = x3 + 5
(c)   ​​  1x ​​
f (x) = __
(d)   ​​  1___
f (x) = ____    ​​ 
​  2x  ​

3. Find  f '(a) for the following  f (x), given the values of a.


f (x) = x + 3, given a = 2
(a)  
f (x) = 2x2 – 3, given a = 3
(b)  
___
f (x) = 2​​√ 3x ​​,  given a = 2
(c)  
f (x) = x2 – 6x + 1, given a = 1
(d)  
1  ​​,  given a = –1
f (x) = –​​ __
(e)  
x2

387

Pre-Calculus and Calculus.indb 387 13/7/2017 5:57:31 PM


Chapter 13 • Differentiation

13.3 The Derivative as a Function


Below is an illustration of how the gradient of a curve can be obtained.
y

Q (x + ∆x,
y + ∆y)

∆y
P (x, y)
∆x
x

Take a general point P on the curve with the coordinates (x, y) for which
the gradient needs to be found. Draw a line through the point for which the
gradient needs to be found which also passes through a nearby point Q on
the curve. The x-coordinate of point Q should be slightly larger than the
x-coordinate of point P.
The x-coordinate of Q would be higher by ∆x (∆x is symbolically used to
represent a slight change in the x-value and do not have any particular
quantity). The x-coordinate of point Q is x + ∆x. The y-value of point Q
would also be slightly changed by ∆y and the y-coordinate of Q is y + ∆y.
The coordinates of point Q are then (x + ∆x, y + ∆y).
Since point Q is on the curve y = x2, y + ∆y = (x + ∆x)2.
Expanding (x + ∆x)2, (x + ∆x)2 = x2 + 2x(∆x) + (∆x)2.
Hence, y + ∆y = x2 + 2x(∆x) + (∆x)2.
Since y = x2, x2 + ∆y = x2 + 2x(∆x) + (∆x)2.
Hence, ∆y = 2x(∆x) + (∆x)2.
Dividing both sides of the expression by ∆x, the approximate gradient,
___ ∆y
​​  ∆x  ​​ is obtained.
___ ∆y
​​  ∆x  ​​ = 2x + ∆x

Since ∆x is a very small value when compared to 2x, the value of 2x + ∆x


tends to 2x when ∆x is shrunk to be a value which is very small and very
Note close to 0. This is known as taking the limit of ∆x to 0.
By definition, taking ∆x
When ∆x is taken closer and closer to its limiting value of 0,
close to 0,
∆y dy ∆y
___ dy
___
​​  ∆x  ​​ = ___
​​    ​​ ​​  ∆x  ​​ = ___
​​   ​​ and 2x + ∆x = 2x (by close approximation).
dx dx
dy
Hence, ___
​​   ​​ = 2x (the gradient function of y = x2).
dx
The gradient of the curve y = x2 could thereafter be determined for any x
value.
388

Pre-Calculus and Calculus.indb 388 13/7/2017 5:57:32 PM


Chapter 13 • Differentiation

For example, when x = 2, the gradient of the curve y = x2 is found using the
dy
gradient function as ___
​​    ​​ = 2(2) = 4.
dx
Example 4
Find the derivative of y = 3x.

SOLUTION
Let point Q be (x + ∆x, y + ∆y).
y = 3x
y + ∆y = 3(x + ∆x)
y + ∆y = 3x + 3(∆x)
Since y = 3x,
3x + ∆y = 3x + 3(∆x)
∆y = 3(∆x)
Dividing both sides of the expression by ∆x,
___ ∆y
​​  ∆x  ​​ = 3
∆y dy
When ∆x is taken to its limiting value of 0, ___
​​ ∆x  ​​ = ___
​​   .​​
dx
dy
Hence, ___
​​   ​​ = 3.
dx

13.4 Differentiation Rules


13.4.1 Differentiation of General Functions
13.4.2 Sum and Difference Rule
13.4.3 Product Rule
13.4.4 Quotient Rule
13.4.5 Chain Rule
13.4.6 Power Rule

Instead of using definitions or first principles to find the derivative of a


function, we can also use some techniques to differentiate functions easily.
These techniques are very simple to use especially when involving difficult
and complicated functions.

13.4.1  Differentiation of General Functions

Theorem 13.1

If  f (x) = k for all values of x, where k is a constant, then  f '(x) = 0.

For example,
if  f (x) = 3, then  f '(x) = 0.

389

Pre-Calculus and Calculus.indb 389 13/7/2017 5:57:32 PM


Chapter 13 • Differentiation

Theorem 13.2

If  f (x) = xn where n is a real number, then  f '(x) = nxn – 1.

For example,
if  f (x) = x5, then  f '(x) = 5x4.

Theorem 13.3

If  f (x) = kxn where k is a constant and n is a real number,


then  f '(x) = knxn – 1.

For example,
if  f (x) = 3x8, then  f '(x) = 3(8)x7 = 24x7.

Example 5
Find the derivatives of the following functions.
(a)  f (x) = 6 (c)  f (x) = __ ​​  12  ​​ 
x
3 __
(b) y = 5x9 (d) y = ​​√ 
  x  ​​

SOLUTION
(a)  f (x) = 6 (c)  f (x) = __ ​​  12  ​​ = x–2
x
  f '(x) = 0  f '(x) = –2x–2 – 1 = –2x–3 = –​​ __ 2  ​​ 
x3
3 __ ​  31 ​ 
__
(b) y = 5x (d)
9
y = ​​√    x  ​​ = ​​x​​  ​​

dy dy ​  1 ​  – 1 2 ​ 
​​   ​​ = ​​  31 ​​​​ x​​  3 ​​ = __ ​​  31 ​​​​ x​​  3​​ = ___
​​  1 __2 ​​ 
__
–​ __
___​​   ​​ = 9(5)x9 – 1 = 45x8 ___ __
dx dx 3​x​​  ​ 3 ​ ​

13.4.2  Sum and Difference Rule

Theorem 13.4

If  f (x) = u(x) ± v(x), where u(x) and v(x) are functions which are
differentiable with respect to x, then
 f '(x) = u'(x) ± v'(x).
We can also write
___d d d
​​    ​​[ u(x) ± v(x)] = ___
​​    ​​[ u(x)] ± ___
​​    ​​[ v(x)].
dx dx dx

390

Pre-Calculus and Calculus.indb 390 13/7/2017 5:57:32 PM


Chapter 13 • Differentiation

Example 6
Differentiate the following functions with respect to x.
​  4 ​ 
​​  3​1 __ ​​  2  ​​ + 1
__
(a)  f (x) = x–3 – 2 (b) y = 3​​x​​  5​​ – ____  ​​ + __
√ x  ​ x2

SOLUTION
(a)  f (x) = x–3 – 2
d d 3
  f '(x) = ___ ​​     (​​ x–3) – ​​ ___   (​​ 2) = –3x–4 – 0 = –​​ __4  ​​ 
dx dx x
​  4 ​ 
​​  3​1 __ ​​  2  ​​ + 1
__
(b) y = 3​​x​​  5​​ – ____  ​​ + __
√ x  ​ x2

​​   [  ​​​​  __ ​   ​ (x​)​​  ​]​​  + ​​     (​​ 2x–2) + ___


dy d d 1 –​ __21 ​  ___ d d
___​​   ​​ = ___ ​​    ( ​​ 3​​x​​  ​ 5 ​ ​​) – ___
4
__
​​     (​​ 1)
dx dx dx 3 dx dx
= 3​​ (__ ​  54 ​ )​​​​x  ​​  –​ 5 ​ ​​ – __ ​​  31( 2 ​ )​​​​x  ​​  ​​ + 2(–2)x + 0
1
__
 ​​​​   –​ __ 1 –​ __23 ​  –3

3
= ​​ ___ 12 ​​ x​​  –​ __51 ​ ​​ + __ ​​  61 ​​​​ x​​  2​​ – 4x–3
–​ __ ​ 
5
= ___​​  12__1  ​​ + ___ ​​  1 __3 ​​ – __ ​​  43  ​​ 
​  5 ​  ​  2 ​  x
5​x​​  ​ 6​x​​  ​

13.4.3  Product Rule

Theorem 13.5
Note
If  f (x) = u(x)v(x), where u(x) and v(x) are functions which are
differentiable with respect to x, then Sometimes, this rule is
written as:
 f '(x) = u'(x)v(x) + v'(x)u(x). dy
y = uv  ___
​​    ​​ = udv + vdu
We can also write dx
___d d d
​​    ​​  [u(x)v(x)] = v(x)​​ ___   ​​[u(x)] + u(x)​​ ___  ​​ [v(x)].
dx dx dx

Example 7
Note
Differentiate the following functions with respect to x using the product ___d
​​    ​​  [u(x)v(x)]
dx
rule. d d
≠ ___
​​    ​​  [u(x)] × ___
​​     ​​  [v(x)]
(a)  f (x) = (3x2 + 7)(6 __– 5x) dx dx
(b)  f (x) = (5x3 + 2)(​​√ x  ​​ + 1)
SOLUTION
(a)  f (x) = (3x2 + 7)(6 – 5x)
  f '(x) =
 (3x2 + 7)(–5) + (6 – 5x)(6x)
= –15x2 – 35 + 36x – 30x2
= –45x2 + 36x – 35
__
(b)  f (x) = (5x3 + 2)(​​√ x  ​​ + 1)
d __ __ d
  f '(x) = (5x3 + 2)​​ ___   (​​ √ ​​  x  ​​+ 1) + (​​√ x  ​​ + 1)​​ ___   (​​ ​​5x3 + 2)
dx dx
= (5x3 + 2)​​ (____  ​ ​​  + (​​√ x  ​​ + 1)(15x2)
√ x  ​)
__
​  2​1 __
5 __​ 25 ​  ___ 1__  ​​ + 15​​x​​  __​ 25 ​ ​​ + 15x2
= ​​ __ 2  ​​​​x ​​  ​​ + ​​  ​  x  ​

5
35 __
1__
= ___​​  2 ​​​​ x​​  2​​ + ___
​   
 ​
​​  √
​  x    ​​​ + 15x
2
391

Pre-Calculus and Calculus.indb 391 13/7/2017 5:57:32 PM


Chapter 13 • Differentiation

13.4.4  Quotient Rule


Note
Sometimes, this rule is Theorem 13.6
written as:
dy vdu – udv u(x)
y = __
u
​​  v ​​  ___
​​    ​​ = ________
​​   ​​ 
  If  f (x) = ​​ ____ ​​ 
, where u(x) and v(x) are functions which are differentiable
dx v2 v(x)
with respect to x, and v(x) ≠ 0 then
v(x)u'(x) + u(x)v'(x)
 f '(x) = ________________
  
​​   ​​. 
Note [v(x)]2
We can also write
d d d
v(x)​ ___  ​[ u(x)] – u(x)​ ___  ​[ v(x)]
]​​  =    
___
 ​ ​​ ≠ ​​ 
​    ​[ u(x)]
​​   [ ​​​​   ​ 
dx v(x) ] ___ ​​   [ ​​​​   ​ 
___d ____ u(x) dx
_______ ___d ____ u(x) dx
______________________ dx
d
 ​​   ​  ​​     2 ​​.
​     ​[v(x)] dx v(x) [v(x)]
dx

Example 8
Differentiate the following functions with respect to x using the quotient
rule.
2x2 – 3
(a)  f (x) = ______
​​  2x + 3 ​​  
(b)  f (x) = (1 + x)–1(x – 1)

SOLUTION
2x2 – 3
(a)  f (x) = ​​ ______
2x + 3 ​​ 
d d
(2x + 3)​ ___   (​ 2x2 – 3) – (2x2 – 3)​ ___   (​ 2x + 3)
dx
_________________________________ dx
  f '(x) =     
​​      ​​
(2x + 3)2
(2x + 3)(4x) – (2x2 – 3)(2)
= ​​ _____________________
       ​​
(2x + 3)2
8x2 + 12x – 4x2 + 6
________________
= ​​        ​​
(2x + 3)2
4x2 + 12x + 6
= ​​ ___________
    ​​ 
(2x + 3)2
2(2x2 + 6x + 3)
= ​​ _____________
    ​​ 
(2x + 3)2
(b)  f (x) = (1 + x)–1(x – 1)
x–1
= ​​ _____
1 + x   ​​
d d
Note (1 + x)​ ___   (​ x – 1) – (x – 1)​ ___   (​ 1 + x)
dx
____________________________ dx
  f '(x) =     ​​      ​​
Alternatively, we can use (1 + x)2
product rule to differentiate (1 + x)(1) – (x – 1)(1)
__________________
functions in Example 8. =    ​​      ​​
(1 + x)2
​​  2  2 ​​ 
= _______
(1 + x)

392

Pre-Calculus and Calculus.indb 392 13/7/2017 5:57:32 PM


Chapter 13 • Differentiation

Example 9
2x2 – 3
Differentiate  f (x) = ______
​​  3x – 2 ​​ 
using the product rule.

SOLUTION
2x2 – 3
 f (x) = ​​ ______
3x – 2 ​​ 
= (2x2 – 3)(3x – 2)–1
d d
 f '(x) = (2x2 – 3)​​ ___   (​​ 3x – 2)–1 + (3x – 2)–1___
​​     (​​ 2x2 – 3)
dx dx
–3(2x2 – 3) ______ 4x
= ​​ _________  ​​ 
 + ​​  3x – 2 ​​ 
(3x – 2)2
–3(2x2 – 3) + 4x(3x – 2)
____________________
=    ​​      ​​
(3x – 2)2
6x2 – 8x + 9
= ​​ __________  ​​ 

(3x – 2)2

13.4.5  Chain Rule

Theorem 13.7

If  f (x) and g(x) are two differentiable functions at the points g(x)
and x respectively, then the composite function  f   g =  f (g(x)) is also
differentiable at the point x.
If y =  f (g(x)) =  f (u) and u = g(x), then
dy
___ dy du
​​   ​​ = ___
​​   ​​ × ___
​​    ​​
dx du dx

Consider the following example. Let y = (2 + 5x)10, (note that y is a


composite function).
We can write y =  f (u) = u10, where u = g(x) = 2 + 5x.
Differentiating y with respect to u and u with respect to x, we have
___dy du
​​   ​​ = 10u9 and ___
​​    ​​ = 5.
du dx
Using Theorem 13.7, we have
___dy dy ___ du
​​    ​​ = ___
​​   ​​ × ​​    ​​
dx du dx
= 10u9 × 5
= 10(2 + 5x)9 × 5
= 50(2 + 5x)9

393

Pre-Calculus and Calculus.indb 393 13/7/2017 5:57:32 PM


Chapter 13 • Differentiation

Example 10
dy
Find ___
​​   ​​of the following functions using the chain rule.
dx
(a) y = (x + 3)6 (c) ​​  2   ​​ 
y = ________
​  1 ​ 
__
(3x2 – 1​)​​  5​
______
(b) y = (4x5 – 3x2 + 2x – 1)3 (d)
y = √ 
​​ 2x2 + 1 ​​ 

SOLUTION
(a) y = (x + 3)6
Let u = x + 3, then y = u6.
du dy
Hence, ___ ​​    ​​ = 1 and ___
​​   ​​ = 6u5.
dx du
Using chain rule,
dy dy ___ du
​​ ___ ​​ = ___
​​   ​​ × ​​    ​​
dx du dx
= 6u5 × 1
= 6(x + 3)5
(b) y = (4x5 – 3x2 + 2x – 1)3
Let u = 4x5 – 3x2 + 2x – 1, then y = u3.
du dy
Hence, ___ ​​    ​​ = 20x4 – 6x + 2 and ___
​​   ​​ = 3u2.
dx du
Using chain rule,
dy dy ___ du
​​ ___ ​​ = ___
​​   ​​ × ​​    ​​
dx du dx
= 3u2(20x4 – 6x + 2)
= 3(4x5 – 3x2 + 2x – 1)2(20x4 – 6x + 2)
(c) y = ________
​​  2   ​​ 
​  1 ​ 
__
(3x2 – 1​)​​  5​
1 ​ 
–​ __
Let u = 3x2 – 1, then y = 2​​u​​  5​​.
du dy 2 ​​​​ u​​  –​ __56 ​ ​​.
Hence, ___ ​​    ​​ = 6x and ___
​​   ​​ = –​​ __
5
dx du
dy dy du
​​ ___ ​​ = ___
​​   ​​ × ___ ​​    ​​
dx du dx
= –​​ ____2   ​​ × 6x
__6
​   ​ 
5​u​​  5​
12x
= –​​ _________   __6 ​​ 
​   ​ 
5(3x – 1​)​​  5​
2
______
(d) y = √  ​​ 2x2 + 1 ​​ 
​  1 ​ 
__
Let u = 2x2 + 1, then y = u​​ ​​ 2​​.
du dy 1 –​ __21 ​ 
Hence, ___ ​​    ​​ = 4x and ___​​   ​​ = __
​​   ​​​​ u​​  ​​.
dx du 2
dy dy ___ du
​​ ___ ​​ = ___ ​​   ​​ × ​​    ​​
dx du dx
= __ ​​  21 ​​​​ u​​  –​ 2 ​ ​​ × 4x
__1

2x
394 = ________
​​  ______    ​​ 
​√ 2x2 + 1 ​ 

Pre-Calculus and Calculus.indb 394 13/7/2017 5:57:32 PM


Chapter 13 • Differentiation

13.4.6  Power Rule


The power rule is a shorter and simpler version of the chain rule. It is
stated in the following theorem and illustrated in the next example.

Theorem 13.8

If  f (x) is differentiable at the point x and y = [ f (x)]n for any real number
n, then dy
___
​​    ​​ = n[ f (x)]n – 1 f '(x).
dx
We can also write
___d
​​     ​​[  f (x)]n = n[ f (x)]n – 1 f '(x).
dx

We can differentiate the functions in Example 10 using the power rule.

Example 11
Using the power rule, differentiate the following functions.
(a) y = (x + 3)6

(b) y = ________
​​  2   ​​ 
​  1 ​ 
__
(3x – 1​)​​  5​
2

SOLUTION
(a) y = (x + 3)6
dy d
​​ ___ ​​ = 6(x + 3)5​___
​     (​​ x + 3)
dx dx
= 6(x + 3)5
1 ​ 
(b) y = ________
​​  2   ​​  = 2(3x2 – 1​​)​​  5​​
–​ __
​  51 ​ 
__
(3x2 – 1​)​​  ​
dy
​​ ___ ​​ = 2​​ (–​ __ 1 ​  ​​( 3x2 – 1​​)​​  –​ __56 ​ ___
5)
d
​​​​     (​​ 3x2 – 1)
dx dx
2  ​​(3x2 – 1​​)​​  –​ __56 ​ ​​(6x)
= –​​ __
5
12x
_________
= –​​    __6 ​​ 
​   ​ 
5(3x – 1​)​​  5​
2

395

Pre-Calculus and Calculus.indb 395 13/7/2017 5:57:32 PM


Chapter 13 • Differentiation

The chain rule or power rule can also be combined with other rules of
differentiation. The following examples show how these rules are used
together with the product rule and the quotient rule.

Example 12
Differentiate the following functions with respect to x.
(a) y = (8x + 5)4(x3 + 3)12
(3x + 1)5
(b) y = ________
​​   ​​ 
(2 – x)8

SOLUTION
(a) y = (8x + 5)4(x3 + 3)12
dy d d
​​ ___ ​​ = (8x + 5)4(12)(x3 + 3)11​___
​     (​​ x3 + 3) + (x3 + 3)12(4)(8x + 5)3​___
​     (​​ 8x + 5)
dx dx dx
= 12(8x + 5)4(x3 + 3)11(3x2) + 4(x3 + 3)12(8x + 5)3(8)
= 4(8x + 5)3(x3 + 3)11[9x2(8x + 5) + 8(x3 + 3)]
= 4(8x + 5)3(x3 + 3)11(80x3 + 45x2 + 24)
(3x + 1)5
(b) y = ________
​​   ​​ 
(2 – x)8
d d
(2 – x)8___
​     (​ 3x + 1)5 – (3x + 1)2___
dy ________________________________ ​     (​ 2 – x)8
___ dx dx
​​   ​​ =      ​​      ​​
dx (2 – x)16
15(2 – x)8(3x + 1)4 + 8(3x + 1)5(2 – x)7
= _______________________________
​​ 
        ​​
(2 – x)16
15(2 – x)(3x + 1)4 + 8(3x + 1)5
= ​​ _________________________
        ​​
(2 – x)9

Example 13 _____

√ 
x–1 dy
Find the derivative of y = ​​ _____
​  x + 1  . Hence, evaluate ​​ ___ ​​ when x = 2.
 ​ ​​ 
dx
SOLUTION_____

√   ​)​​​    ​​
​  1 ​ 
__

 ​ ​​ = (​​​  ​  x + 1 
x–1
_____ x–1 2
_____
y = ​​ ​  x + 1 

 ​)​​​    ​​​​   (  ​)​​  


1 ​ 
–​ __
dy
​​  1(
___ x – 1 2___ d _____ x–1
​​   ​​ = __  ​​​​​   _____
​      ​​​​  ​   
dx 2 x + 1 dx x + 1
From Example 8(b),
​​   (  ​ ​​  = _______
dx x + 1 ) (1 + x)2
d _____ x–1
___
  ​​​​  ​    ​​  2   ​​ 
Therefore, _____
 ​)​​​    ​​​​  2  2 ​​  √ _____
1 ​ 
–​ __
dy 1 _____
​​  2(
___ x – 1 2_______ x + 1 _______ 1 _____1 
​​   ​​ = __  ​​​​​   ​  x + 1  = ​​ ​  x – 1 ​ ​​ ​​  
     ​​  = ____________
​​      ​​
dx (1 + x) (1 + x) ​√ x – 1 ​​ 
2 3
__
(1 + x)​​  ​
​  2 ​ 

dy 1   ​​ 
When x = 2, ___ ​​   ​​ = ____ ​​  ___ .
dx √ ​  27 ​ 

396

Pre-Calculus and Calculus.indb 396 13/7/2017 5:57:32 PM


Chapter 13 • Differentiation

Exercise 13.4
1. Differentiate the following functions with respect to x.
5
__
​   ​  __ __
(a) y = x​​ ​​ 2​​ (d) y = (​​√ x ​​ + 1)(​​√ x ​​ – 1)
(b)  f (x) = 4x3 – 3x2 + 4 (e)  f (x) = (2x + 1)2
__ x2 + 3x3
​​  1 2 ​​ + √
(c) y = ___ ​​  x ​​   (f)   f (x) = _______
​​   ​​ 

2x x2

dy
2. Calculate ___
​​   ​​ for the following functions using the product rule or quotient rule where appropriate.
dx
x2 + 4x + 3
(a) y = (3x2 + 1)(3x2 + 7x) (d) y = _________
​​  2  ​​ 
x + 2x – 3
x3 – 2
(b) y = (8x + 5)12(x3 + 7)13 (e)  f (x) = _____ ​​  2  
 ​​
x +2
__
​  x  ​+ 1
(f) y = ______

(c)  f (x) = (x + 1)(x2 – 1) ​​  1 – ​  __  ​​ 
√ x  ​

3. Differentiate the following functions using the chain rule or power rule.
(a) y = (3x + 2)10
(b) g(y) = ________
​​  1   ​​ 
(3y + 5)2
1 ​ 
–​ __
(c)  f (x) = (3 – x​​)​​  3​​
(d) y = ________ 1   ​​ 
​​  ______
​√ 25 – ​x​​  2​ ​ 
3
_____
(e) y = ​​√ 
  x + 5 ​​ 
2

_____
f (t) = (5t + √  ) 5
(f)   ​​ t2 + 1 ​​ 

4. Find the derivatives


_____ of the following functions using appropriate rules.
​√ x + 2 ​ 
2
(a) y = _______
​​  x    ​​

(x – 3)3
(b)  f (x) = ________
​​    ​​ 
(3x + 4)2
1
__
(c)   f (x) = (3x + 1)3(3 – x​​)​​ –​ 3 ​ ​​
x
(d) y = ________
​​  ______    ​​ 
​√ 25 – ​x​​  2​ ​ 
__________
_____
(e) y = √ 
​​ 1 + √ ​  ​x​​  2​ + 3 ​ ​​ 

_
(f)  f (t) = √ 
​​ t ​​(  2t – 3)
2 3

5. Using the chain rule or power rule, differentiate y = (​​​  x + __ ​ 1x ​)​​​   ​​ and y = (​​​  ​√ x ​ + ___   ​​​   ​​with respect to x.
x   ​​ )
6 __ 3
​  ​√1 __

​ ​  ( ​ 1x ​)​​​   (​​​​​  √ x   ​ ​ )​​​   ​​.


6 __ 3
d
Hence, find ___  ​​​​​   x + __ ​  ​√1 __
​  x  ​ + ___
dx

397

Pre-Calculus and Calculus.indb 397 13/7/2017 5:57:32 PM


Chapter 13 • Differentiation

13.5 Second and Higher-order Derivatives


dy
If y =  f (x) is a differentiable function, then ​___ ​   ​​ =  f '(x) is called the first
dx
d2y
derivative of y or  f (x) (can also be denoted by y'), ​​ ___2 ​​ =  f ''(x) is called the
dx
d3y
second derivative of y or  f (x) (can also be denoted by y''), ​​ ___3 ​​ =  f '''(x) is
dx
called the third derivative of y or  f (x) (can also be denoted by y''') and so
dny
on. Generally, ​​ ___n  ​​ =  f  n(x) is the nth derivative of y or  f (x) and is obtained
dx
by differentiating  f (x), n times.
Example 14
Find the second and third derivatives for each of the following functions.
(a)  f (x) = 5x3 + 3x2 + 2x + 5
–2x
(b) y = ______
​​  3x –  
1 ​​ 
SOLUTION
(a)  f (x) = 5x3 + 3x2 + 2x + 5
 f '(x) = 15x2 + 6x + 2
 f ''(x) = 30x + 6
 f '''(x) = 30
–2x
(b) y = ______
​​  3x –   1 ​​ 
dy _________________
(3x – 1)(–2) + 2x(3)
​​ ___ ​​ =    ​​      ​​
dx (3x – 1)2
= ​​ _______ 2   ​​ 
(3x – 1)2
= 2(3x – 1)–2
dy 2
​​ ___2  ​​ = –4(3x – 1)–3
dx
= _______​​  –4  3 ​​ 
(3x – 1)
d 3
y
​​ ___3  ​​ = 12(3x – 1)–4
dx
​​  12  4 ​​ 
= _______
(3x – 1)
Exercise 13.5
1. Find y''' for y = 3x3 – 2x2 + x + 1.
d2y 1+x
2. Find ___
​​  2  ​​ for y = _____
​​  1 – x  
​​.
dx
x2
3. Find  f ''(x) for  f (x) = ______
​​  _____   ​​. 
​  x – 2 ​ 

4. Find  f '''(x) for  f (x) = (x2 + 3)2.
398

Pre-Calculus and Calculus.indb 398 13/7/2017 5:57:33 PM


Chapter 13 • Differentiation

13.6 Implicit Differentiation


13.6.1 First Derivative of Implicit Functions
13.6.2 Second Derivative of Implicit Functions

13.6.1 First Derivative of Implicit Functions


In the previous sections, we have learnt how to differentiate functions
where the equation of the function is arranged in such a way that y terms
are on the left hand side and the x terms are on the right hand side of
the equation, such as y = 2x3 + 3x. Sometimes, we have to deal with
equations which are in implicit form such as x2 + y = x – y, where y is not
dy
easily written as a subject. To find ___ ​​    ​​for such function, we need to use
dx
a method called implicit differentiation. To use this method, we need to
differentiate every term with respect to x using the appropriate rules. For
example,
d d d
(a) ​​ ___   ​​(xy) = x ​​ ___  ​​ (y) + y ​​ ___  ​​ (x)   Using product rule
dx dx dx
dy
= x ​___
​    ​​ + y(1)
dx
dy
= x ​___
​    ​​ + y
dx
(b) 2y = x 2

Differentiating both sides with respect to x,


d d
​​ ___   ​​(2y) = ___ ​​    ​​ (x2)
dx dx
dy
2​​ ___  ​​ = 2x
dx
dy
​​ ___ ​​ = x
dx
The following examples illustrate how the method of implicit differentiation
is carried out.

Example 15
dy
Find ___
​​   ​​for the following functions.
dx
(a) x2 – y2 = 2xy
(b) xy2 + x2y = 3y – x

SOLUTION
(a) x2 – y2 = 2xy
Differentiating every term with respect to x,
2x – 2y ​​ ___ ​​ = 2​​ [x ​ ___  ​  + y(1)]​​  
dy dy
dx dx
dy dy
2x – 2y ​​ ___ ​​ = 2x ​​ ___ ​​ + 2y
dx dx

399

Pre-Calculus and Calculus.indb 399 13/7/2017 5:57:33 PM


Chapter 13 • Differentiation

dy
Collecting all the terms with ___ ​​   ​​on the left hand side and the rest on
dx
the right hand side, we have
dy dy
2y ​___
​   ​​ + 2x ​​ ___ ​​ = 2x – 2y
dx dx
dy
(2y + 2x)​​ ___ ​​ = 2(x – y)
dx
dy x – y
​​ ___ ​​ = _____ ​​   
 ​​
dx x + y
(b) xy2 + x2y = 3y – x
​​ [x ​ ___  ​ (y2) + y2(1)]​​  + [​​  x2 ___
​    ​ + y ​ ___  ​ (x2)]​​  = 3​​ ___ ​​ – 1
d dy d dy
dx dx dx dx
dy dy dy
x(2y)​​ ___ ​​ + y2 + x2 ​​ ___ ​​ + y(2x) = 3​​ ___ ​​ – 1
dx dx dx
dy
(2xy + x2 – 3)​​ ___ ​​= –1 – y2 – 2xy
dx
dy 1 + y2 + 2xy
​​ ___ ​​ = __________ ​​   ​​ 
dx 3 – 2xy – x2

Example 16
Find the first derivative of the following functions.
y
(a) ​​ __x ​​ – x2y = 4
(b) (x + y)5 – 7x2 = 0

SOLUTION
y
(a) ​​ __x ​​ – x2y = 4
​​ [y ​ ___  ​ (x–1) + x–1 ​ ___  ​ (y)]​​  – [​​  x2 ​ ___  ​ (y) + y ​ ___  ​ (x2)]​​  = ___
d d d d d
​​     (​​ 4)
dx dx dx dx dx
y(–x–2) + x–1 ​​ ___ ​​ – (​​  x2 ​ ___  ​+ 2xy)​​  = 0
dy dy

dx dx
y 1 ___ dy dy
–​​ __2  ​​ + __
​​  x ​​ ​​   ​​ – x2 ​​ ___ ​​ – 2xy = 0
x dx dx
dy __ y
​​ ___(  ​​​​  ​  1 ​ – x2)​​  = 2xy + __ ​​  2  ​​ 
dx x x

​​ ___ ​​ = (  ​​  × ​​  ​ 


​x​​  2​ ) ( 1 – x3 )
   ​ ​​  
dy 2x3y + y x
​​  _______
​   ​   _____
dx
dy y(2x3 + 1)
​​ ___ ​​ = ________
​​   ​​ 
dx x(1 – x3)
(b) (x + y)5 – 7x2 = 0
​    ​ (x + y)]–​​   ___
5(x + y)4​[​  ___
d d
​​     (​​ 7x2) = 0
dx dx
5(x + y)4​(​  1 + ___ ​    ​)​​  – 14x = 0
dy
dx
dy 14x
1 + ___ ​​   ​​ = _______
​​     ​​ 
dx 5(x + y)4
dy 14x
​​ ___ ​​ = _______ ​​     ​​ 
–1
dx 5(x + y)4
400

Pre-Calculus and Calculus.indb 400 13/7/2017 5:57:33 PM


Chapter 13 • Differentiation

Example 17
dy
​​  2x1– 1 ​​ 
If y2 = ______ , prove that y3 + ___
​​   ​​ = 0.
dx

SOLUTION
y2 = ______ ​​  2x 1+ 1 ​​ 
Differentiating both sides with respect to x,
dy
2y ​​ ___ ​​ = –(2x – 1)–2(2)
dx
dy –(2x – 1)–2
​​ ___ ​​ = _________
​​  y   
​​

dx
​  ​  1 ​ ​​  
dy
(2x – 1)2 ( y )
​​ ___ ​​ = –​​ _______ 1   ​​  
​ __
dx
dy –y4
​​ ___ ​​ = ___ ​​  y   ​​ ​​  2x1– 1 ​​ 
Since y2 = ______
dx
dy
​​ ___ ​​ = –y3
dx
dy
y3 + ___
​​   ​​ = 0
dx

13.6.2  Second Derivative of Implicit Functions


The higher order derivatives for implicit functions can also be obtained
using implicit differentiation. The next example shows how the second
order derivative is evaluated.

Example 18
d2y
Using implicit differentiation, find ___
​​  2  ​​for the function 3x2 + y2 = 2 in
dx
terms of x and y.

SOLUTION
3x2 + y2 = 2
dy
First, we calculate ___ ​​   ​​by differentiating both sides with respect to x.
dx
dy
6x + 2y​___ ​   ​​ = 0
dx
dy –3x
​ ​   ​​ = ____
___
​​  y    ​​
dx
dy2
Next, to obtain ___ ​​  2  ,​​ we differentiate both sides with respect to x again.
dx
dy dy
d2
y y(–3) – (–3x)​ ___ ​ –3y + 3x ​ ___ ​
___ _____________ dx __________ dx
​​  2  ​​ =   
​​   ​​  = ​​   ​​   
dx y2 y2
d2y dy –3x
To express ___
​​  2  ​​in terms of x and y, we substitute ___ ​​   ​​ = ____
​​  y    .​​
dx dx

( y ) _________
–3x
___ –3y2 – 9x2
_________
d y _____________
2 –3y + 3x​   ​      
​ ​ ​  y   
​ __________
  –3(y2 + 3x2)
___
​​  2  ​​ =    ​​  2 ​​  = ​​  2 ​​   = ​​   ​​   
dx y y y3
d2y –6
Since 3x2 + y2 = 2, we can also write ___
​​  2  ​​ = ___
​​  3 ​​ .
dx y 401

Pre-Calculus and Calculus.indb 401 13/7/2017 5:57:33 PM


Chapter 13 • Differentiation

Example 19
  ​​ + ​​​  ​   ​ ​​​   ​​ = 1.
dx2 ( dx )
2
dy
___
2
dy
___
If x – y = 1, prove that y ​​ 
2 2

SOLUTION
x2 – y2 = 1
Differentiating both sides with respect to x,
dy
2x – 2y ​​ ___ ​​ = 0
dx
dy x
​​ ___ ​​ = __ ​​   ​​
dx y
dy
d2y _________y(1) – x​ ___ ​
___ dx
​​  2  ​​ = ​​   ​​   
dx y2
y x ​ __x
​​    ​​  ​  ​   ​ ​​  
y y2 ( y )
= ​​ __2  ​​ – __
x2
= __ ​​  1y ​​ – __
​​  3 ​​ 
y

y ​​ ___2  ​​ + (​​​  ___ ​   ) ​ ​​​   ​​ = y ​( ​  3 ​ )​​  + (​​​  __


2
dy dy
​ ​​​   ​​
2
x2 x 2
​  1y ​ – __
​  __ ​  y )
dx dx y
​ ​​​   ​​
x2 x 2
= 1 – __ ​​  2 ​​  + (​​​  __
​  y )
y
=1

Exercise 13.6
dy
1. Find ___
​​   ​​ for the following implicit functions.
dx
x
(a) x2 – y2 = 8 (d) xy2 – __
​​  y ​​ = y
(b) x2y2 = 4 (e) (x + y)2 + (x – y)2 = y2 + x2
(c) x2y2 = x2 + y2 (f) 2x2 + xy = x – 1

d2y
2. Find ___
​​  2  ​​for the functions in Questions 1(a) and 1(b).
dx

d2y
3. Find ___
​​  2  ​​at the point (1, 1) for the curve x2 + 4y2 – 5xy = 0.
dx
d2y
4. Find the value of ___ ​​  2  ​​for the curve x2y + 3y – 4 = 0 at the points
dx
​  43 ​ )​​,  and
(a) ​​ (0, __
(b) (–1, 1).

​   ​)​​​   ​​ = 0.
5. If y2 = 2xy + 2x, prove that (x – y)​​ ___2  ​​ + 2​​ ___ ​​ – (​​​  ___
2
d2y dy dy
dx dx dx
_____

√ 
6x dy d2y
6. Given that the equation of a curve is y = ​​ _____ , find the values of ___
​  x +  2 ​ ​​  ​​   ​​ and ___
​​  2  ​​ when x = 4.
dx dx

402

Pre-Calculus and Calculus.indb 402 13/7/2017 5:57:33 PM


Chapter 13 • Differentiation

13.7 Derivatives of Exponential and


Logarithmic Functions
13.7.1 Derivatives of Exponential Functions
13.7.2 Derivatives of Logarithmic Functions
13.7.3 Logarithmic Differentiation

13.7.1  Derivatives of Exponential Functions

Theorem 13.9

The derivatives of the exponential functions are as follows:


d
(i) ​​ ___  ​​( ex) = ex
dx
d du
(ii) ​​ ___  ​​( eu) = eu ​___
​    ​​, where u =  f (x)
dx dx
d x
___
(iii) ​​    ​​( a ) = ax ln a, where a > 1
dx
d du
(iv) ​​    ​​( au) = (au ln a) ​​ ___  ​​, where u =  f (x) and a > 1
___
dx dx

The proofs of the above theorem are as follows:


d
(i) ​​ ___  ​​( ex) = ex
dx
Using the first principle, if  f (x) = ex, then
f (x + h) – f (x)
_____________
 f '(x) = ​​  lim​ ​​​    ​​    ​​ 
h→0 h
ex + h – ex
= ​​  lim​ ​​​ ​​ _______     ​​

h→0 h
ex(eh – 1)
= ​​  lim​ ​​​ ________
​​      ​​

h→0 h
eh – 1 Note
= ex ​​  lim​ ​​​ _____ ​​      ​​

h→0 h eh – 1
​​  lim​ ​​​ ​​ _____   
​​ = 1 can be proved
h
eh – 1 h→0
Since ​​  lim​ ​​​ _____
​​      ​​ = 1,  f '(x) = ex.
  by numerical approximation
h→0 h
with tables or by L’Hôpital’s
d du
(ii) ​​ ___  ​​ (eu) = eu​___ ​    ​​, where u = f (x) Rule (to be covered in
dx dx
Section 13.11).
Let y = eu and u = f (x).
dy du
Then, ___ ​​   ​​ = eu and ___ ​​    ​​ = f '(x).
du dx
Using chain rule, we have
dy dy ___ du
​​ ___ ​​ = ___ ​​   ​​ × ​​    ​​
dx du dx
dy ​du
​​ ___ ​​ = e­u f '(x) = eu ___ ​    ​​
dx dx

403

Pre-Calculus and Calculus.indb 403 13/7/2017 5:57:33 PM


Chapter 13 • Differentiation

d
(iii) ​​ ___  ​​( ax) = ax ln a, where a > 1
dx
Changing the base, we have ax = ex ln a.
dy
Let y = ex ln a, then ___ ​​   ​​ = (ln a)ex ln a.
dx
dy
Substituting ex ln a = ax into ___ ​​   ​​, 
dx
dy
___
​​   ​​ = a ln a x
dx
d du
(iv) ​​ ___  ​​( au) = (au ln a)​​ ___  ​​, where a > 1
dx dx
Let y = a and u =  f (x). u

dy du
Then, ___ ​​   ​​ = au ln a and ___
​​    ​​ =  f '(x).
du dx
Using chain rule, we have
dy dy ___ du
​​ ___ ​​ = ___ ​​   ​​ × ​​    ​​
dx du dx
dy du
​​ ___ ​​ = (au ln a) f '(x) = (au ln a) ​​ ___  ​​
dx dx

Example 20
dy
Find ___
​​   ​​for each of the following exponential functions.
dx
(a) y = e​​ ​​ 3x ​​ _____
2

(b) y = 3​​e​​  ​√ x – 1 ​​​  _____


(c) y = 3e2x + 1 – ​​e​​  ​√ x – 1 ​​​ 
(d) y = 72x + 1

SOLUTION
(a) y = e​​ ​​ 3x ​​
2

dy d
​​ ___ ​​ = ​​e​​  3x ___
​​​​     (​​ 3x2) = 6x​​e​​  3x ​​
2 2

dx dx
_____
(b) y = 3​​e​​  ​√ x – 1 ​ ​​
dy _____
 d
_____
​​ ___ ​​ = 3​​e​​  ​√ x – 1 ​___
​​​​    √  ​​​​  x – 1 ​​ 
dx dx
= 3​​e​​  ​√ x – 1 ​(​​​​   _______ )​​  
_____
​  _____ 1   ​ 
2​√ x – 1 ​ 
_____
3​e_____
​​  ​√ x – 1 ​ ​
= ​​ _______    ​​ 
2​√ x – 1 ​ 
_____
(c) y = 3e2x + 1 – ​​e​​  ​√ x – 1 ​​​ 
_____
dy
___ ​e​​  _____
_______ ​
​√ x – 1 ​ 
​​   ​​ = 6e2x + 1
– ​​    ​​ 
dx 2​√ x – 1 ​ 
(d) y = 72x + 1
dy
​​ ___ ​​ = 2(72x + 1) ln 7
dx

404

Pre-Calculus and Calculus.indb 404 13/7/2017 5:57:33 PM


Chapter 13 • Differentiation

Example 21
Using appropriate rules, differentiate the following functions with
respect to x.
x2 + 2 _____
(a)  f (x) = x3e–5x (b)  f (x) = ________
​​  3x – 1   ​​   (c) y = (x2 – 2​​e​​  ​√ x + 2 ​ ​​​​)​​ 10​​
2e –1
SOLUTION
(a)  f (x) = x3e–5x
Using product rule,
d d
 f '(x) = x3 ​___
​     (​​ e–5x) + e–5x ​___
​     (​​ x3)
dx dx
= –5x3e–5x + 3x2e–5x
x2 + 2
(b)  f (x) = ________
​​  3x – 1   ​​ 
2e –1
Using quotient rule,
d d
(2e3x – 1 – 1)​ ___   (​ x2 + 2) – (x2 + 2)​ ___   (​ 2e3x – 1 – 1)
dx
_____________________________________ dx
 f '(x) =      ​​       ​​
(2e3x – 1 – 1)2
2x(2e3x – 1 – 1) – (x2 + 2)(6e3x – 1)
__________________________
=     ​​      ​​
(2e3x – 1 – 1)2
2x 6e3x – 1(x2 + 2)
= ​​ ________    ​​ – ___________
​​ 
    ​​ 
2e3x – 1 – 1 (2e3x – 1 – 1​)​​ 2​
_____
(c) y = (x2 – 2​​e​​  ​√ x + 2 ​​​​​  )10
Using product rule,
dy _____ 9
d _____
​​ ___ ​​ = 10 ​​​ (​​x​​  2​– 2e​​  ​√ x + 2 ​)​ ​​​​   ___
​​​​     ​​ ​​ (​​x​​  2​– 2e​​  ​√ x + 2 ​) ​ ​​
dx dx
= 10 (​​​  ​​x​​  2​– 2e​​  ​√ x + 2 ​ ​)​​​​   (​​​​  2x – ______ 1   ​​  e​​  ​√ x + 2 ​ ​)​​  
_____ 9 _____
​  _____
​  x + 2 ​ 

Example 22
Find the first derivatives of the following functions.
​​  12x   ​​
(a) y = 2x + 3 – ___ (b)  f (x) = (x2 + 5x)3
3
SOLUTION
​​  12x   ​​ = 2x + 3 – 3–2x
(a) y = 2x + 3 – ___
3
dy
___
​​   ​​ = 2 ln 2 + 2(3–2x) ln 3
x+3
dx
2 ln 3
= 2x + 3 ln 2 + _____ ​​  2x    ​​ 
3
(b)  f (x) = (x2 + 5x)3
d
 f '(x) = 3(x2 + 5x)2 ​___ ​     (​​ x2 + 5x)
dx
= 3(x2 + 5x)2(2x + 5x ln 5)

405

Pre-Calculus and Calculus.indb 405 13/7/2017 5:57:33 PM


Chapter 13 • Differentiation

13.7.2  Derivatives of Logarithmic Functions


Theorem 13.10

The derivatives of the logarithmic functions are as follows:


d
(i) ​​ ___  ​​( ln x) = __​​  1x ​​
dx
​​  1u ​​ ​(​  ___ ​    ​)​​,  where u =  f (x)
d du
(ii) ​​ ___  ​​( ln u) = __
dx dx
d
(iii) ​​ ___  ​​( loga x) = _____ ​​  1   ​​  = __​​  1 ​​(loga e), where a > 1
dx x ln a x
​​  1   ​​ ( ​    ​ ​​  = __ ​    ​)​​,  where u =  f (x) and a > 1
u ln a dx ) u
​​  1 ​​(loga e)​(​  ___
d du du
(iv) ​​ ___  ​​( loga u) = _____ ​​  ___

dx dx

The proofs of the above theorem are as follows:


d
(i) ​​ ___  ​​( ln x) = __ ​​  1x ​​
dx
Let y = ln x
ey = x
Differentiating both sides with respect to x,
d d
​​ ___   ​​ ey = ___ ​​     ​​ x
dx dx
dy
ey ​___ ​   ​​ = 1
dx
dy 1
​​   ​​ = __
___
​​    ​​
dx ey
= __ ​​  1x ​​

​    ​)​​,  where u =  f (x)


​​  1u ​​ ​(​  ___
d du
(ii) ​​ ___  ​​( ln u) = __
dx dx
Let y = ln u and u =  f (x).
dy 1 du
Then, ___ ​​   ​​ = __ ​​   ​​ and ___ ​​   ​​ =   f '(x).
du u dx
Using chain rule,
Note dy dy ___ du
​​ ___ ​​ = ___ ​​   ​​ × ​​   ​​ 
The proofs of (iii) and (iv) in
dx du dx
​    ​)​​  
​​  1u ​​ ​(​  ___
dy 1 du
Theorem 13.10 are similar ​​ ___ ​​ = __ ­​​   ​​  f '(x) = __
to that of (i) and (ii). dx u dx
Example 23
Differentiate the following functions with respect to x.
y = ln (​​  _____  ​)​​  
x–1
(a) y = ln (7x2) (d) ​  x + 1 
(b)  f (x) = ln (2x3 + 1) (e) y = ln (3x – 2)6
(c) g(x) = 2x3 ln (3x – 2)

SOLUTION
(a) y = ln (7x2)
dy d
​​ ___ ​​ = ___​​  1   ​​ ​​  ___   (​​ 7x2)
dx 7x2 dx
14x
= ​​ ____2 ​​ 
7x
406 __ 2
= ​​  x ​​

Pre-Calculus and Calculus.indb 406 13/7/2017 5:57:33 PM


Chapter 13 • Differentiation

(b)  f (x) = ln (2x3 + 1)


d
  ​​  31   ​​ ​​  
f '(x) = ______ ___
   ​​  (2x3 + 1)
2x + 1 dx
6x2
= ______
​​  3    ​​ 
2x + 1
(c) g(x) = 2x3 ln (3x – 2)
d d
g'(x) = 2x3 ​___ ​      ​​ ln (3x – 2) + ln (3x – 2) ___
​​      ​​ (2x3) Using product rule
dx dx
= 2x3​(​  ______ )​​  + 6x ln (3x – 2)
3
​  3x – 2 ​  2

6x3
= ​​ ______
3x –   2 ​​ + 6x2 ln (3x – 2)

(d) y = ln (​​  _____  ​)​​  


x–1
​  x + 1 

 ​)​​  
dy
   ​​ ​​ (​  x + 1 
d _____ x–1
​​  x –1 1  ​​ ​​  
​​ ___ ​​ = _____ ___
Using quotient rule
dx _____ ​  x + 1 
 ​ dx

​​  x – 1[ ​​​​  _______ ​​  
​(x + 1)​​  ​]
 ​ ​​  = _______
x+1
​  2  2 ​ 
dy x – 1
​​   ​​ ​​ (_____
dx x + 1 ) (x + 1)2
= _____ From Example 8(b), ___ ​    ​​  2   ​​ 

= ​​ ___________
  
2   ​​
(x – 1)(x + 1)
(e) y = ln (3x – 2)6
dy d 
​​  1   ​​ ​​  
​​ ___ ​​ = _______ ___
   ​​ (3x – 2)6 Using power rule
dx (3x – 2)6 dx
​​  1  6 ​​[ 
= _______ (6)(3x – 2)5(3)]
(3x – 2)
18
= ______
​​  3x – 2 ​​ 

Example 24
Differentiate the following functions with respect to x.
(a)  f (x) = log2 3x2 (b) g(x) = e2x log5 (10x – 2)

SOLUTION
(a)  f (x) = log2 3x2
6x
 f '(x) = ___ ​​  2x ​​ log2 e
​​  2  ​​  log2 e = __
3x
(b) g(x) = e2x log5 (10x – 2) Using product rule
d d
g'(x) = e2x ​​ ___   ​​  log5 (10x – 2) + log5 (10x – 2) ​​ ___   ​​  e2x
dx dx
10e2x log5 e
__________
= ​​  10x – 2 ​​   + 2e2x log5 (10x – 2)

407

Pre-Calculus and Calculus.indb 407 13/7/2017 5:57:33 PM


Chapter 13 • Differentiation

13.7.3  Logarithmic Differentiation


Logarithmic differentiation is an alternative method used to differentiate
complicated function of the form  f (x)g(x). We make use of logarithm and its
rules to simplify the function before we apply the method of differentiation.
The procedures are illustrated in the following examples.

Example 25
dy
Find ___
​​   ​​for the following functions.
dx
2x3 –_______
1
(a) y = x2x (b) y = _______________
​​        ​​
(x + 4) ​√ x + 2x  ​ 
2 6 3

SOLUTION
(a) First, we take logarithms of base e on both sides of the equation.
ln y =  ln x2x
ln y = 2x ln x
Then, we differentiate both sides with respect to x.

y (​  dx ) ​ ​​  = 2x ​___


dy
1 ​​ ​​  ___ d d
​​ __ ​     ​​ (ln x) + ln x ​___
​     ​​ (2x)
dx dx
= 2x​(​  __ ​ ​​ + 2 ln x
​  1x )
= 2(1 + ln x)
___dy
​​   ​​ = 2y(1 + ln x)
dx
= 2x2x(1 + ln x) Substitute y = x2x

(b) Taking logarithms of base e on both sides of the equation,


   ​ ​​  
[ (​x​​  ​ + 4​)​​ 6​​√ ​x​​  3​+ 2x   ​]
2x3 –______ 1
ln y = ln ​​  _______________ ​     2

ln y = ln (2x3 – 1) – 6 ln (x2 + 4) – __ ​​ 21 ​​  ln (x3 + 2x) properties


Using logarithm

Differentiating both sides with respect to x,


​ ​​  = ______ – 6​​ (_____ ) ​​  – ________
dy __
​​ ___(
 ​ ​​​ ​  1y )
6x2 2x 3x2 + 2
​​  3    ​​  ​  2    ​  ​​  3    ​​ 
dx 2x – 1 x +4 2(x + 2x)

​​ ___ ​​ = y​[​  ______ ​​  


2x – 1 x2 + 4 2(x3 + 2x) ]
dy 6x2 12x ________ 3x2 + 2
​  3    ​  – _____
​    ​ – ​    ​ 
dx

​​  
(​x​​  2​ + 4​)​​ 6​​√ ​x​​  3​+ 2x [ 
​ 2x – 1 x + 4 2(x3 + 2x) ]
2x3 –______ 1 6x2 12x ________3x2 + 2
= ​​ ______________
      ​​​
​  ______
​  3    ​ – _____
​  2   ​ – ​    ​ 
The answer can be given in terms of y and x or in terms of only x.

408

Pre-Calculus and Calculus.indb 408 13/7/2017 5:57:34 PM


Chapter 13 • Differentiation

Exercise 13.7
1. Differentiate the following exponential functions with respect to x.
(a) y = 6e–3x (f) y = (e2x – e–2x)(e2x + e–2x)
f (x) = e​​ ​​ x + 2x ​​
(b)   (g)   f (x) = (ex + e–x)6
3

f (x) = e2x – e–3x


(c)   (h) y = x2e3x
ex – e–x 2ax
f (x) = ______
(d)   ​​  ex    ​​
  (i) y = ___
​​  5 ​​ 

f (x) = (​​​  ​e​​  3x​ – ____  ​​​​   ​​


   ​)
2
​  13x
__
(e)   (j) y = ​​10​​ 2​√ x ​ ​​
3​e​​  ​

2. Find the derivatives of the following logarithmic functions. Express your answers in the simplest
form. ______
(a) y = ln 4x (g)  f (x) = ln (​​√ 2x – 1 ​​) 
10
__
ln (​√ x ​ – 1)
__________
2
f (x) = ln (3x + 2x) (h)
(b)   y = ​​ 
2

 ​​ __
______
ln (​√ x  ​ + 1​)​​ 2​
f (x) = ln √ 
(c)   ​​ 2x – 1 ​​   (i) y = log2 (2x2 – 5x – 1)
f (x) = ln (1 – x2) + ln (x2 + 1)
(d)   (j)  f (x) = logm (x – 2) – logn (2 – x)

(e)  f (x) = ln [x3(x – 2)] ​  13  ​ )​​  


(k) y = log10 (​​  __
x
(f) y = ln (​​  ______  ​ ​​  
1 – x2 )
1 + 2x
​ 

dy
3. Find ___
​​   ​​ for each of the following functions.
dx
y = ln (​​  _____  ​)​​  
2 – ex
(a) y = ex ln x2 (c) ​  2 + ex 
_____
ln x2
(b) y = 2 ln (​​e​​  ​√ x + 1 ​ ​​) (d) y = ____
​​  ex   ​​ 

dy d
4. Find ___
​​   ​​ for y = [ln (e2x – 2)]5. Hence, evaluate ___
​ ​   ​​{  x3[ln (e2x – 2)]5}.
dx dx
dy dy 1
5. If y = logm x, show that ___ ​​  1   ​​ 
​​   ​​ = ______ or ​​ ___ ​​ = __
​​   ​​ logm e.
dx x ln m dx x
dy
6. Use logarithmic differentiation to find ___
​​   ​​ for the following.
dx
(a) y = x ln x

(b) y = xx
______
(c) y = x2 ​​√ e2x + 3 ​​ 
x3(x2 – 2x + 1)
(d) y = ____________
  
​​      ​​
(x2 – 2)(1 – x3)

409

Pre-Calculus and Calculus.indb 409 13/7/2017 5:57:34 PM


Chapter 13 • Differentiation

13.8 Derivatives of Trigonometric


Functions
13.8.1 Derivatives of sin x, cos x, tan x, sec x, cosec x and cot x
13.8.2 Derivatives of sin u, cos u, tan u, sec u, cosec u and cot u
13.8.3 Derivatives of sinn u, cosn u, tann u, secn u, cosecn u and cotn u

13.8.1 Derivatives of sin x, cos x, tan x, sec x,


cosec x and cot x
Trigonometric functions can be differentiated with respect to the angle x.
Their derivatives can be found using first principles given by
dy f (x + h) – f (x)
 f ′(x) = ___
​​   ​​ = ​​  lim​ ​​​ ____________
​​    h  ​​.

dx h → 0
Two standard results will be used without proof:
 ​ ​​  = 1 and ​​  lim​ ​​​ ( ​)​​  = 0
h→0( h )
sin h 1 – cos h
​​  lim​ ​​​ ​​  _____
​      ​​  ________
​       
h→0 h

Example 26
Find the derivative of y = sin x by using first principles.

SOLUTION
___d sin (x + h) – sin x
_______________
​​     (​​ sin x) = ​​  lim​ ​​​   ​​    ​​ 
dx h→0 h
2 cos (​  x + __ ​ 12  ​h)​  sin (​  __ ​  21 ​ h)​  Using factor formula:
____________________ sin A – sin B
= ​​  lim​ ​​​   
​​    ​​

h
​​ A + B  sin ​​ _____
A – ​​
B 
h→0
= 2 cos _____
⎡ ⎤
​  21 ​ h)​ 
2 ​​  2  

⎢  ⎥ 
sin (​  __
= ​​  lim​ ​​​ ​​   cos (​  x + __ ​ 12  ​h)​   ________
​  __     ​ ​​  

h→0
⎣ ​  21 ​ h ⎦
= cos x
​ 2  ​)​​  = cos x
since ​​  lim​ ​​​ cos (​​  x + __
h
h→0

sin __
______
​  21 ​ h
sin h
and ​​ lim​ ​​​ ​​  __     ​​ = 1
  ​​  lim​ ​​​ _____
​​      ​​ = 1
​  1 ​ h
h→0 h→0 h
2
d
\ ___
​​     (​​ sin x) = cos x
dx

410

Pre-Calculus and Calculus.indb 410 13/7/2017 5:57:34 PM


Chapter 13 • Differentiation

Similarly, the derivatives of other trigonometric functions can be obtained


using first principles. The derivatives are summarized below.

Theorem 13.11

The derivatives of trigonometric functions are


d
​​ ___  ​​  (sin x) = cos x
dx
d
​​ ___  ​​  (cos x) = –sin x
dx
d
​​ ___  ​​  (tan x) = sec2 x
dx
d
​​ ___  ​​  (cot x) = –cosec2 x
dx
d
​​ ___  ​​  (cosec x) = –cosec x cot x
dx
d
​​ ___  ​​  (sec x) = sec x tan x
dx

Example 27
d
Using first principles, prove ___
​​     ​​  (cos x) = –sin x.
dx
SOLUTION
___ d cos (x + h) – cos x
​​     ​​  (cos x) = ​​  lim​ ​​​ _______________ ​​      ​​ 
dx h→0 h
–2 sin (​  x + __ ​ 12  ​h)​  sin (​  __ ​  21 ​ h)​  Using factor formula:
cos A – cos B
_____________________
= ​​  lim​ ​​​    ​​       ​​
h→0 h ​​ A +
= –2 sin _____ B  sin ​​ _____
B 
A – ​​
2 ​​  2  
⎡ ⎤
​  21 ​ h)​ 
⎢  ⎥ 
sin (​  __
= ​​  lim​ ​​ ​  –sin   ( ​  x + ​ 2  ​h)​   ​  __
1
__ ________
 ​  
 ​​  
h→0
⎣ ​  21 ​ h ⎦
= –sin x
since ​​  lim​ ​​​ sin (​​  x + __ ​ 2  ​)​​  = sin x
h
h→0

__
​  21 ​ h
___
and ​​ lim​ ​​​ sin ​​  __   ​​ = 1
h→0
​  21 ​ h
d
∴ ___ ​​     ​​ (cos x) = – sin x
dx

411

Pre-Calculus and Calculus.indb 411 13/7/2017 5:57:34 PM


Chapter 13 • Differentiation

Example 28
d
Show that ___
​​     ​​  (tan x) = sec2 x.
dx

SOLUTION
sin x
tan x = ​​ _____cos x    
​​
Using quotient rule,
___d d sin x
​​     ​​  (tan x) = ___ ​​     ​​ ​​ _____   
 ​​
dx dx cos x
d d
cos x ___ ​    ​ sin x – sin x ___
​    ​ cos x
________________________ dx dx
=     ​​       ​​
cos2 x
cos x cos x + sin x sin x
= ​​ ___________________
        ​​
cos2 x
cos2 x + sin2 x
= ​​ ____________
       ​​
cos2 x
​​  12    
= _____ ​​
cos x
= sec x 2

Example 29
Differentiate the following functions with respect to x.
(a) y = 3 cos x (c) y = 3 cos x sec2 x
sin x
(b) y = 3 cos x + 5 cosec x (d) y = ________
​​  1 – cos x  
​​

SOLUTION
(a) y = 3 cos x
dy
​​ ___ ​​= –3 sin x
dx
(b) y = 3 cos x + 5 cosec x
dy
​​ ___ ​​= –3 sin x – 5 cosec x cot x
dx
(c) y = 3 cos x sec2 x
= 3 cos x (​​  _____ ​ ​​  
cos x )
​  1 2   

= 3​​ (_____
​  cos ​ ​​  
x)
1    
= 3 sec x
dy
Hence, ___ ​​   ​​= 3 sec x tan x.
dx

412

Pre-Calculus and Calculus.indb 412 13/7/2017 5:57:34 PM


Chapter 13 • Differentiation

sin x
(d) y = ________
​​  1 – cos x  
​​
Using quotient rule,
d d
(1 – cos x) ___
​    ​ (sin x) – sin x ___
dy __________________________________ ​    ​ (1 – cos x)
___ dx dx
​​   ​​ =     ​​       ​​
dx (1 – cos x)2
(1 – cos x) cos x – sin x (sin x)
= _________________________
​​ 
        ​​
(1 – cos x)2
cos x – cos2 x – sin2 x
__________________
= ​​        ​​
(1 – cos x)2
cos x – (cos2 x + sin2 x)
___________________
= ​​        ​​
(1 – cos x)2
cos x – 1
= ​​ __________    ​​ 
(1 – cos x)2
–(1 – cos x)
= __________
​​   ​​ 
(1 – cos x)2
= –​​ ________ 1    
1 – cos x ​​
​​  cos 1x – 1 ​​ 
= ________

13.8.2 Derivatives of sin u, cos u, tan u, sec u,


cosec u and cot u
Derivation of sin u, cos u, tan u, sec u, cot u, cosec u, where u = f (x), can
be carried out using the chain rule. The chain rule states that, if y = g(u)
dy dy ___ du
and u = f (x) are both differentiable, then ___
​ ​  ​ = ___
​ ​  ​× ​ ​   ​​.
dx du dx
Theorem 13.12

The derivatives of sin u, cos u, tan u, sec u, cosec u and cot u, where
u =  f (x), is given by
d d
​​ ___  ​​  (sin u) = cos u ​___
​    ​​( u)
dx dx
d d
​​ ___  ​​  (cos u) = –sin u ​___
​    ​​( u)
dx dx
d d
​​ ___  ​​  (tan u) = sec2 u ​___
​    ​​( u)
dx dx
d d
​​ ___  ​​  (cot u) = –cosec2 u ​___ ​    ​​( u)
dx dx
d d
​​ ___  ​​  (cosec u) = –cosec u cot u ​___ ​    ​​( u)
dx dx
d d
​​ ___  ​​  (sec u) = sec u tan u ​___ ​    ​​( u)
dx dx

413

Pre-Calculus and Calculus.indb 413 13/7/2017 5:57:34 PM


Chapter 13 • Differentiation

Example 30
Differentiate the following with respect to x.
(a) sin 6x (c) tan 2x (e) cot 8x
(b) cos 3x (d) sec 5x (f) cosec 4x

SOLUTION
d d
(a) ​​ ___  ​​  (sin 6x) = cos 6x ​​ ___  ​​  (6x) = 6 cos 6x
dx dx
d d
(b) ​​ ___  ​​  (cos 3x) = –sin 3x ​​ ___  ​​  (3x) = –3 sin 3x
dx dx
d d
(c) ​​ ___  ​​  (tan 2x) = sec2 2x ​​ ___  ​​  (2x) = 2 sec2 2x
dx dx
d d
(d) ​​ ___  ​​  (sec 5x) = sec 5x tan 5x ​​ ___  ​​  (5x) = 5 sec 5x tan 5x
dx dx
d d
(e) ​​ ___  ​​  (cot 8x) = –cosec2 8x ​​ ___  ​​  (8x) = –8 cosec2 8x
dx dx
d d
(f) ​​ ___  ​​  (cosec 4x) = –cosec 4x cot 4x ​​ ___  ​​  (4x) = –4 cosec 4x cot 4x
dx dx

Example 31
Differentiate the following with respect to x.
sin x5
(a) sin 3x2 + cos 4x3   (b) cos x3 sin 3x     (c) ​​ ______
cos 2x    
​​

SOLUTION
(a) Using sum rule,
d d d
​​ ___  ​​  (sin 3x2 + cos 4x3) = ___
​​    ​​  (sin 3x2) + ___​​    ​​  (cos 4x3)
dx dx dx
d d
= cos 3x2 ​​ ___   ​​  (3x2) + (–sin 4x3) ​​ ___   ​​  (4x3)
dx dx
= 6x cos 3x2 – 12x2 sin 4x3
(b) Using product rule,
u = cos x3, v = sin 3x
du
___ dv
___
​​    ​​ = –3x2 sin x3, ​​    ​​= 3 cos 3x
dx dx
d
___ ___dv du
___
​​    ​​  (cos x3 sin 3x) = u ​​   ​​ + v ​​    ​​
dx dx dx
= cos x3(3 cos 3x) + sin 3x(–3x2 sin x3)
= 3 cos x3 cos 3x – 3x2 sin 3x sin x3
(c) Using quotient rule,
u = sin x5, v = cos 2x
du
___ dv
___
​​    ​​ = 5x4 cos x5, ​​   ​​= –2 sin 2x
dx dx
​​ ___  ​​ ​​  (______ ​)​​  =    
d sin ​x​​  ​ 5 cos 2x(5x 4
cos x5) – sin x5(–2 sin 2x)
______________________________
​  cos 2x    ​​       ​​
dx cos2 2x
5x4 cos 2x cos x5 + 2 sin 2x sin x5
___________________________
= ​​          ​​
cos2 2x

414

Pre-Calculus and Calculus.indb 414 13/7/2017 5:57:34 PM


Chapter 13 • Differentiation

13.8.3 Derivatives of sinn u, cosn u, tann u, secn u,


cosecn u and cotn u
The chain rule used in Section 13.8.2 can now be extended to differentiate
sinn u, cosn u, tann u, secn u, cosecn u and cotn u. The rule is known as the
power chain rule. It states that, if f (u) is differentiable and n is any real
number, then the derivative of f (u)n is given by
___d d
​​    ​​   f (u)n = n f (u)n – 1 ​​ ___  ​​   f (u).
dx dx
d d
For example, if  f (u) = sin u, then ___ ​​    ​​  (sinn u) = n sinn – 1 u ​___
​     ​​  (sin u).
dx dx
Example 32
d
Prove that ___
​​     ​​  (sinn x) = n sinn – 1 x cos x.
dx
SOLUTION
Let y = sinn x = (sin x)n.
Let u = sin x, then y = un.
du dy
Hence, ___ ​​    ​​ = cos x and ___ ​​   ​​ = nun – 1.
dx du
dy dy ___ du
​​ ___ ​​ = ___
​​   ​​ × ​​    ​​ = nun – 1 cos x = n sinn – 1 x cos x
dx du dx

Example 33
Differentiate the following with respect to x.
(a)  y = cos3 5x   (b) y = tan2 3x2   (c) y = sin2 3x + cos5 2x3

SOLUTION
(a) y = cos3 5x
dy d
​​ ___ ​​= 3 cos2 5x ​​ ___   ​​  (cos 5x)
dx dx
= 3 cos2 5x(–5 sin 5x)
= –15 cos2 5x sin 5x
(b) y = tan2 3x2
dy d
​​ ___ ​​= 2 tan 3x2 ​​ ___   ​​  (tan 3x2)
dx dx
= 2 tan 3x2(6x sec2 3x2)
= 12x tan 3x2 sec2 3x2
(c) y = sin2 3x + cos5 2x3
dy d d
​​ ___ ​​= 2 sin 3x ​​ ___   ​​  (sin 3x) + 5 cos4 2x3 ​​ ___   ​​  (cos 2x3)
dx dx dx
= 2 sin 3x(3 cos 3x) + 5 cos4 2x3(–6x2 sin 2x3)
= 6 sin 3x cos 3x – 30x2 cos4 2x3 sin 2x3

415

Pre-Calculus and Calculus.indb 415 13/7/2017 5:57:34 PM


Chapter 13 • Differentiation

Example 34
dy π
Find the value of ___
​​   ​​for each of the following, when x = __​​  3 ​​ .
dx
(a) y = sin 2x – x cos x (b) y = 4 tan 2x – 3 cos x2
SOLUTION
dy d d
(a) ​​ ___ ​​= ​2 cos 2x – x ​ ___   (​ cos x) + cos x ​ ___   (​ x)​
dx dx dx
= 2 cos 2x – [x(– sin x) + cos x]
= 2 cos 2x + x sin x – cos x
π dy 2π π π π
When x = __ ​​  3 ​​ , ___ ​​   ​​ = 2 cos __ ​​  3 ​​ + ​​ __ __ __
3 ​​  sin ​​  3 ​​  – cos ​​  3 ​​  = –0.5931
dx
dy d
(b) ​​ ___ ​​= 8 sec2 2x – 3(–sin x2) ​​ ___   ​​  (x2) = 8 sec2 2x + 6x sin x2
dx dx
 sin (
3 ​ )​​​   ​​
π ___ dy 2π π π 2
When x = ​​ __ ( )
2 __ __ __

 ​​
3 dx , ​​   ​​= 8 sec ​​  3  ​​ + 6​​  ​  3  
 ​ ​​ ​​​  ​ 
8 π 2
= _________
​​    2 ​​  + 2π sin __ ​​ 9 ​​ 
( 3 ​ )​​​   ​

​​  cos ​ ___
= 32 + 5.59
= 37.59

Example 35
dy
Find ___
​​   ​​for the following equations.
dx _________
(a) y = sin (5x + 2) (d) y = √
​​  4 + sin 2x  

_____
cos 2x
(b) y = _____
​​  3 + 5x  
​​ ​​ 1 – x2 ​ 
(e) y = √  tan x​
(c) y = cos2 (1 – x2)
SOLUTION
(a) y = sin (5x + 2)
du
Let u = 5x + 2  ​​ ___  ​​ = 5
dx
dy
___
y = sin u  ​​   ​​ = cos u
du
By chain rule,
___ dy dy ___ du
​ ​   ​​ = ___​​   ​​ × ​​    ​​ = (cos u)(5) = 5 cos (5x + 2)
dx du dx
cos 2x
(b) y = ______
​​  3 + 5x   ​​
du
Let u = cos 2x  ​​ ___  ​​= –2 sin 2x
dx
dv
v = 3 + 5x  ___ ​​   ​​ = 5
dx
By quotient rule,
du dv
dy _________ v ___
​    ​ – u​ ___ ​ –2(3 + 5x) sin 2x – 5 cos 2x
___ dx dx _______________________
​ ​   ​​ = ​​   ​​   = ​​         ​​
dx v2 (3 + 5x)2

416

Pre-Calculus and Calculus.indb 416 13/7/2017 5:57:34 PM


Chapter 13 • Differentiation

(c) y = cos2 (1 – x2)


du
Let u = 1 – x2  ___ ​​    ​​ = –2x
dx
dy
y = cos u  ___
2
​​   ​​= –2 cos u sin u
du
By chain rule,
dy dy ___
___ du
​​   ​​ = ___
​​   ​​ × ​​    ​​= (–2 cos u sin u)(–2x) = 4x cos (1 – x2) sin (1 – x2)
dx du dx
_________
(d) y = √ 
​​ 4 + sin 2x  

du
Let u = 4 + sin 2x  ___ ​​    ​​= 2 cos 2x
dx
__ dy __
___ 1 1 ​ 
–​ __
y = √  ​​ u  ​ ​​   ​​ = ​​  2 ​​ ​​ u​​  2​​
du
dy __ 1 cos 2x
​   ​​ = ​​  2 ​​​​ u​​  –​ 2 ​ ​​(2 cos 2x) = __________
___ 1
​​  _________
__
​    

dx ​√ 4 + sin 2x   ​
_____
​​ 1 – x2 ​​ 
(e) y = √  tan x
_____
du 1 1 ​ 
–​ __ x
Let u = ​​√ 1 – x2 ​​   ___​​    ​​ = __ ​​  2 ​​ (1 – x2​​)​​  2​​(–2x) = –​​ _______
_____
   ​​ 
dx ​√ 1 – ​x​​  2​ ​ 
dv
v = tan x  ​​ ___ ​​ = sec2 x
dx
dy
___ dv
___ du
___
​   ​​ = u ​​   ​​ + v ​​    ​​

dx dx dx
_____
x tan x
=√ ​​  1 – x2 ​​ sec2 x – _______​​  _____2   ​​ 
​√ 1 – ​x​​  ​ ​ 
(1 – x2) sec 2
x – x tan x
= ​​ ___________________
      _____  ​​
​√ 1 – ​x​​  2​ ​ 

Exercise 13.8
1. Find the derivatives of y = tan x and y = cosec x using first principles.

2. Find the first derivatives of the following functions.


​​ 17 ​​ x
(a) y = cos __ (g) y = e2x tan 2x
(b) y = sin 3x + tan 3x (h) y = x2 sec 4x
(c) y = sec 4x + cosec 3x (i) y = sin x cos x tan 2x
(d) y = sin (3x + 7) (j) y = sin2 (2x + 3)
(e) y = sin (2x2 + 3x + 2) (k) y = cos3 (2x2)
(f) y = sin 2x tan 2x (l) y = 2 cosec2 (2x – 7)

3. Find the second derivatives of the following functions.


(a) y = sin 2x cos 2x
(b) y = tan2 3x
(c) y = –cosec 2x

417

Pre-Calculus and Calculus.indb 417 13/7/2017 5:57:34 PM


Chapter 13 • Differentiation

13.9  Derivatives of Inverse Trigonometric


Functions

Theorem 13.13

The derivatives of inverse trigonometric functions are


___ d 1   ​​ 
​​    ​​ sin–1 x = _______
​​  _____
dx ​√ 1 – ​x​​  2​ ​ 
___ d –1   ​​ 
​ dx  ​ cos–1 x = _______ ​​  _____
​√ 1 – ​x​​  2​ ​ 
d
​​  1  2 ​​ 
___
​ dx   ​ tan–1 x = _____
1+x
___d _______ 1   ​​ ​​   d
___
​​    ​​ sin u = ​​  _____
–1
  ​​( u), where u =  f (x)
dx ​√ 1 – ​u​​  2​ ​  dx
___d –1   ​​ ​​   d
​​    ​​ cos–1 u = _______ ​​  _____ ___
  ​​( u), where u =  f (x)
dx 2 dx
​√ 1 – ​u​​  ​ ​ 
d d
___
​​    ​​ tan–1 u = _____ ​​  1  2 ​​ ​​   ___
   ​​(u), where u =  f (x)
dx 1 – u dx

Example 36
Differentiate the following with respect to x.
(a) sin–1 (3x + 5) (b) cos–1 (​​  __ ​  1x ​– 2)​​   (c) tan–1 (sin x)
SOLUTION
d 1 
(a) ​​ ___   ​​ sin–1 (3x + 5) = ____________ ​​    ___________  ​​ (3)
dx ​√ 1 – (3x + ​5)​​ 2​ ​ 
3
= ​​ __________________
   _________________    ​​
√​  1  – (9​x​​  ​+ 30x + 25) ​
2

3
= ________________
​​     _______________    ​​
√​  –(9​
   x​​  ​+ 30x + 24) ​
2

​  1x ​– 2)​​  = –​​ ____________ –1 ​  ​​  


2 ( x2 )
(b) ​​ ___   ​​ cos–1 (​​  __
d 1 
  __________  ​​ ​​  ​ ___
√ 
dx
​ 1 – (​​  __ ​  1x ​– 2)​​​   ​ ​ 
= ​​ _________________
______________ 1   ​​
√ 
  
​x​​  ​​ 1  
2
– (​  ​  2  ​ – __
1
__
​  4x ​+ 4)​ ​ 
​x​​  ​
_________________
= ​​     ______________ 1   ​​
​x​​  ​​ –​ 
2
  
1
__
√ 
 2 ​ (1 – 4x + 3​x​​  2​) ​
​x​​  ​
1 
= ​​ _______________
   _____________  ​​
x​√    –(3​x​​  2​– 4x + 1) ​
d 1    
(c) ​​ ___   ​​ tan–1 (sin x) = ________
​​  ​​ (cos x)
dx 1 + sin2 x
cos x
= ________
​​      ​​
1 + sin2 x

Exercise 13.9
1. Differentiate the following with respect to x.
  ​​  
cos x
____
( e​ ​ ​x ​ )
(a) sin–1 (ln x2) (b) cos–1 (e3x – 1) (c) tan–1 ​​  ​  2
 ​

418

Pre-Calculus and Calculus.indb 418 13/7/2017 5:57:34 PM


Chapter 13 • Differentiation

13.10 Linearization and Differentials


13.10.1 Linearization
13.10.2 Differentials

13.10.1 Linearization
By the definition of differentiation, we know that at any given point on the
graph of function y =  f (x) with x = a,
f (x) – f (a) _________
f (x) – f (a)
→a
 ​​​ _________
f '(a) = x​​  lim​ ​​  x – a    ​​≈ ​​  x – a   
  ​​, given that x and a are close.

f (x) – f (a)
Hence,  f '(a) ≈ _________
​​  x – a    ​​, which can also be written as:

 f (x) ≈  f (a) +  f '(a)(x – a)


which is the linear approximation to a function at the point x = a.
y =  f (x)
y

y = L(x) when x = a
(a,  f (a))

x
0 a

Linearization is to find the linear approximation to a function at a given


point, with the gradient equals to the gradient of the tangent at the point
(a,  f (a)), which is defined as:
L(x) =  f (a) +  f '(a)(x – a)
where the gradient of the line  f '(a) is the first derivative of the function
y =  f (x) at x = a.

Example 37
Find the linearization of the function y = x2 + 2x – 3 at x = 2.
SOLUTION
Since  f (x) = x2 + 2x – 3,  f (2) = 5.
The first derivative  f '(x) = 2x + 2, hence  f '(2) = 6.
y = x2 + 2x – 3
y y = 6x – 7

(2, 5)
x

The linearization at x = 2:
L(x) = 5 + 6(x – 2) = 6x – 7
419

Pre-Calculus and Calculus.indb 419 13/7/2017 5:57:35 PM


Chapter 13 • Differentiation

Example 38
Find the linearization of function  f (x) = sin (x – 1) at x = 3.
Hence, approximate the value of sin (3.01) and sin (2.98).
SOLUTION
 f (3) = sin (2) ≈ 0.9093
 f '(x) = cos (x – 1)
 f '(3) ≈ –0.4161
The linearization at x = 3:
L(x) =  f (3) +  f '(3)(x – 3)
= 0.9093 – 0.4161(x – 3)
= –0.4161x + 2.158
y

y = –0.4161x + 2.158
y = sin (x – 1)

Using the linearization,


sin (3.01) ≈ L(3.01) = –0.4161(3.01) + 2.158 = 0.9055
sin (2.98) ≈ L(2.98) = –0.4161(2.98) + 2.158 = 0.9180
Remarks:
Without using linearization, the function values when x = 3.01 and
x = 2.98 are 0.9051 and 0.9174 respectively. With more values estimated
with linearization and comparing the actual function values, we can
obtain the table below.
x L(x) = –0.4161x + 2.158  f (x) = sin (x – 1) Error = | f (x) – L(x)|
2.7 1.0344 0.9917 0.0427
2.8 0.9929 0.9738 0.0191
2.9 0.9513 0.9463 0.0050
2.98 0.9180 0.9174 0.0006
3 0.9097 0.9093 0.0004
3.01 0.9055 0.9051 0.0004
3.1 0.8681 0.8632 0.0049
3.2 0.8265 0.8085 0.0180
From the above table, we can observe that linear approximation is more
accurate when the value of x is closer to the point chosen (for instance,
when x = 3). In general, the error will get larger when x is further away
from the point chosen for linearization.

420

Pre-Calculus and Calculus.indb 420 13/7/2017 5:57:35 PM


Chapter 13 • Differentiation

13.10.2 Differentials
Calculus plays a very important role in the development of modern
mathematics and engineering. As an essential component of calculus,
differentiation is based on the idea of differentials. There are many ways
to understand the notation of differentiation. One of them can be presented
in a geometrical way as seen below.
y y =  f (x)

Q
 f (x + Δx)
dy Δy
P
 f (x)
Δx
(dx)
x
x x + Δx
(x + dx)
Note
Imagine that the point Q is approaching the point P as Δx is getting smaller
dy d f (x)
and smaller. The difference between the corresponding function values Rigorously, ___
​​   ​​ or ​​ _____
  
​​ 
dx dx
Δx
will also get smaller and smaller. The ratio ​​ ___ Δy  ​​will be approaching to the
means performing an
operation (differentiation)
gradient of tangent at point P. Hence, for a differentiable function y =  f (x),
on a differentiable function
the derivative  f '(x) can be defined as y =  f (x), which doesn’t mean
Δ f (x) _____ d f (x) ___ dy dy being divided by dx.
 f '(x) = ​​  lim​ ​​​ _____
​​  Δx    ​​ = ​​ 
      ​​ = ​​   ​​  .
Δx → 0 dx dx
dy
In the above notation, ___
​​   ​​ means the (first) derivative of y with respect to x,
dx
and graphically, it gives the gradient of tangent of the curve y =  f (x) at any
specific point. The notions of dx and dy are called differentials.
We can also rewrite the above expression in the following way:
dy =  f '(x)dx
Here, the differential dx is a independent variable and dy is a dependent
variable.
The precise meaning of differential varies under different circumstances.
Conventionally, dx means infinitesimal (infinitely small) increment of x,
and dy means the change of value of y correspondingly, which is also
considered to be very small.

421

Pre-Calculus and Calculus.indb 421 13/7/2017 5:57:35 PM


Chapter 13 • Differentiation

Example 39
Given a function y =  f (x) = x3 + 2x, find the values of ∆y and dy when
x changes from 1 to 1.01.
SOLUTION
When x changes from 1 to 1.01, ∆x = 1.01 – 1 = 0.01 ≈ dx.
 f (1) = 13 + 2(1) = 3
 f (1.01) = (1.01)3 + 2(1.01) = 3.050301
∆y =  f (1.01) –  f (1) = 0.050301
Since  f '(x) = 3x2 + 2,  f '(1) = 3(1)2 + 2 = 5
Hence, dy =  f '(x) dx = 5(0.01) = 0.05

Example 40
The height and radius of a circular cone are the same. The measurement
of its height is 10 cm with maximum error of ±0.05 cm.
(a) Use differential to estimate the maximum error of the volume of the
cone.
(b) What is the percentage (relative) error of the volume?
SOLUTION
1 2
(a) Volume of cone V = ​​ __
3 ​​  πr h.
Since the height and radius are the same for the cone,
V = __ ​​  31 ​​  πr2(r) = ​​ __ 1 3
3 ​​  πr
V' = πr2
When r = 10, V' = π(10)2 = 314.2
When dx = 0.05,
dV = V'dr = 314.2(0.05) = 15.71 cm3
​​  31 ​​  πr3 = __
V = __
(b)  ​​  31 ​​  π(10)3 = 1047.2 cm3
15.71
Percentage error = ______ ​​ 1047.2  ​​ 
× 100% = 1.50%

422

Pre-Calculus and Calculus.indb 422 13/7/2017 5:57:35 PM


Chapter 13 • Differentiation

Exercise 13.10
1. Find the linearization L(x) of the following functions at the given points.
f (x) = x3______
(a)   + x2, x = 3
f (x) = √ 
(b)   ,x=4
​​ 2x + 1 ​​ 
f (x) = e , x = 1
(c)   3x – 3

2. Find the linear approximation of the function y = ln (2x + 1) at x = 0 and use it to approximate the
values of ln (0.98) and ln (1.01).
______ ____ ____
3. Find the linear approximations of the function y = √  , at x = 2, √ 
​​ 4x2 – 1 ​​  ​​ 14.8 ​​ and √
​​  15.2 ​​. 

4. Find the differential dy of the following functions.


(a) y = (​​​  3x – __
​ 1x ) ​ ​​​   ​​
6

_____
(b) y = √​​  x2 – x   ​​

5. Find the differential dy and evaluate it for the given values of x and dx.
(a) y = ln (2x – 1), x = 2, dx = 0.01
(b) y = 2 cos (ex), x = 0, dx = –0.02

6. The surface area of a sphere is measured to be 23 cm2 with a possible largest error of ±0.1 cm2. Use
differentials to estimate the largest error of the surface area of the sphere.

423

Pre-Calculus and Calculus.indb 423 13/7/2017 5:57:35 PM


Chapter 13 • Differentiation

13.11 L’Hôpital’s Rule


For a simple function such as  f (x) = 2x – 1, it is easy to analyze the
behavior of it near to a given point. For example, when the value of x
is approaching 2, we know the function value is also approaching
 f (2) = 3, which is simply the corresponding function value.
y
 f (x) = 2x – 1
3

x
0 2
–1

This analysis can be presented mathematically:


​​  lim​ ​​​   f (x) =  f (2) = 3
x→2

The limit of a function being equal to the function value is related to a


concept called continuity, which has been covered in Chapter 12, Section
12.5. However, for some functions, this analysis would appear to be more
difficult. For example, a function F(x) is given as
sin x
F(x) = ____
​​  x    ​​. 
Suppose we would like to know the behavior of F(x) near x = 1,
sin x _____ sin 1
​​  lim​ ​​​ F(x) = ​​  lim​ ​​​ ____
​​  x    1  = sin 1 = F(1).
 ​​ = ​​   ​​ 
x→1 x→1

We know when the value of x is approaching to 1, the function values


would be approaching to sin 1 ≈ 0.8415. However, if we want to analyze
the behavior of function near to another point, x = 0, the same computation
would fail because the function is undefined at x = 0.
Solution to this problem involves a famous rule called the L’Hôpital’s rule.
Note
a can be any real number, Theorem 13.14
positive or negative infinity.
Given functions  f (x) and g(x) such that either
 ​​​   f (x) = x​​  lim​
​​ xlim​
→a →a
 ​​​ g(x) = 0 or x​​ lim​
→a
 ​​​   f (x) = x​​  lim​
→a
 ​​​ g(x) = ∞
Note then
f (x) f '(x)
We can only apply ​​ xlim​ ​​​ ____
→ a g(x)
​​   ​​ = ​​ xlim​  ​​​ ____
→ a g'(x)
​​   ​​  ,
L’Hôpital’s rule to a
f ′(x)
quotient which satisfies the provided x​​  lim​ ​​​ ____
​​ 
→ a g′(x)
 ​​ exists.
conditions, which sometimes
can simply be presented as
0 ∞
the form __
​​  0 ​​  or ​​ __
∞ ​​.

424

Pre-Calculus and Calculus.indb 424 13/7/2017 5:57:35 PM


Chapter 13 • Differentiation

sin x
To find ​​  lim​ ​​​ F(x) = ​​  lim​ ​​​ ____
​​  x    ​​, 
x→0 x→0

since ​​  lim​ ​​​ sin x = ​​  lim​ ​​​ x = 0, we can apply L’Hôpital’s rule:
x→0 x→0
d
___
​    ​  (sin x) cos (0)
sin x
____ dx
________ cos x _______
​​  lim​ ​​​ F(x) = ​​  lim​ ​​​ ​​  x     ​​ = ​​  lim​ ​​​ ​​   ​​ 
 = ​​  lim​ ​​​ _____
​​  1 ​​ 
 = ​​ 
1 ​   ​= 1
x→0 x→0 x→0 d
___ x→0
​    ​  (x)
dx
sin x
which means even though F(x) = ​​  ____ x   
 ​​is undefined at the point x = 0,
when x is approaching 0, the function value will be approaching to 1. This
is illustrated in the graph below.
y

undefined when x = 0

sin x
y = ​​ ____
x   ​​ 

Example 41
Find the following limits.
1 – cos x ln x
(a) ​​ lim​ ​​​ ________
​​  x    ​​
  (b) ​​ xlim​  ​​​ ____
→ ∞ 2x
​​   ​​ 
x→0

SOLUTION
(a) Since ​​  lim​ ​​​ (1 – cos x) = 0 and ​​ lim​ ​​​ x = 0, the limit of this function is
x→0 x→0
0
__
of the form ​​  0 ​​ .
Applying L’Hôpital’s rule,
d
___
​     ​ (1 – cos x)
1 – cos x
________ dx
____________ sin x __ 0
​​  lim​ ​​​ ​​  x    ​​ = ​​  lim​ ​​​   
  ​​   ​​   = ​​  lim​ ​​​ ____
​​  1 ​​   = ​​   ​​  = 0
1
x→0 x→0 d
___ x→0
​     ​ (x)
dx
(b) Since x​​  lim​
→∞

​​​ ln x = ∞ and ​​ 
x→∞
lim​  ​​​ 2x = ∞, the limit of this function is of
the form ​​  ∞ .​​ ∞
__

Applying L’Hôpital’s rule,


___ d
ln x
​     ​ (ln x)
dx ​  1x ​
__
x → ∞ 2x
____
​​  lim​ ​​​ ​​   ​​ = x​​  lim​ _______
 ​​​ ​​   ​​   __
 ​​​ ​​   ​​  = ​​ xlim​
= x​​  lim​
→∞ 2 → ∞ 2x
​​  1   ​​ = 0
 ​​​ ___
→ ∞ ___ d
​     ​ (2x)
dx

425

Pre-Calculus and Calculus.indb 425 13/7/2017 5:57:35 PM


Chapter 13 • Differentiation

Sometimes, the L’Hôpital’s rule may be applied more than once in a


question.

Example 42
x2
Evaluate ​​  lim​ ​​​ ​​ _________
1 – cos 2x  
.​​
x→0

SOLUTION
Since ​​  lim​ ​​​ x2 = 0 and ​​  lim​ ​​​ (1 – cos 2x) = 0, the limit of this function is
x→0 x→0
0
__
of the form ​​  0 ​​ .
Applying L’Hôpital’s rule,
d
​ ___   ​  (x2)
x2
_________ dx
_____________ 2x x
​​  lim​ ​​​ ​​      
​​ = ​​  lim​ ​​​   ​​     ​​ = ​​  lim​ ​​​ _______
​​  ​​ = ​​  lim​ ​​​ _____
     ​​      
​​
x → 0 1 – cos 2x x → 0 ___ d x → 0 2 sin 2x x → 0 sin 2x
​     ​ (1 – cos 2x)
dx
Since ​​  lim​ ​​​ x = 0 and ​​  lim​ ​​​ sin 2x = 0, the limit of this function is also of
x→0 x→0
0
__
the form ​​  0 ​​ .
Applying L’Hôpital’s rule again,
d
​ ___   ​  (x)
x dx
_____
​​  lim​ ​​​ ​​       _________
​​ = ​​  lim​ ​​​ ​​    ​​   ​​  1    
= ​​  lim​ ​​​ _______ ​​ = ______ 1   ​​ 
​​  2 cos __ 1
0 = ​​  2 ​​ 
x → 0 sin 2x x → 0 ___ d x → 0 2 cos 2x
​     ​ (sin 2x)
dx
x
​​  21 ​​ .
2
Hence, ​​  lim​ ​​​ ​​ _________      ​​ = __
x → 0 1 – cos 2x

The L’Hôpital’s rule can also be used to find limits of the form 0  ∞ or
other forms. All we need to do is to rearrange these expressions into a
0 ∞
quotient of the form __
​​ 0 ​​  or __
​​  ∞ ​​.

Example 43
Evaluate ​​  lim​ ​​​ x2 ln x.
x→0

SOLUTION
ln x
​​  lim​ ​​​ x2 ln x = ​​  lim​ ​​​ ____ ​​  1  ​​ 
x→0 x → 0 __
​  2  ​ 
x
__1
Since ​​  lim​ ​​​ ln x = –∞ and ​​  lim​ ​​​ ​​  2  ​​ = ∞, the limit of this function is of the
x→0 x→0 x

form __ ∞ .​​
​​  ∞
Applying L’Hôpital’s rule,
___ d
ln x
____
​     ​  (ln x)
dx
_______ ___ ​  1x ​
__
1 ​​ x2 = –​​ __
1 ​​ (0)2 = 0
​​  lim​ ​​​ ​​  1  ​​ = ​​  lim​ ​​​ ​​    ​​ = ​​  lim​ ​​​ –​​ __
​     ​ (​  ​  2  ​ )​ 
= ​​  lim​ ​​​ ​​  ___
 ​​  
x → 0 __ x → 0 ___ d 1
__ x → 0 –2 x→0 2 2
​  2  ​  ​  3 ​ 
​x​​  ​ dx x x
Hence, ​​  lim​ ​​​ x2 ln x = 0.
x→0

426

Pre-Calculus and Calculus.indb 426 13/7/2017 5:57:35 PM


Chapter 13 • Differentiation

Exercise 13.11
1. Find the following limits.
x–2
(a) ​​ lim​ ​​​ _____
​​  2    ​​ 
x→2 x – 4

x2 – 5x + 6
(b) ​​ lim​ ​​​ _________
​​  x – 3 ​​   
x→3

ex – 1
(c) ​​ lim​ ​​​ _____
​​   ​​ 
x → 0 sin x

x2 + 2x
(d) ​​ lim​ ​​​ ______
​​  tan x    ​​
x→0
__
​√ x ​ –1
(e) ​​ lim​ ​​​ ______
​​  3 __    ​​ 
x → 1 √ 
  ​ x ​ – 1
π–x
 ​​​ ______
(f) ​​ lim​ ​​    ​​ 
x→π 2 sin x
sin (x – 3)
(g) ​​ lim​ ​​​ _________
​​  2  ​​ 

x→3 x –9

2. Evaluate the following limits. (Hint: You may need to apply the L’Hôpital’s rule more than once.)
__ ex
(a) ​​ xlim​
→ ∞ x2
  ​​​ ​​    ​​
1 – cos 2x
(b) ​​ lim​ ​​​ _________
​​   ​​ 

x→0 2x2
1 + cos 4x
(c) ​​ lim​__π ​​​ _________
​​   ​​ 
x → ​ 4 ​  1 + sin 6x

1 – cos x
(d) ​​ lim​ ​​​ ___________
​​   
2x  
​​
x → 0 (x + 1) – e
2

(ln x)3
______
(e) ​​ xlim​
→∞
  ​​​ ​​  x   ​​ 

3. By rearranging into a quotient and applying the L’Hôpital’s rule, find the following limits.
 ​​​ x2e–x
(a) x​​  lim​
→∞

(b) ​​  lim​__π ​​​ tan x – sec x


x → ​ 2 ​ 

(c) ​​  lim​ ​​​ (​​  ____


x → 1 ln x
​  1   ​ – _____ )​​  
​  x –1 1 ​ 

427

Pre-Calculus and Calculus.indb 427 13/7/2017 5:57:35 PM


Limits and Derivatives
Chapter 14:
Applications of
Differentiation

Economics

Calculus is used to help interpret economic information. Business analysts and


governments around the world need to make sense of the huge masses of statistical
data based on thousands of variables received every day to make sound and reasonable
decisions, or propose alternatives, or predict behaviours.
The application of mathematical methods allows these decision makers to analyze complex
data and variables efficiently, by representing them in simpler theoretical relationships.
The use of differential calculus to explain maximization, or determine marginal product,
revenue and cost leads to a certain economics decision.

Pre-Calculus and Calculus.indb 428 13/7/2017 5:57:35 PM


Chapter 14 • Applications of Differentiation

14 A
 pplications of Differentiation
14.1 Equations of Tangent and Normal
14.2 Related Rates
14.3 Maximum and Minimum Values
14.4 Graphing Using First and Second Derivatives
14.5 Optimization Problems
14.6 Applications in Economics and Business

14.1 Equations of Tangent and Normal


dy
Let P (x1, y1) be a point on the curve y =  f (x) and  f ′(x) = ​​ ___ ​​ be the gradient
dx
of the curve at any point. A straight line passing through P and touching the y
curve y = f (x) at P is known as tangent line (or tangent in short) and the
y = f(x)
gradient of the tangent is  f ′(x1). A straight line perpendicular to the tangent
normal tangent
is called normal and the gradient is –​​ ____ 1    ​​ (see Figure 14a). Since the
f ′(x)
equation of a straight line with gradient m and passing through the point P(x1, y1)

(x1, y1) is given as y – y1 = m(x – x1), then the equation of the tangent to the
x
curve at P is 0
y – y1 =  f ′(x1)(x – x1), Figure 14a Tangent and Normal at P
and the equation of the normal is
1   ​​  
y – y1 = –​​ ____ (x – x1).
f ′(x)
Example 1
Find the equation of the tangent to the curve y = 2x2 at the point (2, 8).
SOLUTION
Let f (x) = 2x 2.
Using the first principles,
f (x + h) – f (x)
____________
f ′(x) = ​​  lim​ ​​​    ​​    ​​

h→0 h
At the point (2, 8),
f (2 + h) – f (2)
____________
f ′(2) = ​​  lim​ ​​​    ​​    ​​

h→0 h
2(2 + h)2 – 2(22)
______________
= ​​  lim​ ​​​ ​​      ​​ 
h→0 h
2(4 + 4h + h2) – 8
= ​​  lim​ ​​​ ​​ _______________
     ​​ 
h→0 h
8h + 2h 2
= ​​  lim​ ​​​ ​​ _______  ​​   
h→0 h
= ​​  lim​ ​​​  (8 + 2h)
h→0
=8
Therefore, the equation of the tangent at the point (2, 8) on the curve is
y – 8 = 8(x – 2)
y – 8 = 8x – 16
y = 8x – 8 429

Pre-Calculus and Calculus.indb 429 13/7/2017 5:57:36 PM


Chapter 14 • Applications of Differentiation

Example 2
Find the equations of the tangent and normal to the circle y2 + x2 – 25 = 0
Note at the point (3, –4).
Use implicit differentiation
SOLUTION
Differentiating y2 + x2 = 0 with respect to x,
dy
2y​​ ___ ​​ + 2x = 0
dx
dy
2y​___ ​   ​​ = –2x
dx
dy x
​​ ___ ​​ = –​​ __y ​​
dx
dy –3 __ 3
At the point (3, –4), ___ ​​   ​​ = ___
​​   ​​ = ​​   ​​ 
dx –4 4
Therefore, the equation of the tangent to the circle at the point (3, –4) is
3
y + 4 = __ ​​ 4 ​​ (x – 3)
4y + 16 = 3x – 9
4y – 3x + 25 = 0
Gradient of the normal at the point (3, –4) is –​​ __ 4  ​​.
3
Therefore, the equation of the normal is
y + 4 = –​​ __ 4
3 ​​ (x – 3)
3y + 12 = –4x + 12
3y + 4x = 0

Example 3
1  ​​x3 – __
Find the coordinates of the points on the curve y = ​​ __ ​​  21 ​​ x2 – 12x + 2
3
where the tangent is parallel to the x-axis.
SOLUTION
1  ​​x3 – __
Differentiating y = ​​ __ ​​  21 ​​ x2 – 12x + 2 with respect to x,
3
dy
​​ ___ ​​ = x2 – x – 12
dx
Since the tangent is parallel to the x-axis, then
dy
​​ ___ ​​ = 0
dx
Therefore,
x2 – x – 12 = 0
(x – 4)(x + 3) = 0
x = 4  or  x = –3
1  ​​(4)3 – __
When x = 4, y = ​​ __ ​​  21 ​​ (4)2 – 12(4) + 2
3
= –32​​ __ 2  ​​
3
1
When x = –3, y = ​​  3 ​​ (–3)3 – __
__
​​  21 ​​ (–3)2 – 12(–3) + 2

= 24​​ __ 1  ​​
2
The coordinates of the points are (​​  4, –32​ __ 3 ​ )​​  and (​​  –3, 24​  2 ​ )​​. 
2 __ 1
430

Pre-Calculus and Calculus.indb 430 13/7/2017 5:57:36 PM


Chapter 14 • Applications of Differentiation

However, some functions are not differentiable, that is , they do not have
a tangent line. Below are some cases.
(i) y
y = |x|

The graph is not differentiable for all real values of x.


(ii) y

x
a

The graph is not differentiable at x = a.


(iii) y

x
a

The graph is not differentiable at x = a as the gradient of a vertical


tangent line is undefined.

431

Pre-Calculus and Calculus.indb 431 13/7/2017 5:57:36 PM


Chapter 14 • Applications of Differentiation

Exercise 14.1
1. Find the equations of the tangent and normal for each of the following curves at the given point or
given value of x.
(a) y = x2, (2, 4)
(b) y = 3x + 2,
2
(4, 50)
(c) y = 3x2 – x + 1, x = 0
(d) y = –2x2 – 4x + 3, x = 1
(e) y = –x3 + 9x, x = –3

2. Find the equation of the tangent to the curve y = 3x2 – 4x2 + 2x – 10 at the point where the curve cuts
the y-axis.

3. Find the points on the curve y = x3 – 9x2 + 20x where the tangents are parallel to the line y = –4x + 3.

4. The tangent to the curve y = x2 + 5x – 2 at the point (1, 4) intersects the normal to the same curve at
the point (–3, –8) at point P. Find the coordinates of P.

5. The normal to the curve y = 2x3 – 12x2 + 23x – 11 at the point where x = 2 intersects the curve again
at the points Q and R. Find the coordinates of Q and R.

6. The tangent to the curve y = px2 + 1 at the point (1, q) is parallel to the line y – 6x = 2. Find the values
of p and q.

7. Find the equations of the tangent and normal of the following curves at the given points.
(a) y = x cos x at the point (π, –π)
π
(b) y(1 + 2 sin x) = 1 at the point x = __
​​  6 ​​ 

8. Find the gradient of the curve y = 3x(1 – x) at the point x = a. Hence, if a = 1, show that the equation
of the tangent to the curve at that particular point is y = 3 – 3x.

432

Pre-Calculus and Calculus.indb 432 13/7/2017 5:57:36 PM


Chapter 14 • Applications of Differentiation

14.2 Related Rates


14.21 Rates of Change
14.2.2 Connected Rates of Change

14.2.1 Rates of Change


dy
If the variable y changes with time, then ___
​​   ​​ is the rate of change of y with
dt
respect to time.
Example 4
Given that A = 2t3 – 3t, find the rate of change of A with respect to t
when t = 3.
SOLUTION
dA
​​ ___ ​​ = 2(3)t2 – 3
dt
= 6t2 – 3
When t = 3,
___dA
​​   ​​ = 6(32) – 3
dt
= 51

Example 5
Suppose a car has traveled (5 + 2t 2) kilometers after t hours, find the
rate of change of the distance traveled when t = 3.
SOLUTION Note
Let f (t) = 5 + 2t 2. The rate of change of
d
Rate of change of distance traveled = __ ​​    [​​ f (t)] distance traveled is the speed
dt of the car.
= f '(t)
Using the first principles,
f (t + h) – f (t)
f '(t) = ___________
   ​​    ​​ 
h
5 + 2(t + h)2 – (5 + 2t2)
___________________
= ​​  lim​ ​​​ ​​      ​​

h→0 h
5 + 2(t + 2ht + h ) – 5 – 2t2
2
_______________________
2
= ​​  lim​ ​​​ ​​         ​​
h→0 h
5 + 2t2 + 4ht + 2h2 – 5 – 2t2
______________________
= ​​  lim​ ​​​ ​​         ​​
h→0 h
4ht + 2h2
= ​​  lim​ ​​​ ​​ ________   ​​ 
h→0 h
= ​​  lim​ ​​​  4t + 2h
h→0
= 4t
When t = 3, f '(t) = 12.
Hence, the car is traveling at a rate of 12 km/h.

433

Pre-Calculus and Calculus.indb 433 13/7/2017 5:57:36 PM


Chapter 14 • Applications of Differentiation

Example 6
An inverted cone of radius 5 cm and height 10 cm with its axis held
vertical is completely filled with water. Water is then drained out from
a small hole at the bottom. If it takes 5 seconds for the water to be
completely drained, find the rate of change of the volume of water in
the cone.
SOLUTION
​​ 13  ​​πr2h
Volume of water in the cone = __
1
= ​​ __
3 ​​  × π × 5 × 12
2

= 100π cm3
dV
Rate of change of volume of water = ___
​​   ​​ 
dt
= 100π cm3 ÷ 5 seconds
= 20π cm3/s
The volume of water is decreasing at a rate of 20π cm3/s.

14.2.2  Connected Rates of Change


The chain rule and implicit differentiation are frequently used to solve
problems on connected rates of change. Suppose the variable y is related
dy dx
to the variable x by an equation, then we can calculate ___ ​​   ​​ in terms of ___
​​   ​​ , x
dt dt
and y by differentiating the equation with respect to t.
Consider the equation x2 – y2 = 25.
Differentiating both sides with respect to t, we obtain
dx dy
2x ​​ ___ ​​ – 2y ​​ ___ ​​ = 0 Implicit differentiation
dt dt
dy x ___ dx
​​ ___ ​​ = __ ​​   ​​ ​​   ​​ 
dt y dt
dx dy
If x, y and ___ ​​   ​​ are known, then ___ ​​   ​​ can be calculated.
dt dt
We can also differentiate the equation using the chain rule.
Differentiating both sides with respect to x, we have
dy
2x – 2y ​​ ___ ​​ = 0
dx
dy __x
___
​​   ​​ = ​​  y ​​
dx
dy dy ___ dx
Then, ​​ ___ ​​ = ___ ​​   ​​ × ​​   ​​   Chain rule
dt dx dt
x dx
= __ ​​  y ​​ ​​ ___ ​​ 
dt
The following steps can be used as a guide when solving problems on
related rates.
Step 1:  Write down the expression (equation) that relates the variables.
Step 2: Differentiate the equation implicitly with respect to time (or use
the chain rule).
Step 3:  Substitute all known values in the equation obtained.
434 Step 4:  Solve for the required rate of change.

Pre-Calculus and Calculus.indb 434 13/7/2017 5:57:36 PM


Chapter 14 • Applications of Differentiation

Example 7
The area of a circle is increasing at a rate of 6 cm2 s–1. Find the rate of
increase of the radius when the radius is 1.5 cm.
SOLUTION
Let A be the area of a circle.
A = r2
___ dA
​​   ​​ = 2 r
dr
dA dr
Given ___
​​   ​​ = 6π, we need to find __ ​​    ​​.
dt dt
dA dA __ dr
Using chain rule, ___ ​​   ​​ = ___
​​   ​​ × ​​    ​​
dt dr dt
dr
6π = 2πr × __ ​​    ​​
dt
dr ___
__ 6π
​​    ​​ = ​​  2πr   ​​
dt
3
= __ ​​  r ​​
When the radius is 1.5 cm, the rate of increase of the radius of the circle
3
= ​​ ___
1.5   ​​ = 2 cm s .
–1

Alternative method:
We can differentiate A = r2 implicitly with respect to t.
___dA dr
​​   ​​ = 2πr ​__​    ​​
dt dt
dA dr 6π
Substituting ​​ ___ ​​ = 6π, we have __ ​​    ​​ = ​​ ___  ​​ .
dt dt 2πr
dr 3
When r = 1.5 cm, __ ​​    ​​ = ​​ ___
   ​​ = 2 cm s–1.
dt 1.5

Example 8
Water is poured into an inverted cone of depth 10 cm and radius 5 cm
at a rate of 8 cm3 s–1. Find the rate at which the depth of the water in the
cone is increasing when the depth is 4 cm.

SOLUTION 5 cm
Let r = radius of the base of the cone
h = depth of the cone
r​​   ​​ = ___
__ 5
​​    ​​ 
h 10 10 cm
h
r = ​​ __ 2 ​​ 
Volume of the cone, V = ​​ __ 1 2
3 ​​ πr h

3 ( 2 ) ​​h
2
1 ​​ π​​​  __ h
= ​​ __ ​   
 ​ ​​​​ 
π 3
= ​​ ___
12  ​​ h
dV π 2
​​ ___  ​​ = ​​ __  ​ h ​
dh 4
435

Pre-Calculus and Calculus.indb 435 13/7/2017 5:57:36 PM


Chapter 14 • Applications of Differentiation

dV dV ___ dh
Using chain rule, ___
​​   ​​ = ___
​​    ​​ × ​​   ​​ 
dt dh dt
π dh
8 = __ ​​  4 ​ h2​ × ___​​   ​​ 
dt
dh ___
___ 32
​​   ​​ = ​​  2  ​​ 
dt πh
dh 32
When the depth is 4 cm, ___ ​​   ​​ = ___​​  ​​  2  ​​ cm s–1.
  ​​ = __
dt π(42) π
Hence, the rate at which the depth of the water in the cone is increasing
2
is ​​ __
π  ​​ cm s .
–1

Example 9
A particle moves along the curve y = x2 + 2x. Find a point on the curve
where both x and y coordinates of the particle change at the same rate.

SOLUTION
y = x2 + 2x
___dy dx dx dx
​​   ​​ = 2x ​​ ___ ​​ + 2​​ ___ ​​ = (2x + 2) ​​ ___ ​​ 
dx dt dt dt
dy dx
When ___​​   ​​ = ___​​   ​​ ,
dt dt
dx dx
​​ ___ ​​ = (2x + 2) ​​ ___ ​​ 
dt dt
2x + 2 = 1
2x + 1 = 0
x = –​​ __ 1  ​​
2
1 3
When x = –​​  2 ​​ , y = –​​ __
__
4  ​​  .

2 ​ , –​  4  ​)​​. 


Hence, the point on the curve is (​​  –​ __
1 __ 3

Example 10
The rate of bacteria growth in a medium is exponential. The bacteria
count in the medium is given by the function, y = 1000ekt where the
value k is a numerical constant and t is the time in minutes.
(a) What is the initial count of bacteria?
(b) Given that there was a bacteria count of 5000 after 20 min, find k.
(c) What is the rate of bacteria growth after 25 min? Correct to the
nearest whole number.

SOLUTION
(a) For t = 0, y =
 1000e0
= 1000
Initial bacteria count = 1000
(b) Since y = 5000 when t = 20, 5000 = 1000ek(20)
e20k = 5
ln e20k = ln 5
20k = ln 5
____
ln 5
436 k = ​   ​ 
20

Pre-Calculus and Calculus.indb 436 13/7/2017 5:57:36 PM


Chapter 14 • Applications of Differentiation

dy
___
(c) ​   ​ = 1000ke kt
dt
____
( )
ln 5 ____
= 1000 ​  ​  20 ​  ​ ​e​(​ ​  20 ​  )​t​
ln 5

= 50 ln 5 e​ ​(​ ​  20 ​  )​t​
____
ln 5

When t = 25 min,
dy
___ ___
ln 5
​   ​ = 50 ln 5 ​e​ ​20 ​(25)


dt _____
5 ln 5
= 50 ln 5​ e​ ​
​  4 ​  

≈ 602
Hence, the rate of bacteria growth is 602 per minute.

Exercise 14.2
1. The sides of a cube is increasing at a rate of 6 cm s–1. Find the rate of increase of the volume when
the length of its side is 9 cm.

​​ 12 ​​ cm s–1. Find the rate of increase of the area when the
2. The radius of a circle is increasing at a rate of __
radius is 4 cm.
3. The area of a square is increasing at a rate of 8 cm2 s–1. Find the rate of increase of the length of its
side when the area is 64 cm2.
4. The volume of a sphere is increasing at a rate of 16π cm3 s–1. Find the rate of increase of the radius
when the radius is 8 cm.

5. The surface area of a sphere is increasing at a rate of 4 cm2 s–1. Find the rate of increase of the radius
when the surface area is 64π cm2.
6. A hollow right circular cone of height 18 cm and radius 12 cm is held vertex downwards beneath a
tap which is leaking at a rate of 2 cm3 s–1. Find the rate of the rise of water level when the depth is
6 cm.
7. A water tank in the shape of an inverted circular cone has a radius of 20 m and a height of 12 m. Water
is pumped into the tank at a rate of 40 m3 per minute. Calculate the rate of change of the height of
water in the tank when the depth of water is 4 m.

8. A tank of water has 300 grams of salt dissolved in it. Pure water starts flowing into it while the salt
solution is drained away from the tank. The mass of salt, M in the tank after t seconds from the start
of the process is given by the formula M = M0e–0.02t.
(a) What is the value of M0?
(b) What is the rate of change of the mass of salt in the tank at time t seconds?
(c) What is the rate of change of the mass of salt in the tank after 25 seconds?
(d) When the mass of salt in the tank is 100 grams, what is the rate of change of the mass?

437

Pre-Calculus and Calculus.indb 437 13/7/2017 5:57:36 PM


Chapter 14 • Applications of Differentiation

9. A bank has an investment scheme which has a value of investment V after T years given by the
formula V = V0 e0.03T, where V0 is the initial amount invested. $10,000 was invested with the bank at
the beginning of the year.
(a) What is the rate of increase of the value of the investment after T years?
(b) After how long will the rate of increase of the value be $500 per year?
(c) After two years of the first investment, another $10,000 is invested. What would be the rate of
increase of the total value of the investments 5 years after the first investment?

10. An object is heated to a temperature of 70º C and then allowed to cool. After T minutes of cooling in
an environment where the temperature is constant at 30º C, the temperature of the object is given by
the formula θ = A + 40e–kT.
(a) Find the value of A.
(b) If the temperature after 15 minutes is 35º C, show that the value of k is 0.2 ln 2.
(c) What is the rate of change of the temperature of the object after 20 minutes of cooling?

11. In a chemical reaction, a substance X is involved which has a mass of x grams at time t seconds. The
mass of the substance X is given by the formula x = 50(1 – e–0.04t) at time t seconds.
(a) What is the mass of the substance X at the start of the reaction?
(b) What is the rate of change of the mass of substance X in terms of t?
(c) After how long will the rate of change of its mass be 0.8 grams per second?
(d) When the mass of x is 20 grams, what is the rate of change of its mass?

12. A radioactive substance decays into other substances such that T minutes from the start of monitoring
the substance, its mass in grams is M = 120e– 0.08T.
(a) After how long will the amount of the substance be half the amount of the substance at the start?
(b) What is the rate at which the substance is decaying at T = 10 min?
(c) What is the rate of change of the mass when M = 100 g?

438

Pre-Calculus and Calculus.indb 438 13/7/2017 5:57:36 PM


Chapter 14 • Applications of Differentiation

14.3 Maximum and Minimum Values


14.3.1 Critical Points
14.3.2 Increasing and Decreasing Functions
14.3.3 First Derivative Test
14.3.4 Increasing and Decreasing Functions
14.3.5 Second Derivative Test
14.3.6 Concavity and Point of Inflection

14.3.1  Critical Points


A point on a curve is called a critical point if the gradient at that point is
zero or does not exist.

Definition 14.1

A point (c, f (c)) in the domain of a function y = f (x) is called a critical


point for f if f '(c) = 0 or f ′(c) is not defined.

From the definition, we can see that there are two types of critical points.
dy
(i) When ​​ ___  ​​ = 0: This is when the tangent to the curve is parallel to the
dx
x-axis. This point is also known as the stationary point. The examples
of stationary points are shown below.
dy
___ dy
___ dy
___ dy
___
​​   ​​ = 0 ​​   ​​ = 0 ​​   ​​ = 0 ​​   ​​ = 0
dx dx dx dx

dy
(ii) When ___ ​​   ​​ is not defined: In this case, the tangent to the curve can be
dx
parallel to the y-axis (​​  ___ ​    ​ = ∞)​​  or ___
dy dy dy
​​    ​​ does not exist. ​​ ___  ​​does not exist
dx dx dx
means that the derivative on the right hand side is not equal to the
derivative on the left hand side of the point. This usually happens to
absolute functions. The examples of these critical points are shown
below.
___dy ___dy ___dy
​​   ​​= ∞ ​​   ​​= ∞ ​​   ​does not exist
dx dx dx

Note
dy dy Critical point must be
The critical point can be obtained by solving ___
​​   ​​ = 0 or ___
​​   ​​ = ∞. The value defined in the domain of the
dx dx
of y obtained is called the critical value. function y = f (x).

439

Pre-Calculus and Calculus.indb 439 13/7/2017 5:57:36 PM


Chapter 14 • Applications of Differentiation

Example 11
Find the critical points for the following curves.
(a) y = x3 + ______
7x2 – 5x + 2
​​ x2 – 16 ​​ 
(b)  f (x) = √ 

SOLUTION
(a) y = x3 + 7x2 – 5x + 2
dy
​​ ___ ​​ = 3x2 + 14x – 5
dx
dy
When ___ ​​   ​​= 0, 3x2 + 14x – 5 = 0
dx
(3x – 1) (x + 5) = 0
x = __ ​​  31 ​​  or x = –5

​​  31 ​​ , y = (​​​  __ ​  31 ​ )​​​   ​​ + 7​​​ (​ __


1 ​  ​​​​   ​​ – 5​​  __
3) (​  3 ​ )​​  + 2
3 2
When x = __ 1

1 __ 7 __ 5
= ​​ ___ 27  ​​ + ​​  9 ​​  – ​​  3 ​​  + 2
31
= ___ ​​  27 ​​ 
When x = –5, y = (–5)3 + 7(–5)2 – 5(–5) + 2
= –125 + 175 + 25 + 2
= 77
Hence, the critical points are (​​  __ ​  27 ​ )​​ and (–5, 77).
31
​  31 ​ , ___
______ ​  1 ​ 
__
(b)  f (x) = √ 
​​ x2 – 16 ​​ 
= ​​(x2 – 16)​​  2​​
1 ​ 
x
​​  21 ​​ (x2 – 1​​6)​​  2​​(2x) = ________
–​ __
 f '(x) = __ ​​  ______    ​​ 
​√ ​x​​  2​– 16 ​ 
For this type of  f '(x), we have to consider two cases for the critical
points.
x
Note Case 1: When  f '(x) = 0, ________ ​​  ______    ​​ 
=0
​√ ​x​​  ​– 16 ​ 
2

x=0 ______
______ x = 0 into
By substituting
 f (x) = √  x2 – 16 ​​, we obtain
​​ ____ Since x = 0 is not in the domain of  f (x) = √  ​​ x2 – 16 ​​ 
, then
 f (x) = √ 
​​ –16 ​​ which is not
the critical point does not exist at x = 0.
______
defined. Case 2: When  f '(x) is not defined, √ 
​​ x2 – 16 ​​ 
=0
x2 – 16 = 0
x = ±4
______
​​ x2 – 16 ​​. 
x = ±4 are in the domain of  f (x) = √ 
______
When x = 4,  f (x) = √
​​  42 – 16 ​​ 
= 0.
______
When x = –4,  f (x) = √​​  (–4)2 – 16 ​​ 
= 0.
Hence, the critical points are (4, 0) and (–4, 0).

440

Pre-Calculus and Calculus.indb 440 13/7/2017 5:57:36 PM


Chapter 14 • Applications of Differentiation

Example 12
Find the critical points of the curve 3x2 + y2 – 6x + 4y + 6 = 0.
SOLUTION
3x2 + y2 – 6x + 4y + 6 = 0
dy dy
6x + 2y ​​ ___ ​​ – 6 + 4 ​​ ___ ​​ = 0
dx dx
dy
(2y + 4) ​​ ___ ​​= 6 – 6x
dx
dy 6 – 6x
​​ ___ ​​ = ______ ​​   
 ​​
dx 2y + 4
6(1 – x)
= ​​ _______   ​​
2(y + 2)
3(1 – x)
= ​​ _______
y + 2 ​​   
dy 3(1 – x)
When ___
​​   ​​ = 0, _______
​​  y + 2 ​​   =0
dx
3(1 – x) = 0
x=1
When x = 1, 3(1) + y2 – 6(1) + 4y + 6 = 0
2

y2 + 4y + 3 = 0
(y + 3)(y + 1) = 0
y = –3 or y = –1
dy
When ___​​   ​​does not exist, y + 2 = 0
dx
y = –2
When y = –2, 3x2 + (–2)2 – 6x + 4(–2) + 6 = 0
3x2 – 6x + 2 = 0
Using quadratic formula,
____________
6±√​    
(–6) – 4(3)(2) ​
2
_________________
x = ​​     ​​ 
2(3)
___
​  12 ​ 
6±√
= _______
​​   ​​ 

2(3)
__
3±√​  3 ​ 
= ​​ ______
3 ​​   
__
​√ 3 ​ 
= 1​ ± ​ ___ ​​ 
3 __
Hence, the __critical points of the curve are (1, –3), (1, –1), (​​  1 – ​  3 ​,  –2)​​  
​√ 3 ​ 
___

​  3 ​,  –2)​​. 
and (​​  1 + ___
​  3 ​ 

14.3.2  Increasing and Decreasing Functions


Consider a continuous function f (x) given in an interval [a, d] as shown
in Figure 14b. The function f is said to be increasing on the interval
[a, b], decreasing on [b, c] and increasing again on [c, d]. Take note that if
x1 and x2 are any two numbers between a and b with x1 < x2 and f(x1) < f(x2),
then the function is called increasing function.
441

Pre-Calculus and Calculus.indb 441 13/7/2017 5:57:37 PM


Chapter 14 • Applications of Differentiation

decrease
f (x2) increase

f (x1) increase

x
a x1 x2 b c d

Figure 14b

Definition 14.2

Let f be continuous on the interval [a, b] and x1 and x2 be any two


numbers in [a, b].
(a) The function f is increasing on the interval [a, b] if  f (x1) <  f (x2)
whenever x1 < x2.
(b) The function f is decreasing on the interval [a, b] if  f (x1) >  f (x2)
whenever x1 < x2.

In order to find the intervals where a function is decreasing or increasing,


we need to use the following theorem.

Definition 14.3

Let f be continuous on the interval [a, b] and differentiable on (a, b).


(a) If  f ′(x) > 0 for every x on (a, b), then f is increasing on the interval
[a, b].
(b) If  f ′(x) < 0 for every x on (a, b), then f is decreasing on the interval
[a, b].
(c) If  f ′(x) = 0 for every x on (a, b), then f is constant on the interval
[a, b].

Figure 14b shows the gradient and shape of the increasing and decreasing
function. Note that in Figure 14c(i), gradients of both curves are positive in
the given interval, indicating that both are increasing while in Figure 14c(ii),
gradients of both curves are negative, indicating that both are decreasing.
y y dy
___
​​   ​​ < 0
___ dy dx
​​  dx ​​ > 0

x x
Figure 14c(i) Figure 14c(ii)

442

Pre-Calculus and Calculus.indb 442 13/7/2017 5:57:37 PM


Chapter 14 • Applications of Differentiation

Example 13
Given f (x) = x3 + 3x2 – 7, find the values of x where the critical point(s)
are and determine the interval(s) where the function f (x) is
(a) increasing, (b) decreasing.
SOLUTION
 f (x) = x3 + 3x2 – 7
 f ′(x) = 3x2 + 6x
When f '(x) = 0, 3x(x + 2) = 0
x = 0  or  x = –2
The critical points are at x = 0 and x = –2.
These two points divide the function into three intervals, that is, (–∞, –2),
(–2, 0) and (0, ∞).
We use Theorem 14.3 to find the increasing and decreasing interval by
testing the sign of f ′(x) for any test value k chosen from each interval.
For the interval (–∞, –2) let k = –3.
f (–3) = 3(–3)2 + 6(–3)
=9
>0
 f (x) is increasing in the interval (–∞, –2).
For (–2, 0), let k = –1.
f ′(–1) = –3
<0
 f (x) is decreasing in the interval (–2, 0).
For (0, ∞), let k = 1.
f (1) = 9
>0
 f (x) is increasing in the interval (0, ∞).
Hence, f is increasing in the intervals (–∞, –2) and (0, ∞) and decreasing
in the interval (–2, 0).
For simplicity, the above procedure can be summarised in a table as
shown below.
Interval (–∞, –2) (–2, 0) (0, ∞)
Test value, k –3 –1 1
Sign of f ′(x) +ve –ve +ve
f (x) increasing decreasing increasing

443

Pre-Calculus and Calculus.indb 443 13/7/2017 5:57:37 PM


Chapter 14 • Applications of Differentiation

Example 14
​  1 ​ 
__
If y = (x2 – 4​​)​​  3​​, find the interval(s) where the function is increasing and
decreasing.
SOLUTION
​  1 ​ 
__
y = (x2 – 4​​)​​  3​​
dy 1 2 2 ​ 
–​ __
​​ ___ ​​ = __ ​​  3 ​​ (x – 4​​)​​  3​​(2x)
dx
2x
= ​​ ________   __2 ​​ 
​   ​ 
3​(x2 – 4)​​  3​
dy 2x
When ___ ​​   ​​ = 0, ________
​​    __2 ​​  =0
dx ​  3 ​ 
3​(x – 4)​​  ​
2

x = 0
___dy
​​   ​​does not exist when x2 – 4 = 0
dx
(x + 2)(x – 2) = 0
x = –2 or 2
There are four intervals to be considered, (–∞, –2), (–2, 0), (0, 2) and
(2, ∞).
Interval (–∞, –2) (–2, 0) (0, 2) (2, ∞)
Test value, k –3 –1 1 3
Sign of f (x) –ve –ve +ve +ve
f (x) decreasing decreasing increasing increasing
Hence, f is increasing in the intervals (0, 2) and (2, ∞) and decreasing
in the intervals (–∞, –2) and (–2, 0).

14.3.3  First Derivative Test


Consider a graph of a function as shown in Figure 14d.
y
dy does not exist
=0
dx

dy
=0
dx

x
a c1 c2 c3 c4 c5

Figure 14d

From Figure 14d, it is found that when the function y =  f (x) changes from
dy dy
increasing to decreasing, there exists a point where ___ ​​   ​​ = 0 or ​​ ___ ​​ does not
dx dx
exist. This point is called a maximum point and the value of y is called
the maximum value. In Figure 14d, in the interval (a, c2), the function has
a maximum value at c1 and the gradient is zero. In the interval (c2, c4), the
dy
function has a maximum value at c3 and ___​​   ​​ at this point does not exist.
444 dx

Pre-Calculus and Calculus.indb 444 13/7/2017 5:57:37 PM


Chapter 14 • Applications of Differentiation

Similarly, when the function changes from decreasing to increasing, there


exists a minimum point. In the interval (c1, c3) and (c3, c5), the function
has minimum values at c2 and c4 respectively. The gradient is zero at
dy
x = c2, but ___
​​   ​​ does not exist at x = c4. The maximum and minimum values
dx
are known as relative maximum and relative minimum respectively.
Collectively, they are known as relative extremum and are also referred
to as local extremum.

Theorem 14.1 Note


(i) A critical point is not
If a function f has a relative extremum at x = c, then f '(c) = 0 or f '(c) necessarily a relative
does not exist. extremum but a relative
extremum is always a
The relative extremum for a function can be found by using the first critical point.
derivative test. (ii) If   f '(x) = c exists and
 f '(x) ≠ 0, then  f (c) is not
Theorem 14.2 a relative extremum.

Let  f  be continuous on an open interval (a, b) that contains the critical
point c.
(i) If  f '(x) changes from positive to negative at c, then  f  has a local
maximum at x = c.
(ii) If  f '(x) changes from negative to positive at c, then  f  has a local
minimum at x = c.
(iii) If  f '(x) < 0 (or  f '(x) > 0) for all x in the interval except for x = c,
then f  has neither a local minimum nor a local maximum at x = c.

Theorem 14.2 is illustrated by the following graphs.


(i) Relative maximum
y y
f '(c) does not exist
f '(c) = 0

x x

445

Pre-Calculus and Calculus.indb 445 13/7/2017 5:57:37 PM


Chapter 14 • Applications of Differentiation

(ii) Relative minimum


y y
f '(c) = 0

f '(c) does not exist


x x

(iii) Neither relative maximum nor relative minimum
y y y y

x x x x

Example 15
Let  f (x) = x3 – 3x + 2. Find all relative extremum for f.
SOLUTION
 f (x) = x3 – 3x + 2
 f '(x) = 3x2 – 3
When  f '(x) = 0, 3(x2 – 1) = 0
x = ±1
We have two critical points and three intervals to consider. The steps
can be presented in a table form.
Interval (–∞, –1) (–1, 1) (1, +∞)
Test value, k –2 0 2
Sign of f '(x) +ve –ve +ve
Using the first derivative test, relative maximum is at x = –1 since f '(x)
changes from positive to negative at x = –1 and relative minimum is at
x = 1 since f '(x) changes from negative to positive at x = 1.
The relative maximum value is f (–1) = (–1)3 – 3(–1) + 2 = 4.
The relative minimum value is f (1) = (1)3 – 3(1) + 2 = 0.

14.3.4  Second Derivative Test


The second derivative can also be used to investigate the nature of the
critical points of a function. Sometimes, it is easier and shorter to use this
test as compared to the first derivative test.
Theorem 14.3

Suppose  f  has a second derivative at the critical point c.


(i) If  f ''(c) > 0, then  f  has a local minimum at c.
(ii) If  f ''(c) < 0, then  f  has a local maximum at c.
(iii) If  f ''(c) = 0, then the test is inconclusive.
446

Pre-Calculus and Calculus.indb 446 13/7/2017 5:57:37 PM


Chapter 14 • Applications of Differentiation

Example 16
The stationary points of the curve 3x2 + y2 – 6x + 4y + 6 = 0 are (1, –3)
and (1, –1). Find the nature of the stationary points using the second
derivative test.
SOLUTION
3x2 + y2 – 6x + 4y + 6 = 0
From Example 12
dy 3(1 – x)
​​ ___ ​​ = _______
​​  y + 2 ​​ 

dx
dy
dy
2 (y + 2)(–3) – 3(1 – x)​ ___ ​
___ ____________________ dx
​​  2 ​​ =   
​​     2 ​​
dx (y + 2)
At the point (1, –3),
___ d2y (–3 + 2)(–3) – 3(1 – 1)(0)
​​  2 ​​ = ____________________
  
​​      ​​
dx (–3 + 2)2
=3
> 0  minimum
Hence, (1, –3) is a minimum point.
At the point (1, –1),
d2y (–1 + 2)(–3) – 3(1 – 1)(0)
​​ ___2 ​​ = ____________________
  
​​      ​​
dx (–1 + 2)2
= –3
< 0  maximum
Hence, (1, –1) is a maximum point.

Example 17
Find the maximum and minimum points of the curve y = 2 cos x – cos 2x
π
for 0  x  __
​​  2 ​​ .

SOLUTION
y = 2 cos x – cos 2x
dy
​​ ___ ​​= –2 sin x + 2 sin 2x
dx
dy
When ___ ​​   ​​= 0, –2 sin x + 2 sin 2x = 0
dx
–sin x + sin 2x = 0
–sin x + 2 sin x cos x = 0
sin x (2 cos x – 1) = 0
sin x = 0  or  cos x = __ ​​  21 ​​ 
π
x = 0 x = __
​​  3 ​​ 
___d2y
​​  2 ​​ = –2 cos x + 4 cos 2x
dx

447

Pre-Calculus and Calculus.indb 447 13/7/2017 5:57:37 PM


Chapter 14 • Applications of Differentiation

d 2y
When x = 0, ___ ​​  2 ​​ = –2 cos 0 + 4 cos [2(0)]
dx
=2
> 0  minimum
y = 2 cos 0 – cos 2(0)
=1
Hence, (0, 1) is a minimum point.
π dy
2
π 2π
When x = __ ​​  3 ​​ , ​​ ___2 ​​ = –2 cos __ ​​  3 ​​  + 4 cos ​​ __
3 ​​ 
dx
= –3
< 0  maximum
π 2π
y = 2 cos __ ​​ 3 ​​  – cos ​​ __3 ​​ 
3
= __
​​  2 ​​ 

( 2 ​ )​​ is a maximum point.


π 3
​​  __
​  3 ​ , ​ __

14.3.5  Concavity and Point of Inflection


Concave Upwards and Concave Downwards
Figure 14e shows graphs of increasing functions.
y y

x x
a c b a c b
(i) (ii)
Figure 14e

Figure 14f shows graphs of decreasing functions.


y y

x x
a c b a c b
(i) (ii)
Figure 14f

We observe that the graphs bend in different directions. In Figure 14e,  f ′(c)
exists and  f ′(c) > 0, but in Figure 14f,  f ′(c) exists and  f ′(c) < 0. If tangent
lines are drawn to the curves at several points, it is observed that:
(i) in Figure 14e(i) and Figure 14f(i), the curves lie above the tangent
lines and the gradient increases from a to b. The function f is said to
concave upwards.
(ii) in Figure 14e(ii) and Figure 14f(ii), the curves lie below the tangent
448 lines and the gradient decreases from a to b. The function f is said to
concave downwards.

Pre-Calculus and Calculus.indb 448 13/7/2017 5:57:37 PM


Chapter 14 • Applications of Differentiation

Definition 14.4

Let f  be a differentiable function on the interval (a, b).


(i) If the graph of f lies above all of its tangents on (a, b), then it is said
to concave upwards.
(ii) If the graph of f lies below all of its tangents on (a, b), then it is said
to concave downwards.

Definition 14.5

Let f  be a differentiable function on the interval (a, b).


(i) The graph of f is said to concave upwards if and only if  f ′(x)
increases on (a, b).
(ii) The graph of f is said to concave downwards if and only if
 f ′(x) decreases on (a, b).

Hence, the second derivative  f ″ can be used to determine the concavity of
the graph f by the following theorem, which is called concavity test.

Definition 14.6

Let f be a differentiable function on an interval (a, b).


(i) If  f ″(x) > 0 for all x in (a, b), then the graph of  f  is concave upwards
on (a, b).
(ii) If  f ″(x) < 0 for all x in (a, b), then the graph of f is concave
downwards on (a, b).

Point of Inflection
Point of inflection is a point on the graph which separates a part of the
graph that concave upwards from a part that concave downwards. It means
that the point of inflection exists at the point where  f ''(x) changes from
positive to negative or from negative to positive. Therefore, it only exists
at the point where  f ''(x) = 0 or  f ''(x) does not exist. The gradient of this
point can be of any value or does not exist.

Definition 14.7

A point P(c, f (c)) on a curve y = f (x) is called a point of inflection if


f is continuous at P and the curve changes from concave upwards to
concave downwards or from concave downwards to concave upwards
at P.

449

Pre-Calculus and Calculus.indb 449 13/7/2017 5:57:37 PM


Chapter 14 • Applications of Differentiation

The following are examples of graphs with different types of points of


inflection.
dy
(i) Point of inflection where ___
​​   ​​ = 0.
dx
y y

x x

dy
(ii) Point of inflection where ___
​​   ​​ does not exist.
dx
y y y

x x x

dy dy
(iii) Point of inflection where ___
​​   ​​ > 0 or ___
​​   ​​ < 0.
dx dx
y y

dy
___ dy
​​   ​​ > 0 ___
​​   ​​ < 0
dx dx

x x

Example 18
Determine all relative extremum and point of inflection for the
function  f (x) = x3 – 6x2 + 9x + 1.
SOLUTION
 f (x) = x3 – 6x2 + 9x + 1
 f '(x) = 3x2 – 12x + 9
When  f ′(x) = 0, 3x2 – 12x + 9 = 0
x2 – 4x + 3 = 0
(x – 3)(x – 1) = 0
x = 3 or x = 1

450

Pre-Calculus and Calculus.indb 450 13/7/2017 5:57:37 PM


Chapter 14 • Applications of Differentiation

 f ''(x) = 6x – 12
When x = 3,  f ''(x) = 6(3) – 12 Note
=6 If  f ″(c) = 0, we have to use
> 0  relative minimum the first derivative test.
 f (3) = 33 – 6(32) + 9(3) + 1 = 1
When x = 1,  f ''(x) = 6(1) – 12
= –6
< 0  relative maximum
 f (1) = 1 – 6(1) + 9(1) + 1 = 5
Hence, (3, 1) is a relative minimum and (1, 5) is a relative maximum.
When  f ''(x) = 0, 6x – 12 = 0
x = 2
When x < 2,  f ''(x) < 0.
When x > 2,  f ''(x) > 0.
There is a change in sign of  f ''(x), indicating that the point of inflection
exists at x = 2.
When x = 2,  f (2) = 23 – 6(22) + 9(2) + 1 = 3
The point of inflection is (2, 3).

Example 19
For the function y = 3x4 – 4x3, find the points of inflection.
SOLUTION
y = 3x4 – 4x3
___dy
​​   ​​ = 12x3 – 12x2
dx
___d2y
​​  2 ​​ = 36x2 – 24x
dx
d2y
When ___ ​​  2 ​​ = 0, 36x2 – 24x = 0
dx
3x2 – 2x = 0
x(3x – 2) = 0
​​ 23  ​​
x = 0 or __

Interval (–∞, 0) ( ​  32 ​ )​​  


​​  0, __ ( ​  32 ​ , ∞)​​  
​​  __

Test value, k –1 1 
​​ __
3 ​​ 1
Sign of f ″(x) +ve –ve +ve
2  ​​.
There exist points of inflection at x = 0 and x = ​​ __
3
When x = 0, y = 0.
​  32 ​ )​​​​   ​​ – ​​4(​  __
​​  32 ​​ , y = 3​​​ (__ ​  32 ​ )​​​​   ​​
4 3
When x = __
16
= –​​ ___
27 ​​ 
Hence, the points of inflection are (0, 0) and (​​  __ 27 ​ )​​. 
16
​  32 ​ , –​ ___
451

Pre-Calculus and Calculus.indb 451 13/7/2017 5:57:37 PM


Chapter 14 • Applications of Differentiation

Example 20
3
_____
  x – 1 ​​ 
A curve is given by the function  f (x) = ​​√  .
(a) Find the first and second derivatives of  f .
(b) Find the coordinates of the critical point(s).
(c) Determine the nature of the points, whether they are maximum,
minimum or point of inflection.
SOLUTION_____
3 1
__
(a)  f (x) = ​​√   x – 1 ​​ = (x – 1​​)​  ​ 3​  ​ ​​
  f '(x) = __​​  31 ​​ (x – 1​​)​  – ​ ​  3 ​ ​​ = ________
2
__
​​  1   ​​ 
__2
3(x – 1​)​  ​ 3​  ​ ​
f ''(x) = – ​​ __ 2 ​​   2  __​ 5 ​  ​​ 
________
5
– ​ __
  9 ​​ (x – 1​​)​  ​  ​​ = –  9(x
3 ​ 
– 1​)​  3​  ​

(b)  f '(x) does not exist at x = 1.


When x = 1, y = 0.
Note Hence, the critical point is (1, 0).
No change in  f ′ sign
(c)
 no extremum point Interval
Sign of f′(x) +ve +ve
There is no extremum point.
For  f ''(x) does not exist, x = 1.
Interval
Sign of f ″(x) +ve –ve
Since there is a change in sign of  f '', (1, 0) is a point of inflection.

Exercise 14.3
1. Determine all the stationary points of the following curves and state their nature (maximum, minimum
or point of inflection).
(a) y = x2 – 6x + 8
(b) y = 3x4 + 16x3 + 24x2 + 3
(c) y = –x3 + 3x2
(d) y = 2x3 – x4
(e) x2 + y2 – 4x + 6y = –12

2. Determine the critical points and their nature of the following curves.
3 __​ 5 ​  ​  2 ​ 
__
​  2 ​ 
__
(a) y = __
​​  5 ​​ ​​ x​​  3​​ – 6​​x​​  3​​ (b) y = ​​(x2 – 16)​​  3​​

3. Find the stationary point(s) and their nature for the following curves for 0  x  2 .
(a) y – x = 2 cos x (b) y(2 – sin x) = sin x

452

Pre-Calculus and Calculus.indb 452 13/7/2017 5:57:37 PM


Chapter 14 • Applications of Differentiation

14.4 Graphing Using First and Second


Derivatives
14.4.1 Polynomial Functions
14.4.2 Rational Functions

14.4.1  Polynomial Functions


In this section, we will concentrate on sketching the graph of polynomial
functions up to degree four.
The following steps can be used as a guide.
Step 1: Locate all critical points ( f (x) = 0 and  f '(x) does not exist).
Step 2: Find all intervals where f is increasing ( f '(x) > 0) and where f is
decreasing ( f '(x) < 0).
Step 3: Find all relative extremum.
Step 4: Find all intervals where f is concave upwards ( f ''(x) > 0) and
where f is concave downwards ( f ''(x) < 0).
Step 5: Find all inflection points ( f ''(x) = 0 or  f ''(x) does not exist).
Step 6: Find the y-intercept and the point(s) where the graph cuts the
x-axis (can be omitted if the point(s) is/are difficult to find).
Step 7: Sketch the graph by first plotting all the points mentioned above,
followed by joining the points according to the basic shapes as
shown below.
(i) increasing and (ii) decreasing and
concave upwards concave upwards

(iii) increasing and (iv) decreasing and


concave downwards concave downwards

453

Pre-Calculus and Calculus.indb 453 13/7/2017 5:57:37 PM


Chapter 14 • Applications of Differentiation

Example 21
For the function  f (x) = x4 – 8x2, find
(a) the stationary points,
(b) the intervals where  f  is increasing or decreasing,
(c) the relative maximum and relative minimum points,
(d) the intervals where  f  is concave upwards and  f  is concave
downwards,
(e) the points of inflection.
Hence, sketch the graph of  f (x).

SOLUTION
(a)  f (x) = x4 – 8x2
  f '(x) = 4x3 – 16x
When  f ′(x) = 0, 4x3 – 16x = 0
4x(x2 – 4) = 0
x = 0 or x = ±2
When x = 0,  f (x) = 0.
When x = 2,  f (x) =  (2)4 – 8(2)2
= –16
When x = –2,  f (x) =  (–2)4 – 8(–2)2
= –16
Note Hence, the stationary points are (–2, –16), (0, 0) and (2, –16).
dy (b) Interval (–∞, –2) (–2, 0) (0, 2) (2, ∞)
 f (x) is increasing when ___
​​   ​​ > 0.
dx
f '(x) –ve +ve –ve +ve
dy
 f (x) is decreasing when ___
​​   ​​ < 0. f (x) increasing: (–2, 0) and (2, ∞)

dx
f (x) decreasing: (–∞, 2) and (0, 2)

(c) Relative maximum: (0, 0)
Relative minimum: (–2, –16) and (2, –16)
(d)  f ''(x) = 12x2 – 16
When  f ''(x) = 0, 12x2 – 16 = 0
3x2 – 4 = 0
2__   ​​ 
x = ± ​​ ___
​  3 ​ 

(​​  –∞, –​  ​√ 3 ​    ​ )​​   (​​  –​  √ ​  2__   ​  ​​   ​​  ​ ___
2__   ​ , ∞ ​​  
​  3 ​  ​√ 3 ​ ) ( ​√ 3 ​  )
___ 2__ ___ 2__   ​ , ___
Interval

f ''(x) +ve –ve +ve


2__   ​  ​​  and ​​  ___
f (x) concave upwards: (​​  –∞, –​ ___
​  3 ​ ) (​  ​√ 3 ​    ​ , ∞)​​  
2__

f (x) concave downwards: (​​  –​ ___   ​​  
​  2__   ​)
2__   ​ , ___
​  3 ​  √
√ ​  3 ​ 

454

Pre-Calculus and Calculus.indb 454 13/7/2017 5:57:38 PM


Chapter 14 • Applications of Differentiation

(​  ​√ 3 ​    ​ )​​​   ​​ – 8​​​ (​  ​√ 3 ​    ​ )​​​   ​​ = –​​  9 ​​  .
4 2
2__   ​​ ,   f (x) = ​​​  ___
2__ 2__ 80
(e) When x = ​​ ___ ___ ____
​  3 ​ 

2__   ​  ​​​   ​​ – 8​​​  –​ ___ 2__   ​  ​​​   ​​ = –​​ ____
( √  ​ ) ( √  ​ )
4 2
2__   ​​ ,   f (x) = ​​​  –​ ___ 80
When x = –​​ ___ 9 ​​  .
​  3 ​ 
√ ​  3  ​  3 

Points of inflection: (​​  – ​ ___  ​ )​​  and (​​   ​ ___  ​  ​​  


 ​ 9 )
80
2__   ​ , –​ ____ 80
2__   ​ , –​ ____
9
​  3 
√  ​ ​√ 3 
y-intercept: When x = 0, y = 0.
x-intercept: When y = 0, x2(x2 – 8) = 0 __
x = 0 or x = ±​​√ 8 ​​ 
Hence, the graph is
f (x)
f (x) = x4 – 8x2

x
–2 0 2

–16

Example 22
Sketch the graph of y = x3 − 3x – 2.
SOLUTION
y = x3 − 3x – 2
___dy
​​   ​​ = 3x2 – 3
dx dy
When ___​​   ​​= 0, 3x2 – 3 = 0
dx
x2 – 1 = 0
x = ±1
Interval (–∞, –1) (–1, 1) (1, ∞)
dy
​​ ___ ​​ +ve –ve +ve
dx
The curve has a relative maximum at x = –1 and a relative minimum at
x = 1.
When x = −1, y = (−1)3 – 3(−1) – 2 = 0.
Hence, the maximum point is (−1, 0).
When x = 1, y = 13 – 3(1) – 2 = –4.
Hence, the minimum point is (1, –4).
f (x) increasing: (–∞, –1) and (1, ∞)
f (x) decreasing: (–1, 1)

455

Pre-Calculus and Calculus.indb 455 13/7/2017 5:57:38 PM


Chapter 14 • Applications of Differentiation

dy
___
2
​​  ​​ = 6x
dx2
d2y
When ___ ​​  2 ​​ = 0, 6x = 0
dx
x=0
Interval (–∞, 0) (0, ∞)
f ''(x) –ve +ve
The curve has a point of inflection at x = 0.
When x = 0, y =
 03 – 3(0) – 2
= −2
Hence, (0, −2) is the point of inflection.
f (x) concave downwards: (–∞, 0)
f (x) concave upwards: (0, ∞)
y-intercept: When x = 0, y = 0.
x-intercept: When y = 0,
x3 – 3x – 2 = 0
(x + 1)(x2 – x – 2) = 0
(x + 1)(x + 1)(x – 2) = 0
x + 1 = 0  or  x – 2 = 0
x = –1 x=2
Hence, the graph is
y

x
–1 0 1 2

–2

y = x3 – 3x – 2
–4

456

Pre-Calculus and Calculus.indb 456 13/7/2017 5:57:38 PM


Chapter 14 • Applications of Differentiation

14.4.2  Rational Functions


Besides the steps mentioned in curve sketching for polynomial functions,
for rational functions, we need to locate all points of discontinuities and
find the vertical and horizontal asymptotes, if any. We will discuss these in
the following examples.

Example 23
x2
A rational function is given as y = ​​ _____ . Find
   ​​ 
x2 – 9
(a) the stationary point (if any),
(b) the relative maximum or relative minimum points,
(c) the intervals where  f  is increasing or decreasing,
(d) the vertical and horizontal asymptotes,
(e) the points of inflection (if any),
(f) the intervals where  f  is concave upwards and  f  is concave
downwards.
Hence, sketch the graph of  f (x).

SOLUTION
x2
(a) y = _____
​​  2    ​​ 
x –9
dy (x2 – 9)(2x) – x2(2x)
​​ ___ ​​ = ________________
   ​​      ​​
dx (x2 – 9)2
–18x
= _______
​​  2   ​​ 
(x – 9)2
dy –18x
When ___
​​   ​​ = 0, _______
​​  2   ​​ =0
dx (x – 9)2
x=0
0
When x = 0, y = ​​ _____ 0 – 9 ​​ = 0.
The stationary point is at (0, 0).
(b)
Interval (–∞, 0) (0, ∞)
dy
​ ___
dx ​
+ve –ve

(0, 0) is a maximum point.


(c)  f (x) increasing: (–∞, 0)
  f (x) decreasing: (0, ∞)
Note
x2
(d) y = _____ is discontinuous if x2 – 9 = 0.
​​  2    ​​ 
x –9 We can also obtain the
x2 – 9 = 0 asymptotes using limits.
x = ±3
Hence, x = 3 and x = –3 are vertical asymptotes.
For horizontal asymptote,
x2 x2
as x → +∞, y = _____ → __
​​  2    ​​  ​​  2 ​​  = 1.
x –9 x
Hence, y = 1 is a horizontal asymptote.
457

Pre-Calculus and Calculus.indb 457 13/7/2017 5:57:38 PM


Chapter 14 • Applications of Differentiation

d2y (x2 – 9)2(–18) – (–18x)(4x)(x2 – 9)


(e) ​​ ___2 ​​ = ____________________________
   
​​      ​​
dx (x2 – 9)4
(x2 – 9)[(x2 – 9)(–18) – (–18x)(4x)]
____________________________
= ​​         ​​
(x2 – 9)4
–18x2 + 162 + 72x2
= ​​ ________________
       ​​
(x2 – 9)3
54(x2 + 3)
= _________
​​  2  ​​ 
(x – 9)3
d2y
​​ ___2 ​​ does not exist at x = ±3.
dx
Interval (–∞, –3) (–3, 3) (3, ∞)
d 2y
​ ___2  ​ +ve –ve +ve
dx
Since x = ±3 are the vertical asymptotes, there is no inflection point.
f (x) concave upwards: (–∞, –3) and (3, ∞)
(f)  
  f (x) concave downwards: (–3, 3)

y
x = –3 x=3

_____x2
y = ​ ​x2 – 9 ​​  

1 y=1

x
–3 3

Example 24
(x – 2)2
It is given that  f (x) = ______
​​   ​​. 

x3
(a) Find the horizontal and vertical asymptotes of  f.
(b) Determine the coordinates of stationary points and state whether
the point is a maximum or minimum point.
(c) Sketch the graph of  f.

458

Pre-Calculus and Calculus.indb 458 13/7/2017 5:57:38 PM


Chapter 14 • Applications of Differentiation

SOLUTION
(x – 2)2
(a)  f (x) = ______ ​​   ​​   
x3
x – 4x + 4
2
= _________
​​   ​​   
x3
= ​​ __ 4 __
1 ​​  – __ 4
x ​​  x2  ​​ + ​​  x3  ​​ 

→ +∞ ( x
​  4  ​  ​​  = 0
x x3 )
​​ xlim​
→ +∞
 ​​​   f (x) = x​​  lim​  ​​​ ​​  __ ​  1 ​  – __​  42  ​ + __

→ –∞ ( x
​  4  ​  ​​  = 0
x x3 )
​​ xlim​
→ –∞
 ​​​   f (x) = x​​  lim​ ​  1 ​  – __
 ​​​ ​​  __ ​  42  ​ + __
The horizontal asymptote is y = 0.
The vertical asymptote is x = 0.
1  ​​ + __ 8 12
(b)  f '(x) = –​​ __ ​​    ​​ – ___​​   ​​ 
x2 x3 x4
–x2 + 8x – 12
= ​​ ___________
    ​​ 
x4
At stationary points,  f '(x) = 0.
–x2 + 8x – 12
___________
​​     ​​  =0
x4
–x2 + 8x – 12 = 0
x2 – 8x + 12 = 0
(x – 2)(x – 6) = 0
x = 2 or x = 6
2 48
  __
​​  244 ​​ + ___
f ''(x) = ​​  3  ​​ – ___ ​​  5 ​​ 
x x x
(2 – 2)2
When x = 2,  f (x) = _______ ​​   ​​   
23
=0
48
 f ''(x) = __ ​​  23  ​​ – ___
​​  24 ​​ + ___
​​   ​​ 
2 24 25
= __ ​​  41 ​​ 
>0
(2, 0) is a minimum point.
(6 – 2)2
When x = 6,  f (x) = _______ ​​   ​​   
63
= ___
​​  27 2  ​​ 

48
 f ''(x) = __ ​​  23  ​​ – ___
​​  24 ​​ + ___
​​   ​​ 
6 64 65
= –​​ ____ 1
324   ​​ 
<0
\ (​​  6, ___ 2  ​  ​​ is a maximum point.
​  27 )

459

Pre-Calculus and Calculus.indb 459 13/7/2017 5:57:38 PM


Chapter 14 • Applications of Differentiation

(c) y

(   ​ )​​  
2
​​  6, ___
​  27

x
0 2 6

Exercise 14.4
1. Sketch the graphs of the following functions.
(a) y = –x2 + x + 3
(b) y = x3 – 12x
–x3 + 2
(c) y = ______
​​   ​​   
x4
(d) y = x + __ ​​  1x ​​
(e) y = 4x2 + __ ​​  1x ​​

2. The equation of a curve is given as y = 3x4 + 4x3.


(a) Show that (0, 0) is one of the stationary points.
(b) Find the other stationary point.
(c) Determine the nature of the two stationary points.
(d) Sketch the curve.
x2
3. Given that y = _____
​​  1 +  x ​​, find
(a) the vertical and horizontal asymptotes, if any,
(b) the critical point(s) and determine the maximum or minimum points,
Hence, sketch the graph of the function.
2x
4. Sketch the graph of the curve y = _____
​​     ​​ 
by showing all the relevant points and asymptotes, if any.
1 + x2

460

Pre-Calculus and Calculus.indb 460 13/7/2017 5:57:38 PM


Chapter 14 • Applications of Differentiation

14.5 Optimization Problems


Some optimization problems can be solved using the theory regarding
maximum and minimum values which we have learnt so far. The most
important and relevant facts to be remembered are, for a function y =  f (x),
dy d2y
(i) if ​​ ___ ​​ = 0 and ​​ ___2 ​​ < 0, then y =  f (x) is maximum.
dx dx
dy d2y
(ii) if ​​ ___ ​​ = 0 and ​​ ___2 ​​ > 0, then y =  f (x) is minimum.
dx dx
When solving optimization problems, the following steps can be used as
a guide.
Step 1: Understand the problem and identify the unknowns, the given
quantities and the conditions.
Step 2: Draw a diagram, introduce variables for the unknown quantities
and label on the diagram.
Step 3: Write all the facts given and express the relationships (in equation
form) among the variables.
Step 4: Identify the variable to be maximised or minimised and express
it in terms of single variable function.
Step 5: Find the stationary points and test them for relative extremum
using the second derivative test.
Step 6: Test the end points for extremum value, if relevant.

Example 25
In the diagram below, ABCD is a rectangle. APB and CQD are
semicircles. It is given that the area of ABCD is 200 cm2.
(a) Show that the perimeter of the diagram, P, is given by
P = (​​  _____  ​+ 2x)​​  cm.
200π
​  x   
(b) Find the minimum perimeter of the diagram.
P

A B

x cm

D C

Q
SOLUTION
(a) Let AB = CD = y cm.
Given the area of ABCD is 200 cm2,
xy = 200
200
y = ____
​​  x    ​​
200 1 ____ 100
Radius of semicircle = ____
​​  x    ​​ × __
​​  2 ​​  = ​​  x    ​​ cm

461

Pre-Calculus and Calculus.indb 461 13/7/2017 5:57:38 PM


Chapter 14 • Applications of Differentiation

100 100
Note Perimeter, P = π ​​ ____ ____
x    ​​ + x + π ​​  x    ​​ + x
P = (​​  _____  ​+ 2x)​​  cm  (shown)
Perimeter of a circle is 2πr. 200π
​  x   
(b) P = 200πx–1 + 2x
dP
​​ ___ ​​ = –200πx–2 + 2
dx
dP –200π
When ___ ​​   ​​ = 0, ______
​​  2 ​​   + 2 = 0
dx x
2x2 = 200π
x2 = 100π__
x = 10​​√ π ​​ 
d 2P
___ 400π
_____
​​  2 ​​ = 400πx–3 = ​​  3 ​​   
dx x
__ d2P _____ 400π
When x = 10​​√ π ​​ , ___ ​​  2 ​​ = ​​  __   ​​  > 0  minimum
dx (10​√ π ​)  3
200π __
Minimum perimeter = _____ ​​ 10​  __  

​​ + √ π ​​ )
2(10​​
√ π ​ 
__ __
= 20​​√ __π ​​ + 20​​√ π ​​ 
= 40​​√ π ​​ 
= 70.90 cm

Example 26
A open cylindrical container has a circular base of radius r cm. Given
that the total surface area of the container is 300π cm2,

2 ​ )​​  cm ,
(a) show that the volume of the container V = (​​  150πr – ​ ___
πr3 3

(b) find the maximum value of V.

SOLUTION
Note (a) Let the height of the container be h cm.
2πrh + πr2 = 300π
Area of circular base = πr2
2rh = 300 – r2
Area of curved surface of
150 __r
cyclinder = 2πrh h = ​​ ____
r   ​​ – ​​  2  ​​
Volume of the container, V = πr2h
V = πr2​(​  ____ 2  ​)​​  
150 r
​  r   ​ – ​ __

V = (​​  150πr – ​ ___ 2 ​ )​​  cm  (shown)


πr3 3

462

Pre-Calculus and Calculus.indb 462 13/7/2017 5:57:38 PM


Chapter 14 • Applications of Differentiation

πr3
(b) V = 150πr – ___
​​  2 ​​ 
dV 3πr2
​​ ___ ​​ = 150π – ​​ ____2 ​​ 

dr
dV 3πr2
When ___ ​​   ​​ = 0, 150π – ​​ ____
2 ​​ 
 = 0
dr
3πr 2
​​ ____ 2 ​​ 
 = 150π
r2 = 100
r = 10 cm
d2V
​​ ___2 ​​ = –3πr
dr
d2V
When r = 10, ​​ ___2 ​​ = –3π(10)
dr
= –30π
< 0  maximum
π(10)3
Maximum value of V = 150π(10) – ___
​​  2 ​​ 
 = 1000π cm3

Exercise 14.5
1. A rectangle has a perimeter of 20 cm. Determine the lengths of the sides of the rectangle when its area
is a maximum.
2. A farmer wishes to fence up a rectangular enclosure of area 128 m2. One side of the enclosure is
formed by part of a river. Find the least possible length of fencing required for the other three sides.
3. The diagram shows a piece of rectangular cardboard of length 8 cm and breadth 5 cm. Equal squares
of sides x cm are removed from each corner, and the edges are then folded up to form an open box of
volume V cm3. x x
(a) Show that V = 4x3 – 26x2 + 40x. x x
(b) Find the maximum volume of the box.
(c) Determine the dimensions of the box. 5 cm

x x
x x
8 cm

4. A closed cylindrical can has a base radius of r cm. Given that the total outer surface area is 24 cm2,
(a) show that the volume of the can V cm3 is given by V = r(12 – r2),
(b) determine the value of r for the volume to be a maximum.
5. Prove that the diameter of the base of a cylinder is equal to the height of the cylinder if the volume is
fixed and the surface area is a minimum.
6. A right triangle fits nicely inside a semicircle of radius r cm as shown in the diagram. If A is the area
of the triangle, deduce that A = 2r2 cos3 α sin α. Hence, find the maximum possible area of the right
triangle.

α O
r
463

Pre-Calculus and Calculus.indb 463 13/7/2017 5:57:38 PM


Chapter 14 • Applications of Differentiation

14.6 Applications in Economics and


Business
Differentiation can also be applied in business and economics.
In this section, we will be discussing about calculating the cost, revenue
and profit of a business.

Summary

• C(x) is the cost function which is the total cost to produce x units of
product.
• The total cost consists of the fixed cost and the variable cost.
C(x) = fixed cost + variable cost
Note • The fixed cost is the cost incurred when no unit of product is produced,
that is, fixed cost = C(0).
Sometimes, the letter q is C(x)
used to represent the quantity • C(x) = ​​ ____
x     ​​is the average cost function which is the cost to produce
instead of the letter x. one unit of the product.
• C′(x) is the marginal cost function which is the additional cost incurred
as a result of producing and selling one more unit of the product at a
production level of x units.
C′(x) also represents the instantaneous rate of change of cost relative
to production at a given production level.
• p(x) is the price or demand function which is the unit price offered for
x units of product demanded.
• R(x) = p(x) × x is the revenue function.
R(x)
• R(x) = ____
​​  x    ​​is the average revenue function.
• R′(x) is the marginal revenue function which is the additional revenue
derived from selling one more unit of the product.
R′(x) also represents the instantaneous rate of change in total revenue
given a change in the number of units sold.
• Π(x) = R(x) – C(x) is the profit function.
Π(x)
• Π(x) = ​​ ____x     ​​is the average profit function
• Π′(x) = R′(x) – C′(x) is the marginal profit function which is the profit
made when one additional unit of product is produced and sold.

464

Pre-Calculus and Calculus.indb 464 13/7/2017 5:57:38 PM


Chapter 14 • Applications of Differentiation

Example 27
Given that the cost function (in $) is C(x) = 50 + 10x – 0.1x2, find
(a) the fixed cost,
(b) the average cost function,
(c) C(10) and explain what it represents,
(d) the marginal cost function,
(e) C′(10) and explain what it represents.
SOLUTION
(a) C(x) = 50 + 10x – 0.1x2
C(0) = 50
Hence, the fixed cost is $50.
C(x)
(b) The average cost function C(x) = ​​ ____ x     ​​
50
= ___
​​  x ​​ + 10 – 0.1x Note
50 The exact cost of producing
(c) C(10) = ___
​​  10 ​​ + 10 – 0.1(10) = 14
the 11th unit is
At a production level of 10 units, the average cost of producing a C(11) – C(10)
unit is $14. = [50 + 10(11) – 0.1(11)2] –
[50 + 10(10) – 0.1(10)2]
(d) The marginal cost function C'(x) = 10 – 0.2x
= 7.9
(e) C′(10) = 10 – 0.2(10) = 8 which is quite close to 8.
At a production level of 10 units, the production cost is increasing The marginal cost function is
at a rate of $8 per unit of product. In other words, the cost of just an approximation to the
producing the 11th unit of product is $8. exact cost of the (x + 1) unit.

Example 28
Given that the demand function (in $) is p(x) = 300 – 0.003x, find
(a) the revenue function,
(b) the average revenue function,
(c) R(100) and explain what it represents,
(d) the marginal revenue function,
(e) R′(100) and explain what it represents.
SOLUTION
(a) The revenue function R(x) = p(x) × x = 300x – 0.003x2
R(x)
(b) The average revenue function R(x) = ____
​​  x    ​​
= 300 – 0.003x Note
(c) R(100) = 300 – 0.003(100) = 299.7 The exact revenue of
At a production level of 100 units, the average revenue of producing producing the 101th unit is
a unit is $299.70. R(101) – R(100)
(d) The marginal revenue function R′(x) = 300 – 0.006x = [300(101) – 0.003(101)2] –
[300(100) – 0.003(100)2]
(e) R′(100) = 300 – 0.006(100) = 299.4
= 299.397
 At a production level of 100 units, the revenue gained from which is quite close to 299.4.
producing the 101th unit of product is $299.40.

465

Pre-Calculus and Calculus.indb 465 13/7/2017 5:57:38 PM


Chapter 14 • Applications of Differentiation

Understanding the relationship between the cost, revenue and profit


functions.
Example 29
Given that the revenue function, R(x) = 200x – 0.002x2 and the cost
function, C(x) = 80,000 + 50x + 0.001x2, find
(a) the profit function and explain what it represents,
(b) the average profit function,
(c) the marginal profit function.
SOLUTION
(a) The profit function Π(x) = R(x) – C(x)
= (200x – 0.002x2) – (80,000 + 50x + 0.001x2)
= –80,000 + 150x – 0.003x2
At a production level of x units, the profit made after x units are
sold is $(–80 000 + 150x – 0.003x2).
Π(x)
(b) The average profit function Π(x) = ____
​​  x     ​​
–80,000
= _______
​​  x     
​​+ 150 – 0.003x
(c) The marginal profit function Π'(x) = 150 – 0.006x

Example 30
Given that the cost function is C(x) = 90,000 – 30x + 0.003x2 and the
Note revenue function is R(x) = 400x – 0.002x2, find
We can use the same concept (a) the minimum cost and the corresponding quantity,
as taught in Section 14.3 to (b) the maximum revenue and the corresponding quantity,
find maximum and minimum (c) the maximum profit and the corresponding price.
values.
SOLUTION
(a) C(x) = 90,000 – 30x + 0.003x2
C'(x) = –30 + 0.006x
For minimum cost C′(x) = 0
–30 + 0.006x = 0
30
x = _____
​​  0.006    ​​ 
= 5000
C′′(x) = 0.006 > 0  minimum
∴ The minimum cost, C(5000) = 90,000 – 30(5000) + 0.003(5000)2
= $15,000
when 5000 units are produced.
(b) R(x) = 400x – 0.002x2
R'(x) = 400 – 0.004x
For maximum revenue R'(x) = 0
400 – 0.004x = 0
400
x = ​​ _____
0.004  ​​ 
= 100,000
R''(x) = –0.004 < 0  maximum exits.
∴ The maximum revenue R(100,000)
= 400(100,000) – 0.002(100,000)2
= $20,000,000
466
when 100,000 units are produced.

Pre-Calculus and Calculus.indb 466 13/7/2017 5:57:38 PM


Chapter 14 • Applications of Differentiation

(c) Π(x) = R(x) – C(x)


= (400x – 0.002x2) – (90,000 – 30x + 0.003x2)
= –90,000 + 430x – 0.005x2
Π'(x) = 430 – 0.01x
For maximum profit Π'(x) = 0
430 – 0.01x = 0
430
x = ____
​​  0.01  ​​ = 43,000
Π''(x) = –0.01 < 0  maximum
∴ The maximum profit Π(43,000)
= –90,000 + 430(43,000) – 0.005(43,000)2
= $9,155,000
R(x)
The price function P(x) = ​​ ____
x     ​​
= 400 – 0.002x
∴ At maximum profit, the price of the product should be
P(43,000) = 400 – 0.002(43,000) = $314.

Example 31
The cost function, C(x) and revenue function, R(x) of producing and
selling x units of pens (in $) are as follows.
C(x) = 2x + 7000
x2
R(x)= –​​ _____
1000    ​​ + 10x
Find
(a) the maximum revenue and the quantity, cost and profit at this level,
(b) the maximum profit and the quantity, cost and revenue at this level,
(c) the price of a pen at maximum profit,
(d) the number of pens that need to be produced to break even.
SOLUTION
x2
(a) R(x) = –​​ _____
1000   ​​ + 10x
2x
R′(x) = –​​ _____
1000  ​​ + 10
2x
When R′(x) = 0, –​​ _____ 1000  ​​ + 10 = 0
x = 5000
_____ 2
R′′(x) = –​​  1000
   ​​ < 0  maximum
50002
Maximum revenue R(5000) = –​​ _____ 1000 ​​ + 10(5000)
= $25,000
Quantity x = 5000 pens
Cost C(5000) = 2(5000) + 7000 = $17,000
Profit function Π(x) = R(x) – C(x)
x2
= –​​ _____
   ​​ + 10x – (2x + 7000)
1000
x2
= –​​ _____
   ​​ + 8x – 7000
1000
50002
Π(5000) = –​​ _____
1000 ​​ + 8(5000) – 7000 = $8000
467

Pre-Calculus and Calculus.indb 467 13/7/2017 5:57:39 PM


Chapter 14 • Applications of Differentiation

x2
(b) Π(x) = –​​ _____
1000    ​​ + 8x – 7 000
2x
Π′(x) = –​​ _____
1000  ​​ + 8
2x
When Π′(x) = 0, –​​ _____
1000   ​​ + 8 = 0
x = 4000
2
_____
   ​​ < 0  maximum
Π′′(x) = –​​  1000
40002
Maximum profit Π(4000) = –​​ _____ 1000 ​​ + 8(4000) – 7000
= $9000
Quantity x = 4000 pens
Cost C(4000) = 2(4000) + 7000 = $15 000
40002
Revenue R(4000) = –​​ _____ 1000 ​​ + 10(4000)
= $24,000
R(x) x
(c) Price function p(x) = ____​​  x    _____
 ​​ = –​​     ​​ 
1000 + 10
At maximum profit, the price of a pen
4000
P(4000) = –​​ _____
1000 ​​ + 10 = $6
(d) To break even (no loss or gain),
C(x) = R(x)
x2
2x + 7000 = –​​ _____ 1000    ​​ + 10x
x2 – 8000x + 7,000,000 = 0
(x – 1000)(x – 7000) = 0
x = 1000 or 7000
∴ To break even, 1000 or 7000 pens need to be produced.

Note
The above information can be represented graphically as below.
y ($)
Maximum revenue → 25,000 C(x) = 2x + 7000
Revenue at → 24,000
maximum profit Profit at
maximum
Maximum revenue
profit = 8000
Cost at Break even points
= 9000
maximum revenue
→ 17,000 C(x) = R(x)
Cost at → 15,000
maximum profit

_____x2
Minimum cost → 7000 R(x) = –​​  1000
   ​​ + 10x
(fixed cost) Quantity at Quantity at
maximum maximum revenue
profit
x (Number of pens)
0 1000 4000 5000 7000 10,000

468

Pre-Calculus and Calculus.indb 468 13/7/2017 5:57:39 PM


Chapter 14 • Applications of Differentiation

Example 32
The total cost (in $), to produce x units of product A per week is given
by the following cost function:
C(x) = 4x3 – 18x2 + 24x + 5
Find
(a) the average cost function,
(b) the marginal cost function,
(c) the average cost when 10 units of product A are produced,
(d) the minimum cost and the number of units of product A that need to
be produced per week.
SOLUTION
(a) The average cost function,
C(x)
C(x) = ____ ​​  x    ​​
4x3 – 18x2 + 24x + 5
_________________
​​ 
=    x  ​​

5
= 4x2 – 18x + 24 + __ ​​ x ​​
(b) The marginal cost function C′(x) = 12x2 – 36x + 24
(c) When 10 units of product A are produced,
5
the average cost, C(10) = 4(10)2 – 18(10) + 24 + ___ ​​ 10  ​​ 
= $244.50
(d) When C′(x) = 0, 12x – 36x + 24 = 0
2

x2 – 3x + 2 = 0
(x – 1)(x – 2) = 0
x = 1 or x = 2
C′′(x) = 24x – 36
C′′(1) = 24(1) – 36 = –12 < 0  maximum
C′′(2) = 24(2) – 36 = 12 > 0  minimum
Minimum cost C(2) =  4(2)3 – 18(2)2 + 24(2) + 5
= $13
when 2 units of product A are produced.

Example 33
The demand function (in $), for x units of an electronic component is
given by p(x) = 500 – 0.1x. Find
(a) the revenue function,
(b) the average revenue function,
(c) the marginal revenue function,
(d) the marginal revenue when x = 20,
(e) the maximum revenue and the number of units of the electronic
component that need to be sold.
SOLUTION
(a) The revenue function R(x) = p(x) × x
= (500 – 0.1x)x
= 500x – 0.1x2

469

Pre-Calculus and Calculus.indb 469 13/7/2017 5:57:39 PM


Chapter 14 • Applications of Differentiation

(b) The average revenue function,


R(x)
R(x) = ____ ​​  x    ​​
500x – 0.1x2
= ___________
​​  x   ​​ 
= 500 – 0.1x
(c) The marginal revenue function R′(x) = 500 – 0.2x
(d) When x = 20, the marginal revenue, R′(20) = 500 – 0.2(20)
= $496
(e) When R′(x) = 0, 500 – 0.2x = 0
0.2x = 500
x = 2500
R″(x) = –0.2 < 0  maximum
The maximum revenue R(2500) = 500(2500) – 0.1(2500)2
= $625,000
when 2500 units of the electronic component are sold.

Example 34
The revenue generated (in $), by selling x units of calculators is given
by R(x) = 100x – 0.2x2. The cost, in $, to produce x units of calculators
is given by C(x) = 60x + 200. Find
(a) the profit function,
(b) the average profit function,
(c) the marginal profit function,
(d) the average profit when 50 units of calculators are sold,
(e) the maximum profit and the number of units of calculators that
need to be sold.
SOLUTION
(a) The profit function Π(x) = R(x) – C(x)
Note = 100x – 0.2x2 – (60x + 200)
= 40x – 0.2x2 – 200
Common mistake: (b) The average profit function,
Π(x)
Π(x) _______________
40x – 0.2x2 – 200 200
= 100x – 0.2x2 – 60x + 200 Π(x) = ____
​​  x    ​​ 
 ​​ =    x  = 40 – 0.2x – ____
​​  ​​  x    ​​
= 40x – 0.2x2 + 200
(c) The marginal profit function Π′(x) = 40 – 0.4x
200
 40 – 0.2(50) – ​​ ____
(d) When x = 50, the average profit Π(50) = 50 ​​ 
= $26
(e) When Π′(x) = 0, 40 – 0.4x = 0
0.4x = 40
40
x = ___
​​  0.4  ​​ 
= 100
Π′′(x) = –0.4 < 0  maximum
 40(100) – 0.2(100)2 – 200 = $1800
Π(100) =
∴ The maximum profit possible is $1800 when 100 units of
calculators are sold.

470

Pre-Calculus and Calculus.indb 470 13/7/2017 5:57:39 PM


Chapter 14 • Applications of Differentiation

Example 35
The average cost function and demand function (in $), of a
2000
company that manufactures computer chips are C(x) = ​​ ____ x   
 ​​+ 4 and
p(x) = 40 – 0.002x respectively, where x is the number of computer
chips manufactured. Find
(a) the cost function,
(b) the maximum revenue and the number of computer chips sold to
achieve this,
(c) the maximum profit,
(d) the maximum profit if each computer chip sold is taxed $5.
SOLUTION
Note
(a) The cost function C(x) = xC(x) = 2000 + 4x
C(x)
(b) The revenue function R(x) = p(x) × x Since C(x) = ____
​​  x   ​​, 
= 40x – 0.002x2
then C(x) = xC(x).
R′(x) = 40 – 0.004x
When R′(x) = 0, 40 – 0.004x = 0
0.004x = 40
40
x = _____
​​  0.004   ​​ 
= 10,000
R′′(x) = –0.004 < 0  maximum
R(10,000) =  40(10,000) – 0.002(10,000)2 = $200,000
∴ The maximum revenue is $200,000 when 10,000 of computer
chips are sold.
(c) Π(x) = R(x) – C(x)
= 40x – 0.002x2 – (2000 + 4x)
= 36x – 0.002x2 – 2000
Π′(x) = 36 – 0.004x
When Π′(x) = 0, 36 – 0.004x = 0
0.004x = 36
36
x = ​​ _____    ​​ 
0.004 = 9000
Π′′(x) = –0.004 < 0  maximum
Π(9000) =  36(9000) – 0.002(9000)2 – 2000 = $160,000
∴ The maximum profit is $160,000.
(d) If each computer chip sold is taxed $5, then the cost function
becomes
C(x) =  2000 + 4x + 5x
= 2000 + 9x
Π(x) =  40x – 0.002x2 – (2000 + 9x) = 31x – 0.002x2 – 2000
Π′(x) = 31 – 0.004x
When Π′(x) = 0, 31 – 0.004x = 0
0.004x = 31
31
x = ​​ _____    ​​ 
0.004 = 7750
Π′′(x) = –0.004x < 0  maximum
Π(7750) =  31(7750) – 0.002(7750)2 – 2000 = $118,125
∴ The maximum profit is $118,125 if each computer chip sold is
taxed $5.
471

Pre-Calculus and Calculus.indb 471 13/7/2017 5:57:39 PM


Chapter 14 • Applications of Differentiation

Example 36
The demand function of a pot is q = 1000 – 10p, where q is the quantity
and p is the price in $.
(a) Find the quantity to achieve maximum revenue.
(b) Find the maximum revenue.
(c) Find the price of a pot at maximum revenue.
SOLUTION
(a) First, find the price function in terms of p.
q = 1000 – 10p
10p = 1000 – q
1000 – q
p = ________
​​  10 ​​   
q
= 100 – ___​​ 10  ​​ 
q2
The revenue function, R(q) = 100q – ___ ​​  10  ​​ 
2q
R′(q) = 100 – ___
​​ 10 ​​ 
q
= 100 – ​​ __
5 ​​ 
q
When R′(q) = 0, __
​​ 5 ​​  = 100
q = 500
1  ​​ < 0  maximum
R′′(q) = –​​ __
5
To achieve maximum revenue, 500 pots need to be sold.
5002
(b) The maximum revenue, R(500) = 100(500) – ____
​​  10 ​​ 
= $25,000
(c) The price of a pot at maximum revenue,
500
p(500) = 100 – ____​​  10 ​​ = $50

Exercise 14.6
1. The total cost (in $), to manufacture x units of a product per day is given by the following cost
function:
C(x) = 8x3 – 36x2 + 48x + 10
Find
(a) the average cost function,
(b) the marginal cost function,
(c) the average cost when 20 units of the product are manufactured,
(d) the minimum cost and the number of units of the product that must be manufactured per day.

472

Pre-Calculus and Calculus.indb 472 13/7/2017 5:57:39 PM


Chapter 14 • Applications of Differentiation

2. The demand function for a product is p(x) = 800 – 0.2x, where x is the number of units demanded and
p(x) is the price per unit (in $). Find
(a) the revenue function,
(b) the average revenue function,
(c) the marginal revenue function,
(d) the marginal revenue when x = 100,
(e) the maximum revenue and the number of units sold.

3. A company has the following cost and revenue functions:


C(x) = 140x + 100
R(x) = 200x – 0.3x2
where C(x) is the cost (in $), R(x) is the revenue (in $), and x is the number of units of product. Find
(a) the profit function,
(b) the average profit function,
(c) the marginal profit function,
(d) the average profit when 100 units of the product are sold,
(e) the maximum profit and the number of units of the product sold.

4. A company makes toys. The demand function and average cost function (in $), are p(x) = 80 – 0.004x
4000
and C(x) = _____
​​  x    ​​+ 8 respectively, where x is the number of toys made. Find
(a) the cost function,
(b) the maximum revenue and the number of toys that need to be sold to achieve this,
(c) the maximum profit,
(d) the maximum profit if each toy sold is taxed $2.
x2
5. The cost function of a product (in $) is C(x) = ​​ __
2 ​​ + 6x + 800, where x is the number of units of product
manufactured. Find the minimum average cost and the number of units of the product that need to be
manufactured to achieve this.

6. Given that the average cost function C(q) = 2q2 – 20q + 50, show that when the average cost is
minimum, the marginal cost is the same as the average cost.

7. The demand function and cost function (in $), of a product are as follows:
p(q) = 1000 – 0.05q
C(q) = 150q + 7000
where q is the number of units of the product.
(a) Determine the maximum revenue.
(b) Calculate the selling price and cost price for each unit of the product to ensure maximum
revenue.

8. Given the demand function and the average cost function (in $), of a spare part are
100 2000
p(x) = ____ __ _____
​  x   ​​​ and C(x) = 1 + ​​  x   
​​  √  ​​

(a) Show that at maximum profit, the marginal revenue equals the marginal cost.
(b) Find the number of spare parts that need to be sold to ensure maximum profit.
(c) Determine the price of a spare part at maximum profit.

473

Pre-Calculus and Calculus.indb 473 13/7/2017 5:57:39 PM


Chapter 14 • Applications of Differentiation

9. A company produces and sells pots each year with cost function, C(x) = 1000 + 5x and demand
function, p(x) = 60 – 0.004x, where x is the number of pots and C(x) and p(x) are in $.
Determine
(a) the revenue function and the maximum revenue,
(b) the profit function, the maximum profit and the number of pots that need to be produced and sold
each year to achieve maximum profit,
(c) the selling price to ensure maximum profit,
(d) the maximum profit, if each pot sold is taxed $3.

10. The profit function P (in $) for producing x items of merchandise of a company is given by
P = 2400x – 6x2.
(a) Determine whether the company made a profit or suffered a loss if it only managed to produce
500 items of merchandise.
(b) Calculate the maximum profit and the maximum number of items of merchandise that need to
be produced.

11. For a company, the demand function is p(x) = 100 – 5x and the average cost function is
C(x) = 20x + 50. Find
(a) the revenue function, cost function and profit function,
(b) the quantity and price that will maximise the revenue,
(c) the profit and revenue that will maximise the profit.

12 The demand function of a blender is q = 2 000 – 20p, where q is the quantity and p is the price in $.
(a) Find the maximum revenue.
(b) Find the price of a blender at maximum revenue.

13. The revenue function (in $) and the cost function (in $) of a factory producing q units of flat-screen
television are given as follows:
R(q) = 500q – 2q2
C(q) = 200q + 300.
Find
(a) the marginal revenue and demand function,
(b)  the number of flat-screen television that need to be produced and sold to achieve maximum
profit,
(c) the maximum profit and the selling price at maximum profit.

474

Pre-Calculus and Calculus.indb 474 13/7/2017 5:57:39 PM


03_Pre Calculus_Ans.pdf 1 14/7/2017 4:00:47 PM

Episteme Foundation Textbooks

PRE-CALCULUS &
CALCULUS FOR
PRE-UNIVERSITY AND
UNIVERSITY
STUDENTS
THE SINGAPORE APPROACH

© Fundación Episteme & Singapore Asia Publishers Pte Ltd. ALL RIGHTS RESERVED.
ANSWERS

Chapter 1 16. (a) 1 (d) 4


(b) 5 (e) 225
(c) 27
Exercise 1.1
17 448 109 17. 9 : 10
1. (a) ​ ___
9 ​   (c) ​ ____
333 ​   (e) ​ ____
66 ​  18. 0.75 m
7 5
(b) ​ ___
33  ​   (d) ​ __9 ​  19. Yes, the two vehicles traveled at the same speed.
2 ​ , 5.144, ___
2. Rational: –​ __ ​  22 Exercise 1.2
5 7 ​ , 3.142, 0, 0.010101... 1 ​  
__
__ 1. (a) 32 (c) ​ 
3 (e) 0.04
Irrational: 5π, e , ​√7 ​,  5.3215467...
2
(b) 3 (d) 10 (f) –125
3. (a) False (d) False (g) True
1 ​   3 16 2
(b) False (e) True (h) False 2. (a) ​ __
t (c) ​ ____
2 ​ y (e) ​ _____
x – 1 ​ 
(c) True (f) False y (n + 2)
u5
___
(b) ​  2  ​   (d) ​ _____
y – 1 ​   (f) –​ ______
 ​  
4. (a) 100 2v n2
(b) –2, 100
Exercise 1.3
​  11
(c) 0.11, ___ __ 1
6 ​,  –5​  3 ​ , –2, 100, 5.123, 4.919191... 1. (a) 5 (c) 4 (e) –4
2e 7
(d) 2π, ___
​ 3 ​,  0.012345... (b) ​ __
2 ​   (d) –1 (f) –3
(e) All the numbers 2. (a) 1.683 (c) 6.229
5. (a) 15 (f) –17 (b) –2.807 (d) 0.107
(b) –5 (g) –9
Exercise 1.4
(c) 6 (h) 21 ___ __ 3 __
___
​√15 ​  ​√3 ​ ​ √  4 ​ 
(d) –18 (i) –28 1. (a) ​ ____
 ​  
3 __   (c) ​ ______
4__  ​     (e) 7 – 2​√10 ​ 
(e) 17 (j) 6 __
(b) 39​
√ 2 ​   (d) 6​√3 ​ – 13​√2 ​ 
6. 695 m __
2. (a) 2 – ​​√2 ​​   4 ​​​​ ( ​ __
(c) –​​ __ )
7. 5 km west 7 √2 ​ + 3  ​​
__ __
11  ​   99 3​√3 ​ – √
(b) ​​ ________
​ 5 ​  1 ​​​​ ( 5 + ​ __
(d) ​​ __
8. (a) ​ ____
100
____
(d) ​  20 ​  22 ​​    
2 √ 5 ​  )​​
__
5 33 ​√y ​  __
(b) –​ __
9 ​   (e) ​ ___
2 ​ 
5
__
3. (a) ​  x ​ (d) ​ ___
y   ​ (g) –8​√3 ​ 
__ __ __
2
(c) ​ ___ _ 2​√3 ​  ​√6 ​ + 2​√2 ​ 
51  ​  (b) x yz ​√z ​  
4 4
(e) ​ ____   
3x ​ (h) ​ ________ ​ –2  
___
67
9. –​ ___ ​ 
3
3 __ __ ​√2x  ​
2
____
   
30 (c) 2xy4 ​√  x ​​ √y ​   (f) ​  3x    ​  
9
10. –​ __
4 ​ 
Chapter 2
151
11. ​ ____
40 ​ 
12. (a) 415.822 (d) –0.700756 Exercise 2.1
(b) –57.477 (e) –30.1 1. (a) −4x3 + 5x + 10
(c) –13.4772 (b) 6a4b + 3a3b2 − 3a2b3 + 14ab4
13. $18.81 2. (a) 8y4 – 13y3 + 5y2 + 15y + 12
(b) –4a2 – ab + 3a + 5b
14. (a) 50 : 1 (f) 1:1
(c) 5m3n2 – 7m2n3 + 6mn4 – 6m4n
(b) 64 : 1 (g) 250 : 1
(c) 74 : 1 (h) 20 : 1 3. (a) 4x4 + 6x3 – 19x2 + 4x + 13
(d) 5 : 1 (i) 5:1 (b) 6a5 – 8a4 – 8a3 + 5a2 – 2a – 3
(e) 1 : 2 (j) 70 : 1 (c) –3r2 – 9rs
(d) 13m3 – m2 – 6mn2 – 4n3
15. (a) No (d) Yes
(e) –24p4q3 + 18p5q2 – 15p3q4 + 27p2q
(b) Yes (e) No
(f) 2y2 – 6x2y3 + 8x2
(c) No A1

Pre-Calculus and Calculus.indb 1 13/7/2017 5:57:39 PM


ANSWERS

(g) 10x4 – 4x2 + 33x Exercise 2.2


(h) 18a3 – 36a2 + 9a 1. (a) 2xy(2x – 4y2 + 3y3)
(y) –10x + 5xy + 4y (b) 2(2x + y)(2x – y)
5
(j) 16x3 – 12x2 + 6x – 15 + __
​ x ​ (c) (a – c)(b + d)
4. (a) 25x4 – 4y4 (d) (2a – b)2
(b) 36a2 – 84ab + 49b2 (e) (2r + 5s)(4r2 – 10rs + 25s2)
(c) 81x4y2 + 18x3y3 + x2y4 (f) (c + 3)(c – 1)
(d) 10r2 + 18rs – 88s2 (g) (4m – n)(16m2 + 4mn + n2)
(e) 20x4 – 41x2y2 + 9y4
2. (a) (3x – 4)(x – 3)
(f) 27v3 + 64
(b) (4a + 4b – 3c)(4a + 4b + 3c)
(g) 343a6 – 27b6
(c) 2(3x2 – 4y3)(3x2 + 4y3)
5. (a) 5x3 – 15x2 + 10x (d) c2(x – y2)(x2 + xy2 + y4)(x + y2)(x2 – xy2 + y4)
(b) –6a4 – 2a3 + 10a2 (e) 2(p – 3q)(2m – 3n)
(c) y2 – 100
(f) (5p2 – 2q2)(25p4 + 10p2q2 + 4q4)
(d) 64b2 – 9c2
(e) y2 + 14y + 49 (g) (7b + 2)(3b – 7)
(f) 9r2 – 30rs + 25s2 3. (a) (5r2 + 3s2)2
(g) x3 – 8 (b) (x – y + 1)(x + y + 3)
(h) a3 + 1 (c) (r – s)(r2 + rs + s2 – r – s)
(i) p2 – p – 30
(d) (m – n)(m – n – p)
(j) 15m2 + 14mn – 8n2
(e) (a – b)(a + 2b – a2 – ab – b2)
9
​ 12
6. (a) 4x2 + ___ ___
5 ​ x + ​ 25  ​  4. (a) (3v2 + 2w2)2
3 1  ​n  6 (b) (r + s – rs – 2)(r + s + rs + 2)
(b) 9m4 + __
​ 2 ​ m2n3 + ___
​ 16
(c) (m – 4)(m + 1)(m2 + 3m + 4)
(c) 9x2 – 12xy + 4y2 + 30x – 20y + 25 (d) (7x2 – 18y + 30z)(3x2 + 6y – 10z)
(d) 10x2 – x + xy – ___ 1  ​  y
​ 10 (e) (x – 4y + 4)(x2 – 8xy – 4x + 16y2 + 16y + 16)
(f) s3(r2 + 4)(r – 2)(r + 2)
(e) 25a2 – 10ab + b2 + 45a – 9b – 10
(f) a2 – b2 – 4b – 4 (g) (5c4 + 2d 4)2
(h) (2r – s)(4r2 + r + 2rs + 3s + s2)
7. (a) q(x) = 1; r(x) = –x2 + 2x – 2
5. (1 – m)(1 – n)
3 17 9 105
(b) q(x) = 3x3 + __
​ 2 ​ x2 – ___
​  4 ​ x – __
​ 8 ​ ; r(x) = –​ ____
16 ​  6. 5x2(x + 5)(x – 4)
17 49 7. 4 – 3a
(c) q(x) = 3x2 + 7x + ___ ​  2 ​;  r(x) = ___ ​  2 ​ 
8. 2(3a – 2)(a – 2)
(d) q(x) = x2 + 1; r(x) = 1
(e) q(x) = 3x2 + 2x + 8; r(x) = 5x + 18 9. (x + 3) units
10. (a – 2) units and (a – 3) units
8. (a) q(u) = u2 + 3u – 2; r(u) = 6
(b) q(u) = u3 – 2u2 + 7u – 15; r(u) = 30
(c) q(u) = 2u4 – 2u3 + 2u2 – u + 1; r(u) = 2
5 28 55 Chapter 3
(d) q(u) = __ ​ 19  ​u – ___
​ 3 ​ u2 + __ ​ 27 ​ ; r(u) = ___
​ 27 ​ 
17 49
(e) q(u) = 3u2 + 7u + ___ ​  2 ​;  r(u) = ___
​  2 ​  Exercise 3.1
x–5
1. (a) x + 7y (f) ​ _____
x + 4 
 ​
9. (a) q(x) = 2x2 + x + 3; r(x) = 5
(b) x – 9y (g) –x
(b) q(x) = x2 – 4x + 8; r(x) = –13
3x2 h2
(c)
3
q(x) = __
3
​ 2 ​ x4 – __
5
​ 4  ​x3 – __
​ 8 ​ x2 + ___
5
​ 16  ​ x – ___
5 101
​ 32  ​;  r(x) = ____
​  32 ​  (c) ​ ___
5 ​   (h) –​ __
2 ​ 
(d) 2x2 1 ​ 
(i) –​ __
(d) q(x) = 7x5 + 8x4 + 8x3 + 8x2 + 20x + 25; 3
r(x) = 22 r+1 7
(e) ​ _____
r + 5 ​  
__
(j) –​  4 ​ 

A2

Pre-Calculus and Calculus.indb 2 13/7/2017 5:57:39 PM


ANSWERS

x x–1
2. (a) ​ ___  ​   (f) ​ _____
x + 2 
 ​ Exercise 3.3
_____
ab
x​√x + 1 ​ + x2
x+y a+3 1. (a) ​​ __________ 2 ​​ 

(b) ​ _____
x – z ​   (g) ​ ______
4a + 2 
 ​  x+1–x _______
​ ​(x + 4)​ ​ ​ 
5x + 20 + √
________________
3
x 1
(c) ​ _____
x + 7 ​   (h) ​ ______
3x – 3y ​ 
(b) ​​     
21 – x ​​ 
__
x a+b x (​√x ​ – 1)
2
_________
(d) ​ ____
x – y ​   (i) ​ _____  
​ (c) ​​   ​​ 
x – 1______  
a–b
3 x+y+2 3x – x​√2x + 1 ​ 
____________
(e) ​ __
n ​ (j) ​ ________
y   ​
  (d) ​​    – x 
4_____ ​​

_____
(e) 3x​(​  √
​ x + 2 ​ – √ )​​
​ x + 1 ​  
Exercise 3.2
1. (a) 2x7
3 Chapter 4
(b) ​ ___ 6 ​ 
5y
4
(c) ​ _____ Exercise 4.1
x – 3 ​ 
1. (a) x = 5 (f) x = 2
b2 + 3b + 2
(d) ​ __________
3 ​
   15
(b) x = –5 (g) x = –​ ___
4 ​ 
(e) m–1
(c) x = –9 22 ​ 
(h) x = –​ ___
(f) 4x3 – 24x2 + 48x – 32 3
3 (d) x = 2 (i) y = –53
2. (a) ​ _____
x + y ​   (c) x
20
y–2 (e) x = –5 (j) z = ___
​ 13 ​ 
x2 – 2x
(b) ​ _____
y – 1 ​   (d) ​ ______
x – 4 ​  23
2. (a) x = –60 (f) x = – ___
​ 11 ​ 
4x – 3
3. (a) ​ ______
 ​   72
7 (b) x = – ___
​ 41 ​   ​ 42
(g) x = ___
19 ​ 
2y +3
(b) ​ ______
3 ​   5
​ 12

(c) x = – __
​ 4 ​   (h) x = – ___
11 ​ 
(c) 3 30
(d) x = ___
​  7 ​   (i) y = 20
(d) 4
26

x2 + 3
(e) ​ _____  ​

  (e) x = – ___
​   ​  
11 (j) x = 16
x2
x2 + 8 375 14 ​ 
(f) ​ ______    
​ 3. (a) a = ____
​  8 ​    (f) x = – ___
​ 13
x2 – 2x
37 7

x2 + 2
(g) ​ ______   ​  (b) y = – ___
​  5 ​   (g) x = ___
​ 3 ​ 
x2 – 16
25
18 – 2b (c) x = __
​  7 ​   ​ 12
(h) x = – ___
7 ​ 
(h) ​ _______  ​ 
b2 – 9 60
8 (d) x = –​ ___
7 ​   (i) x = 1
(i) ​ _____________
      ​
x2 + 2xy – 15y2 25
(e) x = ___
​ 51 ​   ​ 13 ​ 
(j) x = __
10y – 6
(j) ​ _______  
 ​ 
25y2 – 4
Exercise 4.2
7 – 5x
4. (a) ​ _____________
      ​ 3
x3 – 4x2 + x + 6 1. (a) x = 0  or  x = __
​ 5 ​ 
4
(b) ​ _____ 5 5
x – 1 ​  x = __
(b) ​ 7 ​   or  x = –​ __
7 ​ 
(c) 5x + 10 3
x2 – 3xy x = __
(c) ​ 4 ​   or  x = 9
(d) ​ _______
x – y   


x = 0  or  x = 1
(d)
1
(e) ​ _____
x – 2 ​  x = 3  or  x = 2
(e)
1 x = –7  or  x = 3
(f)
(f) ​ _____
x – 1 ​  x = –8  or  x = –2
(g)
x3 – 14x2 + 5x + 20
(g) ​​ ________________
  
5x2
 ​​  (h) ​ 12 ​   or  x = –2
x = __ A3

Pre-Calculus and Calculus.indb 3 13/7/2017 5:57:40 PM


ANSWERS

​ 13 ​   or  x = 2
2. (a) x = – __ 7. (a) A = 2, B = 5, C = 7
___ ___
3 5–√
​ 89 ​  _______
5+√
​ 89 ​ 
(b) ​ 13 ​   or  x = – __
x = __ ​ 5 ​  7 _______
__
(b) x = –1, ​​ 2 ​​ , ​​  4 ​​   or ​​ 
4 ​​   
7
x = – __
(c) ​ 13 ​ 
​ 2 ​   or  x = – __ (c) x = –1.90, –1.87, 1.87 or 1.90
4 ​​ i, __
8. x = –​​ __ 4
(d) ​ 15 ​   or  x = 5
x = __ 3 ​​  3 ​​ i, –5i or 5i
(e) x = –8  or  x = 8
Exercise 4.4
(f) ​ 43 ​   or  x = – __
x = __ ​ 43 ​  1. (a) x = 9 (x = 2 is not a solution.)
16 (b) x = 2  or  x = –2
(g) x = ___
​  3 ​   or  x = 3
(c) x = –3 (x = –35 is not a solution.)
3. (a) k = –13
(d) x = 9 (x = –2 is not a solution.)
(b) x=7
(e) t = 2 (t = –11 is not a solution.)
4. (a) m = 5
(f) x = –3  or  x = –4
(b) x = –13
(g) x = 3 (x = 12 is not a solution.)
5. (a) x = 0  or  x = –25
(h) x = 9 (x = 4 is not a solution.)
(b) x = 36  or  x = 0
(c) x = –6  or  x = –8 2. (a) x = 10
(d) x = 10  or  x = 9 (b) x = –27
(e) x = 6  or  x = –5 (c) x = –34.5
(f) x = 9  or  x = –5 (d) x = 255
__ __
(g) x=5 (e) x = –​​√3 ​​   or  x = √
​​ 3 ​​ 
(h) x = 12  or  x = –5 (f) x = –2  or  x = 2
(i) x = 1  or  x = –8
___ ___
5+√
_______ ​ 17 ​  5 – ​√17 ​ 
_______ Exercise 4.5
(j) x = ​  4 ​    or  x = ​ 
4 ​  

1. (a) x = 6
6. x = 10  or  x = 5
(b) x=2
7. (a) x = –2  or  x = 0 (c) y = –1
16
(b) ​​  45 ​​   or  x = ___
x = __ ​​  5 ​​  (d) x = –1
–i – 5 i–5 (e) x = –1  or  x = 2
(c) x = _____
​​  2 ​​   ____
 or  x = ​​   ​​ 
2  
___ ___ (f) t=2
​ 15 ​ 
√ ​√15 ​ 
(d) x = –1 – ____ ​​  3 ​​i or x

  = –1 + ____
​​  3 ​​i 
  2. (a) x ≈ 2.209
(b) x ≈ –0.170
Exercise 4.3 (c) x ≈ 0.688
1. (a) (3x – 4)(x + 1)(x2 + 3x + 7) 3. (a) 0.250 (b) 51.8 min
___ ___
4
__ __ 3 ____ ​ 19 ​ 
√ __3 ____ ​√19 ​ 
(b) x = –1, ​​ 3 ​​ , –​​  2 ​​  – ​​  2 ​​i 
  or –​​   ​​  + ​​   ​​i 
2 2   Exercise 4.6
2. (a) p = –11 1. (a) y = 25
___ ___
5 7 – ​√33 ​  _______ 7+√
​ 33 ​  (b) x = 7.718
(b) x = –​​ __ ​​ , _______
​​   ​​ 
 or ​​   ​​    9
3 4 4 (c) u = ___
​ 22  ​ 
5 7
3. x = –​​ __ __
2 ​​ , –1 or ​​  3 ​​ 
(d) No solution (t = –2 is not a solution.)
(e) x=4
4. (a) a = – 11, b = –6
__ __
8 (f) x = 1  or  x = 1000
(b) x = –​​√2 ​​ , √ ​​ 2 ​​ or __
​​  5 ​​ 
(g) x = 3  or  x ≈ 1.316
5. x = –3, –4, 2 or 3 (h) x ≈ 0.371
3
6. x = –4, __
​​ 2 ​​  or 6 2. (a) $10 000(1.03)n
(b) 7 years
A4 (c) 24 years

Pre-Calculus and Calculus.indb 4 13/7/2017 5:57:40 PM


ANSWERS

Exercise 4.7 Exercise 5.3


1. 34, 35 and 36 1. (a) x = 2, y = 3
2. 120 students (b) x = 0, y = 0
3. 20 years (c) x = 0, y = –5
4. 105 km (d) x = 4, y = 4
5. 1.2 hours (e) Infinite number of solutions
6. 80 kg (f) No solution
7. 97.5 (g) Infinite number of solutions
8. Length = 15 m, width = 7 m (h) No solution
9. Angel: 12 years old (i) No solution
Christine: 21 years old (j) No solution
10. 6
11. 18 Exercise 5.4
12. 9.5 km/h and 14.5 km/h 1. (a) x = –1, y = 5
13. Angel: 16 years old (b) x = –3, y = –2
Kate: 7 years old (c) Infinite number of solutions
14. 12 am the next day (d) No solution
15. 5 hours (e) x = __​ 23 ​ , y = 2
16. 15 hours
(f) x = __​ 15 ​ , y = –3
17. 120 liters
18. David: 16 years old 2. (a) x = 4, y = 3
Chris: 64 years old (b) x = –​ __ 1 ​ , y = 4
2
19. 51, 53, 55 and 57 (c) t = –9, s = –45
20. $6000 at 9% annual simple interest 6
$64 000 at 14% annual simple interest (d) t = __
​ 5 ​ , s = 1
21. 3 seconds (e) u = 3, v = 2
22. Length = 15 cm, width = 7 cm
Exercise 5.5
23. (b) 25π cm2
1. (a) x = 0, y = 5
24. 142 and 143
(b) x = –3, y = 2
25. 36 cm
(c) x = 1, y = –3
26. 30 years old
(d) ​ 23 ​ , y = __
x = __ ​ 13  ​
(e) Infinite number of solutions
Chapter 5 (f) No solution
2. (a) x = 12, y = 6
Exercise 5.1
(b) ​ 27 ​ , z = __
x = __ ​ 47 ​ 
1. (a) 2
1 ​​ 
(b) ​​ __ (c) ​ 12 ​ , v = 2
u = __
2
5
(c) –3 (d) s = 1, t = – __ ​ 3 ​ 
3
(d) ​​ __
2 ​​ 
(e) y = 3, z = 2

(e) –​​ __2 ​​  Exercise 5.6


3
1. (a) x = 1, y = 3 (d) m = 2, n = 1
(f) –​​  14 ​​ 
__
31 1 ​​   3
(b) p = ___
​​  8 ​​ , q = –​​ __
4 (e) a = __ ​​  12 ​​ 
​​  2 ​​ , b = __
(c) s = –7, t = –13 (f) x = 3, y = 2
A5

Pre-Calculus and Calculus.indb 5 13/7/2017 5:57:40 PM


ANSWERS

Exercise 5.7 x = 1, y = 1, z = 1
(c)
1. 8 and 15 a = 4, b = 3, c = 2
(d)
2. 10 and 25 x = –2, y = 3, z = 1
(e)
3. 47 x = 1, y = –2, z = 1
(f)
4. 58
Exercise 6.3
5. 10 cm by 15 cm
1. (a) x = 1, y = –2, z = –1
6. 8 cm by 12 cm 86 25
11 ​,  x = –​ ___
(b) x1 = ___
​  7 ​,  x2 = –​ ___
7 3 7 ​ 
7. 43 years old
8. Alex: 11 years old Exercise 6.4
Aaron: 16 years old 1. (a) a = 1, b = 1, c = –1
9. 40 chicken sandwiches and 60 ham sandwiches (b) p = 1, q = 4, r = –2
10. 120 girls and 150 boys
11. $30 000 at 8% annual simple interest Exercise 6.5
$60 000 at 10% annual simple interest 1. Food X: 4 mg
12. $22 000 at 5% annual simple interest Food Y: 5 mg
$28 000 at 8% annual simple interest Food Z: 5 mg
13. Matthew: 2 hours 2. (a) L = S + M
Eugene: 3 hours 100S + 50M + 50L = 1300
50S + 50M + 25L = 825

[ ][ ] [ ]
14. Ronie: 3 days 1 1 –1 S 0
Romeo: 6 days (b) ​​  ​ 100​ 
​  ​  50​ ​  ​  50​​   ​ ​​​​  ​ M​ ​  ​  ​​ = ​​  ​ 1300​ ​   ​  ​​
15. 70% alcohol solution: 4 liters 50 50 25 L 825
20% alcohol solution: 6 liters Small: $4
16. 40% muriatic acid solution: 10 liters Medium: $7
20% muriatic acid solution: 10 liters Large: $11
17. Bus A: 80 km/h 3. (a) 10A + 7B + 4C = 2640
Bus B: 60 km/h 10A + 5B + 2C = 2000
18. Train A: 40 km/h 5A + 4B + 3C = 1580
Train B: 60 km/h
19. 40 VIP tickets and 160 non-VIP tickets
20. 17 two-point shots and 5 three-point shots

[
10 7 4 A
(b) ​​  ​10​ ​  ​  5​ ​  2​​  
5 4 3 C
][ ] [ ] 2640
​ ​​​​  ​ B​ ​  ​​ = ​​  ​2000​​  
1580
​ ​​

(c) Cookie A: 100


21. 10 and 20 Cookie B: 120
22. 37 Cookie C: 200
4. (a) z = 5000 + y
x + y + z = 20 000
Chapter 6 0.035x + 0.045y + 0.045z = 850

Exercise 6.1
(b) ​​  ​   
0
[
  
1​ ​  ​ 
0.035 0.045 0.045 z
–1
1​  ​  ​ 
1
][ ] [ ] x 5000
1​​ ​  ​​​​  ​y ​  ​​ = ​​  ​ 20 000​
850
 ​ ​  ​​

1. (a) x = 0, y = –2, z = 2 (c) Investment A = $5000


(b) x = 2, y = –1, z = 1 Investment B = $5000
Investment C = $10 000
(c) x = –3, y = 2, z = 0
(d) x = 2, y = –2, z = 1
(e) x1 = 0, x2 = 1, x3 = –1 Chapter 7
(f) x = 1, y = 2, z = –1
Exercise 7.1
Exercise 6.2
1. (a)
1. (a) u = 8, v = 2, w = 0 5
(b) s = 2, t = 1, u = –1 (b)
A6

Pre-Calculus and Calculus.indb 6 13/7/2017 5:57:40 PM


ANSWERS

(c) –7 0
Exercise 7.3

(d) ​​  34 ​​ 


1. (a) x < __
–3 –1
1 ​​ 
(b) –3 < x < –​​ __
(e) 3
6 10
2 ​​  < x  6
(c) –​​ __
(f) 3
–1
2. (a) x > –2
(g)
–5 0 2 (b) x<4
(h) (c) x<7
–2 8
3
(i) (d) x  __
​  2 ​ 
5 9
10
(e) x  ___
​  7 ​ 
(j)
3 8
2. (a) (–∞, 4] x  58
(f)

4 ​  21
x  ___
(g) 2 ​ 
3
(b) [–3, 8) x  –​ __
(h) 2 ​ 
–3 8 3. (a) x  –5.625
(c) (0, ∞) (b) x > –3
(c) x  –8.16
0
(d) x < 2.625
(d) [–5, ∞) (e) –5.2 < x < 0.4
–5 (f) –28  x  –12
(e) (–∞, 6)
Exercise 7.4
6 ​​  13 ​​ 
1. (a) x < –5, x > __
(f) (–∞, –4] (b) –1  x  3
–4 (c) –3  x < –1
1 ​​ 
(d) x < –2, x > –​​ __
(g) [3, 7] 5
3 7 2. (a) –3 < x < 2
(h) (–2, 9) (b) x  –1, x  4
(c) x < –2, x > 2
–2 9
(d) –4 < x < 4
(i) (0, 3] (e) x  0, x  2
0 3 (f) x  –3
3. (a) [–2, 5) (g) x < –3, x > 5
(h) 2.268 < x < 5.732
(b) {–2, –1, 0, 1, 2, 3, 4, 5, 6, 7, 8}
(c) (–5, 5]  {6, 7, 8} Exercise 7.5
(d) [–2, 5]  {6, 7, 8} 2 ​​  hours
1. 1​​ __
3
(e) {–2, –1, 0, 1, 2, 3, 4} 2. (a) –2
4. (a) [–7, 18] (b) (i) $1970
(b) { } (ii) y=5
(c) (0, 9) 3. 4 marks
(d) [9, 18] 4. (a) $1900
(e) (0, 9) (b) $73 847
A7

Pre-Calculus and Calculus.indb 7 13/7/2017 5:57:40 PM


ANSWERS

5. (a) 28 cm
5
(
(f) Focus: ​​  ​ ___
12  ​,  0  ​​)
(b) 29 cm
5
6. 6.73 cm Directrix: x = –​​ ___
12  ​​ 
7. $20 Vertex: (0, 0)
8. $417 6. (a) x2 = 8y (c) x2 = –4y
9. 8 years (b) y = –8x (d)
2
y2 = 12x
10. $1619 7. (a) Vertex: (3, 0)
Focus: (4, 0)
Chapter 8 (b) Vertex: (1, –4)
Focus: ​​  ​ __
3
(
2 ​ , –4  ​​ )
Exercise 8.1 (c) Vertex: (–12, 3)
1. (a) V (0, 0), F (2, 0), y = 0, x = –2 Focus: ​​  –11​ __ ( 3
)
4 ​ , 3  ​​
( )
(b) V (0, 0), F ​​  0, __
3
​  4 ​   ​​, x = 0, y = –​​ __
3
4 ​​  (d) Vertex: (1, 0)

( )
(c) V (0, 0), F ​​  0, –​ __
3 __ 3
2 ​   ​​, x = 0, y = ​​  2 ​​ 
Focus: (3, 0)
(e) Vertex: (0, 3)
( )
(d) V (0, 0), F ​​  –​ __ 1 ​ , 0  ​​, y = 0, x = __
4 ​​  14 ​​  Focus: (0, 5)
(f) Vertex: (2, –1)
( ) ( )
(e) V ​​  ​ __
3
5
2 ​ , 0  ​​, F ​​  –​ __ 13
___
6 ​ , 0  ​​, y = 0, x = ​​  6 ​​  Focus: (2, 2)
(f) V (2, 0), F (3, 0), y = 0, x = 1 (g) Vertex: (–3, 1)
(g) V (–2, –1), F (–5, –1), y = –1, x = 1 Focus: (–3, 0)
(h) V (–4, 1), F (–4, 0), x = –4, y = 2 (h) Vertex: ​​  –​ __ (
1 ​ , –1  ​​
2 )
3. (a) (y – 2)2 = 20x
(b) (x – 1)2 = 8(y + 2) Focus: ​​  –4​ __ ( 1 ​ , –1  ​​
2 )
(c) (x – 4)2 = 8(y – 1) __ 9
8. (y + 2)2 = –​​  4 ​​ (x + 4)
5. (a) Focus: (0, 1)
Directrix: y = –1 Exercise 8.2
Vertex = (0, 0) 1. (a) C (3, – 5), 4 units (c) C ​​  –​ __
2 (
1 ​ , 0  ​​, __
)
​​  12 ​​  units
___

(
(b) Focus: ​​  0, ___ 1  ​   ​​
​  20 ) (b) C (2, –3), 5 units 1
(
(d) C ​​  –3, ​  2 ​   ​​, ​​ ___
__
)√ 43
​  2 ​ ​​  units
1 2. (a) x2 + y2 + 6x – 8y + 16 = 0
Directrix: y = –​​ ___
20  ​​  (b) x2 + y2 + 12y + 11 = 0
Vertex: (0, 0) (c) x2 + y2 + 2x – 5 = 0

(c) Focus: ​​  ​ __


2 ( )
1 ​ , 0  ​​ 3. (a) (x – 2)2 + (y – 3)2 = 4
(b) (x – 2)2 + (y – 3)2 = 9
1 ​​ 
Directrix: x = –​​ __ (c) (x – 2)2 + (y – 3)2 = 13
2
4. (a) (x + 1)2 + (y – 2)2 = 8
Vertex: (0, 0)
(b) (x – x1)(x – x2) + (y – y1)(y – y2) = 0
3
(
(d) Focus: ​​  –​ __
2 ​ , 0  ​​ ) 5. (a) (x – 2)2 + (y – 1)2 = 25
(b) (x – 1)2 + y2 = 1
3
Directrix: x = __
​​  2 ​​ 
7. (x – 2)2 + (y – 2)2 = 5
Vertex: (0, 0) 8. (a) Equation of tangent: 4y = –3x + 66

( 1 ​   ​​
(e) Focus: ​​  0, –​ __
2 ) Equation of normal: 3y = 4x + 12
(b) Equation of tangent: y = 3x + 1
Equation of normal: 3y = –x – 3
​​  12 ​​ 
Directrix: y = __
Vertex: (0, 0)
A8

Pre-Calculus and Calculus.indb 8 13/7/2017 5:57:40 PM


ANSWERS

___
9. (a) ​​√37 ​​ units (b) Vertices: (±4,
___
0)
(b) 3 units Foci: ​​  ±​√12 ​,  0  )​​
(
___
(c) ​​√91 ​​ units Latus rectum: 2 units
Major axis: 8 units
10. (x – 2)2 + (y – 3)2 = 1
Minor axis: 4 units
11. (a) (x – 1)2 + (y – 2)2 = 25 (c) Vertices: (±2,
__ 0)
(b) (x – 3)2 + (y – 4)2 = 25 Foci: (​​  ±​√3 ​,
   0 )​​
25 Latus rectum: 1 unit
(c) (x – 1)2 + (y + 2)2 = ___
​​  2 ​​  Major axis: 4 units
12. (a) C (1, –2), 2 units Minor axis: 2 units
(b) C (1, 1), 2 units (d) Vertices: (0, ±4)
___ ___
(c) C (0, –3), ​​√11 ​​ units Foci: (​​  0, ±​√12 ​  )​​
Latus rectum: 2 units
(
(d) C ​​  –​ __
4 )
1 ​ , –1  ​​, 1 unit
____
Major axis: 8 units
(e) C (8, –6), 10​​√3 ​​  units Minor axis: 4 units
(e) Vertices: (0, ±1)
13. (a) 11x2 + 11y2 – 119x – 13y + 90 = 0 __
(b) 2x2 + 2y2 + x – 11y – 1 = 0 (  ​√3 ​ 
Foci: ​​ 0, ±​ ___ ​   ​​
2 )
(c) x2 + y2 – 6x – 7y + 15 = 0
​​ 12 ​​  unit
Latus rectum: __
14. (x – 2)2 + (y – 1)2 = 5
Major axis: 2 units
15. (x – 5)2 + (y – 4)2 = 25
Minor axis: 1 unit
16. (x – 6)2 + (y – 8)2 = 25 y2
x2 __
17. (x – 5)2 + (y – 7)2 = 49 5. (a) ​​ ___
16  ​​ + ​​  7 ​​  = 1
18. (7, 4) and (8, 3) x2 ___ y2
(b) ​​ __
9 25  ​​ = 1
 ​​
  + ​​ 
19. (a) (5, 1) and (2, 2)
x2 ___ y2
3 __
(b) (1, –1) and ​​  ​ __
___
1
(
2 ​ , –​  2 ​   ​​ ) (c) ​​ ___
32 36  
 ​​ + ​​    ​​ = 1

20. (a) ​​√42 ​​ units 6. (a) Center: (2, 1)__ __


Foci: (2 – √ ​​ 5 ​​,  1) and (2 + √ ​​ 5 ​​,  1)
(b) 3 units
Vertices: (–1, 1) and (5, 1)
21. (a) Equation of tangent: 5y = –x + 26 Major axis: 6 units
Equation of normal: y = 5x Minor axis: 4 units
(b) Equation of tangent: 5y = –2x – 27 (b) Center: (3, –3) ___ ___
Equation of normal: 2y = 5x – 5 Foci: (3 – √ ​​ 15 ​​,  –3) and (3 + √ ​​ 15 ​​,  –3)
___
22. (a) C (3, 3), ​​√18 ​​ units Vertices: (–1, –3), (7, –3)
(c) y = x Major axis: 8 units
23. A (–1, –2) and B (2, 4) Minor axis: 2 units
Equation of circle: 4x2 + 4y2 – 6x + 7y – 40 = 0 (c) Center: (1, 3)

Exercise 8.3
Foci: ​​ 1 – ____ ( 
​  1__ ) ( 
   ​, 3  ​​ and ​​ 1 +​ ____
2​√3 ​ 
1__
2​√3 ​  )
   ​, 3  ​​

x2
1. (a) ​​ __
9 ​​  + y = 1
2 Vertices: ​​ 1 – ___ ( 
​__3 ​ 
√ ) ( 
​  1__   ​,  3  ​​ and ​​ 1 + ___
​ 3 ​ 
√ )
​  1__   ​,  3  ​​

x y2 x2 ____ y2 2​√3 ​ 
____
(b) ​​ ____  
289 225 ​​ + ____
​​    ​​
  = 1 or ____
​​ 
225 289  ​​ = 1
  ​​ + ​​  Major axis: ​​  3 ​​   units

x2 ___ y 2
Minor axis: 1 unit
(c) ​​ ___
16  ​​ + ​​  12  ​​ = 1 (d) Center: (–10, 5)__ __
4. (a) Vertices: (0,__±3) Foci: (–10 – 5​​√3 ​​ , 5) and (–10 + 5​​√3 ​​,  5)
Foci: (​​  0, ±​√5   ​  )​​ Vertices: (–20, 5) and (0, 5)
8 Major axis: 20 units
Latus rectum: __ ​​ 3 ​​  units
Minor axis: 10 units
Major axis: 6 units
Minor axis: 4 units
A9

Pre-Calculus and Calculus.indb 9 13/7/2017 5:57:40 PM


ANSWERS

(e) Center: (–2, 0) __ __ (b) Center: (0, 0)


Foci: (–2 – √ ​​ 3 ​​,  0) and (–2 + √ ​​ 3 ​​,  0) Foci: (0, –5) and (0, 5)
Vertices: (–4, 0) and (0, 0) Vertices: (0, –3) and (0, 3)
Major axis: 4 units 3 3
Asymptotes: y = –​​ __ __
4 ​​ x and y = ​​  4 ​​ x
Minor axis: 2 units
32
(f) Center: (3, 1) Latus rectum: ___ ​​  3 ​​ units
Foci: (3, –3) and (3, 5)
Vertices: (3, –4) and (3, 6) (c)
Center: (0,___ 0) ___
Major axis: 10 units Foci: (–2​​√10 ​​,  0) and (2​​√10 ​​,  0)
Minor axis: 6 units Vertices: (–2, 0) and (2, 0)
(g) Center: (–1, 2) __ __ Asymptotes: y = –3x and y = 3x
Foci: (​​  –1, 2 – 2​√3   ​  )​​ and (​​  –1, 2 + 2​√3   ​  )​​ Latus rectum: 36 units
Vertices: (–1, –2) and (–1, 6) y2
6. (a) x2 – __ ​​  3 ​​  = 1
Major axis: 8 units
Minor axis: 4 units y2 ___ x2
(b) ​​ __
9 ​​  – ​​  16  ​​ = 1
7. Center: (–4, 0)
x2 ____ y2
Major axis: 8 units (c) ​​ __
4 144  ​​ = 1
 ​​
  – ​​ 
Minor axis: 4 units
7. (a) Center: (–1, 3) __ __
Exercise 8.4 Foci: (–1 – √ ​​ 5 ​​,  3) and (–1 + √ ​​ 5 ​​ , 3)
1. (a) Vertices: (–3, 0) and (3, 0) Vertices: (0, 3), (–2, 3)
___ ___
Foci: (  ​ 13 ​ , 0  )​​ and (​​  ​√13 ​,  0  )​​
​​ –√ Asymptotes: y = 2x + 5 and y = –2x + 1
2 ​​ x and y = __ (b) Center: (8, –6)
Asymptotes: y = –​​ __ 3 ​​  23 ​​ x __ __
Foci: (8 – 2​​√2 ​​,  –6) and (8 + 2​​√2 ​​ , –6)
(b) Vertices: (0, –3) and (0, 3) Vertices: (6, –6) and (10, –6)
___ ___ Asymptotes: y = x – 14 and y = –x + 2
Foci: (  ​​ 0, –​√13 ​  )​​ and (​​  0, ​√13 ​  )​​
3 3 (c) Center: (–1, 0) __ __
Asymptotes: y = –​​ __ __
2 ​​ x and y = ​​  2 ​​ x Foci: (  ​​ –1, –​√5 ​  )​​ and (​​  –1, √ ​ 5 ​  )​​
Vertices: (–1, –1) and (–1, 1)
(c) Vertices: (–8, –1) and (2, –1)
Foci: (–16, –1) and (10, –1) Asymptotes: y = __ ​​  12 ​​ x + __
​​  12  ​​ and y = –​​ __ 1  ​​x – __
1
2 ​​  2 ​​ 
Asymptotes: 5y = 12x + 31 and 5y = –12x – 41 (d) Center: (–3, 1) ___ ___
(d) Vertices: (0, –5) and (0, 5) Foci: (  ​​ –3, 1 – 2​√26 ​  )​​ and (​​  –3, 1 + 2​√26 ​  )​​
___ ___
(  5​√10 ​ 
Foci: ​​ 0, –​ _____
 ​   ) ( 
 ​​ and ​​ 0, ​ 
3
5​√10 ​ 
_____
 ​   ​​
3 ) Vertices: (–3, –9) and (–3, 11)
Asymptotes: y = 5x + 16 and y = –5x – 14
Asymptotes: y = –3x and y = 3x (e) Center: (3, 0)
(e) Vertices: (1, –3) and (1, –1) Foci: (–2, 0) and (8, 0)
___ ___ Vertices: (0, 0) and (6, 0)
Foci: (  ​​ 1, –2 – √ ​ 10 ​  )​​ and (​​  1, –2 + √ ​ 10 ​  )​​
Asymptotes: 3y = x – 7 and 3y = –x – 5 Asymptotes: y = __ ​​  43 ​​ x – 4 and y = –​​ __ 4 ​​ x + 4
3
2. (a) 72y2 – 9x2 = 8 8. (b) Center: (1, 1)
x2 ___ y2 ___ ___
(b) ​​ ___ 16 65  ​​ – ​​    ​​ = 1 Vertices: (1 – √
​​ 12 ​​,  1) and (1 + √
​​ 12 ​​,  1)
___ ___
(c) y2 – 9x2 = 9
(d) x2 – 12y2 = 16 √ 12
5
(c) –​​ ___ √ 12
5
​    ​ ​​  and ​​ ___
​    ​ ​​  

x2 ___ y2
3. ​​ ___ 
 ​​
16 4  – ​​   ​​  = 1

5. (a) Center: (0, 0) ___ ___


Foci: (  ​​ 0, –​√26 ​  )​​ and (​​  0, √
​ 26 ​  )​​
Vertices: (0, –1) and (0, 1)
Asymptotes: y = –​​ __ 1 ​​ x and y = __ ​​  15 ​​ x
5
Latus rectum: 50 units

A10

Pre-Calculus and Calculus.indb 10 13/7/2017 5:57:41 PM


ANSWERS

Chapter 9 2. Quadrant Base angle Sign


sin 30º I 30º +ve
Exercise 9.1 cos 120º II 60º –ve
3π 7π
1. (a) ​​ ___
4 ​​   (f) ​​ ___
4 ​​  tan 190º III 10º +ve
7π π tan 350º IV 10º –ve
(b) ​​ ___
9 ​​   (g) –​​ __
3 ​​  cos (–220º) II 40º –ve
7π 5π
(c) –​​ ___
4 ​​   (h) ​​ ___
4 ​​ 
cos 330º IV 30º +ve

50π 25π 3.
(d) ​​ ____
3 ​​    (i) –​​ ____
18 ​​ 
Base angle Sign Value
__
sin 120º 60º +ve ___​√3 ​ 
5π 43π ​​   ​​ 
(e) ​​ ___
2 ​​   (j) –​​ ____
36 ​​ 
2
cos 240º 60º –ve 1 ​​ 
–​​ __
2. (a) 135º (f) –150º 2
(b) –240º (g) –540º tan 350º 10º –ve –0.176
(c) 308.57º (h) –308.57º sec 220º 40º –ve –1.305
(d) 330º (i) 660º
4. (a) 0.940
(e) –315º (j) 756º (b) 0.5
2π 11π 89π (c) 0.766
3. (a) ​​ ___
9 ​​   (c) ​​ ____
9 ​​    (e) ​​ _____
1440  ​​  (d) 0.5
2π π π
(b) ​​ ___
3 ​​   (d) ​​ ___
12  ​​   (f) ​​ __
9 ​​ 
(e) 1.192
5
4. (a) 143.24º (d) 77.14º ​​  12
5. (a) sin θ = ___ ___ ___12
13 ​​ , cos θ = ​​  13  ​​,  tan θ = ​​  5 ​​ ,
(b) 1145.92º (e) 270º 13 13 5
sec θ = ___
​​  5 ​​ , cosec θ = ___
​​  12 ​​ , cot θ = ___
​​  12  ​​ 
(c) 15º (f) 67.5º
3 3 5
7π 5π 28π (b) sin θ = __ ​​  45 ​​ , tan θ = __
​​  5 ​​ , cos θ = __ ​​  4 ​​ , sec θ = __
​​  4 ​​ ,
5. (a) ​​ ___
36 ​​   (c) –​​ ____
6 ​​   (e) –​​ _____
45 ​​ 
5


(b) ​​ ___
83π
(d) ​​ ___
13π
(f) –​​ __ cosec θ = __ ​​  43 ​​ 
​​  3 ​​ , cot θ = __
5 ​​   36 ​​   180 ​​ 
7
6. (a) –135º (c) –36º (e) –20º ​​  24
(c) sin θ = ___ ___ ___24
25 ​​ , cos θ = ​​  25  ​​,  tan θ = ​​  7 ​​ ,
(b) 85.94º (d) 19.10º
25 25 7
π π π sec θ = ___
​​  7 ​​ , cosec θ = ___
​​  24 ​​ , cot θ = ___
​​  24  ​​ 
7. (a) ​​ __
4 ​​  rad (b) ​​ __
3 ​​  rad (c) ​​ __
4 ​​  rad
6. (a) 3.6397 (b) 8.4853 (c) 11.7368
8. (a) 20º (b) 67º (c) 60º
7. (a) 179.9805 (c) 7.7236
Exercise 9.2 (b) 115.8163 (d) 112.8807
__
1. (a) x = 5​​√2 ​​ 
__ 1 ​​  
8. (a) ​​ __ 1__   ​​  
(c) –​​ ___ (e) 1
(b) x = 4​​√2 ​​  2 ​ __
√ 3 ​ 
__ __
(c) x = 3​​√2 ​​  ​ 2 ​ 
√ ​ 2 ​ 

___ (b) –​​ ___
2 ​​  
___
(d) –​​  2 ​​   (f) 2
(d) x = 2​​√34 ​​ 
2. (a) r = 10 Exercise 9.5
(b) q = 24 1. 75.39 m
(c) p + q = 31 2. 40.41 m
(d) r + s + q = 42 3. 373.21 m
4. 52.85 m
3. (a) No
5. 40.89º
(b) No
(c) Yes
Exercise 9.3
1. (a) 60º (d) 48º (g) 40º
(b) 60º (e) 19º (h) 60º
(c) 40º (f) 50º (i) 27º
A11

Pre-Calculus and Calculus.indb 11 13/7/2017 5:57:41 PM


ANSWERS

Chapter 10 (  )
​  12 ​ , ∞  ​​
(m) Domain: ​​ –__
Range: (0, ∞)
Exercise 10.2 (n) Domain: 
1. (a) One-to-one relation Range: [0, ∞)
A function (o) Domain: 
(b) Many-to-one relation Range: [0, ∞)
A function (p) Domain:  \ {–1}
(c) Many-to-many relation Range: (–∞, – 1)  [0, ∞)
Not a function
(d) One-to-many relation ​​  13 ​​ 
2. (a)  f  (2) = __
Not a function   f (–1): undefined
(e) One-to-one relation   f (a + h): ________ ​​  1   ​​ 
A function a+h+1
(f) Many-to-one relation (b)  f (2): undefined
A function   f (–1): undefined ________
2. (a) A function
(b) Not a function
  √
f (a + h): ​​ ________ ​ 
a+h+2
a+h–2
 ​ ​​ 

(c)  f (2) = 2
(c) Not a function
(d) A function   f  (–1) = –​​ __ 1 ​​ 
2
(e) Not a function |a + h|
(f) A function   f (a + h) = ________ ​​    ​​ 
a+h–1
3. (a) A function (d)  f (2) = 5
(b) A function   f (–1) = –4
(c) Not a function   f (a + h) = (a + h)2 + 2(a + h) – 3
4. (a) A function
3. (a) A function
(b) Not a function
Domain: [0, 4]
Exercise 10.3 Range: [0, 5]
1. (a) Domain: [3, ∞) (b) A function
Range: [0, ∞) Domain: 
(b) Domain: (–∞, 3] Range: (0, ∞)
Range: [0, ∞) (c) A function

[ 
Domain:  \ {0}
(c) Domain: ​​ –__ )
​  12 ​ , ∞  ​​ Range:  \ {0}
Range: [0, ∞) (d) A function
(d) Domain:  Domain: 
Range: [0, ∞) Range: 
(e) Domain:  \ {0} 4. (a) Domain: 
Range:  \ {0} Range: 
(f) Domain:  \ {–2} (b) Domain: 
Range:  \ {0} Range: (0, ∞)
(g) Domain:  \ {1} (c) Domain: 
Range:  \ {0} Range: (–1, ∞)
(h) Domain: (–4, ∞) (d) Domain: [–1, ∞)
Range: (0, ∞) Range: (–∞, 1]
(i) Domain:  \ {–1, 1} (e) Domain: (–1, ∞)
Range: (–∞, –1)  (0, ∞) Range: 
(j) Domain: (–∞, –1)  (1, ∞) (f) Domain: (3, ∞)
Range: (0, ∞) Range: 
(k) Domain: (–∞, –1)  [1, ∞) (g) Domain:  \ {1}
Range: [0, ∞) \ {1} Range:  \ {2}
(l) Domain:  \ {–1} (h) Domain: 
Range:  \ {–2} Range: [2, ∞)
A12

Pre-Calculus and Calculus.indb 12 13/7/2017 5:57:41 PM


ANSWERS

(i) Domain: [1, ∞) (b) ( f  – g)(x) = x – 2 – x2


Range: [2, ∞) Domain: 
(j) Domain: (–∞, 0]
Range: [0, ∞) (c) (g  h)(x) = x
(k) Domain:  Domain:  \ {0}
Range: (–∞, 0)
(l) Domain: 
(d) ​​ __
g
(  )
​   ​  ​​(x) = x3
h
Domain:  \ {0}
Range: [2, ∞)
(m) Domain: (–2, ∞) 6. (a)  f  g = 4 – x2
Range:  (b) g  f  = (1 – x)2 – 3
(n) Domain: 
(c)  f  h = 1 – __ ​​ 2x ​​
Range: [0, ∞)
(o) Domain:  (d) h  h = x
Range:  (e)  f  g  h = 4 – __ ​​ 42  ​​ 
x
5. (a) Odd 7. k(x) = x2
(b) Even __
8.  f  g  h = √ ​​ ex  ​​
(c) Even
Domain: 
(d) Odd
9. (a)  f  g  h = __ ​​  12  ​​ + 4
Exercise 10.4 x
Domain:  \ {0}
​​  12  ​​x + 9, Df  g = , Rf  g = 
1. (a)  f  g = __ (b) x = 0, y = 4
g  f  = __​​  12  ​​x + 8, Dg  f = , Rg  f =  10. h(x) = 3 – x
Domain:  \ {3}
(b)  f  g = (3x + 7)2 + 2, Df  g = , Rf  g = [2, ∞)
g  f  = 3x2 + 13, Dg  f = , Rg  f = [13, ∞) Exercise 10.5
1. (a) Not one-to-one
(c)  f  g = x + 3, Df  g = [–2, ∞), Rf  g = [1, ∞) (b) Not one-to-one
_____ __
g  f  = √ , Dg  f = , Rg  f = [​​√3 ​​,  ∞)
​​ x2 + 3 ​​  (c) Not one-to-one
(d) One-to-one
​​  2 1 
(d)  f  g = __________
(x + 3)2 + 1 f  g (  1
, D = , Rf  g = ​​ 0, ​ ___
 ​​  ]
10  ​  ​​  
x–2
f –1(x) = ln ​​ _____
​  3 ​  
 ​​ (  )
g  f  = ​​​ _____(  x +1 )
​  2 1   ​  ​​ ​​ + 3, Dg  f = , Rg  f = (3, 4]
2 ​​D​f ​​ = (2, ∞)
​​R​f ​​ = 
–1

–1

(e) One-to-one
(e)  f  g = _____​​  1   ​​, Df  g = , Rf  g = (0, 1]
|x| + 1   f –1(x) = 1 – x
g  f  = ​​  1   ​​, Dg  f = , Rg  f = (0, ∞)
_____ ​​D​f ​​ = R
​​ f​ ​​ = 
–1 –1

|x + 1| (f) One-to-one
3 ___
x+1
_____   f –1(x) = ​​√  –x  ​​
2. (a)  f (x) = ​​  x     ​​, g(x) = x2 or 
​​D​f ​​ = R
​​ f​ ​​ = 
–1 –1

x +12
 f (x) = _____
​​  2 ​​,   g(x) = x 3. (a) One-to-one
x
_____ x–3
3
(b)  f (x) = ​​√  x + 1 ​​  , g(x) = 2x or    f –1(x) = _____
​​  2 ​​   
3 __
 f (x) = ​​√  x  ​​, g(x) = 2x + 1 ​​D​f ​​ = –1

(c)  f (x) = x2, g(x) = x – 5 (b) One-to-one


|x|   ​​  1x ​​
f –1(x) = __
(d)  f (x) = _____
​​  x +  1 ​​, g(x) = x + 2  or 
​​D​f ​​ =  \ {0}
–1

|x – 1|
 f (x) = _____
​​  x    ​​, g(x) = x + 3
  (c) One-to-one
  f –1(x) = 5 – x2
3. ( f  g)(3) = 8 ​​D​f ​​ = [0, ∞)
–1

(g  f )(–1) = 0
(d) One-to-one
4.  f (x) = x 3 ___
  f –1(x) = ​​√  x ​​ 
5. (a) ( f  + g)(x) = x2 + x – 2
Domain:  ​​D​f ​​ = –1

A13

Pre-Calculus and Calculus.indb 13 13/7/2017 5:57:41 PM


ANSWERS

ex – 2
4. (b)  f –1(x) = _____
​​  3 ​​    3. (a) Df = , Rf = (0, ∞)

(c) Df = R​ ​​ f ​​ = ​​ –__
–1
(  )
​  23 ​ , ∞  ​​
(b) Df = , Rf = (–2, ∞)
(c) Df = , Rf = (0, ∞)
Rf = D​ ​​ f ​​ = 
–1

x+3 (d) Df = , Rf = (3, ∞)


5. (a)  f –1(x) = _____ ​​  x – 2 ​​  (e) Df = (–2, ∞), Rf = 
(b) ​​  24
c = ___ 7 ​​ 
(f) Df = (2, ∞), Rf = 
(g) Df = (–2, ∞), Rf = 
6.  f –1(x) = ln (x – 1) – 2
​​D​f ​​ = (1, ∞)
–1
(h) Df = (–2, ∞), Rf = 
​​Rf​ ​​ = 
–1 4.  f   g = x2, ​​Df​  g​​ = (0, ∞), ​​R​ f  g​​ = (0, ∞)
4x g   f  = 2x, ​​Dg​   f​​ = , ​​Rg​   f​​ = 
7. (a)  f  f = _____
​​  x +  1 ​​  , x ≠ –1 and 1
5. (a)  f (x) = ex – 1, g(x) = 2x or 
x
(b)  f –1(x) = _____ ​​  x – 2 ​​   f (x) = ex, g(x) = 2x – 1
(b)  f (x) = 2x + 2, g(x) = ln x
​​D​f ​​ =  \ {2}
–1

(c) g(x) = x2 + 2x Exercise 11.8


8. (a) Not one-to-one 1. (a) y = 3 cos x (d) y = cos 4x
(b) y = 3 sin 4x (e) y = sin 6x
(b) g–1(x) = __ ​​  12 ​​  ln x (c) y = 2 tan 2x
​​D​g ​​ = (0, ∞) –1
Exercise 11.9
(c) g–1  f  = __ ​​  12 ​​  ln (2 – x2) 1. θ = __
π
​​  3 ​​ 
9. (a) Df = , Rf = [2, ∞) 2. 0.287
Dg = [–2,_____ ∞), Rg = [0, ∞) 3. x = 0.306
(b) g  f = √
​​ x2 +_____
4 ​​  4. 1
gfg=√ ​​ x + 6 ​​  Exercise 11.10
(d) [0, ∞) 1. (a) 0.648 (d) –10.018
10. (a) Df = , Rf = [0, ∞) (b) 2.164 (e) –0.381
Dg = Rg =  (c) 1 (f) –0.100
(b)  f  g = 2|x + 1| 2. x = 0.549
(c) Not one-to-one
(d) (–∞, –1] or [–1, ∞) Chapter 12

Chapter 11 Exercise 12.2


1. (a) 1
Exercise 11.4 (b) 7c + 3
7 (c) 0
2. (b) x = __ ​​  3 ​​  (d) –2__
13 (e) ​​√7 ​​ 
3. (b) x = –​​ ___ 4 ​​  (f) 3
6. x = –2 (g) Does not exist
​  13 ​​  or x = 7
7. x = –​__ 1 ​​ 
(h) ​​ __
6
Exercise 11.7 (i) 0
1. (a) log3 y = x 2 ​​ 
(j) ​​ __
9
(b) ​​ __1 ​​  log y = x 1 ​​ 
2 3 (k) –​​ __
2
(c) ​​ __5 [  (  ) ]
1 ​​  ​​ ln ​ __ y
​  2  ​  ​– 1  ​​ = x (l) ​​ _____ 1 __ ​​ 
14​√3 ​ 
1 ​​ a y = x
2. (a) ​​ __
y
__
(b) ​​a​​ 2  ​​​ = x
2 (m) 29
3
(n) ​​ __
2 ​​ 
A14

Pre-Calculus and Calculus.indb 14 13/7/2017 5:57:41 PM


ANSWERS

2. ​​  lim​– ​​​   f (x) = 5, ​​ lim​+ ​​​   f (x) = 16 (h) x = 1, x = –1, y = –1


x→2 x→2
​​  lim​ ​​​   f (x) does not exist. (i) x = 0
x→2 2. x = –3, x = 3, y = –1, y = 1
3. ​​  lim​ –​​​   f (x) = e–2, ​​  lim​ +​​​   f (x) = e–2 3. (a) (–4, 4)
x → –2 x → –2
​​  lim​ ​​​   f (x) = e–2 (b) x = –4, x = 4
x → –2
(c) No vertical asymptote
4. ​​  lim​– ​​​   f (x) = 0, ​​ lim​+ ​​​   f (x) = 0 4x
x→0 x→0 (d)  f –1(x) = ________
​​  ______    ​​ 
​​  lim​ ​​​   f (x) = 0 ​√​x2​ ​+ 1 ​ 
x→0
​​D​f ​​= (–4, 4)
–1

5. ​​  lim​+ ​​​   f (x) = 3, ​​ lim​+ ​​​   f (x) = 2


x→4 x→0
​​  lim​ ​​​   f (x) = 2 Exercise 12.5
x→3
1. (a) Continuous
6. ​​  lim​ ​​​   f (x) does not exist.
x → –2 (b) Continuous
​​  lim​ ​​​   f (x) = 0 (c) Not continuous
x→0
​​  lim​ ​​​   f (x) = 1 (d) Not continuous
x→2
(e) Continuous
1 ​​ 
7. (a) ​​ __
6 (f) Continuous at x = –1
(b) 0 Not continuous at x = 1
(c) Does not exist 2. Continuous for all x  
(d) ​​ __1 ​​  3. k = 0
4
(e) 0 4. (a) k = 1 (b) Not continuous
(f) 2 5. Continuous
6. (a) Continuous
Exercise 12.3
(b) {x: (–∞, 0)  [3, ∞)}
1. (a) +∞
7. (a) (–∞, –3]  [3, ∞)
(b) –∞
(b) k=9
(c) –∞
(d) +∞ 8. (a) 1 (b) Continuous
(e) –∞ 9. (a)   f (a) is defined, x​​ lim​ →a

​​​  f (x) exists and
(f) Does not exist ​​ xlim​
→a

​​​  f (x) =  f (a).
(g) 0 (b) (ii) Not continuous at
(h) 0
(i) +∞ ​​  12 ​​ , q = –1
10. p = __
(j) 1
(k) –∞
(l) 0 Chapter 13
1 ​​ 
(m) –​​ __
2 Exercise 13.2
3
__ 1. (a) 2 (b) 2x
(n) ​​  2 ​​ 
1   ​​  
2. (a) ​​ ______
_____ 1  ​​ 
(c) –​​ __
(o) 4 ​ x + 1 ​ 
√ x2
(p) +∞ 1___
(b) 3x2 (d) –​​ ______    ​​ 
1 ​​ 
(q) ​​ __ __ 2x​√2x  ​
6
(r) 0 3. (a) 1 (c) ​​__ √
3
​  2 ​ ​​    (e) –2
2. 2 (b) 12 (d) –4

Exercise 12.4 Exercise 13.4


1. (a) x = 1, x = –1, y = 0 5 __​ 32 ​ 
1. (a) ​​ __
(b) x = 2, y = –1 2 ​​​​ x​ ​​ (d) 1
(c) x = 0, y = 1 (b) 12x2 – 6x (e) 4(2x + 1)
(d) x = –2, y = 0 (c) ​​ ____ 1__ __ 1
(e) x = 3, x = –3, y = 0 2​√ x  ​​ ​ – ​​  x3  ​​   (f) 3
(f) y = 1, y = –1
(g) x = 0, y = 1, y = –1 A15

Pre-Calculus and Calculus.indb 15 13/7/2017 5:57:41 PM


ANSWERS

2. (a) 36x3 + 63x2 + 6x + 7 y2 – x2 2y


2. (a) ​​ ______  ​​    (b) ​​ ___2 ​​ 
(b) (8x + 5)11(x3 + 7)12(408x3 + 195x2 + 672) y3 x
(c) 3x­2 + 2x – 1 3. 0
–2   ​​  8
(d) ​​ ______ 4. (a) –​​ __ 9 ​​   (b) –1
(x – 1)2
dy 1 ___ d2y 1
x(x3 + 6x + 4)
(e) ​​ ____________
    ​​  6. ___
​​   ​​ = ___
​​  12  ​​ , ​​  2 ​​ = –​​ ___
32  ​​ 
(x2 + 2)2 dx dx
__ 1  __  ​​ 
(f) ​​ __________ Exercise 13.7
​ x  ​(1 – ​√x  ​)2

1. (a) –18e–3x
3. (a) 30(3x + 2)9 (b) (1 + 6x2)​​ex​ + 2x ​​
3

–6
(b) ​​ ________
   ​​  (c) 2e2x + 3e–3x
(3y + 5)3 (d) 2e–2x
1   ​​ 
(c) ​​ ________ 4
__ (e) 6e6x – ____ ​​  26x   ​​ 
​3(3 – x)​3​
​   ​ 
3e
x (f) 4(e4x + e–4x)
(d) ​​ ________  __3 ​​  (g) 6(ex + e–x)5(ex – e–x)
​   ​ 
​(25 – x2)​2​
(h) xe3x(3x + 2)
2x
(e) ​​ ________   __2 ​​  2 ​​  ax ln a
​   ​ 
​3(x2 + 5)​3​ (i) ​​ __
5
( 
_____
(f) 5(5t + √ ​​ t2 + 1 ​​) ​ 5 + ______
4​
​  _____
t
   ​  
​√​t2​ ​+ 1 ​  ) ​​ 1__ 2​√__x ​ 
___
(j) ​​  √
​ x   ​​ ​1  ​​0​ ​​ ln 10
4. (a) ​​ ________ –2   ​​  1 ​​
_____
2. (a) ​​ __
x
x2​√​x2​ ​+ 2 ​ 
3(x – 3)2(x + 10) 6x + 2
_______
(b) ​​  2     
​​
(b) ​​ ______________
    ​​  3x + 2x
(3x + 4)3
(c) ​​ ______ 1
2(3x + 1)2(41 – 12x)
(c) ​​ _________________
       ​​ 2x – 1 ​​ 
​  4 ​ 
__
4x3
​3(3 – x)​3​ (d) ​​ ____________
      ​​
(x2 – 1)(x2 + 1)
25
(d) ​​ ________   __3 ​​  2(2x – 3)
(e) ​​ ________ ​​ 
​   ​ 
​(25 – x2)​2​
x(x – 2)
x
(e) ​​ ___________________
   _________________  _____  ​​ 2(x2 + x + 1)
2​√(​    x​ ​+ 3)(1 + √
2
​ ​x2​ ​+ 3 ​)  ​ _____________
(f) ​​   
    ​​
(1 – x2)(1 + 2x)
(2t – 3) (26t
2 2 2
– 3)
(f) ​​ _______________
   _ ​​  
(g) ​​ ______
10
2​√t  ​ 2x – 1 ​​ 
(  ) ( 
​ 1x ​  ​​ ​​​​ 1 – __ ) ( 
​ 12  ​   ​​, ​​​ 3√
__
) (  x–1
1__  ​   2​​ ​​​​ _____
)
__ __ __ __
5
5. ​​​ 6x + __ ​ x  ​ + ___
​  √
​ x  ​ ​  __​ 32 ​  ​  ​​,
(​√x  ​+ 1) ln (​√x  ​+ 1) – (​√x ​ – 1) ln (​√x ​ – 1)
__________________________________
x (h) ​​          __ __ __  ​​
​2x​​  ​ 2​√x  ​(​√x  ​– 1)[ln (​√x  ​+ 1)]2
4x – 5
3(x – 1)(5x2 + 8x + 5)(x + 1)2(x2 + 1)5
______________________________
​​     (i) ​​ __________ log e
  ​​  
   17  ​​
___ 2x2 – 5x – 1 2
​   ​ 
​2x​​  2 ​ 1 log e + _____
(j) ​​ _____
x – 2 ​​   m
​​  2 1– x ​​  
logn e
Exercise 13.5
–3
1. 18 (k) ​​ ___
x ​​  log10 e
​​  4  3 ​​ 
2. ______
(1 – x) 3. (a) ex ​​  ​ __(
2 ​+ ln x​ 2​ ​  ​​
x )
3x2 – 16x + 32
____________
3. ​​    __5  ​​ 
​   ​ 
1
______
(b) ​​  _____    ​​ 
​4(x – 2)​2​ ​ x + 1 ​ 

____________ –4ex
4. 24x (c) ​​        ​​
(2 – ex)(2 + ex)
2 – x ln x 2
Exercise 13.6 (d) ​​ _________ xex    ​​

x y – y4
1. (a) ​​ __y ​​ (d) ​​ ___________
2xy3 + x – y2
   ​​ 
( 10​e2x
4. ​​ ______
​  2x  
e –2
​​
)
 ​  ​​[ln (e2x – 2)]4,
[  ]
–y x
(b) ​​ ___
x ​​   (e) –​​ __y ​​ x2[ln (e2x – 2)]4 ​​ ______
10x​e2x
​  2x  
​​
 ​+ 3 ln (e2x – 2)  ​​
e –2
x(1 – y2) 1 – 4x – y
(c) ​​ _______  ​​   (f) ​​ ________ x    ​​

y(x2 – 1)
A16

Pre-Calculus and Calculus.indb 16 13/7/2017 5:57:41 PM


ANSWERS

6. (a) 2xln x – 1 ln x Exercise 13.11


(b) xx(1 + ln x) 1. (a) ​​ __1 ​​ 
(c) ( ​e2x
2 ​ + ______
y ​​  ​ __
​​
x ​  e2x + 3 ​  ​​ ) (b) 1
4

(d) y ​​  ​ __ (


3 _____ 2 _____2x 3​x2​ ​
+ _____
x ​ + ​  x – 1 ​ – ​  x2 –  2 ​  ​ 
1 – x3
  ​  ​​ ) (c) 1
(d) 2
3
Exercise 13.8 (e) ​​ __2 ​​ 
1. sec2 x, –cot x cosec x 1 ​​ 
(f) ​​ __2
1 ​​  sin __
2. (a) –​​ __ ​​  17 ​​ x
7
(g) ​​  16 ​​ 
__
(b) 3 cos 3x + 3 sec2 3x
(c) 4 tan 4x sec 4x – 3 cot 3x cosec 3x 2. (a) ∞
(d) 3 cos (3x + 7) (b) 1
(e) (4x + 3) cos (2x2 + 3x + 2) 4 ​​ 
(f) 2 sin 2x (sec2 2x + 1) (c) ​​ __
9
(g) 2e2x (tan 2x + sec2 2x) 1 ​​ 
(h) 2x sec 4x (1 + 2x tan 4x) (d) –​​ __
2
(i) tan 2x (cos2 x – sin2 x) + 2 sin x cos x sec2 2x (e) 0
(j) 4 sin (2x + 3) cos (2x + 3) 3. (a) 0
(k) 12x cos2 (2x2) sin (2x2) (b) 0
(l) –8 cosec2 (2x – 7) cot (2x – 7)
1 ​​ 
(c) ​​ __
3. (a) –8 sin 4x 2
(b) 18 sec2 3x (sec2 3x + 2 tan2 3x)
(c) –2 cosec 2x (cot2 2x + 2 cosec2 2x) Chapter 14
Exercise 13.9
Exercise 14.1
1. (a) ​​ ____________
  
2   ​​
__________
x​√1 – ​4(ln x)​2​ ​  1. (a) y = 4x – 4, x + 4y = 18
(b) y = 24x – 46, x + 24y = 1204
–3e3x – 1
(b) ​​ _________
_______    ​​  (c) y = –x + 1, y = x + 1
​√1 – ​e6x ​ – 2​ ​ 
(d) y = –8x + 5, x = 8y + 25
–​ex​ ​  (sin x + 2x cos x)
2

_________________
(c) ​​         ​​ (e) y = –18x – 54, x + 3 = 18y
​e2x
​ ​+ cos2 x
2

2. y = 2x – 10
Exercise 13.10 3. (2, 4), (4, –8)

( 
1. (a) 33x – 63
1 ​​ x – __
(b) ​​ __
5 4. ​​ –__ ) 16
​  13 ​ , –​ ___
3 ​   ​​
3 ​​  3 ​​ 
5. (1, 2), (3, 4)
(c) 3x – 2
6. p = 3, q = 4
2. L(x) = 2x
ln (0.98) = –0.02 7. 12y – x, y + 12x = 75
ln (1.01) = 0.01 8. (a) y = –x, y = x – 2π
__ __
3. L(x)
____ = 2.066x – 0.258 (b) 24y + 6​​√3 ​​ x = 12 + √ ​​ 3 ​​π

__ __
​​ ____
√ 14.8 ​​ ≈ 3.846 6​​√3 ​​ y – 24x = 3​​√3 ​​ – 4π
​​ 15.2 ​​ ≈ 3.899
√ 9. 4y + 2x = 3; 2y – 4x + 1 = 0
4. (a) dy = 6​​​ 3x – __ (  ) ( 
​ 1x ​  ​​ ​​​​ 3 + __
5
)
​ 12  ​   ​​ dx
​x​ ​
dy
10. ​​ ___  ​​= 3 – 6a
2x – 1 dx
(b) dy = _______​​  _____    ​​ dx
2​√​x2​ ​ – x ​  Exercise 14.2
5. (a) dy = ______ ​​  2x2– 1 ​​  dx, 0.00667 1. 1458 cm3 s–1
(b) dy = –2e sin (ex) dx, 0.0337
x 2. 4π cm2 s–1
6. 57.805 cm2 ​​  12 ​​  cm s–1
3. __

4. ___
​​  161  ​​ cm s–1
A17

Pre-Calculus and Calculus.indb 17 13/7/2017 5:57:42 PM


ANSWERS

5. ___ 1  ​​ cm s–1


​​  8π 3. (b) 10 cm3
1  ​​ cm s–1 (c) 6 cm × 3 cm × 1cm
6. ___
​​  8π 4. (b) 2 __
9
7. ____
​​  10π    ​​ m/min 3​√3 ​  2 2
6. ____
​​  8 ​​r 
  cm

8. (a) 300 (c) –3.64 g/s


Exercise 14.6
(b) –6e–0.02t (d) –2 g/s 10
1. (a) 8x2 – 36x + 48 + ___ ​​  x ​​ 
9. (a) 300e0.03T
(b) 24x – 72x + 48
2
(b) 17 years
(c) $2528.50
(c) $677/year
(d) $26, 2 units
10. (a) 30
2. (a) 800x – 0.2x2
(c) –0.347 ºC/min
(b) 800 – 0.2x
11. (a) 0 g (c) 22.9 s (c) 800 – 0.4x
(b) 2e–0.04t (d) 1.20 g/s (d) $760
12. (a) 8.66 min (e) $800,000, 2000 units
(b) –4.31 g/min 3. (a) 60x – 0.3x2 – 100
(c) –8 g/min 100
(b) 60 – 0.3x – ____
​​  x   ​​ 
Exercise 14.3 (c) 60 – 0.6x
1. (a) (3, –1): minimum (d) $29
(b) (–2, 19): point of inflection (e) $2900, 100 units
(0, 3): minimum 4. (a) 4000 + 8x
(c) (0, 0): minimum (b) $400,000, 10,000 toys
(2, 4): maximum (c) $320,000
(d) (0, 0): point of inflection (d) $302,250
​​ __ ( 
3 27
​  2 ​ , ___ )
​  16 ​   ​​: maximum 5. $46, 40 units
(e) (2, –2): maximum 7. (a) $5,000,000
(2, –4): minimum (b) Selling price = $500
Cost price = $150.70
2. (a) (0, 0): maximum
(4, –9.071): minimum 8. (b) 2500 spare parts
(b) (0, 6.350): maximum (c) $2
(4, 0): minimum 9. (a) 60x – 0.004x2, $225,000
(–4, 0): minimum (b) 55x – 0.004x2 – 1000, $188,062.50, 6875 pots
__
3. (a) ​​ __( 
π π
​  6 ​ , ​ __
6  ​
  + )​
√ 3 ​   ​​: maximum
(c) $32.50
(d) $168,000
(  )
__
5π 5π
​​ ___​  6 ​,  ​ ___ 6  ​
  – ​
√ 3 ​   ​​: minimum 10. (a) Loss of $300,000
(b) $240,000, 200 items
__
(  )
π
(b) ​​ ​  2 ​ , 1  ​​: maximum 11. (a) R(x) = 100x – 5x2
​​ ___ ( 
3π 1
​  2 ​,  –​ __ )
3 ​   ​​: minimum
C(x) = 20x2 + 50x
Π(x) = 50x – 25x2
Exercise 14.4 (b) 10 units, $50
2. (b) (–1, –1) (c) Maximum profit = $25
(c) (–1, –1): minimum Maximum revenue = $95
(0, 0): point of inflection 12. (a) $50,000
3. (a) x = –1 (b) $50
(b) (–2, –4): maximum 13. (a) R′(q) = 500 – 4q
(0, 0): minimum p(q) = 500 – 2q
(b) 75 units
Exercise 14.5 (c) Maximum profit = $10,950
1. 5 cm by 5 cm Selling price = $350
2. 32 cm
A18

Pre-Calculus and Calculus.indb 18 13/7/2017 5:57:42 PM

You might also like